®
v
v
v
v

 







501
501
GMAT
®
QUESTIONS
501_00_FM_i-x_501_master.qxd 4/29/13 12:23 PM Page i501_00_FM_i-x_501_master.qxd 4/29/13 12:23 PM Page ii
Blank Page
501
501
GMAT
®
QUESTIONS
NEW YORK
®
501_00_FM_i-x_501_master.qxd 4/29/13 12:23 PM Page iii
Copyright © 2013 LearningExpress, LLC.
All rights reserved under International and Pan American Copyright Conventions.
Published in the United States by LearningExpress, LLC, New York.
Library of Congress Cataloging-in-Publication Data
501 GMAT questions.—First edition.
pages cm
ISBN 978-1-57685-920-9
1. Graduate Management Admission Test—Study guides. 2. Management—
Examinations, questions, etc. 3. Business—Examinations, questions, etc. 4. Verbal
ability—Examinations, questions, etc. 5. Mathematics—Examinations, questions, etc.
6. Universities and colleges—United States—Graduate work—Examinations—Study
guides. I. LearningExpress (Organization) II. Title: Five hundred one GMAT
questions. III. Title: Five hundred one GMAT questions.
HF1118.A14 2013
650.076—dc23
2013001029
Printed in the United States of America
9 8 7 6 5 4 3 2 1
First Edition
ISBN 978-1-57685-920-9
*GMAT is a registered trademark of the Graduate Management Admission Council
®
(GMAT
®
),
which was not involved in the production of, and does not endorse, this product.
501_00_FM_i-x_501_master.qxd 4/29/13 12:23 PM Page iv
The content in this book has been reviewed and
updated by the LearningExpress Team in 2019.
501_00_FM_i-x_501_master.qxd 4/29/13 12:23 PM Page v
Contents
Introduction vii
SECTION 1 GMAT Writing Section 1
1 Analytical Writing Assessment 3
SECTION 2 GMAT Verbal Section 25
2 Critical Reasoning 27
3 Sentence Correction 121
4 Reading Comprehension 235
SECTION 3 GMAT Integrated Reasoning Section 299
5 Graphics Interpretation 301
6 Two-Part Analysis 331
7 Table Analysis 371
8 Multisource Reasoning 397
SECTION 4 GMAT Quantitative Section 415
9 Arithmetic 417
10 Algebra 441
11 Geometry 473
vi
501 GMAT
®
Questions
501_00_FM_i-x_501_master.qxd 4/29/13 12:23 PM Page vi
501_00_FM_i-x_501_master.qxd 4/29/13 12:23 PM Page vii
Introduction
Congratulations! By purchasing this book, you’re investing in your
future by taking a step toward obtaining an MBA degree, which in turn will
sharpen your business and analytical skills. 501 GMAT
®
Questions will
familiarize you with the Graduate Management Admission Test (GMAT
®
)
and its format.
When you take the test, remember that the GMAT is a computer adap-
tive test (CAT), which means that as you answer each question, a harder
question or an easier question will appear. Therefore, you will not be able
to skip a question and come back to it later, or change your answer to any
previous questions. If you find that you are having trouble with a particu-
lar question, try to eliminate some of the wrong answer choices, reread the
passage one more time, and then make your best guess. Do not leave any
questions unanswered.
How to Use this Book
This book is divided into four distinct sections, each representing a sec-
tion on the official GMAT: Writing, Verbal, Integrated Reasoning, and
Quantitative.
Review the instructional sections in each chapter before answering the
questions in that chapter. The answers to all questions are at the end of each
corresponding chapter. Each answer is fully explained, so if you have trou-
ble with a particular question, you will be able to figure out how to arrive
at the correct answer. Take your time; refer to the instructions as frequently
as you need to, and be sure you understand the answer explanations at the
end of each chapter. This book provides the practice; you provide the ini-
tiative and perseverance. Good luck!
viii
501 GMAT
®
Questions
501_00_FM_i-x_501_master.qxd 4/29/13 12:23 PM Page viii
501
501
GMAT
®
QUESTIONS
501_00_FM_i-x_501_master.qxd 4/29/13 12:23 PM Page ix
501_00_FM_i-x_501_master.qxd 4/29/13 12:23 PM Page x
Blank Page
501_01_1-24_501_master.qxd 4/29/13 12:26 PM Page 1
SECTION 1
GMAT
Writing Section
Strong writing skills are a prized asset in the business world. The abil-
ity to make a cohesive argument and communicate it clearly is something
that you can use in any facet of the business world. The GMAT Analytical
Writing Assessment aims to test these skills.
501_01_1-24_501_master.qxd 4/29/13 12:26 PM Page 2
Blank Page
1
Analytical
Writing Assessment
501_01_1-24_501_master.qxd 4/29/13 12:26 PM Page 3
The Analytical Writing Assessment (AWA) is designed to measure
your ability to think critically and to cohesively express your ideas. The
AWA consists of one 30-minute essay: Analysis of an Argument.
The test will use topics of general interest connected to business or a
variety of other subjects to generate your prompts. No specific knowledge
of the essay topic is necessary to write a high-scoring essay—only your
capacity to write analytically will be measured.
You will be asked to analyze the reasoning behind an argument that is
provided and then write a critique of that argument. You will not be asked
to present your own views on the subject.
All AWA prompts share a similar format that includes a backdrop for the
prompt, the prompt statement, and instructions. It looks like this:
The following appeared in the editorial section of a corporate
newsletter:
The common notion that workers are generally apathetic about man-
agement issues is false, or at least outdated: a recently published survey
indicates that 79 percent of the nearly 1,200 workers who responded to
survey questionnaires expressed a high level of interest in the topics of
corporate restructuring and redesign of benefits programs.
Discuss how well reasoned you find this argument. In your discussion
be sure to analyze the line of reasoning and the use of evidence in the
argument. For example, you may need to consider what questionable
assumptions underlie the thinking and what alternative explanations
or counterexamples might weaken the conclusion. You can also dis-
cuss what sort of evidence would strengthen or refute the argument,
what changes in the argument would make it more logically sound,
and what, if anything, would help you better evaluate its conclusion.
4
501 GMAT
®
Questions
501_01_1-24_501_master.qxd 4/29/13 12:26 PM Page 4
Developing Your Essay
When developing your essay, consider the following:
What questionable assumptions underlie the thinking behind
the argument?
What alternative explanations or counterexamples might
weaken the conclusion?
What sort of evidence could help strengthen or refute the
argument?
The AWA tests your ability to construct a thoughtful and unified critique
of a specific statement based on a specific line of reasoning. An effective way
to approach your argument is to create a brief, organized outline. Within
this outline, create:
5
an introduction that states a clear position in response to the
prompt with some brief support
a body that elaborates on each supportive idea with relevant
details and examples or experiences that define each supportive
idea
a conclusion that ties together all the parts of your argument
Improving Your Score
In order to improve your score, be aware of proper grammar, spelling, and
vocabulary. Using varied vocabulary and sentence structure (as long as both
are used effectively) will have a positive impact on your score.
Another way to boost your score is to employ some tactics that will sep-
arate your essay from the masses. Some ways to do this:
Correctly use relevant facts, statistics, and historical examples.
(Almost one-third of freshmen entering high school will
graduate.)
Make well-known literary references. (To quote Dickens, “It was
the best of times, it was the worst of times,” for everyone who
remained employed at the peak of the recession.)
Craft relevant and intelligent metaphors. (“All the world’s a
stage, and upon it our elected officials are merely players.”)
A smart way to make your position stronger is to offer thoughts on the
other side of the argument. Briefly addressing these counterpoints and
exposing their weaknesses can only strengthen your argument. It is easier
to make your position appear right after you have dispelled assumptions
from opposing perspectives.
Timing
It is important to keep track of time while developing your argument for the
AWA and to pace yourself accordingly. An incomplete essay will have a neg-
ative impact on the overall scoring of the AWA section of your exam.
501 GMAT
®
Questions
501_01_1-24_501_master.qxd 4/29/13 12:26 PM Page 5
Here is a good guideline to keep in mind for the usage of your allotted
time:
consider your issue or argument (1 minute)
choose your position, develop an outline (4–6 minutes)
create the body of your essay (22–24 minutes)
proofread (1–2 minutes)
Understanding Your Score
Each essay in the AWA section will be given two independent ratings, one
of which may be performed by an automated essay-scoring engine. The auto-
mated essay-scoring engine is an electronic system that evaluates over 50
structural and linguistic features, which include the organization of ideas,
syntactic variety, and topical analysis.
If the two ratings differ by more than one point, a separate and third eval-
uation by an expert reader is required to resolve the discrepancy and deter-
mine the final score.
The two scores are then used to create an average score. Scores range
from 0 to 6, including half-point intervals. These scores are computed sep-
arately from the multiple-choice sections of the GMAT and will not affect
your Verbal, Quantitative, or Total scores.
College and university faculty members trained as readers for the AWA
will consider the following:
the overall quality of your ideas about the argument presented
your overall ability to organize, develop, and express those
ideas
the relevant supporting reasons and examples you used
your ability to control the elements of standard written
English
Business schools that receive your GMAT score will also receive your AWA
score. While most business schools will not place a great deal of importance
on a mediocre score, a good score can help boost your application.
Examinees whose first language is not English will have their essays con-
sidered with some sensitivity to the elements of standard written English.
6
501 GMAT
®
Questions
501_01_1-24_501_master.qxd 4/29/13 12:26 PM Page 6
7
Set 1
Create a response to the following sample prompts.
1. The following appeared in the opinion column of a financial
magazine:
A powerful business tycoon is in a better position to influence the
legislation, and ultimately the destiny, of a society or nation than
any political leader or government official can in their position.
Discuss how well reasoned you find this argument. In your
discussion be sure to analyze the line of reasoning and the use of
evidence in the argument. For example, you may need to consider
what questionable assumptions underlie the thinking and what
alternative explanations or counterexamples might weaken the
conclusion. You can also discuss what sort of evidence would
strengthen or refute the argument, what changes in the argument
would make it more logically sound, and what, if anything, would
help you better evaluate its conclusion.
2. The following appeared as part of a newspaper editorial:
Throughout our society’s history there have been people who
dedicate themselves to changing the norms and set the bar anew
for generations to come. To leave such a mark on society takes
immense ambition, and we remember best the people who broke
the rules to achieve success.
Discuss how well reasoned you find this argument. In your
discussion be sure to analyze the line of reasoning and the use of
evidence in the argument. For example, you may need to consider
what questionable assumptions underlie the thinking and what
alternative explanations or counterexamples might weaken the
conclusion. You can also discuss what sort of evidence would
strengthen or refute the argument, what changes in the argument
would make it more logically sound, and what, if anything, would
help you better evaluate its conclusion.
501 GMAT
®
Questions
501_01_1-24_501_master.qxd 4/29/13 12:26 PM Page 7
3. The following is an excerpt from a memo written by the head of a
governmental department:
Organizations should attempt to remove the large number of
positions and salary levels that categorize employees by skill and
experience, because a flat structure is more likely to foster a
congenial working environment within the company.
Discuss how well reasoned you find this argument. In your
discussion be sure to analyze the line of reasoning and the use of
evidence in the argument. For example, you may need to consider
what questionable assumptions underlie the thinking and what
alternative explanations or counterexamples might weaken the
conclusion. You can also discuss what sort of evidence would
strengthen or refute the argument, what changes in the argument
would make it more logically sound, and what, if anything, would
help you better evaluate its conclusion.
4. The following appeared in the editorial section of a college
newspaper:
In a world of ever-changing economic climates, it seems clear the
global impact of a nation should be weighed by its artistic and
scientific accomplishments rather than its accomplishments in
business and economic progress.
Discuss how well reasoned you find this argument. In your
discussion be sure to analyze the line of reasoning and the use of
evidence in the argument. For example, you may need to consider
what questionable assumptions underlie the thinking and what
alternative explanations or counterexamples might weaken the
conclusion. You can also discuss what sort of evidence would
strengthen or refute the argument, what changes in the argument
would make it more logically sound, and what, if anything, would
help you better evaluate its conclusion.
8
501 GMAT
®
Questions
501_01_1-24_501_master.qxd 4/29/13 12:26 PM Page 8
9
5. The following appeared in a blog about education:
In order to maximize students’ potential, educators should create
an atmosphere of freedom and spontaneity in a learning
environment, rather than maintain a highly structured
environment that emphasizes discipline, punctuality, and routine.
Discuss how well reasoned you find this argument. In your
discussion be sure to analyze the line of reasoning and the use of
evidence in the argument. For example, you may need to consider
what questionable assumptions underlie the thinking and what
alternative explanations or counterexamples might weaken the
conclusion. You can also discuss what sort of evidence would
strengthen or refute the argument, what changes in the argument
would make it more logically sound, and what, if anything, would
help you better evaluate its conclusion.
6. The following appeared as part of an article in a trade magazine:
Because there are already so many personal distractions to be had
at work, such as Facebook and texting, employees should keep
their private lives and personal activities as separate as possible
from the workplace. In this way, an employee can focus on work
and little else.
Discuss how well reasoned you find this argument. In your
discussion be sure to analyze the line of reasoning and the use of
evidence in the argument. For example, you may need to consider
what questionable assumptions underlie the thinking and what
alternative explanations or counterexamples might weaken the
conclusion. You can also discuss what sort of evidence would
strengthen or refute the argument, what changes in the argument
would make it more logically sound, and what, if anything, would
help you better evaluate its conclusion.
501 GMAT
®
Questions
501_01_1-24_501_master.qxd 4/29/13 12:26 PM Page 9
7. The following appeared in a research paper written for an
introductory economics course:
Competition in the marketplace is beneficial to businesses because
it inspires changes and improvements to the status quo. Businesses
that are ill-equipped to compete in a bustling market will not
likely last very long. The true mettle of a company and its product
and services is tested within the open market.
Discuss how well reasoned you find this argument. In your
discussion be sure to analyze the line of reasoning and the use of
evidence in the argument. For example, you may need to consider
what questionable assumptions underlie the thinking and what
alternative explanations or counterexamples might weaken the
conclusion. You can also discuss what sort of evidence would
strengthen or refute the argument, what changes in the argument
would make it more logically sound, and what, if anything, would
help you better evaluate its conclusion.
8. The following appeared in the opinion column of a financial
magazine:
These are times in which people find themselves strapped for cash
while working multiple jobs or unemployed with little notice. The
average job listed online or in local classifieds can sometimes have
up to seventy applicants, if not more. With this in mind, the
upcoming crop of soon-to-be college graduates should consider
financial compensation as the main factor in deciding on a career
path.
Discuss how well reasoned you find this argument. In your
discussion be sure to analyze the line of reasoning and the use of
evidence in the argument. For example, you may need to consider
what questionable assumptions underlie the thinking and what
alternative explanations or counterexamples might weaken the
conclusion. You can also discuss what sort of evidence would
strengthen or refute the argument, what changes in the argument
would make it more logically sound, and what, if anything, would
help you better evaluate its conclusion.
10
501 GMAT
®
Questions
501_01_1-24_501_master.qxd 4/29/13 12:26 PM Page 10
11
9. The following appeared in the media section of a magazine on
trends and lifestyles:
Because there is such a range of diverse cultures that make up the
world, the degree of censorship for improper content and language
in arts and entertainment media varies in different countries.
Discuss how well reasoned you find this argument. In your
discussion be sure to analyze the line of reasoning and the use of
evidence in the argument. For example, you may need to consider
what questionable assumptions underlie the thinking and what
alternative explanations or counterexamples might weaken the
conclusion. You can also discuss what sort of evidence would
strengthen or refute the argument, what changes in the argument
would make it more logically sound, and what, if anything, would
help you better evaluate its conclusion.
10. The following appeared in an article in a consumer products
magazine:
When considering an array of advertising from a smattering of
different cultures from around the globe, one thing appears true:
the ideas of a nation become clear when observing its advertise-
ments. All aspects of a nation’s advertisements draw together to
paint a cultural story that moves with the population.
Explain what you think this quotation means and discuss how true
or false it may be. Develop your argument with reasons and/or
specific examples drawn from history, current events, or your own
experience, observation, or reading.
501 GMAT
®
Questions
501_01_1-24_501_master.qxd 4/29/13 12:26 PM Page 11
Answers—Set 1
Each sample essay below represents a score-6 response.
1. This statement, a powerful business tycoon is in a better position to
influence the destiny of a society or nation than any political leader or
government official, has a good deal of truth to it, but it should be
noted that a powerful business tycoon can easily influence
politicians to do what is in their interests as well. It is well-known
that lobbyists, who work in the interests of powerful businesses,
have a major influence on policy in Washington. Business tycoons,
and corporations, also have the power to greatly influence
elections with their money by funding campaigns, buying airtime,
and producing ads to achieve their personal ends. Furthermore,
tycoons and corporations can influence the destiny of a nation
by developing media and products that change the way people
do things.
Lobbyists are influential people who work for powerful
companies, or groups of companies, to secure their interests in
Washington. Lobbyists will often spend millions to influence
politicians regarding legislation and the positions they take on
certain issues. While the actual tycoons are not out there meeting
directly with politicians, their lobbyists are doing this work for
them. Lobbyists’ influence on legislation is far more profound than
most citizens realize.
Another way that tycoons and corporations with large
amounts of money can influence the destiny of a nation is through
campaign contributions. Perhaps one of the most effective ways to
influence the people’s vote is to control the message sent to the
voting public. Buying airtime and creating campaign press that will
influence the way people perceive a candidate can change the
outcome of an election, particularly if the tycoon happens to be a
media magnate.
On another level, a tycoon or successful corporation can
influence the destiny of a nation on a basic level—creating
something that is integral to the cultural fabric of the nation. This
can be anything from pharmaceuticals to media. Two examples of
this are Dow Chemical and Rupert Murdoch. Dow is an umbrella
company for most of the creators, researchers, and providers of
12
501 GMAT
®
Questions
501_01_1-24_501_master.qxd 4/29/13 12:26 PM Page 12
13
chemical content in everything from cosmetics to pesticides;
therefore, its presence in our daily lives is highly pervasive. Rupert
Murdoch, who owns and operates NewsCorp, is behind Fox News
Channel, which is available to upward of 102 million homes in the
United States. It is easy to influence a nation when you are a daily
part of its life.
While it is not necessarily a healthy truth, it is a truth
nonetheless that tycoons (or otherwise powerful entities) are in a
better position to influence the destiny of a country than any
politician. They are, in fact, in positions to influence the politicians
themselves to a great extent, consequently influencing legislation
and eventually our everyday lives.
2. There are several ways to understand the statement, and not every
angle is favorable or even true. It is not definitive to say that the
people we remember best are remembered because they broke the
rules. A great many people in history who are remembered for
their contributions to society are memorable because their
accomplishments benefited the masses, regardless of how they
achieved the ends. Also, to remember someone for breaking the
rules may not be a positive memory, as is the case with infamous
criminals. It might be more honest to say that the people who we
remember for changing the game did so by breaking the rules.
It is not entirely true that we remember people because they
broke the rules. Many people, like Mother Teresa and Gandhi,
built a lifetime of accomplishments by sticking very closely to the
rules in everything they did. Of course we can think of many, like
Steve Jobs and Rupert Murdoch, who certainly have achieved
monumental success for having done things their way, against the
grain, but it would be incorrect to use them as definitive examples.
Additionally, rule breakers whose accomplishments are
remembered are not necessarily positive forces in our society.
Bernard Madoff would be considered a breaker of rules. He did
everything he wasn’t supposed to do and benefited greatly
financially, yet he destroyed the lives of many people and families
who trusted him. While we may remember him, it isn’t because he
did anything good.
Perhaps a more refined way to state the sentiments would be
to say that “the people we remember best are the ones who played
501 GMAT
®
Questions
501_01_1-24_501_master.qxd 4/29/13 12:26 PM Page 13
the game by their own rules.” Intrinsic in this statement is that the
person in question had an independent mind-set and did not give
up on his or her dreams when things seemed difficult. By bringing
to light a person’s individuality rather than insubordination, this
statement can remain a positive inspiration to others.
In sum, it is not a reliable statement to say we remember
those who broke the rules. To put it this way lends this statement
to much interpretation that isn’t necessarily positive or useful. To
say that we remember those who did things their own way would
be more appropriate.
3. While the fundamental premise of the argument, that a flat structure
is likely to foster a more congenial work atmosphere, may be true, it
could prove disastrous for the organization. Not every position at
every organization requires the same amount of daily hours or the
same skill set from the individual occupying said position. Some
positions may require extensive and specific education, while other
positions at the same organization may not require any secondary
education or training at all. While it might be fair to imply that the
disparity between some people at the very top of the organizational
structure and the rest of the organization’s employees is unjust,
imposing a flat salary structure would be too drastic. Furthermore,
to initiate such a drastic policy could end in a mass exodus of
higher-level employees and spell the end for an organization.
To say to an employee with an applicable secondary degree
that he or she will now be compensated on the same level as
another employee who may lack the same credentials can easily be
taken as an insult. Engineers and technicians who work at the same
automobile factory will have different tasks in their positions. They
may all be equally important in the grand scope of fabricating a
car, but it would be unfair to say that every job at that automobile
plant requires the same skill set and experience level. While the
people who design the cars, the people who test the cars, and the
people who build the cars are all working toward the same end,
they have committed to their craft on different levels. Also,
different employees in the same position may have varying degrees
of experience as well as education.
Another reason why it would be a bad business plan to change
an existing salary structure to a flat rate is that the change would
14
501 GMAT
®
Questions
501_01_1-24_501_master.qxd 4/29/13 12:26 PM Page 14
15
have far too drastic an impact on some employees’ finances. Once
a person has become accustomed to living with certain means, or
salary, it is very difficult to change that salary without causing the
employee great stress. Imagine living on a salary that gets reduced
by a third or even a quarter: Some sacrifices must be grudgingly
made to adapt. A disgruntled employee is rarely a productive
employee.
Lastly, when you have a business full of top employees who
are undervalued, you risk a mass exodus of prime employees. Of
course, there will be a great many employees on the lower scale
who will be happy, but one must consider whether these employees
are more vital than the ones whose pay will be reduced. Once all
the people who design or create your automobiles have left the
organization, you would have to start from the bottom to rebuild.
This could quickly end up costing the company far more than it
ever hoped to save from changing its salary structure.
To be fair, all employees at any organization should be paid
with considerable generosity to maintain a healthy work environ-
ment. People do not generally work hard when they feel
undervalued, and the same goes for the employees who work at the
very top of the salary, and position, pyramid.
4. It is impossible to judge the value of a nation solely on its contribu-
tions to the arts and sciences. Historically, it is the impact of various
factors within a society that shape the standard of living of a nation.
Business and economic progress set the stage for success in arts and
sciences research and accomplishments. A healthy economy and
global business engagement can only increase the internal wealth
and infrastructure of a country, which can advance a nation in its
cultural accomplishments.
To assert that only artistic and scientific accomplishments can
define a nation’s worth is heavily one-sided. While the massive
totem heads found on Easter Island are largely considered a
magnificent accomplishment in art and sculpture, is it fair to say
that this Pacific island nation should be valued as a most accom-
plished nation? A tribal nation that lacked the infrastructure to
prosper as a first world country should hardly be in the same
category as, say, Italy for its respective achievements in art. It is an
incongruous comparison at best.
501 GMAT
®
Questions
501_01_1-24_501_master.qxd 4/29/13 12:26 PM Page 15
In most countries where the economy is growing, people tend
to have more formal education. This is in large part because a
growing economy requires all sorts of people in varying profes-
sions to be successful in their pursuits. These pursuits can be
exporting and importing goods, creating software, or telecommu-
nications. Businesses that engage with the economies of other
nations also open the door to other cultures, and therefore increase
a nation’s understanding of the world and humanity.
Furthermore, a robust economy creates a heartier cash flow
for a nation’s citizens. When food and shelter are stable, it enables
a society to pursue more intellectual fields and consequently
develop ideas and research in the arts and sciences. Countries that
have a high or competitive GDP also tend to be countries with
highly educated citizens and a high percentage of people with
secondary degrees. This type of intellectual slant on society creates
the right environment for success and accomplishment in the arts
and sciences.
So, you see it would be folly to sum up a nation’s value relying
only on a tally of accomplishments in the arts and sciences. While
achievements in the arts and sciences are a good indicator of a
nation’s cultural progress, they are by no means an overall
illustration of standards. A better method of calculating a
nation’s value would be to consider all accomplishments in the
arts and sciences as well as its economic progress and business
achievements.
5. People learn in many different ways and find inspiration in a
variety of environments. Some students require the structure and
discipline of a routine-heavy system of learning, while others may
shut down in this environment. Conversely, a learning
environment with little restrictions and an inclination toward
creativity and spontaneity may be very inspiring to some, but
others may need help focusing on tasks and topics. The latter is a
method that can surely inspire advanced thinking, but either
method could easily alienate either type of learner.
While a free-learning environment would be the most
inspiring, for some students it is a challenge to remain focused on a
task for an extended period of time. A significant portion of
students in today’s classrooms could potentially be classified within
16
501 GMAT
®
Questions
501_01_1-24_501_master.qxd 4/29/13 12:26 PM Page 16
17
a wide range of learning disabilities. These disabilities could be
easily treated with medication, or by creating the boundaries of an
effective routine. For students like these, a free-learning environ-
ment may pose a challenge or even a threat. While some students
will thrive in a free-learning environment, others will unfortu-
nately become alienated by this method.
Other students, who perhaps are more creatively inclined or
experimental-minded, will thrive in an open, spontaneous learning
environment. In preschool this is referred to as “free-play,” where
students can decide what games they play and what they will learn.
In this scenario, the teacher allows the student to lead the way and
supports the student’s exploration. Some students respond to this
freedom with an enhanced desire to learn and a drive to explore
tasks and topics with even greater depth. For students who can
handle this type of setting, it can be a mind-opening experience
that stokes the flames of curiosity. Furthermore, it impresses in the
young student’s mind that learning can be a thrilling experience
and he or she can feel great inspiration by controlling the process.
Particularly advanced students may become bored with tasks and
topics within a routine-heavy and structured environment.
The most successful classrooms and learning environments
will incorporate both styles of education effectively. It falls upon
the educators, as well as the families of the students, to work to
better understand the minds of each and every student in a
classroom, so that an environment that strives not to alienate
anyone is born. This is a task that may seem impossible for a lone
teacher with a classroom full of students, but there can be ways
around this conundrum. For example, a teacher might assign
students to groups that tackle the same exact tasks and topics in
different ways.
In sum, it is not effective to rely on one method of teaching
to maximize all students’ potential. Even though an open learning
environment may prove to be more inspiring and motivational, it
can potentially alienate students who are not prepared for these
scenarios. Also, students who require more responsibility so that
they remain interested in tasks and topics may lose interest in
lessons. The ideal learning environment is one that is sensitive to
students’ individual needs and responds to those needs accordingly.
501 GMAT
®
Questions
501_01_1-24_501_master.qxd 4/29/13 12:26 PM Page 17
6. This statement is essentially great advice, but in reality it may be
difficult to follow. In cases where an individual works in a
profession that requires her or him to be social, it may prove to be
almost impossible to draw such a hard line between professional
and personal aspects of life. Some professions will require so much
of one’s time that it would require one to basically live behind a
facade. Still, in most settings it is the safest approach toward one’s
job whenever possible.
Some professions require people to be socially, and even
emotionally, available to others. One such profession is sales.
Whether a person is selling insurance, cars, or ideas, he or she will
be expected to spend a great deal of time interacting with clients.
In this situation it might prove disastrous to keep your clients at a
distance. Most people who do this type of work walk a fine line
between being genuine and being disconnected. Additionally, it
behooves a good salesperson to know his or her market/clientele in
order to keep providing them with the goods and services that are
most relevant to them. In order to do this, salespeople must be
ready to give of themselves so that they may get some of the others.
Professionals like lawyers, doctors, and journalists may spend
unusual amounts of time at work with others. In these situations a
person may become depressed or unmotivated after long stretches
of time without connecting to anyone. Perhaps getting a drink
with some coworkers at the end of the day and chatting about each
other’s lives and interests may bond them so that they can work
together more effectively. While engaging in this behavior is
always a risk, and could end up in social divisions or rifts, it might
be unavoidable.
Despite the human need to make connections with each
other, there are still some professions in which fraternizing may
become detrimental to one’s position. While teachers may enjoy
creating personal connections with each other, it would be
inappropriate for a teacher to engage in similar relationships with
his or her students. Similarly, while social workers explore their
clients’ personal lives and activities, they should largely remain
an anonymous presence in their clients’ lives to maintain
professionalism.
Employees certainly should strive to maintain their personal
lives and activities as separate from work as possible, but in some
18
501 GMAT
®
Questions
501_01_1-24_501_master.qxd 4/29/13 12:26 PM Page 18
19
cases this is not likely or emotionally healthy. This philosophy is
certainly the easiest and most effective way to reduce potential for
things like prejudice and emotional confusion between coworkers,
but, unfortunately, it is not always the most realistic approach.
7. Competitive businesses tend to grow and adapt at a fast and
flexible rate. In the struggle to remain on top of a particular
market, a business must understand the needs and expectations of
its client base. One method of collecting this data is to assess and
analyze the models of competitive businesses. In doing so a
business learns more about itself in relation to the marketplace,
and also readies itself to compete with rival businesses. Sometimes
this leads businesses to adopt methods and models “outside the
box” in order to stay one step ahead of their rivals. Adapting and
changing the perspectives and expectations of how a business
should be run increases the business’s ability to survive and rise to
the top.
The lifeline of a business is its connection to its clientele.
Success in business is a codependent relationship with its clients.
When several businesses are in competition to serve the same
demographic, it makes perfect sense to monitor the methods of
rival businesses. For example, when Apple develops a new phone, it
is imperative for Samsung and other competitors to find out as
much as possible about this new product before it even hits the
market. The more data these rival companies can collect, the more
likely they can come up with a competitive phone before they lose
a portion of their business to this new product. The only way to
stay new and popular is to constantly try to outperform other
businesses in your marketplace.
In some cases a competitive marketplace will lead a business
to change its internal model on a regular basis. This is good
because change in the marketplace is less likely to happen when
the internal workings of a business remain stagnant. This also
helps reduce bloating of staff and unnecessary protocol. A regular
restructuring and purge of unnecessary positions is vital to
maintaining a healthy business model. Much like the rivalry
between Microsoft and Apple, it makes sense to emulate the model
of a more successful business in a shared marketplace. By creating
competitive products and functioning with a competitive business
501 GMAT
®
Questions
501_01_1-24_501_master.qxd 4/29/13 12:26 PM Page 19
model, a business continues to increase its chances of remaining
relevant to its client base.
More importantly than emulating its rivals, a company must
always be able to conceive of ways to stay one step ahead of them.
Necessity is often an impetus to radical invention, and while no
business wishes to find itself in a desperate situation, it certainly
increases the likelihood of risk-taking. In situations where a
business must struggle against a very competitive market, it pays to
think of unconventional ways of reaching and satisfying the
clientele.
In summation, competition in the marketplace is vital for the
growth and survival of any company. Without competition, a
business risks becoming irrelevant, antiquated, and out of touch.
8. It is an age-old struggle for young people: deciding on a path for
the future. When carving out a career path, what factors should be
most important? Some may say it is a better life decision to do
what makes you happy. Others believe the most important thing in
a career is stability and financial security. Some people are lucky to
find themselves inclined to pursue a career that is lucrative and also
makes them very happy. Still others may dedicate themselves to the
struggle of doing what they love without the promise of steady pay.
I feel that if you do not enjoy your profession, there is no amount
of money that can make you happy.
Many people choose fields of study in college that will guide
them toward a reliable career with opportunities for growth and
exponential financial compensation. Some such fields may include
medicine or law. In either of these fields of study, the possible
careers one may forge are complicated and deeply engrossing.
Most lawyers and doctors are prone to work long, intense hours
under massive amounts of pressure, but the compensation for their
invested time is substantial. Furthermore, doctors and lawyers will
always have a professional place in society. This may all be true,
but one can hardly imagine dedicating one’s life to such professions
without receiving some sort of personal satisfaction.
Others decide to pursue livelihoods doing what they love
most, even if it may not be financially secure. For example, take
the notion of the “starving artist” as an illustration of someone
who pursues a career in a financially unstable field. Actors, writers,
20
501 GMAT
®
Questions
501_01_1-24_501_master.qxd 4/29/13 12:26 PM Page 20
21
and musicians can easily fit this description, although many artists
do get to enjoy a great deal of professional and financial success.
Being able to spend most of one’s time doing what one loves most
may prove to be highly fulfilling, but it often does not guarantee
any sort of financial security for the future. While this professional
fulfillment may be profound, as time passes it is overshadowed by a
lack of stability.
The luckiest people are those who take great joy in the
professions they have chosen in financially viable fields. Doctors
who love to study medicine and lawyers who get a thrill from
practicing law have the best of both worlds. To do what one
loves best and be amply compensated from this work is truly to
have it all.
In the end, a person may pursue a lifetime of work that he or
she is not passionate about, but the financial compensation may
never be enough for this sacrifice. Conversely, even though it may
be thrilling to dedicate your life to pursuing your passion, it may
get old when there is never any stability. The best confluence of
factors would be to find a lucrative way to do something that
makes you happy.
9. Censorship in arts and entertainment varies from country to
country, as cultures and media will vary. In most cases the result is
a confluence of two factors: a country’s media infrastructure (access
to the Internet, cable/satellite, etc.) and that country’s cultural
inclinations (religion, type of government, etc.). A third factor
when considering a government’s control over media content is
relevant demographics. A country’s power of censorship should be
primarily driven by these three factors to provide a fair and
relevant cultural experience for its people.
Available access to media is something that varies greatly
across the globe. While many nations, like the United States, can
easily connect to the Internet from the home, there are many
nations that still largely rely on public access to the Internet, such
as cafes. In cases of countries that lack a pervasive electrical grid
that extends power to the individual homes of its inhabitants (e.g.,
tribal areas of Africa), access to the Internet or TV may be a
greater problem than regulation, thereby limiting the govern-
ment’s ability to control what type of content the people can
501 GMAT
®
Questions
501_01_1-24_501_master.qxd 4/29/13 12:26 PM Page 21
access. Countries that engage in open flow of content exchange,
such as purchasing international media to run on national TV and
radio, will have greater opportunities to shape that nation’s
programming and to fine-tune the message of that content.
Perhaps the greatest regulator of a culture’s media is its
government type and religious inclinations. China’s firewall and
close surveillance of Internet content is a perfect illustration of that
country’s powerful communist administration. To deregulate the
flow of media in China would greatly undermine the staunch
values and focused message of Chinese-generated media. By
contrast, countries like the United States that are founded on
principles like freedom of speech would greatly undermine their
fundamental values by allowing the government too much control
of their media. Religious affiliations and inclinations may have
even more specific regulatory effects in places like Afghanistan and
Saudi Arabia, where it is a violation of the Koran to depict Allah,
and the role of women in society requires them to wear traditional
garb at all times.
A country’s access to media and its cultural orientation are
expressly guided by its internal demographic. While a government
has the ability to restrict or release media, it makes no difference
when no one is watching. There is a basic aspect of supply and
demand, or viewership, with regard to programming. For example,
the most expensive commercial airtime on American TV every
year is during the Super Bowl. Drawing an average of 111 million
viewers in recent years, it is one of the most watched sporting
events in the world.
In conclusion, the extent to which a governing body should
be allowed to regulate public media varies from country to
country. Power over the media should be relevant to the founding
principles of a nation and should not undermine certain funda-
mental tenets of a society. The power a governing body has over a
nation’s media is relative to the peoples’ access to programming
and content. Viewership and demographics play a substantial role
in shaping that media, perhaps more powerfully than the govern-
ing body itself.
10. This statement can be interpreted in various ways. The ideas of a
nation can be a broad array of topics, particularly in a country like
22
501 GMAT
®
Questions
501_01_1-24_501_master.qxd 4/29/13 12:26 PM Page 22
23
the United States, where people feel nationalistic pride to express
their minds. Beyond seeing the desires and needs of a country
based on the supply and demand of its product and service
consumption, advertising also tells us a lot about demographics. To
define the limits and motives of a nation’s advertising is to
recognize its demographics and thereby somehow define the ideas
and preferences of the people within those demographics.
First, we should define exactly what we mean by the ideas of a
nation and how they relate to advertising. If by ideas we mean to
say ideologies, then I do agree that we can ascertain this from a
sampling of advertising. Clearly, if a nation is primarily religious
it isn’t likely that we will see advertising containing overt sexual
innuendo, nudity, or other images that challenge those reli-
gious views.
If we define the ideas of a nation to refer specifically to widely
accepted notions, I believe advertising is not a reliable source of
data. Things like trends, styles, and much-observed topics are the
fabric of pop culture. This cultural fabric is an eclectic quilt of
varying demographics, and the audience will vary from product to
product and media format to media format. Therefore the ideas, in
this sense, would be too broad to define so simply.
Demographic information is a revealing tale of who makes up
the most influential groups of a nation. For example, we can
assume that a country in which a majority of the women in each
household with children leave the home to go to work must have a
reliable infrastructure of childcare. Because of this large demo-
graphic, the consumption of products and services in the field of
childcare will likely be greater than the demand for these products
and services in countries where women of the household tradition-
ally stay at home to raise their families. If there is a negligible
market for this area, it will be reflected as a lack of a country’s
advertising in that field.
Somehow, by factoring together these aspects of a nation’s
advertising, we can grasp a broad notion of its ideas as they relate
to the established demographics present in that nation, but the
previous statement remains far too broad in its verbiage. Perhaps it
would make more sense to say that we can tell the demographics of
a nation by its advertising.
501 GMAT
®
Questions
501_01_1-24_501_master.qxd 4/29/13 12:26 PM Page 23
501_01_1-24_501_master.qxd 4/29/13 12:26 PM Page 24
Blank Page
501_02_25-120_501_master.qxd 4/29/13 12:30 PM Page 25
SECTION 2
GMAT
Verbal Section
To be successful in the business world, you’ll need to recognize how
sentences work, how arguments are constructed, and how written English
is used and understood. These skills will be tested in the GMAT Verbal sec-
tion with three types of multiple-choice questions—Reading Comprehen-
sion, Critical Reasoning, and Sentence Correction.
501_02_25-120_501_master.qxd 4/29/13 12:30 PM Page 26
Blank Page
Critical Reasoning
2
When you take the GMAT, critical reasoning sections will be inter-
spersed throughout the Verbal section. Critical reasoning questions use pas-
sages that are shorter than those in the reading comprehension section, and
passages will have one or two questions. The passage will remain visible on
the screen for each question.
Critical reasoning questions test the reasoning skills involved in con-
structing an argument, evaluating an argument, and formulating or evalu-
ating a plan. You do not need to have any familiarity with the subject matter
of the questions in order to answer them. Furthermore, you will want to be
wary of any prior knowledge that you have about any field written about in
a critical reasoning question because outside knowledge may cause you to
make assumptions about the correct answer.
Following are the directions for the critical reasoning questions. Read
them carefully and understand them clearly so that you will not need to
spend time reviewing them when you sit for the test: Each of the critical rea-
soning questions is based on a short argument, a set of statements, or a plan of
action. For each question, select the best answer of the choices given.
501_02_25-120_501_master.qxd 4/29/13 12:30 PM Page 27
Argument Construction
Examples of argument construction question types are: identify the basic
structure of the argument, draw the argument’s conclusion, identify the
assumption, draw an inference, explain the hypothesis, and identify paral-
lels between arguments with similar structures.
Identify the Basic Structure of the Argument
These questions test you on your ability to recognize components of an
argument and how the argument is put together. This question may actu-
ally quote a speaker and ask you to identify what that speaker is doing.
These questions may also quote a speaker and then list the response of a dif-
ferent speaker. Examples:
X responds to Y by
The phrase in bold plays which of the following roles in the
argument above?
A supports his conclusion by
Draw the Conclusion
These questions ask you to identify the conclusion that most logically fol-
lows from the passage.
Which of the following most logically completes the argument
given below?
The statements above, if true, best support which of the
following as a conclusion?
Which of the following conclusions CANNOT be drawn from
the statements above?
Identify the Assumption
Look for an unstated premise that must be true in order for the argument
to be valid.
The conclusion above is properly drawn if which of the
following is assumed?
The conclusion drawn above is based on the assumption that
Which of the following assumptions would make the
conclusion above properly drawn?
28
501 GMAT
®
Questions
501_02_25-120_501_master.qxd 4/29/13 12:30 PM Page 28
29
The argument above relies on which of the following
assumptions?
Draw an Inference
Look to see what inferences you can draw from the information in the pas-
sage.
Which of the following can be inferred from the argument
above?
If all of the statements above are true, then which of the
following must also be true?
Which of the following is best supported by the passage?
Explain the Hypothesis
These questions ask you to resolve an apparent contradiction in the passage
or to pick the answer choice that would be most helpful to assess the argu-
ment.
Which of the following, if true, helps to explain the
discrepancy described above?
All of the following statements help to explain . . . EXCEPT
Identify Parallels Between Arguments with Similar Structures
These questions ask you to find the answer choice that imitates the argu-
ment in the passage.
Which of the following uses the same method as the author in
the above statements?
Which of the following has the same structure as the above
passage?
Argument Construction Test-Taking Strategies
1. Identify the premises of the argument.
Premises could consist of an actual fact (“larger cars usually burn more
fuel”), a report or study (“the city’s survey shows that the majority of peo-
ple want the X subway line expanded”), or a reason (“most people eat broc-
501 GMAT
®
Questions
501_02_25-120_501_master.qxd 4/29/13 12:30 PM Page 29
coli instead of French fries because they want to lose weight”). Be aware
that critical reasoning questions will often clog the statements with extra-
neous information that is not part of the argument. Mentally set that infor-
mation aside and focus on the premises. Jot them down on scratch paper
if necessary.
2. Identify the conclusion.
The conclusion of the argument is the reason that the argument exists in
the first place. Conclusions exist in various forms, such as a recommenda-
tion for a future course of action (“In order to increase revenue, Company
X should offer a sale during the upcoming holiday”), an opinion (“Farmers
believe that this new irrigation technique will result in greater crop yields”),
or an interpretation (“Polling companies concluded from the survey that
most citizens of Nation Z will vote for Candidate Y”). If the passage does
not have a conclusion and the question asks you to identify the answer
choice that has the correct conclusion, try to paraphrase a conclusion in
your own words and look for the answer choice that best matches the con-
clusion you already formed.
3. Identify the assumption.
An assumption is the gap in the reasoning of the argument. Not all argu-
ments in life have assumptions. For example:
All people are warm-blooded. Laura is a person. Therefore, Laura is warm-
blooded. There is no gap in the reasoning here. However, GMAT
®
critical
reasoning questions use arguments with a gap in the reasoning. This kind
of gap, an assumption, would make the above argument look like this:
All people are warm-blooded. Laura is warm-blooded. Therefore, Laura is a
person. The link between the conclusion and the supporting facts is flawed
because there is an assumption here, which is that all warm-blooded beings
are people. This is obviously not true, but the conclusion assumes that it is.
If you reject the assumption, then you must reject the conclusion.
To identify the assumption, you will want to be familiar with the most
common forms of assumptions on the GMAT
®
. One kind of assumption is
a causal connection, in which the argument assumes that one causal con-
nection in the supporting facts also proves an additional causal connection.
Example: People who enroll in MBA programs are more likely to earn a high
salary in their career. Therefore, enrolling in an MBA program causes a person to
find a better job. The argument assumes that because enrollment in an MBA
30
501 GMAT
®
Questions
501_02_25-120_501_master.qxd 4/29/13 12:30 PM Page 30
31
program causes higher salaries, then enrollment also causes people to find
better jobs.
Another kind of assumption is a similarity between terms in the sup-
porting facts and those in the conclusion. Example: Sixty percent of people
with MBA degrees in New York City indicate that they earn over $100,000 a year.
Therefore, 60% of people with MBA degrees earn over $100,000 per year. The
argument assumes a similarity between people in New York City with MBA
degrees and people overall with MBA degrees.
A third kind of assumption is a shift in the terms in the supporting facts
and the conclusion. Example: People with MBA degrees tend to have more
knowledge about computer information systems. Therefore, people with MBA
degrees know more about conducting business at the corporate level. The argument
shifts from knowledge about computer information systems to knowledge
about conducting business at the corporate level.
The most straightforward way to attack an assumption question is to test
each answer choice against the conclusion. Could the conclusion still be
valid if the assumption were negated? If so, then that answer choice is not
the assumption of the argument.
4. Identify the inferences that go too far.
Inference questions ask you to identify the answer choice with the inference
that must be true, not the choices that might be true. Therefore, be wary of
answer choices that state facts that the passage itself does not actually state.
For example, suppose an answer choice says: Defense attorneys have abused
their positions. If the passage does not say anything about defense attorneys
abusing their position, then this answer choice probably goes too far. You
want to pick the answer that says something that you can definitely con-
clude from the passage and that is within the scope of the passage.
5. In resolve the discrepancy questions, identify other possible causes
for the end result.
Resolve the discrepancy questions tend to be about a plan or a study that
set out to achieve or prove one thing, and the end result was not what was
expected. Think of other reasons that could be the cause for the end result
and then look at the answer choices. Is that answer choice a plausible expla-
nation for the result? Second, does that answer choice thoroughly explain
the end result? Be wary of answer choices that would partially explain the
result, but fail to address the entire issue.
501 GMAT
®
Questions
501_02_25-120_501_master.qxd 4/29/13 12:30 PM Page 31
Set 2
Now it is time to answer GMAT Critical Reasoning practice questions that
have been designed to test your argument construction skills. Good luck!
11. Louis: The financial industry will suffer greatly from the new
government regulations over derivatives. Compliance with these
regulations will cost the industry $5.5 million annually because
companies in the industry will have to double the size of their
proxy statements in order to disclose all the required information.
Companies in the industry will then lose profits and have to lay off
other employees. Therefore, these regulations will have an adverse
effect on the nation’s economy.
Peter: The $5.5 million that companies in the finance
industry will have to spend will be profits for other types of
companies in the private sector. Profits and jobs may be lost in the
finance industry, but they will be gained by other companies.
Peter responds to Louis by
a. agreeing with Louis’s conclusion by offering additional infor-
mation to support it.
b. offering information that suggests that Louis has overlooked a
mitigating consequence.
c. agreeing with Louis’s conclusion but suggesting that the out-
come is positive rather than negative.
d. challenging the tenability of the facts that serve as the basis of
Louis’s argument.
e. demonstrating that Louis’s conclusion is not based on relevant
facts.
32
501 GMAT
®
Questions
501_02_25-120_501_master.qxd 4/29/13 12:30 PM Page 32
33
12. A cellular phone company spent tens of thousands of dollars in the
past year fixing cell towers damaged by extreme weather
conditions. In order to reduce the company’s overall budget this
year, the company plans to cover the base of its towers with a
better type of steel and to move certain towers to different
locations so that fewer overall are needed.
Which of the following would NOT be required for the
cellular phone company to achieve its aim?
a. The cost of the steel would have to be lower than the cost of
the repairs to the cell towers during the previous year.
b. The cost of evaluating the most efficient placing of cell towers
would have to be lower than the cost of the repairs to the cell
towers during the previous year.
c. The cost of covering the towers with the better type of steel
would have to be lower than the cost of the repairs to the cell
towers during the previous year.
d. The cost of the repairs to the cell towers during the previous
year would have to be lower than the price of the steel con-
struction crews’ wages to cover the cell tower bases.
e. The cost of shipping the steel would have to be lower than the
cost of the repairs to the cell towers during the previous year.
13. Kathryn has a higher GPA than Pamela. Madelyn has a higher
GPA than Adrienne. Shane has a higher GPA than Madelyn.
Thus, it follows that Kathryn has a higher GPA than Adrienne.
Which of the following, if introduced into the argument as an
additional premise, makes the argument above logically incorrect?
a. Madelyn has a higher GPA than Katherine.
b. Katherine has a higher GPA than Madelyn.
c. Adrienne has a higher GPA than Pamela.
d. Pamela has a higher GPA than Madelyn.
e. Adrienne has a higher GPA than Shane.
501 GMAT
®
Questions
501_02_25-120_501_master.qxd 4/29/13 12:30 PM Page 33
14. The Braman Rule states that someone who leaves property to
another in a will cannot leave property that the deceased did not
own at the time of death. Humphrey Monroe left Claire McGinnis
a cottage in his will. Claire McGinnis stated in her will that all her
property would be left to her niece, Sharon. Claire McGinnis
died before Humphrey Monroe. Therefore, the cottage was
not inherited by the person whom Claire McGinnis wanted to
inherit it.
Which of the following assumptions would make the
conclusion above properly drawn?
a. Claire McGinnis wanted Sharon to inherit the cottage.
b. Humphrey Monroe wanted Sharon to inherit the cottage.
c. Sharon wanted to inherit the cottage.
d. Claire McGinnis did not want Sharon to inherit the cottage.
e. Humphrey Monroe did not want Sharon to inherit the cottage.
15. A franchise restaurant pays its employees in State A $2.00 per hour
above the minimum wage, and pays its employees in State B $1.00
per hour above the minimum wage. The franchise has three times
as many employees in State A as in State B. The franchise’s total
expenditures on hourly wages for its employees must therefore be
higher in State A than in State B.
The argument above relies on which of the following
assumptions?
a. The franchise employees in State A do not each work signifi-
cantly less hours than the franchise employees in State B.
b. The franchise has a greater number of restaurants in State A
than in State B.
c. The franchise employees in State B each work significantly
more hours than the franchise employees in State A.
d. The law in State A requires the franchise to pay its employees a
higher wage than the law in State B.
e. The franchise is planning to decrease the wages paid to employ-
ees in State A to $1.00 per hour above the minimum wage.
34
501 GMAT
®
Questions
501_02_25-120_501_master.qxd 4/29/13 12:30 PM Page 34
35
16. The cost of buying diamonds in the country of Luvania is 40%
more than the cost of buying diamonds in the country of Oretania.
When one adds the expenses of travel and the tax on purchases by
foreigners in Oretania, buying diamonds in Oretania is still less
expensive for a citizen of Luvania than buying diamonds in
Luvania.
The statements above, if true, best support which of the
following as a conclusion?
a. The ground in Oretania breaks more easily under the force of a
pickaxe than it does in Luvania.
b. The tax on purchases by foreigners in Oretania is less than 40%
of the cost of buying diamonds in Luvania.
c. The cost of traveling to Oretania from Luvania is more than
the 40% of the cost of buying diamonds in Luvania.
d. Diamond mining costs are higher in Luvania than in Oretania.
e. Buying diamonds in Luvania will eliminate some retail jobs in
jewelry stores in Oretania.
17. After reports from three schools of too much chlorine in their
drinking water, the city hired a plumbing company to revamp its
running water system. The plumbing company replaced the
filtration system after the point in which groundwater is collected.
We can conclude that the three schools will no longer have a
problem with too much chlorine in their drinking water.
Which of the following does the argument depend on as an
assumption?
a. Not all faulty water filtration systems will allow too much chlo-
rine into the drinking water.
b. Water filtration systems are not the only source of too much
chlorine in the drinking water.
c. The city has been able to successfully filter water at its water fil-
tration plant.
d. The excess chlorine was not introduced into the schools’ water
supply at some point later than groundwater collection.
e. Not all faulty water filtration systems are likely to produce
excess chlorine in a drinking water supply.
501 GMAT
®
Questions
501_02_25-120_501_master.qxd 4/29/13 12:30 PM Page 35
18. The number of people who quit smoking every year is on the rise.
The most common reasons that people cited in a survey for
quitting smoking are wanting to improve their health, wanting to
save money, and wanting to appear physically appealing. Out of the
people who quit smoking that cited health reasons, 85% of them
also cited wanting to appear physically appealing. The last
question in the survey asked whether physical appearance was
important to the respondents. 100% of survey respondents
answered affirmatively.
Which of the following, if true, does NOT explain the
discrepancy described above?
a. Some respondents felt that although their physical appearance
was important to them, it was not important enough to quit
smoking.
b. Some respondents believed that holding a cigarette enhanced
their physical appearance.
c. Some respondents believed that cigarette smoking would not
have an adverse effect on their physical appearance.
d. Some respondents felt superficial in admitting that their physi-
cal appearance was important to them.
e. Some respondents believed that cigarette smoking had little to
no connection to their physical appearance.
36
501 GMAT
®
Questions
501_02_25-120_501_master.qxd 4/29/13 12:30 PM Page 36
37
19. Rental prices are usually lower on the basement and ground floors
of an apartment building, and highest at the top of the building.
Rental prices are also usually higher for buildings that receive
more sunlight. In addition, rental prices are determined by the
number of bedrooms in an apartment. There are six floors in
building #153. Cathy lives on the sixth floor, David lives on the
fourth floor, Krystal lives on the ground floor, and Oscar lives in
the basement. Out of the four tenants, Krystal pays the highest
rent and Oscar pays the lowest rent.
Which of the following conclusions CANNOT be drawn
from the statements above?
a. Oscar lives in a 1-bedroom, David’s apartment receives the
most sunlight, and Krystal lives in a 3-bedroom.
b. Cathy lives in a 3-bedroom, Oscar lives in a 3-bedroom, and
Krystal lives in a 1-bedroom.
c. Krystal lives in a 1-bedroom, David’s apartment receives the
most sunlight, and Cathy lives in a 3-bedroom.
d. David lives in a 2-bedroom, Cathy’s apartment receives the
most sunlight, and Oscar lives in a 1-bedroom.
e. Cathy lives in a 3-bedroom, Cathy’s apartment gets the most
sunlight, and Krystal lives in a 5-bedroom.
501 GMAT
®
Questions
501_02_25-120_501_master.qxd 4/29/13 12:30 PM Page 37
20. A university study showed that women who spent more money on
their exercise clothes went to the gym more often. However, the
women who spent more money on their exercise clothes did not
lose more weight than the women who spent less money on their
exercise clothes.
Which of the following statements does NOT help to explain
the results of the study?
a. The women who spent more money on their exercise clothes
did not exercise as hard as those who spent less money because
they were too concerned about looking good at the gym.
b. The women who spent less money on their exercise clothes
worked out more frequently than those who spent more; they
just did not work out as frequently at the gym.
c. The women who spent more money on their exercise clothes
focused their workouts more on strength training and less on
fat and/or calorie burning exercise.
d. The women who spent more money on their exercise clothes
were, on average, already at their ideal weight.
e. The women who spent more money on their exercise clothes
had higher memberships in weight loss programs.
21. In the past, doctors would often prescribe antibiotics to patients
with viruses purely for purposes of patient satisfaction. Antibiotics
only successfully treat bacterial infections and do not actually treat
viruses, which must merely run their course. Doctors have since
decreased the percentage of patients for whom they prescribe
antibiotics.
All the following statements help to explain the decrease in
the percentage of patients prescribed antibiotics EXCEPT:
a. Doctors are less concerned with patient satisfaction.
b. Doctors are more concerned about patients developing an
immunity to antibiotics.
c. A larger percentage of patients today are diagnosed with
viruses.
d. A smaller percentage of patients today are diagnosed with ail-
ments that require antibiotics.
e. A larger percentage of patients today are diagnosed with bacter-
ial infections.
38
501 GMAT
®
Questions
501_02_25-120_501_master.qxd 4/29/13 12:30 PM Page 38
39
22. A publishing company derives revenue from three main areas.
Digital products constitute 30% of sales and earn 40% of the
profits. Books constitute 60% of sales and earn 50% of the profits.
Audiobooks and other mediums constitute 10% of sales and earn
10% of the profits.
Which of the following can be inferred from the statements
above?
a. Digital products earn higher profits per sale than books or
other mediums.
b. Digital products earn more profit than books or other medi-
ums.
c. The area of digital products is growing more rapidly than
books.
d. Each digital product is more profitable than each book product.
e. Books earn less profit than digital products and products in
other mediums combined.
23. Major businesses have begun near-shoring, which is a practice that
involves moving jobs from cities with a higher cost of living to
lower cost of living locales. The rationale behind near-shoring is
that employees with a lower cost of living will not demand as high
salaries. High-level positions that involve a lot of face time with
clients will remain in the higher-cost-of-living cities. High-level
employees will travel to the lower-cost-of-living locales when they
need to meet with staff.
Which of the following conclusions, if true, provides the
strongest support for the practice of near-shoring?
a. Employees will be willing to relocate in order to keep their
jobs.
b. The cost of travel for high-level employees will not negate the
savings in moving jobs to locations with a lower cost of living.
c. Clients will not need to travel to the lower-cost-of-living locales.
d. Cities with a higher cost of living will not offer financial incen-
tives to businesses to stay.
e. Clients will not move out of the cities with a higher cost of
living.
501 GMAT
®
Questions
501_02_25-120_501_master.qxd 4/29/13 12:30 PM Page 39
24. In order for humanitarian effort to relieve starving that results
from a drought to succeed, there must be an efficient
transportation system for water and support from several relief
organizations. At least a dozen trucks that hold liquid are required
for an efficient transportation for water. Each relief organization
involved has pledged to supply trucks. Thus, relief will be brought
to drought victims in the country of Y.
Which of the following is the most important item be
ascertained in order to evaluate the argument presented above?
a. the number of relief organizations involved
b. the availability of gasoline for the trucks
c. the availability of water for transportation
d. the amount of time it would take for the trucks to transport the
water to drought victims
e. the amount of time it would take to begin transporting the
water to drought victims
25. Alexandra has more money than Courtney. Marnie has more
money than Courtney. Alexandra has less money than Danielle.
We can conclude that Courtney has less money than Josie.
Which of the following, if true, would make the argument
above logically incorrect?
a. Danielle has more money than Marnie.
b. Danielle has less money than Courtney.
c. Alexandra has more money than Josie.
d. Josie has more money than Danielle.
e. Josie has less money than Marnie.
40
501 GMAT
®
Questions
501_02_25-120_501_master.qxd 4/29/13 12:30 PM Page 40
41
26. Taylor: The requirement that financial institutions register with
the Securities and Exchange Commission all securities offered for
sale to the public is pointless because people never read through all
those piles of paper before they buy securities.
Sophia: I disagree. The registration provides a paper trail that
people can use to hold financial investors responsible in the event
that securities were not accurately represented.
Sophia responds to Taylor’s argument by
a. providing evidence that people who buy securities actually read
the paperwork about them beforehand.
b. shifting the discussion away from the paper that registration
generates and toward the illegal activity that often occurs in
securities sales.
c. pointing out a benefit of registration, which Taylor’s argument
failed to consider.
d. objecting to Taylor’s dismissal of securities purchasers’ financial
literacy.
e. rejecting Taylor’s argument while suggesting that registration
with the Securities and Exchange Commission be required of
all securities.
501 GMAT
®
Questions
501_02_25-120_501_master.qxd 4/29/13 12:30 PM Page 41
27. The two methods available to ABC Car Company to increase fuel
efficiency in its vehicles are installing a new type of engine in its
cars and using a lighter metal for the car frames. The use of a
lighter metal in the car frames will increase efficiency more than
installation of a new type of engine. Therefore, by using a lighter
metal in the car frames, ABC Car Company will be doing the most
it can to increase fuel efficiency in its cars.
If the statements above are true, which of the following must
be true?
a. ABC Car Company has several options for a lighter metal to
use for the car frames.
b. Fuel efficiency cannot be increased more by using both meth-
ods together than by only using a lighter metal in the car
frames.
c. The cars that ABC Car Company manufactures are already
quite fuel efficient.
d. ABC Car Company is unsure whether it should bother with
increasing fuel efficiency.
e. Installing a new type of engine will be more expensive than
using a lighter metal in the car frames.
28. Demand for olive oil has remained constant during the past few
years. Production and prices have also remained the same during
the past few years, yet Crunchy Organics has seen an increase in its
profits from olive oil by more than 20% over last year’s profits.
The conclusion above would be more reasonably drawn if any
of the following were inserted into the argument as an additional
premise EXCEPT:
a. An economic recession has enabled Crunchy Organics to find
workers who will work for lower wages.
b. Crunchy Organics installed insulation and other energy-saving
measures in its facilities, which resulted in significant decreases
in utility bills and a hefty tax refund.
c. Increased rainfall resulted in a more plentiful olive crop.
d. Crunchy Organics was featured in a nationwide news segment
about ethical small businesses, which was viewed by over 10
million people.
e. The demand for olive oil has decreased, and most companies
that produce olive oil have increased their distribution.
42
501 GMAT
®
Questions
501_02_25-120_501_master.qxd 4/29/13 12:30 PM Page 42
43
29. Which of the answer choices best completes the argument below?
More cosmetic companies should follow the marketing plan
of Company G. Women want to see how products look on women
who look like themselves because they can get a better idea of how
the makeup will actually look on them. Women are also tired of
being confronted with images ______.
a. of women who can clearly afford more expensive cosmetics than
the average woman.
b. of supermodels who wear evening makeup when most women
shop for a daytime look.
c. of supermodels who perpetuate the stereotype of unrealistic
beauty.
d. of models who fail to make cosmetics look glamorous.
e. of models wearing makeup that is not of a good quality.
30. Township P conducted a survey in which over half of its residents
admitted that they do not recycle, and the township’s recycling
plant has threatened to lay off employees due to a lack of work. In
order to increase recycling, Township P should install recycling
machines in the town center, in which residents can turn in their
bottles, cans, and other recyclables in exchange for five cents
apiece. This plan would dramatically increase the number of
residents that recycle and save the jobs of many residents.
Which of the following are assumptions made in drawing the
conclusion above?
a. Saving the jobs at the township’s recycling plant would not
result in corresponding layoffs at the township’s trash plant, and
the 5-cent refund would pay for the cost of recycling the addi-
tional materials.
b. The township’s recycling plant can handle the increased quantity
of materials to recycle, and an increase in recycling would save
the jobs that the recycling plant has threatened to eliminate.
c. The recycling machines would induce residents to recycle more, and
the 5-cent refund would pay for the recycling machines over time.
d. The 5-cent refund per item would eventually pay for the cost of
the recycling machines, and the township could afford to pay
the workers that the recycling plant has threatened to lay off.
e. The recycling machines and the 5-cent refund per item would
induce residents to recycle more, and an increase in recycling would
save the jobs that the recycling plant has threatened to eliminate.
501 GMAT
®
Questions
501_02_25-120_501_master.qxd 4/29/13 12:30 PM Page 43
31. Reporter: The Q Virus is a serious disease that can make people
deathly ill. Starting in 1990 and continuing in every subsequent
year, approximately 10,000 people became infected. Before 1990,
there were only about 3,000 cases per year. We can conclude that
the Q Virus has become a dire public health threat.
Public Health Expert: But before 1990, the population was
only one quarter the size that it is now.
The public health expert challenges the reporter’s argument
by doing which of the following?
a. pointing out that the argument makes a conclusion based on a
small sample of the population
b. pointing out that the argument treats facts about some people
in a group as if they were true about all people in that group
c. presenting information that refutes the argument’s assumption
of an incremental increase
d. introducing information that casts doubt on an assumption of
the reporter’s argument
e. questioning the argument’s information about the people in a
particular group
44
501 GMAT
®
Questions
501_02_25-120_501_master.qxd 4/29/13 12:30 PM Page 44
501_02_25-120_501_master.qxd 4/29/13 12:30 PM Page 45
45
32. Computer programming classes are offered in the College of Math
and Science. A middle range percentage of students is enrolled in
courses of the College of Math and Science. Economics classes are
offered in the College of Business. The largest percentage of
students is enrolled in courses in the College of Business. Business
writing courses are offered in the College of Humanities. The
lowest percentage of students is enrolled in courses in the College
of Humanities. Therefore, computer information system classes
should be offered in the College of Business.
The conclusion above would be more reasonably drawn if
which of the following were inserted into the argument as an
additional premise?
a. Courses should be offered in the college in which a greater per-
centage of students is enrolled.
b. Computer information systems courses are more like business
than they are like computer science.
c. More majors in the College of Business require courses in com-
puter information systems than majors in the College of Math
and Science.
d. Computer information systems courses have no relevance to the
humanities.
e. More students major in business because business majors have
the greatest chance of finding a job.
33. The press reports sensationalist stories in order to get people to
pay attention to the news. In order to generate high ratings,
reporters must only report on topics with great scandal that will
compel people to pay attention. The general public is, therefore,
uninformed about basic current events.
The conclusion drawn above is based on the assumption that
a. the press only cares about high ratings.
b. only scandalous topics will generate high ratings.
c. the general public obtains all its news information from news
outlets that generate high ratings.
d. the press should be rewarded for high quality journalism.
e. the press has tried to generate high ratings with quality news,
and has failed.
501 GMAT
®
Questions
501_02_25-120_501_master.qxd 4/29/13 12:30 PM Page 46
46
501 GMAT
®
Questions
34. This year, the police department of City X reported a relatively
substantial increase in petty crime. The voters of City X
congregated at a city council meeting to express their concern
about this problem. A leader of a community action group stood
up and urged the city council to rehire the 300 police officers it
laid off due to budget cuts in the previous year. The city council
president responded that the increase was in the actual number of
victims who bothered to report petty crime to the police.
The city council’s statements, if true, best support which of
the following as a conclusion?
a. The previous presence of an extra 300 police officers deterred
criminals from committing petty crime.
b. The laid-off police officers committed petty crime because they
were no longer earning a salary.
c. The budget cuts were a symptom of an economic downturn,
which is usually a cause behind a rise in petty crime.
d. It is possible to determine how many people were victims of
petty crime in a year.
e. The percentage of victims of petty crime in the past year is no
larger than the percentage of victims of petty crime this year.
501_02_25-120_501_master.qxd 4/29/13 12:30 PM Page 47
47
501 GMAT
®
Questions
35. Which of the answer choices most logically completes the
argument given below?
Retail Expert: Companies that manufacture and sell name
brand products often operate under the common misconception
that manufacturing generic products would not be as profitable
because they sell for a lower price. Even though name brand
products sell for a higher price, the cost of production for generic
products does not include any marketing expenses. Thus, the
profits generated from the manufacture and sale of name brand
products and generic products are usually comparable because
____.
a. the market for generic products is large enough and the cost of
production is sufficiently low enough to offset the lower prices
charged to customers
b. the market for name brand products is large enough and the
cost of production is sufficiently low enough to offset the higher
prices charged to customers
c. the respective markets for name brand products and generic
products are about the same size
d. consumer demand will always exist for name brand products
despite their higher prices
e. most companies are savvy enough to manufacture and sell both
name brand and generic products, and they can purposely bal-
ance out those markets
501_02_25-120_501_master.qxd 4/29/13 12:30 PM Page 48
48
501 GMAT
®
Questions
36. Many people complain that standardized tests do not actually
measure knowledge because people simply direct their studies
toward the actual test and not the subject. However, standardized
tests do measure knowledge because test takers have no way of
anticipating the questions on the test and therefore have to study
the entire subject.
The conclusion above is properly drawn if which of the
following is assumed?
a. Standardized tests merely ask particular types of questions on
which certain test takers excel.
b. Test takers must be able to anticipate test questions on any stan-
dardized test in order to direct their studies toward the actual
test and not the subject.
c. If a standardized test has questions that cover the entire breadth
of a subject, then the test actually measures knowledge.
d. People who direct their studies toward an actual test do not
study an entire subject for the test.
e. Non-standardized tests actually measure knowledge.
37. Many companies offer their employees the option of flex time.
Flex time is the practice of allowing employees to work their
allotted number of hours on whatever schedule that they choose.
Flex time permits employees to work during the most convenient
hours for them, and results in increased productivity because
employees are no longer as distracted by convenience issues.
Any of the following, if true, is a valid reason for a company
to offer flex time to its employees EXCEPT:
a. The majority of workers in any industry report that they often
conduct e-mail correspondence outside of traditional work
hours.
b. Reports show that employees who work from home are often
less productive, and employees who often utilize work from
home options usually opt for flex time instead when their
employers offer it.
c. Employees who work traditional hours often miss time at work
because of health care appointments.
d. The majority of clients report that they prefer to conduct busi-
ness over the telephone rather than via e-mail.
e. Employees often exceed their lunch hours because they are run-
ning personal errands that are more convenient to do midday.
501_02_25-120_501_master.qxd 4/29/13 12:30 PM Page 49
49
501 GMAT
®
Questions
Argument Evaluation
Examples of argument evaluation questions are: analyze the argument,
strengthen the argument, weaken the argument, identify reasoning errors
in the argument, or recognize aspects of the argument’s development.
Flaw Questions
These questions ask you to identify the answer choice that makes the argu-
ment incorrect.
Which of the following is a flaw in the above argument?
The flaw in the argument is that it fails to consider. . . .
Which of the following, if true, identifies a flaw in the
argument above?
Strengthen Questions
This type of question requires that you identify the assumption and pick the
answer choice that would strengthen the assumption.
Which of the following, if true, would most strengthen the
conclusion drawn above?
Which of the following, if true, would lend the most support
to the view . . . ?
Weaken Questions
This type of question requires that you identify the assumption and pick the
answer choice that would weaken the assumption.
Which of the following, if true, would most weaken the
conclusion drawn above?
Which of the following, if true, would most undermine the
argument?
Which of the following, if true, casts the most doubt on the
conclusion above?
Evaluate the Argument
These questions ask you to identify what would be the most useful to assess
the argument in the passage.
Which of the following would be most helpful in evaluating
the argument above?
Which of the following would be most useful to know in
assessing the conclusion above?
501_02_25-120_501_master.qxd 4/29/13 12:30 PM Page 50
50
501 GMAT
®
Questions
Argument Evaluation Test-Taking Strategies
1. Identify the assumption.
Identify whether the assumption is a causal connection, a similarity, or a
shift. Strengthen and weaken questions require that you identify the
assumption and pick the choice that would either strengthen or weaken it.
Ask yourself whether the passage creates a causal connection, similarity, or
a shift between two things that does not actually exist. These two items may
be quite similar, designed to trip you up by assuming that they are in fact
related.
2. Identify the relationships between the numbers.
Many GMAT questions use a lot of quantitative information designed to
cause you to assume relationships between sets of numbers when in fact
there is not one. Critical reasoning questions will often use dollar amounts,
quantitative amounts, percentages, interest rates, and inflation. Do not
assume that dollar or quantitative amounts necessarily correlate with per-
centages. Furthermore, ask yourself whether interest rates or inflation, if
mentioned in the prompt, will actually have a particular effect on the num-
bers in the passage. After you read the question and reread the prompt, dis-
card any extraneous numbers clogging the passage that you do not need to
consider.
3. Remember “except,” “not,” and other opposites.
GMAT questions will occasionally flip the standard format and list a series
of answer choices that strengthen, weaken, or explain the argument. These
questions will ask you to identify the answer choice that does NOT
strengthen, weaken, or explain. Test takers often get tripped up when work-
ing on these questions and start looking for the opposite of what the ques-
tion is asking for. For questions such as these, jot down what the question
is asking for on your scratch paper. For example:
Which of the following, if true, does NOT strengthen the argument above
Remember, that “not strengthen” does not necessarily mean “weaken.”
The correct answer choice may not do either. Note that you should cross off
all answer choices that strengthen the argument and pick the remaining one.
4. Note which argument the question addresses.
Some GMAT questions will include the arguments of multiple parties
instead of just one. Many test takers have a tendency to skim the question
and assume that it is asking you to strengthen, weaken, or point out the flaw
501_02_25-120_501_master.qxd 4/29/13 12:30 PM Page 51
51
501 GMAT
®
Questions
in the last argument in the passage. However, the question may want you
to work with a different party’s argument. For example:
A asserts that the budget is not accurate because it failed to take into account tax
credits.
B concludes that there were no tax credits because of an accounting error.
Which of the following, if true, would strengthen As argument?
Make sure that you are addressing the argument that the question directs
you to because one of the answer choices will likely strengthen B’s argument
and trick you.
5. Right result, wrong reason.
Beware of answer choices that list subject matter that could plausibly
strengthen or weaken an argument, be a flaw in the argument, or point out
something useful to evaluate an argument, but that employ incorrect rea-
soning. If multiple answer choices seem plausible to you at first glance, take
a closer look at the reasoning in them. For example:
A dog food company was concerned about failing to adhere
to its budget in the current calendar year. In order to solve
this problem, the company decided to move its headquar-
ters to a different office building with a lower rent. Upper
management concluded that moving the company’s
headquarters in the next year would result in sufficient
savings to meet its budgetary requirements.
Which of the following, if true, would weaken the
argument?
a. After the company paid for moving costs, it would not
earn back the rental savings from the new location for an
additional two years—correct answer
b. The company failed to consider that the new building
has higher utility bills—incorrect answer
Do you see why? Both answers provide reasons that the new building will
not provide the savings that the company believes it will. However, choice
a specifically negates savings that the company’s plan to move will allegedly
generate. Choice b provides another reason that could plausibly negate the
savings, but it is not conclusive because the answer does not specifically say
that the higher utility bills would cancel out the savings generated by the
lower rent. The reasoning here is not sufficiently thorough to be the best
answer to weaken the argument.
501_02_25-120_501_master.qxd 4/29/13 12:30 PM Page 52
52
501 GMAT
®
Questions
Set 3
Now it is time to answer GMAT Critical Reasoning practice questions that
have been designed to test your argument evaluation skills. Good luck!
38. Vegetarians tend to live longer than people who eat meat. This
does not show that vegetarianism causes people to live longer
because vegetarians as a group tend to exercise more regularly than
meat eaters.
Which of the following, if true, most strengthens the
argument above?
a. A vegetarian who does not exercise regularly is more likely to
start exercising than a person who eats meat.
b. Among vegetarians, most of those who start to exercise regu-
larly early in life tend to maintain the practice later in life.
c. Among people who exercise regularly, those who do not eat
meat tend to live as long as those who eat meat.
d. Vegetarianism tends to cause people to engage in other healthy
habits besides regular exercise.
e. A person who eats meat and gives up regular exercise is much
more likely than a vegetarian to never resume regular exercise.
39. The city of Springfield has received a proposal to upgrade its metro
system. An environmental group has argued that increasing the
number of trains and the metro’s area of coverage within the city will
result in a 20% increase in rider capacity, reducing the need for
people to drive cars. The city of Springfield should grant the
proposal to solve the problem of traffic congestion on the city roads.
Which of the following, if true, most seriously weakens the
argument?
a. Most people in Springfield are unwilling to switch from driving
to public transit.
b. Increasing the metro’s passenger capacity will require the trains
to travel at a slower speed.
c. Most people who drive in Springfield commute to places in the
city that the metro does not currently reach.
d. The proposed upgrade to the metro system will increase air
pollution.
e. The metro system is currently not filled to capacity during the
rush hour commute.
53
501 GMAT
®
Questions
501_02_25-120_501_master.qxd 4/29/13 12:30 PM Page 53
40. President of Fundraising: Donations to our organization have
decreased steadily over the past five years. Many individuals at this
meeting have suggested that the poor economy is the reason. I
disagree. My research has indicated that donors to this cause tend
to donate the same amounts and in the same frequency regardless
of their income. I believe that we need to reexamine our mailings
so that we can find a better way to remind our donors about the
important work that our organization does.
Which of the following, if true, most strongly supports the
fundraising president’s theory?
a. The majority of the donors to this organization have not suf-
fered financially in the downturned economy.
b. The organization’s mailings used to contain prepaid postage on the
return envelopes for donations, but have not for the past five years.
c. The organization has decreased its charitable work over the past
five years in order to increase its fundraising effort.
d. The organization has not updated its mailing list for the last
seven years.
e. The majority of the donors are above age 75 and have difficulty
reading smaller print.
501_02_25-120_501_master.qxd 4/29/13 12:30 PM Page 54
54
501 GMAT
®
Questions
41. Dr. Roberts, a dentist, gave a speech to a dental convention in
which she argued that the recent rise in tooth enamel decay in the
population came from a new brand of toothbrush. This brand used
a new type of plastic for the bristles, which Dr. Roberts claimed
wears down the enamel of the teeth over time. Dr. Roberts
concluded her remarks by stating that the consistent high rate of
soft drink consumption was not a significant contributing factor to
the recent rise in tooth enamel decay.
Which of the following, if true, most strongly supports the
view that the new brand of toothbrush was responsible for the rise
in tooth enamel decay mentioned above?
a. The brand Dr. Roberts mentioned is the only brand to use the
new type of plastic for the bristles.
b. Consistent heavy consumption of soft drinks does not increase
tooth enamel decay over time provided that patients brush
twice a day.
c. This brand of toothbrush has a 70% market share.
d. The majority of Dr. Roberts’s patients used this brand of tooth-
brush for a significant period of time.
e. The company that sold this brand of toothbrush sent hundreds
of thousands of samples to dentists for free giveaways to dental
patients.
42. People between the ages of 40 and 50 purchase more vacations to
Mexico than any other age group. Therefore, the Mexico Tourist
Board should convince hotels and resorts in Mexico to offer more
family friendly packages and services.
Which of the following indicates a flaw in the reasoning above?
a. The author assumes that the majority of people between the
ages of 40 and 50 have children.
b. The author assumes that the majority of people between the
ages of 40 and 50 want family friendly vacations.
c. The author assumes that the majority of people between the
ages of 40 and 50 are married.
d. The author does not supply information about people in other
age groups.
e. The author assumes that the majority of people between the
ages of 40 and 50 can afford a vacation in Mexico.
501_02_25-120_501_master.qxd 4/29/13 12:30 PM Page 55
55
501 GMAT
®
Questions
43. Last year, approximately 1 million smokers quit smoking. This
year, the number of smokers who quit is approximately 3 million.
Thus, we can conclude that the number of smokers who will quit is
increasing and that the number of smokers who will quit next year
will be even higher.
Which of the following statements best supports the
conclusion above?
a. The 1 million smokers who quit last year represented the lowest
number in six years.
b. An increase in government regulation regarding health care
usually results in an increase in health care coverage of smoking
cessation programs.
c. This year, a government grant to a program called Light Up
No More brought the number of smokers who quit temporarily
up from the average number of 1 million per year.
d. This year, health insurance companies raised premiums 20%
for customers who smoke, and they plan to institute a 5%
increase for next year.
e. The estimated number of smokers who quit annually is actually
based on a representative sample of the population rather than
the entire population.
44. Production of paper has always been costly, but the tariffs on a box
of paper currently make domestic paper production less expensive.
Therefore, if we want to obtain paper in the cheapest way possible,
we should produce all our paper in our own country.
Which of the following, if true, most seriously weakens the
argument above?
a. Producing paper domestically will create more jobs.
b. Tariffs on boxes of paper are the same flat cost on every box no
matter the size.
c. Companies that produce a product have a high incentive to
control the cost of production.
d. The final price of a product after adding in the tariff is higher
than the impact that production start-up costs have on the final
price of a product.
e. Foreign paper producers can produce more paper overall than a
domestic producer.
501_02_25-120_501_master.qxd 4/29/13 12:30 PM Page 56
56
501 GMAT
®
Questions
45. The retail industry suffers from the problem of high employee
turnover. Since wages remain stagnant for employees who work in
stores, employees have no incentive to remain with the same
company over time. The Namaste Clothing Co. has a much higher
retail employee retention rate than other retail companies. The
human resources department published a report that stated that
the company’s success is due to qualitative factors that the
company offers, such as a pension plan, comp time for salaried
employees, casual Fridays, water coolers, and high-end computers
with great tech support.
Which of the following, if true, identifies a flaw in the argument
above?
a. The human resources report assumes that employees care about
the qualitative factors listed.
b. The human resources report erroneously lists pension and
comp time for salaried employees as qualitative factors, when
they are actually related to employee compensation.
c. The human resources report lists qualitative factors that would
apply only to company employees who do not work in stores.
d. The human resources report assumes that employees have
remained with the company because of something the company
has offered them.
e. The human resources report did not list all the qualitative
advantages that the company offers its employees.
501_02_25-120_501_master.qxd 4/29/13 12:30 PM Page 57
57
501 GMAT
®
Questions
46. In 2000, women’s clothing retailers reported that 20% of their
sales were for sizes 0–4, 35% of their sales were for sizes 6–12, and
45% of their sales were for sizes 14–16. In 2010, women’s clothing
retailers reported that those percentages were 25%, 45%, and
30%, respectively. We can conclude that the average adult woman
was the same size in 2010 that she was 10 years ago.
Which of the following, if true, best explains the conclusion
above?
a. The amount of clothing purchased in 2010 was greater than the
amount of clothing purchased in 2000.
b. Vanity sizing was much more prevalent in 2000 than in 2010.
c. A greater percentage of slimmer woman bought clothing in
2010 than in 2000.
d. The number of women who purchased clothing in 2000 was
greater than the number of women who purchased clothing in
2010.
e. Most of the designers who designed clothes in 2010 had not yet
earned their degrees in fashion design in 2000.
47. The average law firm pays entry-level attorneys an annual salary of
$70,000. Ninety-five percent of law school students have jobs upon
graduation. Therefore, of that 95%, the average law student will
earn an annual salary of $70,000.
Which of the following, if true, most seriously undermines
the argument above?
a. Most law students who have jobs upon graduation were hired
by small law firms.
b. The average salary offered by law firms varies by geographical
region.
c. Fewer students enroll in law school today than 10 years ago.
d. The law students who do not have jobs upon graduation did not
pass the bar exam.
e. Over half of law students who have jobs upon graduation were
not hired by law firms.
501_02_25-120_501_master.qxd 4/29/13 12:30 PM Page 58
58
501 GMAT
®
Questions
48. Company XYZ manufactures and sells products that help people
quit smoking. This year, Company XYZ reported an increase in
sales overall and in each individual smoking cessation product. The
entire market for products that help people quit smoking also
reported an increase in sales overall. The product with the highest
increase was the patch. The product with the lowest increase was
the gum. We can conclude from this information that the number
of people who have quit smoking has increased this year.
Which of the following, if true, most seriously calls into
question the explanation above?
a. Most people who quit smoking reported that their greatest suc-
cess was with the gum.
b. Most people who quit smoking reported that they did not buy
smoking cessation products from Company XYZ.
c. Most people who quit smoking reported that they relapsed in
approximately 10 months.
d. Most people who quit smoking reported that they bought mul-
tiple smoking cessation products at once.
e. Most people who quit smoking reported that they purchased
whatever smoking cessation product their doctor recom-
mended.
49. The rise of college tuition was widely expected to result in an
overall increase in the total amount that people in this country
spent on college tuition. However, the exact opposite occurred.
The rise in tuition occurred along with a rise in merit scholarships.
The result is that college students in this country actually spend
less on college than previously.
Which if the following would NOT be useful in evaluating
the reasoning above?
a. the total cost of tuition and the monetary amount of the schol-
arships
b. the total cost of tuition and the percentage of students who take
advantage of scholarships
c. the total cost of tuition considering scholarships and the cost of
room and board
d. the total cost of tuition considering scholarships and the break-
down of student attendance at public versus private colleges
e. the total cost of tuition considering scholarships and the num-
ber of students who attend college
501_02_25-120_501_master.qxd 4/29/13 12:30 PM Page 59
59
501 GMAT
®
Questions
50. The state legislature has become concerned with the rise in drunk
driving accidents on state highways. In an effort to curtail this
problem, the legislature raised the driving age from 17 to 18. The
governor lauded this effort, declaring to the press that this new law
will drastically reduce drunk driving accidents.
All the following statements strengthen the argument above
EXCEPT:
a. Seventeen-year-old drivers were involved in over half of the
drunk driving accidents on state highways.
b. A famous study reported that the alcoholism rate in minors was
exorbitantly high in this state.
c. A survey of teenagers found that the majority of teenagers did
not believe that alcohol consumption significantly impaired
their driving.
d. A report issued by the department of highway safety asserted
that less driving experience made teenagers slower to react to
hazards on the highway.
e. The state police’s statistics showed that most drunk driving
accidents occurred between 10
P.M. and 6 A.M., and the depart-
ment of motor vehicles issued only limited drivers licenses to
teenagers, which permitted driving only during daylight hours.
501_02_25-120_501_master.qxd 4/29/13 12:30 PM Page 60
60
501 GMAT
®
Questions
51. Te n p e o p l e w e r e h o s p i t a l i z e d f o r s e v e r e s t o m a c h p r o b l e m s w i t h i n
one day after drinking Zippo’s Cola. The Zippo Cola Company
fervently argued that its cola drink was not responsible for the
stomach problems and that the company was in no way responsible
for anyone’s illness. Instead, Zippo Cola released a statement to the
public that it believed that the company that makes the soda cans,
Carrie’s Canning Inc., used an ingredient in the metal that made
people sick. Margie Smith, one of the patients, told her doctor that
she was convinced that she ate bad shellfish. All 10 patients ate lunch
at the same restaurant that day. The doctors concluded that both the
restaurant and the cola were responsible for the patients’ illness.
Which of the following, if true, would cast the most serious
doubt on Zippo’s argument?
a. Carrie’s Canning, Inc. also supplied cans from the same can
supply to the Bubbles Orange Soda Company, and no one who
drank that soda became ill.
b. All 10 patients ate shrimp scampi as an appetizer at the restau-
rant that day.
c. All 10 patients ate meals at the restaurant that had no overlap-
ping ingredients with each other’s meals.
d. Seven of the patients drank regular Zippo Cola and three of the
patients drank Diet Zippo Cola.
e. Margie Smith’s lunch was a pasta dish with mussels.
501_02_25-120_501_master.qxd 4/29/13 12:30 PM Page 61
61
501 GMAT
®
Questions
52. The Department of Safe Transportation reported that 73% of all
accidents involve people who are supposed to wear glasses or
contact lenses. We can reduce car accidents if department
employees ask drivers whether they wear glasses or contact lenses,
and make a notation on the driver’s license, every time drivers
renew their license.
The argument is flawed because it fails to consider that
a. drivers who are supposed to wear glasses or contact lenses will
wear them while driving.
b. driver’s license renewals will catch a fair number of drivers who
have not notified the Department that they are required to wear
glasses or contact lenses while driving.
c. drivers who are supposed to wear glasses or contact lenses are
not aware of it.
d. a vision test will let drivers know that they need to see an oph-
thalmologist.
e. the departments of transportation in other states will not imple-
ment the same policy.
53. University Z created eight intramural sports teams in order to
combat obesity among the student population. The university
estimated that approximately 55% of students were overweight,
including 35% who were obese. Seventy-five percent of students
signed up and participated regularly in the intramural sports teams
for the entire year. At the end of the year, 55% of students
remained overweight, and 35% remained obese.
Which of the following, if true, best explains why the
university’s plan failed?
a. The students who participated in intramural sports continued
to have extremely poor eating habits that exercise could not
offset.
b. The students who participated in intramural sports were not
active previously.
c. The students who participated in intramural sports included
those who also participated in official university sports teams.
d. The students who participated in intramural sports possessed
various athletic abilities.
e. The students who participated in intramural sports were some-
what active previously.
501_02_25-120_501_master.qxd 4/29/13 12:30 PM Page 62
62
501 GMAT
®
Questions
54. Dr. Roberts, a dentist, gave a speech to a dental convention in
which she argued that the recent rise in tooth enamel decay in the
population came from a new brand of toothbrush. This brand used
a new type of plastic for the bristles, which Dr. Roberts claimed
wears down the enamel of the teeth over time. Dr. Roberts added
that the consistent soft drink consumption was not a significant
contributing factor to the recent rise in tooth enamel decay. She
then concluded her remarks by stating that tooth enamel decay in
patients will significantly decrease if every dentist convinces her
patients not to use this brand of toothbrush.
The argument is flawed primarily because the author
a. assumed that the cause of increased tooth enamel decay in her
patients is the same as in every other dentist’s patients.
b. assumed that consistent soft drink consumption was not a sig-
nificant contributing factor to the recent rise in tooth enamel
decay.
c. assumed that the new type of plastic used for the bristles was
the cause of the rise in tooth enamel decay in the general popu-
lation.
d. assumed that this brand of toothbrush is the only brand to use
the type of plastic for the bristles that Dr. Roberts claims wears
down tooth enamel over time.
e. assumed that because consistent soft drink consumption was not
a significant contributing factor to tooth enamel decay in her
patients, then it was not in any other dentist’s patients.
501_02_25-120_501_master.qxd 4/29/13 12:30 PM Page 63
63
501 GMAT
®
Questions
55. Television Executive: The largest demographic in this country is
senior citizens. Senior citizens are also more likely to not work
than the rest of the population because they have retired, and
therefore spend more time at home. The majority of senior
citizens dislike reality television. We can therefore conclude that
our network should plan fewer reality TV programs in the future.
Which of the following, if true, would most undermine the
television executive’s argument?
a. Reality television programs cost less to produce than other
types of programs.
b. Senior citizens are not the largest demographic that watches
television.
c. A smaller percentage of senior citizens are currently retired
than in previous generations.
d. The reality TV program called The Golden Years is extremely
popular among senior citizens.
e. Retired senior citizens generally spend their daytime hours
engaged in activities such as gardening, walking, and taking care
of grandchildren.
56. A company has decided to implement a new employee evaluation
program in which pay raises will be determined by an employee’s
overall rating given by his or her supervisor. Upper management
concluded that this new evaluation program would result in pay
raises for the employees with the most meritorious job performance.
Which of the following, if true, does NOT cast doubt on
upper management’s belief that the new program will result in pay
raises based on merit?
a. Supervisors’ judgment of what constitutes “exceeds expecta-
tions” is subjective.
b. All employees who meet revenue goals cannot receive “does not
meet expectations” on their employee evaluations.
c. The three ratings of “does not meet expectations,” “meets
expectations,” and “exceeds expectations” do not provide
enough information to differentiate the various levels of
employee job performance.
d. Supervisors who like their employees tend to rate them higher
than supervisors who dislike their employees.
e. The new employee evaluation program records the information
in a database that the human resources department can access
when processing pay raises.
501_02_25-120_501_master.qxd 4/29/13 12:30 PM Page 64
64
501 GMAT
®
Questions
57. A recent economic downturn has reminded people of the
importance of saving, both for emergencies and for retirement.
The average household saved 5% of its income in the past year,
and that number is predicted to grow over the next 10 years. The
result will be that banks will have more money to lend to
customers to start businesses and buy homes. We can conclude
that economic activity will grow over the next 10 years as a result
of this ethic in saving money.
Which of the following would most weaken the argument?
a. People who place a greater percentage of their income into
their savings accounts have been and will be underemployed
over the next 10 years.
b. People who place a greater percentage of their income into
their savings accounts will take out loans to start businesses.
c. People who place a greater percentage of their income into
their savings accounts will also pay down their credit card debt.
d. People who place a greater percentage of their income into
their savings accounts will focus their spending on only neces-
sary home improvements.
e. People who place a great percentage of their income into their
savings accounts will enable the banks to curtail fee increases.
501_02_25-120_501_master.qxd 4/29/13 12:30 PM Page 65
65
501 GMAT
®
Questions
58. Retail stores that pay their sales associates based on commission
pay their employees wages that vary with the sales dollar amount
for which each employee is responsible. In the retail business last
year, stores that paid their employees on commission showed that
worker productivity was 17% higher than that of their competitors
that paid their sales associates an hourly wage.
If, on the basis of the evidence above, it is argued that paying
sales associates on commission increases worker productivity, which
of the following, if true, would most seriously weaken the argument?
a. Retail stores that pay their sales associates on commission have
experienced that costs other than wages, such as rent, utilities,
and inventory processing, make up an increased proportion of
the total cost of operating each store.
b. Retail stores that pay their sales associates on commission have
more troubling hiring workers because many applicants are
concerned about making little money on days when the stores
have few customers.
c. Sales associates who earn money on commission have paychecks
that are 22% higher than those of sales associates who are paid
an hourly wage.
d. The productivity results cited for sales associates in retail stores
are also true for telemarketers.
e. The majority of customers who shop in retail stores report that
they purchase items without assistance from sales associates.
501_02_25-120_501_master.qxd 4/29/13 12:30 PM Page 66
66
501 GMAT
®
Questions
59. College President: This state’s families actually do not pay less for
tuition at public colleges than private colleges. Each family in this
state pays taxes, a portion of which go to state colleges. When you
add the amount of those taxes with the cost of tuition at a public
college, the cost is actually the same as tuition at a private college.
The college president’s argument is flawed because it fails to
consider that
a. the cost of tuition at a private college is higher than the cost of
state taxes paid toward a public college and tuition at a public
college.
b. a portion of taxes that each family in the state pays goes to pub-
lic colleges even if no members of that family attend college.
c. many private colleges offer scholarships to students who
demonstrate superior academic achievement.
d. a portion of every family’s taxes in the state goes to public col-
leges even if members of that family attend a private college, so
the taxes are not actually part of the cost of tuition.
e. the price of tuition at a public college depends on the state’s
budget.
60. Economist: Large price club type stores will eventually drive small,
independently owned stores out of business. Price clubs can price
their goods much lower per item because they sell products in bulk
sizes. Small stores simply cannot compete with those prices.
Which of the following, if true, does NOT weaken the
argument above?
a. Price club stores may be able to offer lower prices, but they only
sell a limited variety of goods, and customers will need to shop
elsewhere to find the remainder of the goods that they want.
b. An increasing majority of customers use public transportation
instead of cars, and live in small apartments rather than houses,
so they cannot transport or store a large number of goods.
c. Many customers desire to support independently owned busi-
nesses, and will shop there rather than at price clubs even if it
means paying higher prices.
d. Independently owned stores accept coupons, and most customers
report that when they factor in coupons, they wind up paying the
same price at independently owned stores as they do in price clubs.
e. Price club customers report that they earn back the price of
their membership fee within the first two months of purchases.
67
501 GMAT
®
Questions
501_02_25-120_501_master.qxd 4/29/13 12:30 PM Page 67
61. Osteoporosis is a disease that results in a severe weakening of
bones. A diet rich in calcium throughout one’s lifetime is commonly
known to be the best prevention against osteoporosis. Sources of
calcium include legumes, leafy green vegetables, and dairy
products. A medical group devoted to raising the awareness of
osteoporosis recommends that people consume more leafy green
vegetables than dairy products in order to raise their calcium intake.
Which of the following, if true, would most strongly support
the position above?
a. Dairy products typically contain fat, and leafy green vegetables
do not.
b. Leafy green vegetables provide many other health benefits,
most notably a higher iron content.
c. Nutritionists recommend five to eight servings of leafy green
vegetables per day, and only three servings of dairy per day.
d. The body’s absorption rate for calcium in leafy green vegetables
is 50%, and for dairy products is 30%.
e. The farming of leafy green vegetables uses fewer environmental
resources than the production of dairy products.
501_02_25-120_501_master.qxd 4/29/13 12:30 PM Page 68
68
501 GMAT
®
Questions
62. Each unit in an apartment complex is supplied with a refrigerator,
stove, microwave, and dishwasher. In order to keep the appliances
functioning, the apartment complex employs a full-time super who
is on call Monday through Friday from 9
A.M. to 5 P.M. Without
repairs, the appliances would eventually stop working and would
be expensive to replace. We can conclude that the apartment
complex should continue to employ the super full-time.
Which of the following, if true, would most seriously weaken
the argument above?
a. Most tenants discover that their appliances are not working
during the evening hours.
b. Apartment complexes that install dishwashers in their units use
a much greater amount of water than those without dishwashers
in their units.
c. Appliances become obsolete in only a few years, which makes
repairing them less practical and more costly then buying new
appliances.
d. Employing a super full-time requires benefits, such as medical
insurance and paid vacation time, in addition to a salary.
e. This apartment complex uses electric stoves, and gas stoves usu-
ally have lower utility bills.
501_02_25-120_501_master.qxd 4/29/13 12:30 PM Page 69
69
501 GMAT
®
Questions
63. The textbook publishing industry reported a consistent annual
increase in e-book sales. For two years, publishers of law textbooks
offered a special promotion, in which the e-book version was
offered free along with the hard copy. Publishers of law textbooks
expected this exposure to e-books to ease law students’ transition
away from hard copies and result in a decline in sales of hard
copies in favor of e-books. Surprisingly, this decline did not
occur.
Which of the following would be the most useful to
determine in order to evaluate why the publishers’ promotion did
not achieve the expected results?
a. the average age of law students and their respective familiarity
with the e-book format before the promotion
b. the weight of the average hard copy law textbook
c. the percentage of courses covered by the textbooks with the
promotion
d. the percentage of law students who previously bought e-books
compared to the percentage of undergraduate students who
bought e-books
e. the majority of law students who had open-book exams and
took their exams on their laptop computers, which required an
exam software that blocked student access to all their computer
files during the exam
501_02_25-120_501_master.qxd 4/29/13 12:30 PM Page 70
70
501 GMAT
®
Questions
64. The press has consistently reported stories about the adverse
effects of soy on people with existing thyroid problems, and about
the environmental problems that stem from farming soybeans. A
national health organization stated that fewer people reported
consuming soy products in place of meat and dairy, and an
increasing number of people reported concern that they might be
allergic to soy. Therefore, we can conclude that soy consumption
will decrease over the next few years.
Which of the following, if true, would most undermine the
conclusion above?
a. Soy is a common ingredient in processed foods that contain
meat.
b. People do not discover a food allergy until after they have had
an adverse reaction to food.
c. Many people suspect that they have a thyroid problem even
when they do not actually have one.
d. People base many of their food choices on the effect they have
on the environment.
e. Meat and dairy products tend to be more expensive than soy
products.
Formulating and Evaluating a Plan of Action
Examples of formulating and evaluating a plan of action questions are: iden-
tify the effectiveness of a plan, identify whether the plan is appropriate to
address the problem at hand, recognize whether another plan would be
more efficient, identify factors that would strengthen a plan, and identify
factors that would weaken a plan.
Identify the Effectiveness of a Plan/Recognize Whether Another
Plan Would Be More Efficient
These questions will ask you to identify whether a proposed plan will
address the actual course of a problem effectively. These questions will also
ask you to recognize whether another plan would provide a better solution
to the problem.
Which of the following is the LEAST important to know to
determine the success of the plan?
501_02_25-120_501_master.qxd 4/29/13 12:30 PM Page 71
71
501 GMAT
®
Questions
Which of the following is NOT important to know for the
plan?
Which of the following would help to explain why the plan
failed?
Strengthen the Plan
These questions will ask you to identify the answer choice that would
strengthen the plan in the passage.
Which of the following, if true, provides the strongest reason
to expect the plan to succeed?
Which of the following, if added to the plan, would be most
effective in helping the above plan?
Weaken the Plan
These questions will ask you to identify the answer choice that would
weaken the plan in the passage.
Which of the following, if true, would provide the most serious
disadvantage to the plan above?
Which of the following, if true, would most likely worsen the
plan above?
Formulating and Evaluating a Plan of Action
Test-Taking Strategies
1. Identify the assumption.
This strategy is in each section of this book for the Critical Reasoning por-
tion of the GMAT
®
, but it bears repeating. Most of the plans in this type
of question are either business related or government related. Do not let
your own knowledge of these subject matters cause you to make assump-
tions about the plan. Instead, find the gap in the reasoning and identify
whether it is a causal connection, shift, or similarity.
2. Identify whether the plan could actually address the problem.
If the passage states that the plan failed, evaluate the plan and see whether
it actually could have addressed the problem. What was the flaw in the plan?
Did the plan fail to consider something? Did it address an effect of the
problem rather than the source? If the answer choices list alternative or
501_02_25-120_501_master.qxd 4/29/13 12:30 PM Page 72
72
501 GMAT
®
Questions
additional plans designed to rectify the problem that the original plan failed
to solve, look for answer choices with plans that have the same flawed rea-
soning and eliminate them.
3. Watch out for the numbers.
Since many of the plans that you will evaluate on the test will have to do
with business or government, they will likely contain numbers. Watch
out for assumed relationships between raw numbers and percentages.
You s ho ul d a ls o be aw ar e o f a ny co mp ar is on s b et ween co mb in at io ns of
different variables. If the numbers are vague and you cannot figure out
how anyone could eliminate answers, remember that the question may
ask you for an answer choice that is the only one for which you can draw
a conclusion.
4. Identify reasons for the plan’s failure or success.
Many formulating and evaluating a plan of action questions will ask you to
identify the answer choice that explains the reason for a plan’s failure or
success. If you see this type of question and then reread the passage, you
could benefit from thinking of your own reasons before looking at the
answer choices, much like thinking of an argument’s conclusion before
reading the conclusions that the answer choices offer in argument con-
struction questions.
501_02_25-120_501_master.qxd 4/29/13 12:30 PM Page 73
73
501 GMAT
®
Questions
Set 4
Now it is time to answer GMAT Critical Reasoning practice questions that
have been designed to test your formulating and evaluating a plan of action
skills. Good luck!
65. Millipoint is a large city that employs many people from the
surrounding suburban communities. Suburban Commute
Transportation Company must expand its services for the
increasing number of people who commute to Millipoint.
Suburban Commute could replace its trains with double-decker
trains, add 10% more ferries, or add 25% more buses. Trains
typically spend the least amount of time in traffic, which is
important because they run every 15 minutes. Ferries have a
capacity of 75 people apiece. Buses have a capacity of about 50
people apiece and tend to spend time in traffic. After considering
all options, Suburban Commute decided to replace its trains with
double-decker trains.
Which of the following, if true, could represent the most
serious disadvantage for Suburban Commute’s plan?
a. Two new highways were constructed that lead into Millipoint.
b. Double-decker trains require twice the amount of boarding
time as single-level trains.
c. A competitor has significantly increased its use of the same train
tracks that Suburban Commute uses.
d. Double-decker trains running against rush hour traffic tend to
be nearly empty.
e. Double-decker ferries are available.
501_02_25-120_501_master.qxd 4/29/13 12:30 PM Page 74
74
501 GMAT
®
Questions
66. The Consumer Watchdog Agency reports that 65% of red snapper
sold in grocery stores and restaurants is actually less expensive
types of fish fraudulently labeled. In an effort to clamp down on
this fraud, the agency has created a new consumer confidence
stamp that would mark packages of red snapper as inspected and
verified by the Consumer Watchdog Agency.
Which of the following, if true, provides the strongest reason
to expect that the Consumer Watchdog Agency will succeed in
decreasing the percentage of less expensive types of fish fraudu-
lently sold as red snapper?
a. Grocery store inventory managers are usually the individuals
who place other fish in the red snapper section of the fresh fish
counter.
b. Restaurant owners have a common practice of passing off other
fish as red snapper.
c. Most customers are unable to tell the difference between red
snapper and other types of fish.
d. The majority of red snapper sold is sold fresh off the boat with-
out packaging.
e. The fishing industry is interested in curtailing the fraud in red
snapper sales.
501_02_25-120_501_master.qxd 4/29/13 12:30 PM Page 75
75
501 GMAT
®
Questions
67. Potter Shipping, Inc. had a relatively high failure rate in delivering
packages on time. In order to address this problem, the CEO
promised drivers a 10% year-end bonus if they succeeded in
delivering 90% of their packages on time.
All the following, if true, would help to explain why the
CEO’s bonus incentive plan failed to achieve a significantly better
on-time delivery rate EXCEPT
a. highway construction projects caused drivers to spend hours of
time sitting in traffic.
b. most company trucks had faulty GPS units.
c. drivers were responsible to pay for their own fuel, which was
expensive, and faster driving consumed greater quantities of
gasoline.
d. other shipping companies paid their drivers a 15% bonus if they
delivered 90% of their packages on time, and many Potter driv-
ers wound up leaving their jobs within six months to work for
competitors.
e. most drivers believed the 10% bonus to be a great incentive to
increase on-time deliveries.
68. A group called People for the Preservation of the Environment has
published a damaging report about the pollution caused by Trikas
Oil Refineries. Although not accused of any illegal activity, Trikas
has decided that it must decrease the air pollution it generates in
order to placate public opinion. In order to accomplish this goal,
Trikas plans on upgrading its refineries with the Zeebot
Purification System.
The adoption of the Zeebot Purification System would most
likely worsen Trikas Oil Refineries’ problem if
a. the Zeebot Purification System costs a significant amount of
money.
b. the Zeebot Purification System malfunctions 10% of the time.
c. the Zeebot Purification System will take six months to imple-
ment.
d. the Zeebot Purification System takes a lot of manpower to
maintain.
e. the Zeebot Purification System results in a significant increase
in water pollution.
501_02_25-120_501_master.qxd 4/29/13 12:30 PM Page 76
76
501 GMAT
®
Questions
69. The government of Woodrock decided five years ago to increase
the number of preventative care medical services covered in the
government’s free health care program for seniors.
Which of the following would be the LEAST important in
determining whether the increased number of preventative care
medical services would be likely to achieve better health for
Woodrock’s senior population?
a. the percentage of seniors who seek preventative care medical
services
b. the success rate of preventative care medical services
c. the cost of preventative care medical services
d. the senior population’s access to facilities that provide preventa-
tive care medical services
e. the percentage of seniors who utilize the government’s free
health care program for seniors
70. In the upcoming elections, the Grow the Economy Now party’s
candidates have proposed a government tax break to individuals
and companies who invest in start-up businesses. The plan would
exempt these individuals and companies from paying any taxes on
the profits from these investments for five years.
All the following would NOT be important to know in
determining whether the tax break would provide an incentive to
invest in start-up businesses EXCEPT
a. the amount of capital required to start a business.
b. the tax bracket of the average individual investor.
c. the most common types of start-up businesses created.
d. the number of years a start-up business requires to earn a profit.
e. the number of candidates in the Grow the Economy Now party
who have a history in investing in start-up businesses.
501_02_25-120_501_master.qxd 4/29/13 12:30 PM Page 77
77
501 GMAT
®
Questions
71. The Lightening Tread Shoe Company, in an effort to boost
revenue, has decided to offer free shipping on its website for all
orders over $22.00. The cost of shipping these orders will reduce
the profit by $4.00 per shipment. However, management believes
that the savings to customers will result in more orders.
Which of the following would be NOT be important to know in
determining whether this plan will result in an increase in revenue?
a. the current price of packing materials
b. whether competitors of the Lightening Tread Shoe Company
also offer free shipping on orders over $22.00
c. changes in the average price of an order
d. changes in postal rates
e. whether customers purchased the same amount of merchandise
spread out in several smaller orders
72. A citywide survey was conducted in the past year, and revealed that
35% of employees do not know where the fire exits are in their
places of employment. Previously, a private safety company would
conduct annual fire drills in all commercial city buildings that
consisted of more than four floors. In order to increase employee
knowledge of the location of fire exits, the city has arranged to pay
fire department employees a bonus to conduct a second annual fire
drill in these buildings.
Which additional plan, combined with the plan just outlined,
will be most effective in helping the city address the high
percentage of employees who do not know the location of the fire
exits in their workplace?
a. hiring the private safety company to conduct annual fire drills
in city buildings with less than four floors
b. paying fire department employees a bonus to conduct a second
annual fire drill in city buildings with less than four floors
c. hiring the private safety company to conduct two annual fire
drills in all city buildings
d. finding a new private safety company to conduct the annual fire
drills
e. threaten to fine companies with over 35% of employees unable
to identify the location of the fire exits
501_02_25-120_501_master.qxd 4/29/13 12:30 PM Page 78
78
501 GMAT
®
Questions
73. Plan: The government of Tulpso was concerned about the high
level of credit card debt that its average citizen carried, and about
the high amount of credit card debt the population carried as a
nation. The government convinced the banks in the credit card
industry to offer a 4% lower APR for existing debt to any customer
who paid off more than the monthly interest. The government
subsidized the difference in interest rates.
Result: Many Tulpso citizens still carry the same amount of
debt as before.
Further Information: The annual rate of inflation since the
government-subsidized reduced interest rate on existing credit
card debt has been below 3%, and every Tulpso citizen who
carried credit card debt paid more than the monthly interest of
their credit card bills.
In light of the further information, which of the following, if
true, does most to explain the result that followed implementation
of the plan?
a. The most recent reduction in APR was only the second of its
kind in the last 20 years.
b. The Tulpso credit card repayment system required citizens to
pay their bills in person.
c. The majority of citizens in Tulspo needed this program to pay
down their credit card debt.
d. The prices of food and other necessities rose to a level that
required citizens to charge similar amounts to what they repaid.
e. The government offered the reduced APR at a time when a
large number of people were poverty-stricken.
501_02_25-120_501_master.qxd 4/29/13 12:30 PM Page 79
79
501 GMAT
®
Questions
74. The population in the country of Xusa has remained stable for the
last 20 years and is expected to remain so. The airline industry has
decided to triple the number of flights offered to current
destinations in the next year.
Which of the following would be most helpful to justify the
airline industry’s plan?
a. The largest segment of the population has reached retirement
age.
b. Businesses have reported a substantial rise in attendance at
trade shows around the nation.
c. The price of airplane fuel has decreased.
d. Companies that produce airplane meals have offered substan-
tially better wholesale prices to airlines.
e. A famous university study has reported that the percentage of
the population with a fear of flying has dropped significantly
over the last 10 years.
75. The customer service department of the E-Z Movie Rental Co.
has reported a 15% increase in customer service complaints
regarding damaged or unplayable discs in the past year. The CEO
announced that employees responsible for packing the discs into
mailer envelopes are required to perform a new 3-step process to
check for scratches and other visible defects on the discs. Over the
next year, the customer service department reported a 5% increase
in customer complaints regarding damaged or unplayable discs.
All the following, if true, would explain why the new 3-step
process failed to achieve the company goal EXCEPT
a. the paper that the mailer envelopes were made of was a particu-
larly abrasive type of paper.
b. the warehouse employees did not comply with the new 3-step
process when mailing out discs.
c. most of the customers who complained about damaged or
unplayable discs had damaged those discs themselves.
d. the majority of the discs’ defects were visible defects.
e. damaged or unplayable discs were not taken out of circulation.
501_02_25-120_501_master.qxd 4/29/13 12:30 PM Page 80
80
501 GMAT
®
Questions
76. The Whigham Insurance Company offers All Risk insurance to
hotels, resorts, and other large structures. All Risk insurance covers
everything that is not listed in the exclusions. There has been a
recent increase in earthquakes and Whigham is concerned that
earthquake coverage will be too expensive to maintain. However,
Whigham would still prefer to avoid listing earthquakes in the list
of exclusions for the future if possible.
Which of the following strategies would be most likely to
minimize Company X’s losses on the policies?
a. insuring only those customers whose buildings did not experi-
ence earthquake damage in the past 20 years
b. insuring only those customers who cannot obtain earthquake
coverage through another insurance provider
c. expanding its marketing campaign to attract customers with
buildings far away from fault lines
d. insuring only those customers who can afford to pay for earth-
quake coverage for their buildings
e. insuring other disasters such as tornadoes, hurricanes, and mon-
soons, in the hopes of maintaining customer loyalty
77. A company that manufactures laundry detergent has decided to
switch from petroleum-based ingredients to plant-based
ingredients. The company believes that the switch will decrease its
environmental footprint and address frequent customer complaints
that the detergent tends to cause the color in clothes to fade
quickly.
Which of the following, if true, most helps to provide a
justification for the company’s planned switch?
a. The cost of petroleum-based ingredients and plant-based ingre-
dients is the same.
b. The company plans to move its manufacturing overseas.
c. The annual sales of laundry detergent have decreased over the
past two years.
d. The company will continue to manufacture stain remover pens
that have stronger ingredients.
e. Consumer spending on higher-quality clothing is anticipated to
grow over the next 10 years.
501_02_25-120_501_master.qxd 4/29/13 12:30 PM Page 81
81
501 GMAT
®
Questions
78. Currently, the market for ice cream is dominated by dairy-based
ice cream. To increase its revenue, the Simply Soy marketing
department developed a new campaign for its line of soy-based ice
cream flavors. The marketing team convinced the product
development team that it should amplify the food coloring used in
all the flavors, and the marketing team would release ads with
pictures of the ice cream designed by a food stylist.
What condition will make the marketing department’s plan
more likely to increase revenue?
a. The food coloring used in the Simply Soy flavors consists of all
natural ingredients.
b. The number of people who are lactose intolerant has increased.
c. People who buy ice cream are not bothered by food that con-
tains artificial colors.
d. The public in general is gravitating toward more plant-based
foods.
e. People tend to base their grocery shopping decisions based on
how appealing the products look.
79. Both universities and the government are concerned about the low
level of student achievement in math and science. In addition, the
government is also concerned about the rising cost of university
tuition. A university suggested that private enterprise could locate
some of their research in university laboratories, and provide
tuition assistance to promising students who aid in the research.
Which of the following, if true, would be most useful to know
in evaluating the university’s suggested plan?
a. the size of the student applicant pool for the research program
b. the science and math aptitude of the student applicant pool for
the research program
c. the value of incentives that universities would offer private
enterprise to conduct research in university laboratories
d. the number of students who would major in math and science as
a result of the plan
e. the type of research that private enterprise would conduct in
university laboratories
501_02_25-120_501_master.qxd 4/29/13 12:30 PM Page 82
82
501 GMAT
®
Questions
80. Construction Company Y always used power drills that plug into
an electrical socket. Power-Z has manufactured a new type of
battery-powered drill that uses so little energy that it is much
cheaper to use. Thrilled that the price of both kinds of drills is the
same, Construction Company Y has decided to replace all its drills
with the new Power-Z drill, and estimates that the replacement
costs will pay off within three years.
Which of the following, if it occurred, would constitute a
disadvantage for Construction Company Y of the plan described
above?
a. construction Company Y would need to buy 350 new drills
b. an increase in the market demand for construction work
c. other companies that manufacture power tools introduced simi-
lar drills at the same price
d. a significant increase in the price of batteries in the next year
e. a shortage of industrial-sized extension cords in hardware stores
81. A national long distance track coach created a new cross-training
program for the team, which includes swimming, weight training,
sprinting staggered with jogging, and yoga. The coach has stated
to the press that the new cross-training program will result in an
Olympic podium sweep by the nation’s team.
Which of the following, if true, could present the most
serious disadvantage for the coach’s cross-training program?
a. The yoga exercises would enhance the runners’ breathing capa-
bilities under physical stress.
b. None of the national team’s competitors have considered this
type of cross-training program.
c. The cross-training program would lead to improvement in
some of the runners on the team, but runners on other teams
would likely have trouble with it.
d. The program requires additional athletic gear, which is very
expensive.
e. The weight training would add muscle to the runners, which
would result in a slight weight gain that would slow down their
times.
501_02_25-120_501_master.qxd 4/29/13 12:30 PM Page 83
83
501 GMAT
®
Questions
82. Winston University had many cases of meningitis the previous
year. The Director of Residence Life presented a plan to prevent
this from happening again this year. All university students covered
by the student health plan would be required to obtain a
meningitis vaccine before moving into the dormitories.
The answer to which of the following questions would be
most important in determining whether implementing the
proposal would be likely to achieve the desired result?
a. the percentage of Winston University students who are covered
by the student health plan
b. the percentage of Winston University students who live in the
dormitories and are covered by the student health plan
c. the percentage of Winston University students who live in the
dormitories
d. the percentage of Winston University students who live off
campus
e. the percentage of Winston University students who did not
receive a meningitis vaccine the previous year
83. A non-governmental organization raised money to donate
thousands of anti-malarial mosquito nets to a country in which
malaria is a serious public health threat. The organization also
operated a chain of free clinics in that country. The organization
transported the nets in April of that year. Four months later, the
president of the organization announced that the severe decline in
malaria outbreaks would continue throughout the remainder of the
year, and adjusted the clinic budget by slashing its orders for
quinine, the treatment for malaria, in half.
Which of the following, if true, would most undermine the
president of the organization’s decision to order half as much
quinine for the year?
a. Quinine is used to treat a few other uncommon ailments.
b. The chain of clinics treats on average only 40% of the country’s
population.
c. The wet season, during which malaria is prevalent, begins in
October and ends in March.
d. The percentage of the population that received the anti-
malarial mosquito nets is unclear.
e. Other clinics operated by different organizations ordered the
same amount of quinine.
501_02_25-120_501_master.qxd 4/29/13 12:30 PM Page 84
84
501 GMAT
®
Questions
84. Foreign language skills are vital in today’s international economy.
Studies have shown that the earlier a student begins to learn a
language, the more proficient that student will become with that
language. The school district of M wants to increase student
foreign language skills. The superintendent of the M school
district concluded that requiring all students to begin studies in a
foreign language in fourth grade instead of seventh grade will
result in a greater number of students who are proficient in a
foreign language.
Which of the following, if true, would cast the most doubt on
the success of the superintendent’s plan?
a. The majority of students who study a foreign language in the
school district already speak a foreign language at home.
b. The instruction time for foreign language study will be signifi-
cantly less for the earlier grades than for grades 7 and above.
c. Students will still have a choice between only two languages
d. Foreign language study for fourth graders will take some
instruction time away from social studies.
e. Most elementary school teachers in school district M do not
have training in a foreign language.
85. Trial courts in the state’s civil division have found that jurors pay
less attention in civil trials than they do in criminal trials, which
jurors find to be more interesting. In order to maintain the
attention of jurors, the county bar association proposed that all
civil litigators use color and/or interactive visual aids.
Which of the following, if true, provides the strongest reason
to expect that the bar association’s plan will succeed in getting
jurors to pay attention during civil trials?
a. Jurors will find the color and/or interactive visual aids as inter-
esting as criminal trials.
b. Jurors will find the color and/or interactive visual aids interest-
ing enough to hold their attention.
c. Judges will allow the civil litigators to use color and/or interac-
tive visual aids.
d. Civil litigators will actually want to use color and/or interactive
visual aids.
e. Criminal litigators will not start to use color and/or interactive
visual aids.
501_02_25-120_501_master.qxd 4/29/13 12:30 PM Page 85
85
501 GMAT
®
Questions
86. The Everglades has an ecological problem because of its
population of Burmese pythons, which are not a native species of
the area and were likely introduced by people who dumped them
when they were no longer desirable as pets. In order to address this
problem, a local ecological group created a procedure for safely
capturing the pythons.
Which of the following, if true, would provide the most
support for the conclusion that the ecological group will capture
most or all of the Burmese pythons in the Everglades?
a. The ecological group has a group of dogs trained to sniff out
the location of the Burmese pythons.
b. The local government has increased the fines for people caught
releasing animals into the Everglades.
c. The Burmese python is one of several snakes in the Everglades
that are not native to the area.
d. Burmese pythons are not venomous.
e. Other environmental groups have also created plans to reduce
the Burmese python population in the Everglades.
87. A state has used the speed limit of 65 miles per hour on its
highways over the last several years. A study has shown that many
drivers pulled over for speeding on these highways were not
driving at an unsafe speed. In order to lower the number of drivers
pulled over for speeding who were not driving at dangerous
speeds, the state has decided to increase the speed limit on its
highways to 70 miles per hour.
Which of the following would be most useful in evaluating
the state’s plan to lower the number of drivers pulled over for
speeding who were not driving at dangerous speeds?
a. the number of drivers who will continue to drive at speeds
greater than 70 miles per hour
b. the percentage of drivers who were pulled over for speeding as
opposed to the percentage of drivers who were pulled over for
other reasons
c. the number of the state’s voters who believe that speeds of 70
miles per hour and under are not unsafe
d. the number of drivers who were pulled over for driving at
speeds between 66 miles per hour and 70 miles per hour
e. the number of drivers on the state’s highways during rush hour
501_02_25-120_501_master.qxd 4/29/13 12:30 PM Page 86
86
501 GMAT
®
Questions
88. The Bridgeview shopping mall’s customer service department has
reported many complaints from customers about a shortage of
parking in the parking deck. The Bridgeview mall drafted a plan in
which it would construct two additional levels to its parking deck,
and would charge each retailer in the mall a $4,000 fee to help pay
for it, to which the retailers all agreed.
Which of the following, if true, would be most useful to
evaluate the benefit to the retailers for agreeing to pay the fee in
order to enable more customers to shop at the mall?
a. the percentage of mall customers that drive to the mall, as
opposed to taking other forms of transportation
b. whether the additional number of parking spaces will alleviate
the parking shortage
c. the actual number of complaints to customer service about a
shortage of parking spaces
d. the cost of the plan to the Bridgeview mall after the retailers
each paid their fee
e. whether the $4,000 fee that each retailer would pay toward the
additional two levels in the parking deck would actually pay off
in sales revenue
89. A municipality is concerned about the number of car accidents that
occurred during the winter. The mayor suggested that it consider
buying and staffing snowplows in order to alleviate the problem.
The municipality purchased four snowplows, and offered a $15 per
hour wage to applicants to drive the snowplows when needed.
Which of the following would be most useful in determining
whether the municipality’s plan to decrease car accidents during
the winter season will actually succeed?
a. whether job applicants will drive the snowplows for $15 per
hour
b. whether most winter car accidents are a result of snow
c. whether most car accidents in the municipality occur during
winter
d. whether four snowplows will be enough
e. whether most cars in the municipality have snow tires
501_02_25-120_501_master.qxd 4/29/13 12:30 PM Page 87
87
501 GMAT
®
Questions
90. Kitchen, Kitchens, Kitchens, Inc.! is a kitchen appliance company
that is looking to increase sales for stainless steel appliances. The
company designed its new line of appliances to be more energy
efficient. Advertisements will show the total of the annual savings
in energy costs plus the average government tax rebate to
homeowners for increasing the energy efficiency of their homes.
Which of the following, if true, raises the most serious doubt
regarding the company’s expected success from its new advertising
campaign?
a. Competitors plan on highlighting the appearance of the appli-
ances in their advertisements.
b. The savings generated by energy savings and the tax rebate are
less than the cost of the new appliances.
c. Stainless steel appliances always have a higher price than appli-
ances in basic colors.
d. Most customers want to buy their kitchen appliances in match-
ing sets.
e. The savings generated by energy efficient appliances and the tax
rebate are negligible.
91. Many universities have elected to turn their undergraduate
programs into 5-year programs, in order to incorporate mandatory
internships for credit. Although students will have an additional
year of study, they will gain valuable working experience that will
look great on their resumes in this recession and this experience
will be worth the cost of the additional year’s tuition.
Which of the following, if true, provides the most support for
the potential success of attracting undergraduate applicants to 5-
year undergraduate programs?
a. Most students take out only a small quantity of money in stu-
dent loans, and find their monthly payments to be affordable.
b. The recession has meant that employers have their pick of stu-
dents, and prefer those with experience.
c. Students have the opportunity to intern full-time during their
summers.
d. Most of the internships are offered in big cities that students
regard to be fun and exciting.
e. About half of undergraduate institutions are planning on offer-
ing these 5-year programs.
501_02_25-120_501_master.qxd 4/29/13 12:30 PM Page 88
88
501 GMAT
®
Questions
92. The We Care Insurance Company wants to expand by offering
new insurance products. One of the members of the product
development team proposed that the company offer pet insurance.
The team agreed that it was a good idea, until the member who
pitched the idea shared an industry study, which stated that pet
insurance clients tend to avoid policies that raise the annual
premium based on the number of claims submitted. Since pet
insurance cannot legally drop a pet’s coverage once the pet
becomes ill or significantly injured, the product development team
is worried that pet insurance will not be profitable after all.
Which of the following strategies would most likely enable
the We Care Insurance Company to offer pet insurance?
a. capping the amount of covered services per pet at the cost of an
annual checkup and shots
b. require pet owners to bring their pets only to veterinary prac-
tices that participate in the We Care Insurance Company’s plan
c. insure only those customers who are wealthy enough to afford
many veterinary services for their pets
d. raise premiums based on the age of the pets
e. insure only pets with no history of medical problems
501_02_25-120_501_master.qxd 4/29/13 12:30 PM Page 89
89
501 GMAT
®
Questions
Answers
Set 2
11. b. Choice b identifies the strategy in Peter’s counterargument.
Peter points out that Louis did not consider that the financial
industry’s losses in profits and jobs will result in profit and job
increases for others. Choice a is incorrect because Peter does not
offer additional evidence to support Louis’s argument. Choice c is
incorrect because Peter does not argue that the regulations on
derivatives will have a negative effect on the national economy. He
argues that the increase in profits and jobs in other industries will
compensate for the loss in profits and jobs in the finance industry.
Choice d is incorrect because Peter does not challenge Louis’s
facts; he supplements them. Choice e is incorrect because Peter
does not challenge the relevance of Louis’s facts.
12. d. Choice d is correct because the cost of repairing the cell towers
does not need to be lower to justify the cost savings of this plan.
Those repairs must cost more than the cost of this plan for the cell
towers. Choices a, b, c, and e are incorrect because they all
mention items that should cost less than the repairs to the cell
towers during the previous year.
13. e. This is a problem that you should attempt to sketch out. Try to
connect the names to make a chain. Katherine has a higher GPA
than Pamela and Adrienne. Shane has a higher GPA than Madelyn,
who has a higher GPA than Adrienne.
P & A < K
A < M < S
Choice e is correct because Adrienne cannot have a higher
GPA than Shane. Choices a and b are incorrect because the
argument does not tell you whether Katherine or Madelyn has a
higher GPA. Choice c is incorrect because the argument does not
tell you whether Pamela or Adrienne has a higher GPA. Choice d
is incorrect because Pamela could have a higher GPA than
Madelyn.
501_02_25-120_501_master.qxd 4/29/13 12:30 PM Page 90
90
501 GMAT
®
Questions
14. a. The argument shows that Sharon did not inherit the cottage
because Claire McGinnis did not own the cottage when she died.
Choice a is correct because we do not know whether Claire would
have left the cottage to her niece, even though Claire left Sharon
the rest of her property. Choices b and e are incorrect because
whether Humphrey Monroe wanted Sharon to inherit the cottage
is irrelevant to the argument’s conclusion. Choice c is incorrect
because whether Sharon wanted to inherit the cottage is irrelevant.
Choice d is incorrect because the assumption is about who Claire
wanted to inherit the cottage, not who Claire did not want to
inherit it.
15. a. Choice a is correct because it is possible that the employees in
State A each work such a low number of hours per week and that
the employees in State B work such a high number that the total
number of hours paid for is actually higher in State B. Choice b is
incorrect because there is not necessarily a correlation between the
number of employees and the number of restaurants since many
employees could work very few hours. Choice c is incorrect
because the statement would actually contradict the argument’s
conclusion. Choice d is incorrect because there is no indication
that state law has anything to do with why employees in State A
are paid a higher wage. Choice e is incorrect because the argument
does not say anything about what might happen in the future.
16. b. Choice b is correct because in order for buying diamonds to
cost 40% more in Luvania, the tax on purchases by foreigners in
Oretania and the cost of transportation to Oretania must be 40%
less than the cost of buying diamonds in Luvania. Choice a is
incorrect because the ease of the ground breaking under the force
of a pickaxe in Oretania is merely a plausible explanation for the
cost difference, but there are also other possible reasons. Choice c
is incorrect because if the transportation cost is more than 40%,
buying diamonds in Oretania would be more expensive. Choice d
is incorrect because the cost of diamond mining is merely a
possible explanation for the cost difference. Choice e is incorrect
because there is no evidence about retail jobs in jewelry stores in
the passage, and the passage also indicated that people would
actually be less likely to buy diamonds in Luvania.
501_02_25-120_501_master.qxd 4/29/13 12:30 PM Page 91
91
501 GMAT
®
Questions
17. d. Choice d is correct because fixing the filtration system at the
point of groundwater collection will not change the excess chlorine
content if the problem starts later in the process. Choice a is
incorrect because whether all faulty water filtration systems allow
excess chlorine into drinking water is irrelevant. Choices b and e
are incorrect because they introduce assumptions that would
disprove the argument. Choice c is incorrect because it states a fact
listed in the argument.
18. d. Choice d is correct because whether survey respondents felt
superficial in admitting that their physical appearance was
important to them does not explain why 100% admitted that it
was, yet 15% of respondents did not cite concern over their
physical appearance as a reason for quitting smoking. Choice a is
incorrect because it reflects the possible reason that respondents
saw a connection between their physical appearance and cigarette
smoking, and did not care enough about it to quit smoking. Choice
b is incorrect because it indicates the possible reason that
respondents saw a positive correlation between their physical
appearance and cigarette smoking. Choices c and e are incorrect
because they reflect the possible reason that respondents did not
see much, if any, connection between their physical appearance
and cigarette smoking.
501_02_25-120_501_master.qxd 4/29/13 12:30 PM Page 92
92
501 GMAT
®
Questions
19. c. This problem appears tricky, but it is actually pretty simple if
you remember not to make any assumptions about the listed
apartment traits and their respective values. The prompt tells you
that a higher rental price results from a higher floor, more
sunlight, and/or a greater number of bedrooms. What we do not
know is how various combinations compare to each other, so it is
important not to assume anything, such as a greater number of
bedrooms having a higher value than greater sunlight. What you
want to look for is the one answer choice that clearly violates the
following setup:
Cathy—6th floor
David—4th floor
Krystal—1st floor, $$$$ (highest rent)
Oscar—basement floor, $ (lowest rent)
Choice c is correct because it violates the setup. If Krystal
lives in a 1-bedroom, David’s apartment has the most sunlight, and
Cathy lives in a 3-bedroom, then how can Krystal pay the highest
rent? She cannot because her apartment would not meet any of the
conditions that could plausibly price her rent the highest. In choice
a, Krystal could pay the highest rent because no one is listed as
having an apartment with more bedrooms than hers. Choice b
appears tricky because Krystal has a smaller apartment than Cathy,
but Krystal’s apartment might receive far more sunlight than the
others. Choice d is incorrect because the conditions provided
support the fact that Oscar pays less rent than the other two
tenants listed in the answer choice. Choice e is incorrect because
even though Cathy’s top floor apartment gets the most sunlight,
the additional two bedrooms in Krystal’s apartment provide a
reason that she pays the highest rent.
20. e. Choice e is correct because a higher membership in weight loss
programs fails to explain why women who spent more money on
exercise clothes and went to the gym more often did not lose more
weight than other women. Choices a, b, c, and d are all reasons
why the women who spent more money on exercise clothes, and
went to the gym more often, did not lose more weight than those
who spent less money on their exercise clothes.
501_02_25-120_501_master.qxd 4/29/13 12:30 PM Page 93
93
501 GMAT
®
Questions
21. e. Choice e is correct because a larger percentage of patients
diagnosed with bacterial infections would not justify a decrease in
the percentage of patients prescribed antibiotics. Choices a, b, c,
and d are all reasons that would explain a drop in the percentage of
patients prescribed antibiotics.
22. a. Choice a is correct because digital products earn a higher
percentage of profits than the percentage they make up of sales.
Choice b is incorrect because books earn the greater percentage of
profits. Choice c is incorrect because the passage says nothing
about growth. Choice d is incorrect because the passage does not
say anything about the number of products in each group. Choice
e is incorrect because books earn the same percentage of profits as
digital products and products in other mediums together.
23. b. Choice b is correct because the cost of travel for high-level
employees would need to be less than the savings generated by
near-shoring in order to justify the practice. Choice a is incorrect
because the prompt does not say anything about businesses
wanting to convince current employees in cities with a higher cost
of living to relocate. Choice c is incorrect because the prompt does
not indicate that businesses would have to pay for any cost of client
travel. Furthermore, even if the clients traveled out of the cities
with a higher cost of living, that would actually further justify
moving employees out of those locations, including higher-level
employees who deal with clients. Choice d is incorrect because
although it provides some support for near-shoring, there is not
enough information about financial incentives to provide a
comparison to the savings generated by near-shoring. Choice e is
incorrect because the clients staying put does not provide an
incentive for near-shoring.
501_02_25-120_501_master.qxd 4/29/13 12:30 PM Page 94
94
501 GMAT
®
Questions
24. c. This question is asking for the MOST important item to
ascertain, so be careful with answer choices that list items that are
relevant but are not the most important. Choice c is correct
because water has to be available to the drought victims in order to
give them relief. Choice a is incorrect because the exact number of
relief organizations involved is irrelevant. Choices b, d, and e all
list items that are important to know but are not the most
important, because without water, the rest of these items will have
no impact on the drought relief plan.
25. b. This is a formal logic argument, so you should try to sketch it
out and make connections between the people. Courtney is the
most commonly mentioned name, so try to see where everyone
falls in relation to her. Courtney has less money than Marnie,
Alexandra, and Josie. Alexandra has less money than
Danielle. Therefore:
C . . . M/A/J
C . . . A . . . D
Choice b is correct because Danielle cannot have less money
than Courtney, since Alexandra has more money than Courtney,
and Alexandra has less money than Danielle. The other choices are
all possible.
26. c. Choice c is correct because Sophia pointed out that Taylor failed
to consider that registration is a benefit after the purchase of
securities, not before. Choice a is incorrect because Sophia does
not provide evidence that people actually read the securities
paperwork. Choice b is incorrect because Taylor’s argument is not
about the volume of paper that registration generates, and Sophia’s
response merely indicates that buyers of securities will have a
remedy against the parties who misrepresented them. Choice d is
incorrect because Taylor’s argument is about the volume of
information that registration generates, not about the financial
literacy of people who buy securities. Choice e is incorrect because
Sophia does not suggest that all securities be registered; she only
states the advantages of registration for those securities that
already must be registered.
501_02_25-120_501_master.qxd 4/29/13 12:30 PM Page 95
95
501 GMAT
®
Questions
27. b. Look for the choice that is required for the argument to be true.
Choice b is correct because both options together cannot result in
greater fuel efficiency than only the use of a lighter metal in the car
frames. Choice a is incorrect because ABC’s options for a lighter
metal would not change the argument that the use of a lighter
metal in the frames would be the most that ABC could do to
increase fuel efficiency. Choice c is incorrect because the cars’ fuel
efficiency can still be increased even if it already is good. Choice d
is incorrect because what ABC wants to do and the most that ABC
can do are two different things. Choice e is incorrect because cost
is not relevant to ABC’s best effort to increase fuel efficiency.
28. e. Choice e is correct because a lower market demand and higher
distribution in the overall industry would be reasons for Crunchy
Organics’ profits to decline. Choice a is incorrect because
decreased wages would enable the company to earn a greater
profit. Choice b is incorrect because lower utility bills and a tax
refund would result in a greater profit. Choice c is incorrect
because a larger olive crop means a greater supply of olives, and
rainfall does not cost the company money. Choice d is incorrect
because a feature about the company as an ethical business could
reasonably result in more consumers buying olive oil specifically
from this company.
29. c. Choice c is correct because women like to see how a product
will look on them, which is a realistic image. Choice a is incorrect
because the affluence of women in cosmetics advertisements is not
relevant to the passage. Choice b is incorrect because the passage
provides no information that most women actually shop for
daytime cosmetics rather than makeup more appropriate for
evening. Choice d is incorrect because glamorous is not the same
as realistic. Choice e is incorrect because the passage does not
indicate that women are dissatisfied with the quality of cosmetics
featured in advertisements.
501_02_25-120_501_master.qxd 4/29/13 12:30 PM Page 96
96
501 GMAT
®
Questions
30. e. Choice e is correct because the passage does not explicitly say
that the installation of the machines in town that give a 5-cent
refund per item will motivate people to recycle. Furthermore, the
passage does not indicate whether any resulting increase in
recycling would be enough to save those jobs. Choice a is incorrect
because the township’s trash plant is irrelevant and the 5-cent
refund will not pay for any costs because residents will pocket it.
Choice b is incorrect because the passage does not question the
recycling plant’s capacity for additional materials. Choices c and d
are incorrect because the 5-cent refund would go to residents, not
for the cost of the machines.
31. d. Choice d is correct because the public health expert provides
information about population size that negates the reporter’s claim.
Choice a is incorrect because the issue is the number of cases of
the virus in relation to the size of the entire population. Choice b
is incorrect because the public health expert does not single out
particular people who are infected with the Q Virus. Choice c is
incorrect because the argument never alleges an incremental
increase, only an increase. Choice e is incorrect because the expert
does not question information about the people, only about the
group as a whole and its relation to the entire population.
32. a. Choice a is correct because the argument draws a conclusion
based on the percentage of students enrolled in each college.
Choices b and d are incorrect because the argument does not say
anything about the qualities of computer information systems
courses. Choice c is incorrect because the argument does not say
anything about course requirements. Choice e is incorrect because
the job market is not relevant to the argument.
33. c. Choice c is correct because people obtaining news from sources
that do not have high ratings would undermine the argument’s
conclusion. Choice a is incorrect because the conclusion does not
rely on what the press cares about. Choice b is incorrect because
the argument already made this statement. Choice d is incorrect
because what the press should do has no role in the argument.
Choice e is incorrect because whether the press has failed to obtain
high ratings with quality news is not an assumption necessary for
the conclusion that people will not obtain news elsewhere.
501_02_25-120_501_master.qxd 4/29/13 12:30 PM Page 97
97
501 GMAT
®
Questions
34. e. Choice e is correct because the city council is arguing that crime
did not increase. Do not let the answer’s use of percentages fool you.
The key is that the percentages did not change year to year. Choice a
is incorrect because this conclusion follows from the argument by the
community action group leader. Choice b is incorrect because the
passage says nothing about the laid-off police officers committing
petty crime. Choice c is incorrect because the city council president
is not stating causes for an increase in petty crime; she is stating that
no increase in petty crime has occurred. Choice d is incorrect
because the city council’s claim that only the number of reported
petty crimes has increased does not indicate that it is possible to
actually find out how many people were actually victims in a year.
35. a. Choice a is correct because a large market and low production
cost are both valid criteria for offsetting a lower price for generic
products. Choice b is incorrect because a large market and low
production cost for name brand products are not needed to offset
higher prices. Higher prices would naturally generate higher
profits, and therefore do not need to be offset at all. Choice c is
incorrect because the passage does not provide any information to
link similar profits with a similar market size. Choice d is incorrect
because the passage says nothing about either of the markets in the
future. Choice e is incorrect because the argument states that
companies that manufacture and sell name brand products are
often under this misconception about the difference in their
profits, which indicates that most companies are not savvy enough
to manufacture and sell both brand name and generic products.
36. b. Choice b is correct because the argument assumes a connection
between anticipating test questions and gearing study toward the
question types. Choice a is incorrect because the argument is
about how people study for standardized tests, not about whether
certain people excel on those tests due to question types. Choice c
is incorrect because the argument does not say anything about tests
covering the entire breadth of a subject. Choice d is incorrect
because the argument concludes that people have to study the
entire subject for tests. Choice e is incorrect because the argument
does not mention non-standardized tests.
501_02_25-120_501_master.qxd 4/29/13 12:30 PM Page 98
98
501 GMAT
®
Questions
37. d. Choice d is correct because telephone correspondence requires
that clients are at work at the same time as employees at the
companies that serve clients. Although employees could schedule
telephone calls during flex time, this answer provides a
circumstance in which flex time is not beneficial, even if it is not
detrimental here. Choice a is incorrect because it shows that e-
mail correspondence occurs at all hours, which could actually make
flex time beneficial. Choice b is incorrect because it shows that flex
time can make a particular group of employees more productive.
Choices c and e are incorrect because they show negative results of
employees working regular hours, which flex time can fix.
Set 3
38. c. The argument establishes a correlation between regular exercise
and longevity, and states that vegetarianism is not the cause of
greater longevity. Choice c is correct because it strengthens the
argument that there is no causal connection between vegetarianism
and longevity by showing that both meat eaters and vegetarians
have the same longevity when they exercise regularly. Choice a is
incorrect because it states an association between vegetarianism
and the likelihood of exercising regularly, which would weaken the
argument. Choice b is incorrect because it does not negate the
causal connection between vegetarianism and longevity. Choice d
is incorrect because it promotes an association between vegetarian-
ism and other healthy habits that fails to weaken a causal
connection between vegetarianism and exercising regularly.
Choice e is incorrect because it merely states a correlation between
meat eating and unhealthy habits.
501_02_25-120_501_master.qxd 4/29/13 12:30 PM Page 99
99
501 GMAT
®
Questions
39. a. Choice a is correct because if most people in Springfield are
unwilling to switch from their cars to public transit, the upgrade
will not significantly reduce the number of drivers on the streets.
Choice b is incorrect because the speed of the metro trains has no
impact on the argument. Choice c is incorrect because the current
limited reach of the metro supports the argument that increasing
its area of coverage would entice drivers to use public transit
instead. Choice d is incorrect because although an increase in air
pollution may counteract additional reasons that the
environmental group may have to support the proposed upgrade
for the metro, air pollution does not have a relationship in this
argument with getting drivers to switch to public transit. Choice e
hints that people do not desire to ride public transit, but does not
weaken any argument that people are unwilling to switch.
40. e. The question is asking for the choice that bests supports the
president of fundraising’s theory that donors are giving less
because they are less aware of the work that the organization does.
Choice e is correct because smaller print, if used in the mailings,
could cause the donors to be unable to read about the
organization’s work, and therefore feel less inclined to donate.
Choice a is incorrect because the fundraising president stated that
the economic downturn did not have a relationship to donation
activity. Choice b is tricky, but it is incorrect because although the
prepaid postage on the donation envelopes has to do with the
mailings, it does not inform the donors about the organization’s
work. Choices c and d are incorrect because they provide different
reasons for the decrease in donations.
501_02_25-120_501_master.qxd 4/29/13 12:30 PM Page 100
100
501 GMAT
®
Questions
41. c. Choice c is correct because it shows that the majority of people
who brush their teeth used this brand of toothbrush. Choice a is
incorrect because it only establishes that no other brands used the
same type of plastic for the bristles, not that a significant number
of people used this brand of toothbrush, which would be necessary
to show the brand’s effect on the tooth enamel of the population.
Choice b is incorrect because it does not address the brand of
toothbrush under discussion, and whether people brushed twice a
day is outside of the scope of the dentist’s theory. Choice d is
incorrect because the majority of Dr. Roberts’ patients do not
provide a basis for making a statement about the general
population. Choice e is incorrect because the toothbrush
company’s promotional activities did not necessarily result in a
significant portion of the population using those toothbrushes.
42. b. Choice b is correct because the author makes a faulty
assumption that people between the ages of 40 and 50 want a
particular style of vacation, when the argument does not provide
any information to that effect. Choice a is a tricky one, but it is
incorrect because the author does not assume that people between
the ages of 40 and 50 have children, but that they want to travel
with their families as opposed to other people. Choice c is
incorrect because the author does not assume that people between
the ages of 40 and 50 have a family that includes a spouse, but
that they want to travel with their family. Choice d is incorrect
because the argument tells us that people between ages 40 and 50
purchase the most vacations to Mexico. We do not need to know
information about other age groups. Choice e is incorrect because
whether the majority of people in a particular age group can afford
a vacation in Mexico is outside of the scope of the argument.
501_02_25-120_501_master.qxd 4/29/13 12:30 PM Page 101
101
501 GMAT
®
Questions
43. d. Choice d is correct because it provides a reason that more
smokers chose to quit this year. A higher cost for the following
year would support the conclusion that even more smokers will be
motivated to quit the following year. Choice a is incorrect because
the previous year having the lowest number does not tell us
whether the following year will have a higher number than the
current year. Choice b is incorrect because the answer does not tell
you whether an increase in government regulation has actually
occurred or will occur in the future. Choice c is incorrect because
it merely provides a possible reason for the increase in the number
of smokers who quit this year. It does not provide any information
about whether that grant will be available for the following year.
Choice e is incorrect because it does not provide any information
about the following year.
44. b. Choice b is correct because fewer larger boxes of paper will
generate lower tariffs and might wind up being less expensive than
domestic paper production. Choice a is incorrect because job
creation is not relevant to cost reduction here. Choices c and d are
incorrect because they actually would strengthen the argument.
Choice e is incorrect because the amount of paper produced
overall does not necessarily have relevance here as a foreign paper
producer likely produces and sells paper to several countries.
45. c. Choice c is correct because the human resources report fails to
recognize that the qualitative benefits it lists generally do not
pertain to store employees. Choice a is incorrect because there is
no information in the prompt that indicates that company
employees do not care about the qualitative factors listed. Choice b
is incorrect because whether pension and comp time for salaried
employees relates to employee compensation does not matter; the
point is whether these items actually help retain store employees.
Choice d is incorrect because the argument states that the
company has done something to retain employees. Choice e is
incorrect because additional qualitative factors that may have
retained employees are irrelevant to the flaw in this argument.
501_02_25-120_501_master.qxd 4/29/13 12:30 PM Page 102
102
501 GMAT
®
Questions
46. c. Choice c is correct because it explains why a greater percentage
of clothing in smaller sizes purchased in 2010 does not show that
the average size of adult women has changed in 10 years. Choices a
and d are incorrect because variations in the amount of clothing
purchased or in the number of adult women who purchased
clothing does not explain the discrepancy between percentages of
various sizes purchased and a lack of change in the average adult
woman’s clothing size. Choice b is incorrect because it supports
the opposite conclusion than the one posited. Vanity sizing would
have to be more prevalent in 2010, not 2000, to explain this
conclusion. This is the type of answer choice designed to trip you
up because you may have assumed that the correct answer would
be vanity sizing before you looked at the answer choices. Choice e
is incorrect because a change in the majority of people who
designed clothes merely hints at the vanity sizing issue mentioned
in choice b, but does not actually indicate any change in the way
the designers designed clothes. Remember that you are looking for
the best answer.
47. e. Choice e is correct because if most law students who have jobs
upon graduation were not hired by law firms, then the average
annual salary of $70,000 provides no information about their
salaries. Choice a is incorrect because the argument provides no
information about salary in relation to the size of the law firm.
Choice b is incorrect because geographical region is outside of the
scope of the argument. Choice c is incorrect because the argument
makes a statement about the percentage of law students, not the
total number. Choice d is incorrect because the argument does
not draw a conclusion about law students who did not pass the
bar exam.
501_02_25-120_501_master.qxd 4/29/13 12:30 PM Page 103
103
501 GMAT
®
Questions
48. d. Choice d is correct because multiple purchases by each person
who attempted to quit smoking would negate the association
between the number of the smoking cessation products purchased
and the number of people who quit smoking. Choice a is incorrect
because the fact that people reported that they had more success
with the gum may imply that they might have failed with the other
products, which would cast some doubt on the argument, but this
statement is not as conclusive as choice d. Choice b is incorrect
because Company XYZ may be one of many companies that
manufacture and sell smoking cessation products. Choice c is a
tricky one, but be careful. Someone who quit smoking and then
relapsed still quit smoking, albeit temporarily, so choice c is
incorrect. Choice e is incorrect because doctor recommendations
are irrelevant in this argument.
49. d. Choice d is correct because the breakdown of student
attendance at public versus private colleges is irrelevant; the actual
total in overall tuition is what matters. Choice a is incorrect
because theoretically a majority of students could have
scholarships, but they could be for such low amounts that they
barely offset the cost of tuition. Choice b is incorrect because even
if scholarships offer a large discount on college tuition, a low
percentage of students who have them could result in scholarships
having a minimal effect on the expense of tuition. Choice c is
incorrect because the conclusion is about college, not college tuition
specifically. Therefore, room and board is a relevant cost of
college. Choice e is incorrect because the number of students who
attend college affects overall college expenditures.
50. e. Choice e is correct because teenagers could only legally drive
during daylight hours and the most drunk driving accidents
occurred during late hours. Choices a and d are statements that
would support the state legislature’s belief that drivers under age
18 significantly contribute to or are less capable of avoiding drunk
driving accidents. Choices b and c are statements about teenagers
and alcohol that would support an inference that teenagers are
responsible for a percentage of drunk driving accidents.
501_02_25-120_501_master.qxd 4/29/13 12:30 PM Page 104
104
501 GMAT
®
Questions
51. a. Remember to read the question closely, because this one asks
you for the answer that would cast the most serious doubt on
Zippo’s argument, NOT the doctors’ argument. Choice a is correct
because it would cast doubt on the argument that an ingredient in
the cans caused the illness. Choices b, c, and e are incorrect
because they are about what the patients ate, which does not
undermine Zippo’s argument. Choice d is incorrect because
whether the patients drank regular or diet Zippo Cola does not
matter: they all got sick.
52. c. Choice c is correct because if most people who need to wear
glasses or contact lenses are unaware of that fact, then they
obviously will not notify the department of transportation when
they renew their driver’s licenses, and the plan will not reduce
accidents. Choices a and b are incorrect because they are
assumptions of the argument, not factors that the argument failed
to consider. Choice d is incorrect because the prompt does not
mention a vision test. Choice e is incorrect the prompt does not
mention anything about out-of-state drivers in relation to car
accidents.
53. a. Choice a is correct because it provides a reason why increased
athletic activity among students did not result in any weight loss
among the student body. Choice b is incorrect because it provides
a reason why the program should have succeeded. Choice c is
incorrect because there is no information regarding what
percentage of students who participated in the intramural sports
were also on official university athletic teams. Choice d is incorrect
because athletic ability does not reveal anything about student
weight before and after participation in the intramural sports.
Choice e is incorrect because students being somewhat active
before participating in intramural sports merely hints that the
increased physical activity was insufficient to result in weight loss.
501_02_25-120_501_master.qxd 4/29/13 12:30 PM Page 105
105
501 GMAT
®
Questions
54. d. Choice d is correct because Dr. Roberts’ conclusion is that the
rate of tooth enamel decay will drop if patients stop using this
brand of toothbrush. However, the problem is actually the type of
plastic in the bristle. If other brands used the same plastic in the
bristles, patients would still suffer the same rate of tooth enamel
decay. Choices a and e are incorrect because they merely assume a
connection between Dr. Roberts’ patients and dental patients in
general. Choices b and c are incorrect because these claims by Dr.
Roberts do not address the brand of the toothbrush in the
conclusion.
55. b. Choice b is correct because the television executive assumes that
since senior citizens are the largest demographic of the population
and spend more time at home due to retirement, they watch the
most television. If another age group watched the most television,
then the executive should gear programming toward that age
group. Choice a is incorrect because the mention of cost is not
directly tied to viewer popularity, which is the subject of the
argument. Choice c is incorrect because it merely states that fewer
senior citizens spend time at home than previously, not that they
necessarily watch less television than other groups. Choice d is
incorrect because it simply mentions one reality TV show popular
among seniors; theoretically, that group may not watch any other
reality programs. Choice e is incorrect because it only provides
reasons that hint that senior citizens do not spend that much time
watching television, and those reasons only apply to daytime.
56. e. Choice e is correct because database storage shows a feature of
the program that is efficient for human resources to use. Choices a
and d are incorrect because supervisors can allow personal likes or
dislikes to color their evaluations of employees. Choice b is
incorrect because employees who meet revenue goals may be
deficient in other job responsibilities. Choice c is incorrect because
differences in merit may vary more than at three levels.
501_02_25-120_501_master.qxd 4/29/13 12:30 PM Page 106
106
501 GMAT
®
Questions
57. a. Choice a is correct because if the people who are saving are
underemployed, their income is probably lower than it was
previously and an increased percentage of income saved could
actually result in less money to spend, which will not increase
economic activity. Choice b is incorrect because loans to start
businesses may generate more economic activity. Choice c is
incorrect because the public paying down its credit card debt
would bring more money to the banks, which would facilitate
increased economic activity. Choice d is incorrect because
spending only on necessary home improvements does not indicate
how much or little economic activity is generated by that activity.
Choice e is incorrect because bank fees are not relevant to the
conclusion.
58. e. Choice e is correct because if the majority of customers
purchase items without help from sales associates, then the
connection between increased worker productivity and sales rates
is broken. Choice a is incorrect because wages becoming a smaller
proportion of total costs does not weaken the argument. The other
costs could have grown faster than the cost of wages. Choice b is
incorrect because hiring is not relevant to the argument. Choice c
is incorrect because sales associates who work on commission
having higher paychecks supports the argument that they are more
productive. Choice d is incorrect because other industries are not
relevant.
59. d. Choice d is correct because the money that state residents pay in
their taxes is not actually a cost of tuition, and families pay tax even
if their children attend private colleges. Choice a is incorrect
because the college president states that the cost of tuition at a
private college equals the cost of tuition at a public college plus the
cost of taxes paid toward public colleges. Choice b is incorrect
because it only explains that everyone pays the taxes. Choice c is
incorrect because scholarships at private colleges could result in
private colleges costing less. Choice e is incorrect because the
state’s budget is not relevant in this argument to the cost that state
families pay toward tuition at a public college.
501_02_25-120_501_master.qxd 4/29/13 12:30 PM Page 107
107
501 GMAT
®
Questions
60. e. Take note that this question asks for which answer does NOT
weaken the argument, NOT which answer strengthens the
argument. Therefore, cross off each answer that weakens the
argument and choose the one that remains. Choice a is incorrect
because customers will shop at other stores for goods if price clubs
do not sell them. Choice b is incorrect because customers who
cannot transport or store bulk sizes of goods are unlikely to shop at
price clubs. Choice c is incorrect because it states that some
customers will not shop at price clubs despite the low prices.
Choice d is incorrect because it negates the claim that price club
prices, the reason that customers would presumably choose to shop
there over other stores, are not actually lower than those in other
stores. Choice e is correct because it does not address the price
comparison between the two stores.
61. d. Choice d is correct because it provides justification for the
medical group’s recommendation of one food item that contains
calcium over another in order to lower one’s chances of
osteoporosis. Choice a is incorrect because the argument does not
say that fat intake has any effect regarding osteoporosis. Choice b
is incorrect because the argument does not concern itself with
other health issues. Choice c is incorrect because the medical
group’s argument is not about the general daily diet, only about
calcium intake. Choice e is incorrect because the effect on the
environment is not relevant to the argument.
501_02_25-120_501_master.qxd 4/29/13 12:30 PM Page 108
108
501 GMAT
®
Questions
62. c. Choice c is correct because if the installation of new appliances
is less expensive than repairing the current ones, the reason for the
complex to employ a full-time super is weakened. Choice a is
incorrect because the time of day that tenants discover problems
with the appliances is not important; what is important is the cost
for the complex to have the appliances fixed. Choice b is incorrect
because the amount of water the appliances use tells us nothing
about whether the complex should continue to have the appliances
fixed. Although you might infer that more water used means a
higher cost, the answer does not provide any information to infer
that the cost from greater water use would negate the savings from
fixing the appliances. Choice d is incorrect because the answer
does not provide any information indicating that the cost of the
super’s benefits would negate the savings of having the super fix the
appliances. Choice e is incorrect because it does not compare the
expense of utility bills with the expense of paying the super. In
addition, tenants commonly pay utility bills, which would mean
that the utility bill savings suggested here are irrelevant to the
argument.
63. e. Choice e is correct because it provides a reason why law
students would not buy the e-book. If a large percentage of
students had open-book exams and took them on the computer
using software that blocked access to computer files during the
exam, these students would not have access to the e-book during
the exam; they would therefore have a reason to continue
purchasing hard copies. Choice a is incorrect because the age and
familiarity with e-books before the promotion would not explain
why the promotion did not succeed in making more students
prefer e-books after the promotion. Choice b is incorrect because
the weight of a hard copy textbook, if heavy, does not explain why
the promotion failed. Choice c is incorrect because regardless of
the percentage of courses covered by the promotion, some
decrease in hard copies purchased would be expected. Choice d is
incorrect because a comparison to undergraduate students is not
relevant to why a decline in the purchase of hard copies of law
textbooks did not occur as a result of the promotion.
501_02_25-120_501_master.qxd 4/29/13 12:30 PM Page 109
109
501 GMAT
®
Questions
64. a. Choice a is correct because the argument states reasons why
people plan not to consume soy and fewer people plan to consume
soy in place of meat and dairy; if soy is abundant in processed
foods that contain meat, people may inadvertently consume an
increased amount of soy. Choice b is incorrect because the
increasing number of people concerned that they might be allergic
to soy would presumably have already had an adverse reaction to
something they ate, believe that soy was the culprit, and plan on
avoiding soy in the future. Choice c is incorrect because suspicion
of a thyroid problem, even if not well founded, is enough to
convince people to avoid soy when they believe that soy will
exacerbate the problem. Choice d is incorrect because it would
actually strengthen the argument. The passage states that soybean
farming causes environmental problems. Choice e is incorrect
because although a price difference is a reason that people would
consume more soy, price difference fails to negate one of the
reasons the passage listed for less future consumption of soy.
Set 4
65. c. Choice c is correct because the advantage of the trains was that
they do not spend much time in traffic. A competitor significantly
increasing its use of the same tracks could result in train traffic
problems. Choice a is incorrect because it merely presents an
advantage of the bus option, not a disadvantage of the train option.
The answer also does not indicate whether the construction of two
new highways will necessarily alleviate traffic on the road. Choice
b is incorrect because the passage does not say how long trains take
to board in the first place, and the amount of boarding time is
unlikely to cause a problem with a train system that runs at 15-
minute intervals. Choice d is incorrect because the low use of the
trains for trips in small demand is not a factor in Suburban
Commute’s capacity needs for peak trips. Choice e is incorrect
because it merely presents an advantage of the ferry option.
501_02_25-120_501_master.qxd 4/29/13 12:30 PM Page 110
110
501 GMAT
®
Questions
66. e. Choice e is correct because the fishing industry’s desire to
decrease fraudulent red snapper sales could help the Consumer
Watchdog Agency. Choice a is incorrect because if grocery store
inventory managers are placing other fish in the red snapper
section of the fresh fish counter, the fish is being sold unpackaged
and customers will not see the stamp. Choice b is incorrect
because restaurants serve prepared, and not packaged, food, so
customers would not see the stamp. Choice c is incorrect because
customers failing to discern red snapper from other fish will not
help solve the fraud problem. Choice d is incorrect because if most
red snapper is sold without packaging, the stamp verification
would not appear to be helpful.
67. e. This is an EXCEPT question, so note that you are looking for
the answer choice that does NOT explain why the plan failed.
Choice e is correct because the drivers finding the 10% bonus to
be an incentive would not explain why they failed to deliver 90%
of their packages on time. Choice a is incorrect because traffic
problems are a plausible reason that delays were beyond the
drivers’ control. Choice b is incorrect because faulty GPS units
could result in drivers arriving at the wrong locations and
therefore delivering packages late. Choice c is incorrect because if
fuel was expensive enough, the cost of driving fast could outweigh
the benefit of the 10% bonus. Choice d is incorrect because if
drivers could earn more money at another company and therefore
the majority left Potter Shipping after six months, the drivers
would not remain for a year to collect the bonus, thereby
eliminating the incentive.
68. e. Choice e is correct because Trikas wants to decrease air
pollution in order to placate public opinion, and a significant
increase in water pollution is likely the negative effect that their air
pollution problem has on public opinion. Choice a is incorrect
because cost is irrelevant to the immediate goals of Trikas listed in
the prompt. Choice b is incorrect because the system may still
significantly reduce air pollution even if it malfunctions 10% of the
time. Choice c is incorrect because the prompt does not provide a
deadline for Trikas to meet its goals. Choice d is incorrect because
the manpower required to maintain the system is irrelevant to the
immediate goals of Trikas.
501_02_25-120_501_master.qxd 4/29/13 12:30 PM Page 111
111
501 GMAT
®
Questions
69. c. The question asks you to identify which answer choice is
LEAST important in determining whether the government’s new
coverage of preventative care medical services will increase senior
health. Choice c is correct because since this program is free to
seniors, the cost of the services does not matter. Choice a is
incorrect because if the percentage of seniors who seek
preventative care is very low, then the program would not improve
senior health. Choice b is incorrect because the program would
not improve senior health if the preventative services detected
health conditions but early detection provided no advantage in
treating those conditions. Choice d is incorrect because if a great
percentage of seniors do not have access to preventative care, the
program will not improve senior health. Choice e is incorrect
because if only a low percentage of seniors use the government’s
program, then the program will not significantly impact the health
of the senior population.
70. d. The question is asking you to identify the answer choice that
would be helpful in determining whether this plan would provide
an incentive to invest in start-up businesses. Choice d is correct
because if the average start-up business did not turn a profit for the
first five years, this tax break would not be advantageous for the
average investor in a start-up business. Choice a is incorrect
because the amount of capital needed does not tell us anything
about the tax break. Choice b is incorrect because the average tax
bracket of an individual investor has no impact on this proposed
tax break. Choice c is incorrect because the type of business is
irrelevant to the tax break. Choice e is incorrect because the
number of Grow the Economy Now party candidates who would
benefit from this tax break has no connection to whether the tax
break would serve as an incentive to investors in general.
501_02_25-120_501_master.qxd 4/29/13 12:30 PM Page 112
112
501 GMAT
®
Questions
71. a. The question asks you to identify which answer choice would
NOT be relevant in determining whether the free shipping offer
would result in an increase in revenue. Choice a is correct because
the price of packing materials would not matter because the
prompt tells you the expected loss in profits. Only changes in this
price would matter, and the answer choice does not mention
changes. Choice b is relevant because competitors providing the
same offer would not make shopping at The Lightening Tread
Shoe Company particularly advantageous for customers. Choice c
is relevant because if the average price of an order dropped by over
$4.00, the company could lose money. Choice d is relevant because
higher postal rates could mean a loss in profits greater than $4.00
in the future. Choice e is relevant because customers splitting their
purchases into several smaller orders could result in the company
losing more money without gaining in merchandise purchases.
72. b. The prompt does not explain whether the majority of the 35%
work in buildings with more than four stories. Therefore, the
additional plan should include those buildings. Choice b is correct
because it includes the smaller commercial buildings AND it
addresses the possibility that the private safety company might be
doing an inadequate job in the buildings it currently covers.
Choices a and c are incorrect because they do not address the
possibility that the private safety company’s job performance is
inadequate. Choice d is incorrect because it does not address the
possibility that commercial buildings smaller than four floors
might be the source of the problem. Choice e is incorrect because
the effectiveness of fines is unclear, especially as the answer does
not provide a monetary amount for the fine. For example, a $1 fine
is unlikely to provide an incentive.
501_02_25-120_501_master.qxd 4/29/13 12:30 PM Page 113
113
501 GMAT
®
Questions
73. d. Choice d is correct because it explains that citizens in the
repayment plan wound up not paying off any debt because they
were charging at the same rate. Choice a is incorrect because the
current repayment plan could have enabled citizens to reduce their
credit card debt no matter how many similar programs the
government had offered in the past. Choice b is incorrect because
the inconvenience of paying bills in person does not explain why
people who participated in the program did not reduce their credit
card debt. Choice c is incorrect because the need for this program
does not explain why it did not succeed. Choice e is incorrect
because the program could still be expected to reduce debt even
though people were living in poverty.
74. b. The question is asking you to find a valid reason for the airline
industry’s decision to triple its flights. Choice b is correct because
an increase in business attendance at trade shows around the
nation would be a reason to expect an increase in travel. Choice a
is incorrect because even if the percentage of retirees has
increased, they will not necessarily travel more. Choices c and d
are incorrect because they only state that an area of operating costs
has decreased, not that there is a greater demand for air travel.
Choice e is incorrect because there is not necessarily a correlation
between a lower percentage of people who fear flying and a greater
demand for air travel.
75. d. The question asks you to identify the answer choice that would
not explain why the new 3-step process failed. Choice d is correct.
Note that the prompt states that the 3-step process checked for
“scratches and other visible defects.” If the majority of the defects
were visible, then the process would have caused employees to
discover them. Choice a is a plausible explanation because it
explains that the mailer envelopes are scratching the discs when
the warehouse employees slide the discs into them, which would
occur after the employees inspected the discs. Choices b and e
would clearly explain why the process failed. Choice c would
explain the failure because the damage to the discs would have
occurred after the company mailed them out.
501_02_25-120_501_master.qxd 4/29/13 12:30 PM Page 114
114
501 GMAT
®
Questions
76. c. Choice c is correct because the strategy would increase the
customer pool to include customers who are less likely to file
claims for earthquake damage. Choice a is incorrect because since
earthquakes are on the rise, customers with buildings that have not
experienced earthquake damage in the past 20 years may be more
likely to experience it in the future. Choice b is incorrect because
customers who cannot obtain earthquake coverage through
another provider probably cannot do so because their likelihood of
experiencing earthquake damage in the future is high. Choice d is
incorrect because people who can afford earthquake coverage will
pay for it; the problem will arise when too many claims are filed in
the future and the company cannot afford to pay all of them.
Choice e is incorrect because insuring other disasters is irrelevant
to the problem of maintaining coverage for earthquakes.
77. e. Choice e is correct because if people are buying higher quality
clothing, they are more likely to want to preserve the dye in them.
Choice a is incorrect because although the same cost is not an
argument against the switch, it does not show that the switch will
have an additional advantage. Choice b is incorrect because
moving manufacturing overseas does not provide an additional
justification regarding the switch to plant-based ingredients.
Choice c is incorrect because a decrease in consumer spending for
laundry detergent overall indicates a declining consumer demand
for the product. Choice d is incorrect because stain remover pens
are not in the scope of the prompt’s discussion of the company’s
plan.
78. e. Choice e is correct because whether the plan will increase
revenue depends on whether the ads and the ice cream’s enhanced
colors entice people to buy the product. Choice a is incorrect
because natural ingredients are irrelevant to the marketing
department’s plan. Choices b and d are incorrect because the
marketing department’s plan does not have anything to do with the
product’s soy base. Choice c is incorrect because the prompt does
not reveal whether the food coloring uses artificial ingredients.
Assuming that it does, and even if people do not mind artificial
ingredients, that fact will not motivate them to buy the product.
501_02_25-120_501_master.qxd 4/29/13 12:30 PM Page 115
115
501 GMAT
®
Questions
79. c. Choice c is correct because the university’s suggested plan would
require private enterprise to have a motive to conduct research in
university laboratories and to offer tuition assistance to students.
Choice a is incorrect because the size of the applicant pool is not
the most relevant consideration, as private enterprise would have
to first participate in the program. Choice b is incorrect because
the suggested plan does not indicate that students have to possess
great achievement levels in math and science in order to
participate. Choice d is incorrect because the universities and the
government are concerned about achievement in math and science,
not necessarily in the number of majors. Choice e is incorrect
because the type of research is not particularly relevant to whether
this suggested plan would succeed.
80. d. Choice d is correct because an increase in the price of batteries
could negate the savings generated by using the new drills. Choice
a is incorrect because the number of drills that Construction
Company Y would need to buy does not matter. Choice b is
incorrect because an increase in the demand for construction work
would bring Construction Company Y more business. Choice c is
incorrect because other companies releasing a similar product at
the same price would not negate the construction company’s
savings. Choice e is incorrect because a shortage of industrial-sized
electrical cords would not matter since the new drills are battery
operated.
81. e. Choice e is correct because the cross-training program would be
disadvantageous if it resulted in slower times. Choice a is incorrect
because it is an advantage of the program. Choice b is incorrect
because what the team’s competitors have not considered does not
mean the program is not advantageous for this team. Choice c is
incorrect because all that matters is the advantages of the runners
for this team. Choice d is incorrect because the price of the athletic
gear does not take away from the goal of the program.
501_02_25-120_501_master.qxd 4/29/13 12:30 PM Page 116
116
501 GMAT
®
Questions
82. b. Choice b is correct because the prompt stated that the plan only
addresses students who are covered by the student health plan.
Furthermore, the prompt stated that students would be required to
have the vaccine before moving into the dormitories, which would
mean that students living off campus would not be affected by the
plan. Therefore, choice a is incorrect. Choice c is incorrect
because it leaves out the variable of students covered by the
student health plan. Choice d is incorrect because it does not
address the issue that the vaccination plan only applies to students
on the student health plan. Choice e is incorrect because it fails to
address the issues of students on the student health plan and
students who live in the dormitories.
83. c. Choice c is correct because if the wet season has not yet begun,
the president of the organization does not yet have confirmation
that the mosquito nets have actually reduced cases of malaria.
Choice a is incorrect because a small need for quinine for other
purposes does not negate cutting orders in half when the large
public health threat that requires quinine is perceived to have
abated. Choice b is incorrect because the perceived success of the
anti-malarial mosquito nets would presumably have an effect on
the 40% of the population that seeks treatment at the
organization’s clinics. Choice d is incorrect because it merely
suggests that the mosquito nets’ success might not be widespread,
but does not give any reason to affirmatively see a failure. Choice e
is incorrect because other clinics simply may not have known
about this organization’s donation of anti-malarial mosquito nets.
501_02_25-120_501_master.qxd 4/29/13 12:30 PM Page 117
117
501 GMAT
®
Questions
84. b. Choice b is correct because less time given to foreign language
study for younger students will offset the benefits that are
supposed to be gained by teaching students a foreign language at a
younger age. Choice a is incorrect because students who already
speak a foreign language at home do not necessarily study the same
foreign language at school. Choice c is incorrect because a choice
limited between two languages is not relevant to proficiency in a
foreign language. Choice d is incorrect because taking instruction
time away from social studies will not negatively affect proficiency
in a foreign language. Choice e is incorrect because the
superintendent’s plan does not necessarily indicate that the
elementary school teachers will not receive training or that they
must be the ones to teach a foreign language.
85. b. Choice b is correct because the plan requires that jurors will
actually find the visual aids interesting in order for the plan to
succeed. Choice a is incorrect because the plan does not require
that jurors find civil trials as interesting as criminal trials. Choice c
is incorrect because judges allowing the visual aids is a requirement
for the plan to work, not something that would actually show that
jurors find the aids interesting. Choice d is incorrect because the
litigators wanting to use the aids will not make the jurors find them
interesting. Choice e is incorrect because what criminal litigators
do is irrelevant to the plan.
86. a. Choice a is correct because it explains how the members of the
ecological group will find the Burmese pythons. Choice b is
incorrect because increased fines for releasing animals does not
explain whether the ecological group’s capturing procedure will
succeed. Choice c is incorrect because other non-native snakes are
not part of the group’s objective. Choice d is incorrect because the
passage already states that the procedure for capture is safe, so
whether the Burmese python is venomous is not an issue. Choice e
is incorrect because the plan is only about this ecological group,
not others.
501_02_25-120_501_master.qxd 4/29/13 12:30 PM Page 118
118
501 GMAT
®
Questions
87. d. Note that the state’s speed limit plan assumes that a speed of 70
miles per hour is safe. Choice d is correct because the number of
drivers who were pulled over under the old speed limit would have
to decrease under the new speed limit in order for the plan to
work. Choice a is incorrect because the number of drivers who will
continue to drive over 70 miles per hour has no bearing on the
state’s change in the speed limit; what matters is the number of
drivers who were driving 70 miles per hour or less. Furthermore,
the state’s plan is about drivers pulled over for driving at that
speed, not drivers who drove that fast in general. Choice b is
incorrect because the state’s plan has nothing to do with other
reasons drivers were pulled over for besides speeding. Choice c is
incorrect because what the voters think about the speed limit is not
relevant. Choice e is incorrect because the passage does not
provide enough information to make any associations between the
density of traffic during rush hour and the speed limit.
88. e. Choice e is correct because each retailer would want to earn the
$4,000 back in sales revenue for the fee to be worth paying. Choice
a is incorrect because the percentage of mall customers who drive
does not necessarily indicate a correlation between increased
revenue and increased parking. Choice b is incorrect because the
purpose of the parking plan for the retailers is to increase revenue,
which could occur even if a parking shortage continued after the
two new levels in the garage were built. Choice c is incorrect
because the number of complaints about the parking shortage only
indicates the problem, not how severe the problem may be. Many
customers who were unable to find parking may not have bothered
to complain to the mall’s customer service department. Choice d is
incorrect because the cost to the mall does not affect the retailers’
decision to pay $4,000 apiece.
501_02_25-120_501_master.qxd 4/29/13 12:30 PM Page 119
119
501 GMAT
®
Questions
89. b. Choice b is correct because the municipality’s plan assumes that
a large cause of car accidents during the winter is snow. Choice a is
incorrect because whether applicants will drive the snowplows for
the offered wage does not address the plan’s assumption that snow
is the primary cause of the car accidents. Choice c is incorrect
because the plan only addresses car accidents that occur during the
winter season. Choice d is incorrect because the number of
snowplows does not address the plan’s assumption that snow is the
major cause of these accidents. Choice e is incorrect because snow
tires are out of the plan’s scope.
90. e. Choice e is correct because the company’s advertising plan is
premised on the assumption that the savings generated will be
sufficient to attract customers to this line of appliances. Choice a is
incorrect because the advertising campaigns of competitors are not
relevant. Choice b is incorrect because it assumes that customers
will only buy these appliances if the savings will actually cancel out
the cost of the appliances. Choice c is incorrect because the price
of appliances in basic colors does not matter regarding this
advertising campaign. Choice d is incorrect because customers
would presumably still see savings in buying a matching set of
these appliances.
501_02_25-120_501_master.qxd 4/29/13 12:30 PM Page 120
120
501 GMAT
®
Questions
91. b. Choice b is correct because the passage states that the benefit to
students of the 5-year programs is job experience that employers
find valuable. In a recession, this reason is even more compelling.
Choice a is incorrect because it says that most students find their
current loans to be affordable, but provides no indication that the
expense of a fifth year of tuition would still be affordable. Choice c
is incorrect because other time opportunities for internship
experience are a reason that students would less likely find 5-year
programs attractive. Choice d is incorrect because it merely
provides an additional reason that students would be attracted to
5-year programs. It does not address job experience. Choice e is
incorrect because the fact that half of undergraduate institutions
plan on offering these programs does not indicate that students
would necessarily enroll in all the available slots in these programs,
which if true, could support an inference that students not in these
programs would be at a competitive disadvantage. Even with this
inference, this statement does not address the value of job
experience and its relationship to the cost of the fifth year of
tuition.
92. d. Choice d is correct because age provides a mechanism for
raising premiums, and since older pets are likely to need more
medical care, the premiums are more likely to cover the cost of
submitted claims. Choice a is incorrect because capping the price
of pet insurance at the cost of an annual checkup and shots simply
does not make sense for either customers or the insurance
company. If the insurance cost equals or is greater than the costs of
the checkup and shots, customers have no reason to buy it because
they might as well pay for those costs out of pocket. If the
insurance cost is less than the cost of the checkup and shots, the
company would lose money on the policies. Choice b is incorrect
because the plan does not indicate that participating veterinary
practices would somehow make offering the policies affordable.
Choice c is incorrect because people who are wealthy enough to
afford many veterinary services would still only buy insurance if
the price provided an advantage, and they would still submit
claims. Choice e is incorrect because the problem is that the
insurance company has to insure the pets after their medical
problems are revealed.
501_03_121-234_501_master.qxd 4/29/13 12:32 PM Page 121
3
Sentence Correction
The GMAT sentence correction questions are designed to test your
knowledge of correct written English, including both grammatical and sty-
listic components. All questions are given in multiple-choice format with
ve
possible
answers.
Each
question
presents
a
sentence,
of
which
all
or
part
has
been
underlined.
The
test
taker
is
then
presented
with
five
different
ver
-
sions
of
the
underlined
part
in
choices a through
e.
Choice
a will
always
be
the
same
version
as
the
underlined
portion
that
is
given
in
the
original
sen-
tence.
Choices
b
,
c,
d,
and e
will
be
variants
of
the
underlined
portion.
These
alternate
choices
test
a
variety
of
aspects,
ranging
from
grammar
and
sentence
structure
to
word
choice
and
redundancy.
The
correct
choice
must
do all the following:
utilize correct English grammar
have correct sentence structure
not contain any diction (word choice) mistakes
not change the original meaning of the sentence
The
questions
will
present
problems
based
on
two
different
categories:
cor-
rect expr
ession and
effective expr
ession.
Correct
expression problems
deal
with
grammar
mistakes,
misplaced
or
improperly
formed
modifiers,
unidiomatic
or
inconsistent
expressions,
and
501_03_121-234_501_master.qxd 4/29/13 12:32 PM Page 122
501 GMAT
®
Questions
faults in parallel construction. Effective expression problems deal with proper
diction and syntax, redundancy, brevity, and needlessly complicated
expressions. In this GMAT sentence correction practice, we have divided
the practice questions into the two categories of correct expression and
effective expression so test takers may easily pinpoint the types of mistakes
to later recognize them on test day. Test takers will encounter, however,
some slight blending of both categories in some of the questions. The real
GMAT sentence correction questions will combine both categories of
errors, so be sure to carefully review both sections. They complement each
other and your understanding of one section will lead to a greater under
-
standing of the other
.
Remember
that
one
question
may
contain
multiple
errors,
so
it
is
impor-
tant
to
read
all
the
choices
before
selecting
your
answer.
Use
the
process
of
elimination
whenever
possible.
If
you
get
stuck
on
one
question,
look
for
subtle
differences
between
the
possible
choices.
Insert
the
selected
part
into
the
sentence
and
read
it
quietly
to
yourself.
Sometimes
an
error
will
sound
more obvious to the ear then it will appear on paper
.
Review
the
material
provided
before
you
begin
to
tackle
the
questions.
It
has
been
carefully
crafted
to
help
you
succeed
on
this
practice
and
the
real
test.
Good luck!
Correct Expression
The following material will help you prepare for the part of the GMAT sen-
tence correction section that focuses on noun-verb agreement, pronouns,
verb tense sequence, correctly formed and placed modifiers, idiomatic
expressions, and faults in parallel construction.
Parts of Speech
In order to correctly structure a sentence, it is important to understand its
basic
components.
Read
the
following
parts
of
speech
and
make
sure
you
understand their roles and limitations.
Noun: person,
place,
or
thing;
the
name
of
anything
dog,
Disneyland,
Mary, Mar
y’s dog, etc.
Verb: action
run, eat, sleep, etc.
1 2 2
501_03_121-234_501_master.qxd 4/29/13 12:32 PM Page 123
501 GMAT
®
Questions
Adjective: word used to describe a noun pretty, tall, incredible, surprising,
good, etc.
Adverb: word used to describe verb, adjective, other adverb, phrases; it
describes how something is done quickly, carefully, patiently, well, very, etc.
Preposition: word that shows relationship between noun/pronoun and
other words in a sentence on, at, through, between, with, etc.
Parts of a Sentence
Subject: It tells the reader what is being talked about. The main word of a
subject is always a noun or pronoun.
Predicate: It tells the reader what
is being said about the subject.
Object: It
tells
the
reader
who
or
what
the
recipient
of
the
action
(verb)
is.
The teacher praised the student.
Subject:
teacher
Predicate:
praised the student
Object:
student
Subject/Verb
Agreement
A
complete
sentence
must
have
a
subject
and
verb.
If
it
is
missing
one
of
these
components, it is considered a fragment and is incorrect. In addition
to ensuring that a sentence has both subject and verb, you must also check
that the subject and verb agree with each other .
*A
singular
subject
requires
a
singular
verb
form,
and
a
plural
sub-
ject requires a plural verb form.
X The
development
of
new
hybrid
cars
have been
a
very
important
step
toward
less-
ening the countr
y’s dependence on oil.
The
previous
sentence
is
incorrect
because
have is
used
in
the
plural
form
to
modify
new
hybrid
cars.
That
is
not
the
subject,
though.
The
complete
1 2 3
501_03_121-234_501_master.qxd 4/29/13 12:32 PM Page 124
501 GMAT
®
Questions
subject is the development of new hybrid cars. Therefore, the subject is sin-
gular. The correct sentence reads:
The development of new hybrid cars has been a very important step toward less-
ening the countr y’s dependence on oil.
You will notice that the sentence correction questions try to hide the sub-
ject. Remember to capture the entire subject! It may be a phrase rather than
a
word, and there may be modifiers after the subject that try to confuse you!
*Beware of phrases that attempt to hide the subject.
X The planes on the tarmac at Los Angeles International Airport comes and leaves
at all hours of the night.
The previous sentence is incorrect because the verbs comes and leaves mod-
ify a singular subject. The real subject, however, is planes. Try to ignore the
prepositional phrases that can follow the subject (on the tarmac, at Los Ange-
les International Airport) when determining the actual subject. The correct
sentence reads:
The planes on the tarmac at Los Angeles International Airport come and
leave
at all hours of the night.
*Beware
of
the
difference
between
compound
subjects
and
additive
phrases.
X Dr
. Jones and his staff goes out for a Christmas dinner ever y December .
X Dr. Jones, accompanied by his staff, go out for a Christmas dinner ever y
December.
Both examples are incorrect. The first sentence features a compound sub-
ject (Dr. Jones and his staff). The compound subject is plural and therefore
requires
a
plural
verb
formation.
The
second
sentence,
however,
adds
his
staff with
the
phrase
accompanied
by.
This
different
structure
that
uses
an
additive
phrase
does
NOT
change
the
verb
formation.
The
correct
sen
-
tences are:
1 2 4
501_03_121-234_501_master.qxd 4/29/13 12:32 PM Page 125
501 GMAT
®
Questions
Dr. Jones and his staff go out for a Christmas dinner ever y December .
Dr. Jones, accompanied by his staff, goes out for a Christmas dinner every
December.
Common Additive Phrases
accompanied by
along with
as well as
in addition to
together with
Collective Nouns
Collective
nouns
are
nouns
that
can
represent
a
group
of
people
or
things,
but
they
function
as
singular
nouns.
Beware
of
collective
nouns
on
the
sen-
tence
correction
portion
of
the
GMAT.
They
are
used
to
trick
you
and
make
you
believe
the
noun
is
plural.
On
the
GMAT,
collective
nouns
are
considered to be singular
. Some common ones are:
audience
band (musical)
board (of dir
ectors)
cabinet (of members)
company
committee
corporation
council
crowd
department
family
government
majority
minority
public
society
team
1 2 5
501_03_121-234_501_master.qxd 4/29/13 12:32 PM Page 126
501 GMAT
®
Questions
X Society have changed over the past 20 years with the advent of technology .
This example is incorrect because society is singular. Although it repre-
sents a group of people, it is a collective noun and must be treated as a sin-
gular noun. Therefore, have must change to has.
Society has changed over the past 20 years with the advent of technology .
Subject Phrases: Always Singular
A sentence may have a phrase or idea as its subject. This phrase is often pre-
sented with a gerund. It is always singular.
X Having twins have taken a toll on Mrs. Smith’
s health.
The
sentence
is
incorrect
because
the
real
subject
is
having
twins,
not
just
twins.
Twins is plural, but having twins is a singular concept. This is an exam-
ple of a subject phrase whose corresponding verb must be singular. The cor-
rect sentence reads:
Having twins has taken a toll on Mrs. Smith’
s health.
Pronouns
A pronoun is a word that replaces a noun or noun phrase. Depending on the
role
of the noun or noun phrase in the sentence, a pronoun may be a sub-
ject pronoun (replacing the subject of a sentence) or an object pronoun
(replacing the object
of a sentence).
Subject Pronouns
I
you
he
she
it
we
they
The teacher praised the student. He praised the student.
In this case, he has been substituted for the teacher. He is a subject pronoun.
1 2 6
501_03_121-234_501_master.qxd 4/29/13 12:32 PM Page 127
Object Pronouns
501 GMAT
®
Questions
me
you
him
her
it
us
them
The teacher praised the student. The teacher praised her .
In this case, her has been substituted for the student. Her is an object
pronoun.
Possessive Pronouns
mine
yours
his
hers
its
ours
theirs
Possessive
pronou
ns
are
used
to
replace
nouns
and
show
ownership
or
pos-
session.
Of all the excellent grades, the teacher praised Jenna’s grade the most. Of all the
excellent grades, the teacher praised hers the most.
In
this
case,
Jennas
grade has
been
replaced
by
hers.
Hers is
a
possessive
pro-
noun.
Possessive Adjectives
my
your
his
her
its
our
their
1 2 7
501_03_121-234_501_master.qxd 4/29/13 12:32 PM Page 128
501 GMAT
®
Questions
The teacher praised the student’ s efforts. The teacher praised her efforts.
In this case, the student’ s efforts has been replaced by her.
Demonstrative Pronouns
this
that
these (plural of this)
those (plural of that)
The time invested by the CEO to research the project was much less than the time
spent by his subordinates. The time invested by the CEO to research the project
was much less than that spent by his subor dinates.
In this case, the second mention of the time has been replaced by that.
Pronouns are often used to avoid repeating the subject or noun they refer
to in the sentence. However, because they often refer to a noun
in
another
part
of
the
sentence,
choosing
the
wrong
pronoun
is
a
common
mistake.
Pronoun
errors
are
very
frequent
on
the
GMAT
sentence
correction
sec
-
tion. It is crucial that you check a pronoun every time you see one!
*Make sure a proper antecedent exists for a pronoun.
X The homeowners pondered several upgrades, which would greatly raise the value
of it.
The previous sentence is incorrect because it does not refer to anything in
the sentence. A reader might assume it refers to the home, but home is only
given to modify the type of owners (homeowners), so home cannot be a log
-
ical antecedent for the pronoun
it. The correct sentence would be:
The
homeowners
pondered
several
upgrades,
which
would
greatly
raise
the
value
of
the home.
*Make sure the pronoun is NOT ambiguous. It must only have ONE
antecedent.
X The dental field has developed a new series of white fillings for anterior teeth,
purportedly giving them stronger r esistance against common wear and tear .
1 2 8
501_03_121-234_501_master.qxd 4/29/13 12:32 PM Page 129
501 GMAT
®
Questions
This sentence is incorrect because them can refer to a number of nouns in
the sentence (series of white fillings, anterior teeth). Again, the reader can
assume what them refers to, but more than one answer is possible. Fix this
problem by rewording the sentence.
The
dental
field
has
developed
a
new
series
of
white
fillings
for
anterior
teeth,
purportedly giving
teeth stronger r esistance against common wear and tear .
*Make sure the pronoun and its antecedent agree.
X Joanna’s promotion to the board of directors is uncertain because she does not meet
all their rigid standar ds.
This sentence is incorrect because their incorrectly refers to the board of
directors. Although directors is plural, the true noun is the singular board (of
directors). Therefore, the sentence should read:
Joanna’s promotion to the board of directors is uncertain because she does not
meet all its rigid standar
ds.
Now
that
you
have
seen
common
pronoun
errors,
let’s
look
at
common
errors with modifiers.
Modifiers
Modifiers
modify
or
describe
a
part
of
the
sentence.
A
noun
modifier
mod-
ifies a noun.
John sat down in his chair
,
worn from endless hours of pr
essure.
The
clause
wor
n from endless hours of pressure is a modifier. Upon initial
glance, a reader may think that it modifies John. This is incorrect; in fact,
it modifies his chair .
*A noun and its modifier should be placed next to each other .
Look at the difference in meaning when the modifier changes location.
John, worn from endless hours of pr essure, sat down in his chair .
1 2 9
501_03_121-234_501_master.qxd 4/29/13 12:32 PM Page 130
501 GMAT
®
Questions
Be very aware of modifiers! They must be as close as possible to the noun
or clause that they modify. Misplaced modifiers are a common trick on the
GMAT sentence correction section. They can lead to ambiguous meaning
and syntax. On some occasions, the GMAT will present a dangling modi
-
fier that
does not
seem
to have a referent.
Dangling Modifiers
X Forced to accept his r
esignation, sadness per
meated the office.
The
clause
sadness
permeated the
office
is
a
complete, independent clause. The
first part of the sentence, however, is not independent because it lacks a sub-
ject. To what does it refer? It cannot be sadness
.
Sadness cannot
be
forced
to
accept
his
resignation. This is illogical. Therefore, forced to accept his resigna-
tion is
considered
a
dangling
modifier
because
the
noun
it
is
trying
to
mod-
ify
is
not
even
in
the
sentence.
It
is
incorrect
and
needs
to
be
fixed.
One
possible correction could be:
Forced to accept his resignation, the staff exuded a sadness that permeated
the office.
Here, the
staff is clearly forced to accept his resignation. This
sentence is
correct.
Restrictive versus Nonrestrictive Clauses
Another important testing point on the GMAT sentence correction por-
tion is the difference between a restrictive (essential) clause and a non-
restrictive (nonessential) clause.
*Restrictive clauses provide essential information about a subject.
The dog that has three legs was adopted by a family today.
In this example, the clause that has three legs is essential to identifying which
dog. Therefore, that
is used and there are no commas.
*Nonrestrictive clauses provide extra, or nonessential, information
about a subject.
1 3 0
501_03_121-234_501_master.qxd 4/29/13 12:32 PM Page 131
501 GMAT
®
Questions
The East Los Angeles Animal Shelter, which was founded in 1983, takes in
handicapped animals and helps them find a home.
In this example, the clause which was founded in 1983 is not essential to iden-
tifying the East Los Angeles Animal Shelter. It is considered extra information
and therefore set off by commas.
*Put commas between nonessential clauses and their nouns. Use
which, NOT
that.
*Do
NOT
put
commas
between
essential
clauses
and
their
nouns.
Use
that, NOT
which.
Active
Voice
versus
Passive
Voice
Sentences
will
either
be
in
the
active
or
passive
voice.
In
the
active
voice,
the
subject
of
the
sentence
performs
the
action.
In
the
passive
voice,
the
subject
of
the
sentence
receives
the
action
or
has
an
action
performed
on
it.
Active V
oice
The teacher
praised the student.
Passive V oice The student was praised by the teacher .
Beware of intransitive verbs, which are verbs that cannot be formed in the
passive voice. Intransitive verbs have no direct object. Look at the follow-
ing example:
Active V oice Josh went to work on Monday mor ning.
This sentence cannot be put in the passive voice, so go (or the past tense
went in this example) is an intransitive verb.
The GMAT generally prefers the active voice to the passive voice. Look for
more details about stylistic preferences with the active voice in the Effec-
tive Expression instructional text.
Verb Tenses and Their Uses
Review the following definitions of the different verb tenses. Make sure you
have a general understanding of their different roles and limitations.
1 3 1
501_03_121-234_501_master.qxd 4/29/13 12:32 PM Page 132
501 GMAT
®
Questions
1 3 2
Simple Present
Uses:
Facts or generalizations The Earth rotates around the sun.
Repeated or usual actions She always forgets to call her mother
on Sundays.
Scheduled events in the near future The family leaves tonight
for their vacation home.
Non-continuous verbs to express an action right now She
has her train ticket in her hand. (see Simple Present versus
Present Progressive for more information on non-continuous
verbs)
Present Progressive
Uses:
Express an action that is occurring at this moment You are
studying verb tenses.
Express longer actions that are in progress now He is
preparing for his speech next month.
*Remember that some verbs (known as non-continuous verbs) are not
expressed in the gerund form.
Simple Present versus Present Progressive
Always use the simple present tense when referring to a general statement
or truth.
Remember that non-continuous verbs are verbs that are almost never
e
xpressed in the gerund -ing form. Some of the major non-continuous verbs
include the following:
To be (location) Ralph
is at the store right now. NOT Ralph is being at the
store right now .
To have (possession) She has $3.00 in her hand. NOT She is having $3.00
in her hand .
501_03_121-234_501_master.qxd 4/29/13 12:32 PM Page 133
501 GMAT
®
Questions
1 3 3
To want The doctor wants additional tests. NOT The doctor is wanting addi-
tional tests .
Present Perfect
Uses:
Express an action that happened at a previous unspecified time
She has met him before.
Express an action that has happened various times in the past
They have had six quizzes this semester.
Present Perfect Progressive
Uses:
Express an action that started in the past and continues until
now, with or without reference She has been taking antibiotics
since Tuesday. They have been behaving badly.
Simple Past
Uses:
Express completed action in the past She finished her test at
noon.
Past facts or generalizations Horses were the principal means of
transportation 100 years ago.
Express habits in the past He played the piano as a child.
Express an action that began and ended in the past They
lived in Chile for one year.
Express a group of completed actions He checked into his
room, took a shower, and met his friends for dinner.
Past Perfect
Uses:
Express the idea that something occurred before something
else in the past They had visited Chile several times before they
moved there.
Past Perfect versus Simple Past
It is important to mention that the past perfect tense is not always used to
indicate
earlier
actions.
If
it
is
obvious
that
the
earlier
event
preceded
the
501_03_121-234_501_master.qxd 4/29/13 12:32 PM Page 134
501 GMAT
®
Questions
1 3 4
other, then the past perfect tense can be substituted for the simple past
tense. Also, if the order of events is not being emphasized, the past perfect
tense is not necessary.
Anthony drove to work and crashed his car in the parking lot.
The family checked the car tires before they began the long drive home.
Notice how the above examples mention two events in an obvious chrono-
logical order, but they don’t use the past perfect tense.
Past Progressive
Uses:
Express a longer action in the past that was interrupted She
was studying for the GMAT exam when the phone rang.
Describe an atmosphere or situation The sun was shining and
the birds were chirping.
Comparing with Adjectives and Adverbs
Comparatives: to be used when comparing two things
Superlatives: to be used when comparing more than two things
Comparing with Adjectives
Adjectives, as you have already learned, describe nouns. When you want to
c
om
pare two nouns with an adjective, you will need to change the adjective
into its comparative form followed by than and the second object you are
comparing. When you want to compare more than two things with an
adjective, you will need to change the adjective into its superlative form.
Comparative: adjective + -er (for adjectives of 1 and 2 syllables)
The new skyscraper is taller than the church.
The stained glass in the church is prettier than the glass on the skyscraper.
Comparative: mor
e + adjective (for longer adjectives)
The new skyscraper has more fascinating architectural elements than the church.
501_03_121-234_501_master.qxd 4/29/13 12:32 PM Page 135
501 GMAT
®
Questions
The church is more historical than the skyscraper.
Superlative: adjective + -est (for adjectives of 1 and 2 syllables)
He was the happiest storeowner on the block.
His business was the quickest to succeed.
Superlative: the + most + adjective (for longer adjectives)
His restaurant had the most interesting dishes in the neighborhood.
His restaurant is the most successful on the block.
Comparing with Adverbs
Adverbs describe adjectives, verbs, and other adverbs. The most common
adverb mistakes occur when describing verbs. Watch for correct adverb use
on the GMAT sentence correction portion, especially when you make a
comparison. When comparing two ideas with an adverb, use the compar
-
ative form
of
the
adverb,
followed
by
than and
the
second
idea
you
are
com-
paring.
When
comparing
more
than
two
ideas
with
an
adverb,
use
the
superlative form.
Comparative: more + adverb + than (for all adverbs)
A Marine is trained to react more quickly than a civilian in times of panic.
John received the promotion because he thinks more creatively than his compe-
tition does.
Superlative: most + adverb (for all adverbs)
Of all Robert’
s work references, Mr. Jacobs spoke the most highly of him.
She visited her parents most often during the summer.
Exceptions with Comparison W ords
*As with all grammar rules, there are exceptions. The following list includes
some common irregular adjectives and adverbs.
1 3 5
501_03_121-234_501_master.qxd 4/29/13 12:32 PM Page 136
501 GMAT
®
Questions
1 3 6
Adjective/Adverb Comparative Superlative
good better the best
well better the best
bad worse the worst
badly worse the worst
many more the most
much more the most
little less the least
far further the furthest
It
is
very
important
on
the
GMAT
sentence
correction
section
for
com-
parative
and
superlative
forms
to
be
correctly
formed.
However,
the
GMAT
exam
will
go
one
step
further
and
make
sure
that
correct
comparisons
are
made.
Look
for
more
information
on
logical
comparisons
in
the
Effective
Expression instructional text.
Subjunctive
Voice
The subjunctive voice tends to appear in two types of situations:
Requests, desires, and suggestions that are structured with
certain verbs and followed by the word that (known as the
command subjunctive)
Conditional sentences
Command Subjunctive
The command subjunctive form is used with certain verbs that tell people
to do things or suggest that
they do things. The initial verb must always be
followed by that.
The state requires that the school be prepared for an earthquake.
Notice how the sentence begins with requires + that, and the
second verb is
in its base form. The base form of a verb is its most simple form, without
to. This is the command subjunctive.
The board proposed that the company go to career fairs to recruit younger per-
sonnel.
501_03_121-234_501_master.qxd 4/29/13 12:32 PM Page 137
501 GMAT
®
Questions
Notice how go is in its base form following the verb proposed + that. There-
fore, the construction of the command subjunctive is:
Command verb + that + subject + command subjunctive form of verb:
She suggests that everyone contemplate the implications of the proposal.
They demand that the company respond with an answer by 5
P.M.
Common verbs that require the command subjunctive:
demand, dictate, insist, mandate, propose, recommend, request, stipulate, suggest
The board recommends that all employees take their time to consider their health
insurance options.
*
There are, however, some verbs that seem like command verbs, but do not
take the
subjunctive form. Instead, they remain in the infinitive (to + base
form of verb) form.
Common verbs that require the infinitive form:
advise, allow, forbid, persuade, want
The lawyer advised his client to get his affairs in order.
He forbade her to drive after midnight.
Other verbs can take either the subjunctive or the infinitive form. Keep in
mind that the GMAT will typically prefer the subjunctive form, if a choice
exists at all. Just remember to check the structure of the subjunctive clause
to make sure it is formed correctly! Notice the different structures that fol
-
low with the same command verb.
Common verbs that can take the subjunctive or infinitive forms:
ask, beg, intend, order, prefer, urge, require
The state requires that the school be prepared for an earthquake. OR
The state requires the school to be prepared for an earthquake.
1 3 7
501_03_121-234_501_master.qxd 4/29/13 12:32 PM Page 138
501 GMAT
®
Questions
Remember that the that construction after the command verb is necessary
to form the subjunctive voice!
Hypothetical Subjunctive and Conditional Sentences
Conditional sentences are also known as if . .
. then constructions. The fol-
lowing examples are given with the if clause first. Notice
in the examples
following the structure that the sentence may also be inverted with the
result clause first and the if clause second.
Used to express a general truth or statement
If + subject + present tense verb, subject + present tense verb
If you drop a ball, it falls.
A ball
falls if you drop it.
Used to express the consequence of an action
If + subject
+ present tense verb, subject + future tense verb (or
may/might/could + verb in base form)
If he breaks his promise, she will be disappointed.
She will be disappointed if he breaks his promise.
If she goes to London to study, her parents might be worried.
Her par
ents might be worried if she goes to London to study.
Used to express the consequence of an
unlikely event
If + subject + past tense verb (hypothetical subjunctive voice), subject +
would/may/might + verb in base form
If he broke his promise, she would be disappointed.
She would be disappointed if he broke his promise.
If he were here, he would know what to do.
He would know what to do if he were here.
Used to express the consequence of an action that never happened
(refers to something in the past)
If + subject + had + past participle, subject + would/may/might + have +
past participle
If he had broken his promise, she would have been disappointed.
She would have been disappointed if he had broken his promise.
1 3 8
501_03_121-234_501_master.qxd 4/29/13 12:32 PM Page 139
501 GMAT
®
Questions
Hypothetical Subjunctive
***The hypothetical subjunctive voice is used to express an unlikely or
unreal condition. It is
always the simple past form of every verb, except in
the verb to be. With to be, it is always were, and never was.
If I were you, I would take the risk.
If she were in your place, I would tell her to quit.
It is common to see this structure with the verb wish:
I wish I were finished with this test!
We wish there were more days to study for this test!
In spoken speech, it is common to hear I wis
h I was. Although it has become
accepted in everyday speech, it is grammatically incorrect, especially on the
GMAT sentence correction section.
Parallelism
Parallelism, or parallel structure, improves the effectiveness of a sentence
b
y wo
rding multiple ideas with a similar structure. A similar structure facil-
itates understanding and can consist of verb and verb, noun and noun,
phrase
and phrase, and
so on. Parallel structure is a very important aspect
of GMAT sentence correction questions and in many cases, it can be the
deciding factor between a right and wrong answer. You will encounter var
-
ious questions that deal with parallelism. It is important to remember that
othe
r errors may exist within a question that seems to touch on parallelism,
so carefully read each choice.
Parallelism may exist in a number of instances that include:
List
She bought eggs, flour, sugar, pineapple, and cream for her cake.
X She bought some eggs, flour, sugar, two pineapples to cut into
chunks, and then some cream for her cake.
Notice how the first example is much easier to understand
because it employs a parallel structure with simple nouns.
Series of Phrases
The conference speakers touched on the importance of conservation,
the dangers of exploitation, and the future of ecotourism.
1 3 9
501_03_121-234_501_master.qxd 4/29/13 12:32 PM Page 140
Idiomatic Expressions
501 GMAT
®
Questions
X The conference speakers talked about how conservation is
important, the dangers of exploitation, and what the future holds
for ecotourism.
Notice how the first example is much more clear and succinct
because the three factors are all in noun form.
Compare and Contrast
It is better to have loved and lost than never to have loved at all.
X It is better to have loved and lost than never know love.
Notice that although the parallel example that employs the
present perfect tense in both clauses is longer
, it sounds better.
Brevity does not always indicate the best answer.
Certain expressions force the items in a sentence to be parallel. It is good
to be aware of these idiomatic expressions. When you see them, make sure
the items they refer to are structured with the same part of speech.
Parallel Indicators
And She decided to r
un
and swim yester day.
Both . . . and She went to both the gym and the pool yester day.
Either . . . or She was going to either r un or swim.
From . . . to She went from r unning at the gym to swimming at
the pool.
Not . . . but She pr
efers not cycling,
but running and swimming.
Not only . . . but also She does not only run, but also swims.
Or On most days, she r uns or swims.
The aforementioned examples all contain some variant of run and swim. In
e
ach example, the ideas of running and swimming are kept in a similar struc-
ture to maintain parallelism.
Now, let us look at some additional idiomatic expressions that may
a
ppe
ar on the GMAT sentence correction test. You should be familiar with
them and be able to spot errors associated with their use.
as many as (to be used with countable nouns)
as much as
(to be used with noncountable nouns)
as (adverb/adjective) as
the number of
(to be used with singular verbs)
1 4 0
501_03_121-234_501_master.qxd 4/29/13 12:32 PM Page 141
501 GMAT
®
Questions
1 4 1
a number of (to be used with plural verbs)
to consider
to regard as
x and y, as well as z
Set 5
Now it is time to answer GMAT sentence correction practice questions that
have been designed to test your correct expression skills. As the questions
progress, you may notice some additional concepts that are not addressed
in this instructional text. These concepts are derived from the effective
expression section. The real GMAT sentence correction questions will
combine concepts from both sections, so pay close attention to unfamiliar
ideas. They will surely be addressed in the effective expression section that
follows. Good luck!
93.
According to a report released by Oxfam, the most successful
nations in the world not only consume the bulk of the world’s
resources, but they are now home to more than half of the world’s
poorest
people.
a. not only consume the bulk of the world’s resources, but they are
now
b. not only consumes the bulk of the world’s resources, but they
are now
c. not
only consumes the bulk of the world’s resources, but they
are now being
d. not only consume the bulk of the world’s resources, but they
have now been
e. not only is consuming the bulk of the world’s resources, but
they are now
501_03_121-234_501_master.qxd 4/29/13 12:32 PM Page 142
501 GMAT
®
Questions
94.
When it ruled that the government couldn’t track an individual by
attaching a GPS device to a vehicle without a warrant, the
Supreme Court curtailed many covert
operations, setting new
standards for future intelligence missions.
a. operations, setting new standards for future intelligence mis-
sions.
b. operations, having set new standards for future intelligence mis-
sions.
c. operations and set new standards for future intelligence mis-
sions.
d. operations and will set new standards for future intelligence
missions.
e. operations, new standards were set for future intelligence mis-
sions.
95.
One of the most important questions facing the American public in
the wake of the global energy crisis is if they can make the
transition to hybrid cars to lessen the dependence on foreign oil.
a. is if they can make the transition
b. are if they can make the transition
c. is whether they can make the transition
d. is whether it can make the transition
e. are if it can make the transition
96.
China’s views of stem cell treatment, which do not place as much
significance on the moral value of the embryo, differ largely from
the United States, which views the embryo as a representation of
human life.
a. the United States
b. that of the United States
c. that embraced by the United States
d. those from the United States
e. those of the United States
1 4 2
501_03_121-234_501_master.qxd 4/29/13 12:32 PM Page 143
501 GMAT
®
Questions
1 4 3
97.
British scientists are working on preserving the world’s most
threatened plant species, their regeneration once they’ve been in
frozen storage, and ways to extract their medicinal properties.
a. are working on preserving the world’s most threatened plant
species, their regeneration once they’ve been in frozen storage,
and ways to extract their medicinal properties.
b. are working on preserving the world’s most threatened plant
species, regenerating them after frozen storage, and extracting
their medicinal properties.
c. are working on preserving the world’s most
threatened plant
species, to regenerate them after frozen storage, and extract
their medicinal properties.
d. are working diligent to preserve the world’s most
threatened
plant species, regenerate them after frozen storage, and extract
its medicinal properties.
e. are working to preserve the world’s more threatened plant
species, regenerate them after frozen storage, and extract their
medicinal properties.
98.
During the bus boycott by the African American community in
1955, contemporaries of Martin Luther King, Jr.’s insisted that he
had had the makings of a great leader.
a. of Martin Luther King, Jr.’s insisted that he had had
b. of Martin Luther King, Jr.’s were insisting that he has
c. of Martin Luther King, Jr.’s insisted that he had
d. of Martin Luther King, Jr. insisted that he has
e. of Martin Luther King, Jr. insisted that he had
99.
Floating above Earth, satellites help transmit information between
newspapers and printing sites, information that affects society.
a. printing sites, information that affects
b. printing sites, with information that affects
c. printing sites, the information affects
d. printing sites that affect
e. printing sites that has affected
501_03_121-234_501_master.qxd 4/29/13 12:32 PM Page 144
501 GMAT
®
Questions
100.
If there was any truth to recent findings, inactivity is as deadly than
smoking.
a. there was any truth to recent findings, inactivity is as deadly
than
b. there was any truth to recent findings, inactivity is as deadly as
c. there’s been any truth to recent findings, inactivity is as deadly
as
d. there is any truth to recent findings, inactivity is as deadly as
e. there is any truth to recent findings, inactivity is as deadly than
101.
In an uncertain economy, the manufacturing sector, which includes
everything from automobiles, energy equipment, and medical
devices, seems to be making a comeback.
a. from automobiles, energy equipment, and medical devices,
seems to be
b. from automobiles and energy equipment to medical devices,
seems to be
c. from automobiles, energy equipment, and medical devices,
seems that
it is
d. from automobiles and energy equipment to medical devices,
seems that it is
e. from automobiles, energy equipment,
in addition to medical
devices, seems to be
102.
As companies grow increasingly dependent on technology,
employees having mastered computer engineering become more
competitive in the job market.
a. having mastered computer engineering become
b. who mastered computer engineering became
c. having mastered computer engineering will be becoming
d. who master computer engineering are becoming
e. who master computer engineering have been becoming
1 4 4
501_03_121-234_501_master.qxd 4/29/13 12:32 PM Page 145
501 GMAT
®
Questions
103.
Like trees have rings to determine their age, so do growth rings on
the scales of some fish.
a. Like trees have rings to determine their age, so do growth rings
on the scales of some fish.
b. Like trees have rings to determine their age, some fish have
growth rings on their scales.
c. Trees, as some fish, have growth rings that determine their age.
d. As trees have rings, growth rings also appear on some fish to
determine their age.
e. Some fish, like trees, have growth rings that determine their
age.
104.
Fossil discoveries suggest that an extinct river turtle survived the
meteorite, which killed off dinosaurs 65 million years ago.
a. meteorite, which killed off dinosaurs 65 million years ago.
b. meteorite that had killed off dinosaurs 65 million years ago.
c. meteorite that killed off dinosaurs 65 million years ago.
d. meteorite, which had killed off dinosaurs 65 million years ago.
e. meteorite killing off dinosaurs 65 million years ago.
105.
Three of the most encouraging ideas to extend the life of satellites
includes restoring power via a mission extension vehicle, refueling
via a type of traveling service station, and to detach working parts
from old satellites to attach them to new ones.
a. includes restoring power via a mission extension vehicle, refuel-
ing via a type of traveling service station, and to detach working
parts from old satellites to attach them
to new ones.
b. include restoring power via a mission extension vehicle, refuel-
ing via a type of traveling service station, and detaching working
parts from old satellites to attach them to new ones.
c. are to restore power via a mission extension vehicle, refueling
via a type of traveling service station, and by detaching working
parts from old satellites to attach them to new ones.
d. includes restoring power via a mission extension vehicle, refuel-
ing via a type of traveling service station, and detaching working
parts from old satellites to attach them to new ones.
e. include restoring power via a mission extension vehicle, using a
traveling service station that refuels them, and detaching work-
ing parts to attach them to new satellites from old ones.
1 4 5
501_03_121-234_501_master.qxd 4/29/13 12:32 PM Page 146
501 GMAT
®
Questions
106.
Thought to be either symbolic or accurate descriptions,
archaeologists have debated cave paintings for some time.
a. archaeologists have debated cave paintings for some time.
b. archaeologists have long debated the significance of cave paint-
ings.
c. much time has been devoted to debating the significance of cave
paintings.
d. cave paintings have been a subject of debate by archaeologists
for some time.
e. cave paintings have debated archaeologists for some time.
107.
Some economic experts argue that if government would not have
intervened in the housing market, the economic crisis would have
been avoided.
a. if government would not have intervened in the housing mar-
ket, the economic crisis would have been avoided.
b. the economic crisis would have been avoided if government
would not have intervene in the housing market.
c. if government had not intervened in the housing market, the
economic crisis could have been avoided.
d. the economic crisis could be avoided if government didn’t inter-
vene in the housing market.
e. the economic crisis would have been avoided if government had
not
intervene in the housing market.
108.
Easter Islanders claim that the island’s ancient, famous statues
walked by themselves, which they allege as proof of their magical
powers.
a. which they allege as proof of their magical powers.
b. which they have alleged as proof of their magical powers.
c. which they alleged as an example of evidence of its magical
powers.
d. which allegedly are proof of the statues’ magical powers.
e. which is alleged proof of the statues’ magical powers.
1 4 6
501_03_121-234_501_master.qxd 4/29/13 12:32 PM Page 147
501 GMAT
®
Questions
109.
Citizens often demand that a presidential candidate release their
tax records in order to determine how much has been taken by the
government.
a. release their tax records in order to determine how much has
been taken by the government.
b. release his or her tax records in order to determine how much
has been taken by the government.
c. releases his or her tax records in order to determine how much
have been taken by the government.
d. release their tax records in order to determine how much have
been taken by the government.
e. releases his or her tax records in order to determine how much
has been taken by the government.
110.
Democrats’ views, which favor progressive taxation and a liberal
philosophy, are generally deemed more revolutionary than
Republicans’.
a. are generally deemed more revolutionary than Republicans’.
b. are generally deemed more revolutionary than Republican.
c. are generally deemed the most revolutionary over Republicans’.
d. are generally deemed the most revolutionary in comparison to
Republican’
s.
e. are generally deemed more revolutionary than Republicans.
111.
Figures from 2011 indicate that a number of unemployed
American citizens with a disability were about the same as in 2010.
a. a number of unemployed American citizens with a disability
were about the same as in 2010.
b. a number of unemployed American citizens with a disability was
about the same as in 2010.
c. the number of unemployed American citizens with a disability
was about the same than in 2010.
d. the number of unemployed American citizens with a disability
were about the same than in 2010.
e. the number of unemployed American citizens with a disability
was about the same as in 2010.
1 4 7
501_03_121-234_501_master.qxd 4/29/13 12:32 PM Page 148
501 GMAT
®
Questions
112.
The market forces to which the movements of a stock price are
subjected are sufficient to surprise even the most experienced
stockbrokers.
a. to which the movements of a stock price are subjected are
b. to which the movements of a stock price is subjected is
c. to which the movements of a stock price are subjected is
d. to which the movements of a stock price is subjected are
e. subjecting the movement of a stock price is
113.
A new definition for autism suggests that a new umbrella category
of “autism spectrum disorder” be established, and in encompassing
previously separate disorders, such as Asperger’s syndrome and
childhood disintegrative disorder, they would achieve more
accurate diagnoses and better treatment.
a. be established, and in encompassing previously separate disor-
ders, such as Asperger’s syndrome and childhood disintegrative
disorder
, they would achieve more accurate diagnoses and bet-
ter treatment.
b. is established, and in encompassing previously separate disor-
ders, such as Asperger’
s syndrome and childhood disintegrative
disorder
, more accurate diagnoses and better treatment would
be achieved.
c. to be established, and in encompassing previously separate dis-
orders, such as Asperger’s syndrome and childhood disintegra-
tive disorder, they would potentially achieve more accurate
diagnoses and better treatment.
d. be established, which would encompass previously separate dis-
orders such as Asperger’
s syndrome and childhood disintegra-
tive disorder, and lead to more accurate diagnoses and better
treatment.
e. is established, and by encompassing previously separate disor-
ders, such as Asperger’s syndrome and childhood disintegrative
disorder
, they would achieve more accurate diagnoses and bet-
ter treatment.
1 4 8
501_03_121-234_501_master.qxd 4/29/13 12:32 PM Page 149
501 GMAT
®
Questions
1 4 9
114.
Surveys indicate that feelings of happiness and satisfaction are
relative; a poor man may wish he was $1 million richer, yet a
millionaire may still feel unsatisfied and wishes he had $2 million
more.
a. may wish he was $1 million richer, yet a millionaire may still
feel unsatisfied and wishes he had
b. may wish he were $1 million richer, yet a millionaire may still
feel unsatisfied and wish he had
c. may wish he was $1 million richer, yet a millionaire will still feel
unsatisfied and wish he had
d. wishes he was $1 million richer, yet a millionaire still feels
unsatisfied and wishes he had
e. may wish he were $1 million richer, yet a millionaire may still
feel unsatisfied and wish he has
115.
A system of underground tunnels at the White House were built
on an impressive scale, with a complex communication network, to
lead to the Deep Underground Command Center in the case a
national emergency.
a. House were built on an impressive scale, with a complex com-
munication network, to lead to the Deep Underground Com-
mand Center in the case a national emergency.
b. House were built on an impressive scale, which has a complex
communication network,
leading to the Deep Underground
Command Center in the event of a national emergency.
c. House, featuring a complex communication network, was built
on an impressive scale to lead to the Deep Underground Com-
mand Center in the event of a national emergency.
d. House was built on an impressive scale with a complex commu-
nication network to lead to the Deep Underground Command
Center should a national emergency occurs.
e. House were built on an impressive scale, with a complex com-
munication network, that lead to the Deep Underground Com-
mand Center in case of a national emergency
.
501_03_121-234_501_master.qxd 4/29/13 12:32 PM Page 150
501 GMAT
®
Questions
116.
A majority of society argues that neither technology nor genetics is
to blame for rampant childhood obesity in society.
a. argues that neither technology nor genetics is to
b. argue that neither technology nor genetics are to
c. argue that they cannot look at technology or genetics as the
d. argue that not technology or genetics is to
e. argues that not technology nor genetics is to
117.
Rather than waiting to die, Oregon’s right-to-die law permits
mentally stable people, which have less than six months to live,
access to medication to accelerate their demise.
a. Rather than waiting to die, Oregon’s right-to-die law permits
mentally stable people, which have less than six months to live,
access to medication to accelerate their demise.
b. Rather than waiting to die, Oregon’s right-to-die law permits
mentally stable people that have less than six months to live to
access medication to accelerate their demise.
c. Rather than waiting to die, the demise of mentally stable people
who have less than six months to live can be accelerated by Ore-
gon’s right-to-die law who permits them access to medication.
d. Rather than waiting to die, mentally stable people who have less
than six months to live are permitted access to medication by
Oregon’
s right-to-die law to accelerate their demise.
e. Rather then waiting to die, mentally stable people’s demise can
be accelerated by Oregon’s right-to-die law, permitting them
access to medication provided they have less than six months
to live.
1 5 0
501_03_121-234_501_master.qxd 4/29/13 12:32 PM Page 151
501 GMAT
®
Questions
118.
The newly established International Criminal Court brought
charges against the warlord for plundering villages, stealing food,
and the kidnapping of children for taking as soldiers and wives.
a. for plundering villages, stealing food, and the kidnapping of
children for taking as soldiers and wives.
b. for plundering villages, to steal food, and kidnap children to
take as soldiers and wives.
c. for his plundering of villages, stealing food, and kidnapping
children to be soldiers and wives.
d. because he plundered villages, stealing food, and kidnapping
children to take as soldiers and wives.
e. for plundering villages, stealing food, and kidnapping children
to take as soldiers and wives.
119.
Although the Olympic athlete’s mother was at home watching her
son on television, the competition was as exciting to her as the
royal attendees sitting in the first row.
a. was as exciting to her as the royal attendees sitting in the first
row
b. was as exciting to her than the royal attendees sitting in the first
row
c. was as exciting to her as to the royal attendees sitting in the first
row
d. was as exciting to her than to the royal attendees sitting in the
first row
e. was as exciting to her then to the royal attendees sitting in the
first row
1 5 1
501_03_121-234_501_master.qxd 4/29/13 12:32 PM Page 152
501 GMAT
®
Questions
120.
Many full veiled Muslim women are outspoken in defending their
patriarchal traditions, especially when confronted with Western
opposition who views them as being heavily
oppressed.
a. full veiled Muslim women are outspoken in defending their
patriarchal traditions, especially when confronted with Western
opposition who views them as being heavily
b. full veiled Muslim women are outspoken in defending her patri-
archal traditions, especially when confronted with Western
opposition who views her as being heavily
c. fully veiled Muslim women are outspoken in defending their
patriarchal traditions, especially when confronted with Western
opposition that views them as being heavy
d. fully veiled Muslim women are outspoken in defending their
patriarchal traditions, especially when confronted with W
estern
opposition that views them as being heavily
e. fully veiled Muslim women are outspoken in defending her
patriarchal traditions, especially when confronted with Western
opposition that views her as being heavily
121.
A handful of companies offer DNA-based tests that supposedly
determine how well a person’s natural athletic abilities are, but
critics insist these tests don’t reveal much more than a standard
performance test.
a. well a person’s natural athletic abilities are, but critics insist
these tests don’t reveal much more than a standard performance
test.
b. good a person’
s natural athletic abilities are, but critics insist
that these tests don’t reveal much more than standard per-
formance tests do.
c. well a person’s natural athletic abilities are, but critics insist
these tests don’
t reveal much beyond a standard performance
test.
d. well a person’s natural athletic abilities are, but critics insist that
these tests don’
t reveal much more than standard performance
tests do.
e. good a person’s natural athletic abilities are, but critics insist
that standard performance tests don’t reveal much less than the
other tests.
1 5 2
501_03_121-234_501_master.qxd 4/29/13 12:32 PM Page 153
501 GMAT
®
Questions
122.
Resigned to accept his disqualification from the Olympic trials,
there was only silence in the locker room.
a. Resigned to accept his disqualification from the Olympic trials,
there was only silence in the locker room.
b. There was only silence in the locker room, resigned to accept
his disqualification from the Olympic trials.
c. Silent in the locker room, resigned to accept his disqualification
from the Olympic trials.
d. Resigned to accept his disqualification from the Olympic trials,
silence permeated the locker room.
e. Resigned to accept his disqualification from the Olympic trials,
the athlete remained silent
in the locker room.
123.
Latin America has no single dominant Spanish dialect: The range
of Spanish encompasses many styles, speeds, and slang, with each a
product of their local influences, from the fast-paced, clear
Colombian variants to the more obvious, melodic Argentinian
alternatives.
a. with each a product of their
b. each products of their
c. each products of
d. with each as a product of its
e. each a product of
1 5 3
501_03_121-234_501_master.qxd 4/29/13 12:32 PM Page 154
501 GMAT
®
Questions
124.
In the eighteenth century, William Shakespeare was regarded as
one of the greatest playwrights of all time, and not until the mid-
nineteenth century was the authorship of the works attributed to
Shakespeare first openly questioned by Joseph C. Hart.
a. In the eighteenth century, William Shakespeare was regarded as
one of the greatest playwrights of all time, and not until the
mid-nineteenth century was the authorship of the works attrib
-
uted to Shakespeare first openly questioned by Joseph C. Hart.
b. William Shakespeare was considered as one of the greatest play-
wrights of all time in the eighteenth century, and not until the
middle of the ninteenth century did Joseph C. Hart
first openly
question his works.
c. In the eighteenth century
, they considered William Shakespeare
as one of the greatest
playwrights of all time, and not until the
mid-ninteenth century did Joseph C. Hart first explicit question
his authorship of Shakespeare’s works.
d. In the eighteenth century,
William Shakespeare was considered
to be one of the greater playwrights of all time, and not until
the mid-ninteenth century did Joseph C. Hart openly question
Shakespeare’s authorship of the works attributed to him.
e. In the eighteenth century, William Shakespeare was regarded
to be one of the greatest playwrights of all time, and not until
the middle of the ninteenth century did Joseph C. Hart first
openly question Shakespeare’s authorship of the works attrib
-
uted to him.
1 5 4
501_03_121-234_501_master.qxd 4/29/13 12:32 PM Page 155
501 GMAT
®
Questions
125.
The Happy Planet Index, which ranks 151 countries, considers
data like life expectancy, overall well-being, and a country’s
ecological footprint on calculating the happiness of a country.
a. which ranks 151 countries, considers data like life expectancy,
overall well-being, and a country’s ecological footprint on calcu-
lating the happiness of a country.
b. which ranks the happiness of 151 countries, considers data like
life expectancy and overall well-being and a country’
s ecological
footprint.
c. which ranks the happiness of 151 countries, considers data such
as life expectancy and overall well-being, as well as the country’s
ecological footprint in its calculations.
d. which ranks 151 countries, considers data such as life
expectancy
, overall well-being, as well as a country’s ecological
footprint when calculating the happiness of a country.
e. which ranks 151 countries, consider data such as life expectancy,
overall well-being, and a country’s ecological footprint to deter-
mine the happiness of a country
.
126.
Results from numerous studies show that if a diabetic plans his or
her diet and exercised with fellow diabetics, they will regain
control quicker and stick to weight goals for a longer period of
time than do
someone who attempts to do so alone.
a. exercised with fellow diabetics, they will regain control quicker
and stick to weight goals for a longer period of time than do
b. exercises with fellow diabetics, he or she regains control more
quickly and sticks to weight goals for a longer period of time
than
c. exercises with fellow diabetics, they regain control quicker and
stick to weight goals for a longer period of time than does
d. exercised with fellow diabetics, he or she will have regained
control more quickly and stuck to weight goals for a longer
period of time than
e. exercises with fellow diabetics, he or she regains control more
quickly and sticks to weight goals for a longer period of time
than does
1 5 5
501_03_121-234_501_master.qxd 4/29/13 12:32 PM Page 156
501 GMAT
®
Questions
127.
Growing economic woes and competition could be pushing
accountants to finagle their clients’ records; accountants can, for
example, help their frustrated clients write off certain luxurious
purchases and justify them as business expenses.
a. clients’ records; accountants can, for example, help
b. clients’ records, as an example, they can help
c. clients’ records, like they can help
d. clients records; which might include that they
e. clients’ records, such as to be helping
128.
A series of troubling reports released by some of the top
environmental organizations indicates that as much as 18,000
pieces of floating plastic, which is resistant to natural
biodegradation, makes up
every square kilometer of the ocean.
a. indicates that as much as 18,000 pieces of floating plastic, which
is resistant to natural biodegradation, makes up
b. indicates that as much as 18,000 pieces of floating plastic, which
is resistant to natural biodegradation, have made up
c. indicate that as many as 18,000 pieces of floating plastic, which
is resistant to natural biodegradation, is making up
d. indicates that as many as 18,000 pieces of floating plastic, which
are resistant to natural biodegradation, make up
e. indicate that
as many of 18,000 pieces of floating plastic, which
are resistant to natural biodegradation,
make up
129.
Restaurateurs are well aware that the frequency of employee
turnover and how servers treat their customers is very indicative of
employee happiness on the job.
a. how servers treat their customers is
b. how servers treat their customers are
c. and servers’ treatment of their customers are
d. and servers’ treatment of their customers is
e. customers’ reactions to their servers are
1 5 6
501_03_121-234_501_master.qxd 4/29/13 12:32 PM Page 157
501 GMAT
®
Questions
130.
The famous rivalry of Coke and Pepsi may never have been if
Coca-Cola hadn’t seized the opportunity presented three times
between 1922 and 1933 and bought
the bankrupt Pepsi Cola
Company.
a. of Coke and Pepsi may never have been if Coca-Cola hadn’t
seized the opportunity presented three times between 1922 and
1933 and bought
b. of Coke and Pepsi may never have been if Coca-Cola hadn’t
seized the opportunity presented three times between 1922 and
1933 and hadn’
t bought
c. between Coke and Pepsi may never have been if Coca-Cola
hadn’t seized the opportunity presented three times between
1922 and 1933 and bought
d. between Coke and Pepsi would never be if Coca-Cola had
seized the opportunity presented three times between 1922 and
1933 and bought
e. between Coke and Pepsi may never have been if Coca-Cola had
seized the opportunity presented three times between 1922 and
1933 and bought
131.
In June 2012, the Rhode Island General Assembly had voted to
repeal a law enacted in 1989 that made lying online a
misdemeanor, citing the illogic of trying to enforce something that
is so often
innocently violated.
a. had voted to repeal a law enacted in 1989 that made lying
online a misdemeanor, citing the illogic of trying to enforce
something that is so often
b. voted to repeal a law enacted in 1989 that
made lying online a
misdemeanor, citing the illogic of trying to enforce something
that is so often
c. had voted to repeal a law enacted in 1989 that made lying
online a misdemeanor, citing the illogic of the trying to enforce
something that is so often
d. voted in favor of to repeal a law enacted in 1989 that has made
lying online a misdemeanor, citing the illogic of attempting to
enforce something that is so often
e. voted for repealing a law enacted in 1989 that had made lying
online a misdemeanor, having cited the illogic of trying to
enforce something that is so often
1 5 7
501_03_121-234_501_master.qxd 4/29/13 12:32 PM Page 158
501 GMAT
®
Questions
132.
However much United States citizens may criticize the perils of
capitalism and that the democracy is corrupted, it is rare to find
organized national movements for a structure other than
democracy.
a. However much United States citizens may criticize the perils of
capitalism and that the democracy is corrupted,
b. Although United States citizens may agree capitalism is danger-
ous and democracy is corrupted,
c. Despite criticism by United States citizens to the extent of the
perils of capitalism and corruption of democracy,
d. However much United States citizens may criticize the perils of
capitalism and the corruption of democracy
,
e. However many United States citizens may criticize the perils of
capitalism and corruption of democracy,
133.
The private lending sector has $8 billion in student loans that are
in default or at such serious risk to default that it does not expect
payments when it is due and has consequently tightened credit
standards for new student
loans.
a. that it does not expect payments when it is
b. that payments will not be expected to be paid when
c. that it
does not expect payments to be made when they are
d. that payments are not expected to be paid when they are going
to be
e. that they do not expect payments when it is
Effective Expression
The following material will help you prepare for the part of the GMAT sen-
tence correction section that focuses on clarity and conciseness in sentences,
effective syntax, redundancy, and proper diction.
Sentence Formation
In order to form an effective sentence, it is crucial that you understand its
components.
Syntax
is
defined
as
sentence
structure. Syntax can make the
difference between an effective and ineffective sentence. The following
terms break down the different parts that make up a sentence and how their
misuse can lead to confusion and ambiguity.
1 5 8
501_03_121-234_501_master.qxd 4/29/13 12:32 PM Page 159
501 GMAT
®
Questions
Independent Clause
A clause that expresses a complete sentence Monica walked on the grass .
Coordinating Conjunction
A word that joins two independent and equal clauses: and, but, so, or, for, nor,
yet
Dorothy had a beautiful rose gar den, and her yar d was a profusion of color
every summer .
*The most common coordinating conjunction is
and.
And can also be
used in a list, such as in the following example:
Dorothy has roses, lilies,
and daisies in her gar
den.
*When you see and in a sentence correction question, make sure it is used
in a list (as in the aforementioned example), or as a coordinating
conjunction that combines two independent clauses. Look at the
following example:
X The language test administer
ed by the school
and touched on important differ -
ences between British English and American English.
This sentence is incorrect because the language test administer ed by the
school
is not an independent clause. Therefore,
and cannot combine
it with the rest of the sentence. Look at
the corrected version, which
changes the first
part of the sentence to an independent
clause by
including the verb was:
The language test was administer
ed by the school
and touched on important
differences between British English and American English.
Conjunctive Adverb/T ransition W ord
A word that introduces a relationship between two independent clauses and
connects them. These words are called conjunctive adverbs, transition
words, and connecting words. If the word is used in the middle of a sen
-
tence,
a
semicolon
is
generally
used
before
the
word.
Every
sentence
must
have
at
least
one
main
clause
(with
a
subject
and
verb),
but
many
sentences
will
have
two
or
more.
In
order
to
logically
combine
clauses
in
a
sentence,
conjunctive
adverbs
(or
transition
words)
are
necessary
to
show
the
rela
-
tionship
between
clauses.
Using
too
many
or
not
enough
conjunctive
1 5 9
501_03_121-234_501_master.qxd 4/29/13 12:32 PM Page 160
501 GMAT
®
Questions
adverbs can lead to errors. Some common conjunctive adverbs and their
uses are given here:
Contrast and Comparison
however, instead, nevertheless, otherwise, yet
On T uesdays, I play racquetball; otherwise, I would go with you.
Cause–Effect/Consequence
as a r
esult,
consequently
, hence, then, therefore, thus
On T uesdays, I play racquetball; therefore, I am unable to go to the meeting.
Addition
also, as well as , furthermore, in addition (to) , moreover
On T uesdays, I play racquetball; furthermore, I lift weights for one hour in
the mor ning.
*On the GMA
T sentence correction section, it
is important
that you use
the correct conjunctive adverb to connect
ideas. Choose a word that
logically fits into the sentence and sensibly connects the ideas.
Look
at the following
example:
X The childr
en ar
e not inter
ested in swimming;
therefore, swimming is their
favorite Olympic sport to watch.
The preceding sentence is incorrect because therefore does not logically
combine the two contrasting ideas. Look at the corrected version with a dif-
ferent, more logical conjunctive adverb.
The childr en ar e not inter ested in swimming; however, swimming is their
favorite Olympic sport to watch.
Dependent (Subordinate) Clause
A clause that does not express a complete sentence though it was wet
Monica walked on the grass,
though it was wet
.
1 6 0
501_03_121-234_501_master.qxd 4/29/13 12:32 PM Page 161
501 GMAT
®
Questions
Subordinating Conjunction
A word that makes a clause dependent: after, although, as, because, before, if,
once, since, than, that, though, unless, until, when, whenever, where, wherever,
while
The man wasn’ t angr y, though he had a right to be.
Though he had a right to be is a dependent clause because of though, a subor-
dinating conjunction.
Restrictive (Essential) Clause
A dependent clause that is necessary to the basic meaning of the completed
sentence. who ar e pr egnant
Women who ar e pr egnant can crave salty or sweet foods. (Notice the
absence of commas.)
Nonrestrictive (Nonessential) Clause
A dependent clause that is not necessary to the basic meaning of the com-
pleted sentence. who growls whenever the phone rings
Elmo, who growls whenever the phone rings , attacked the vacuum cleaner
.
(Notice the commas.)
Appositive
A
phrase
that
makes
a
preceding
noun
or
pronoun
clearer
or
more
definite
by explaining
or identifying it rice pudding and fr uit salad
Candice’s grandfather brought her favorite desserts, rice pudding and fr uit
salad.
Fragment
A
phrase
punctuated
like
a
sentence
even
though
it
does
not
express
a
com-
plete thought.
Timothy saw the car
.
And ran
.
1 6 1
501_03_121-234_501_master.qxd 4/29/13 12:32 PM Page 162
501 GMAT
®
Questions
Correct Syntax
To obtain correct syntax, the first step in an effective sentence, a sentence
must:
Always have at least one independent clause in the sentence.
Chaucer was a narrator .
Join two independent
clauses with a comma and a
conjunction.
Chaucer was a narrator , and he was a pilgrim in his Canterbury
Tales.
Not run two or more independent clauses together without
punctuation; that error is called a run-on.
Not separate two independent clauses with just a comma; that
error is called a comma splice .
X Chaucer was a narrator
, he was a pilgrim in his Canterbury T ales.
The preceding example is incorrect because only a comma is
separating the two independent clauses. A correct sentence
would be:
Chaucer was a narrator
,
and he was a pilgrim in his Canterbury
Tales.
Do not
use a
conjunctive adverb
(words such as
accordingly,
besides,
consequently,
furthermore,
hence,
however,
instead,
moreover,
nevertheless,
otherwise,
then,
therefore,
thus) like a
conjunction.
X Chaucer was a narrator
, mor
eover he was a pilgrim in his
Canterbury T ales.
The preceding example is incorrect because only a comma is
separating the conjunctive adverb. The corrected sentence
would be:
Chaucer was a narrator
; mor
eover, he was a pilgrim in his
Canterbury T
ales.
Semicolons
Semicolons are generally used in the following two circumstances:
Between two independent clauses
Edward joined the basketball team ; remarkably, the 5 ¢4¢¢ young
man excelled at the sport.
1 6 2
501_03_121-234_501_master.qxd 4/29/13 12:32 PM Page 163
501 GMAT
®
Questions
In this example, Edward joined the basketball team and the 5 ¢4¢¢
young man excelled at the sport are both independent clauses that
have been connected by the conjunctive adverb remarkably and
a semicolon.
Between elements in a series that
uses commas already
The possible dates for the potluck dinner ar
e Thursday
, June 5
;
Saturday, June 7
; or Monday
, June 9.
Effective
Sentence
Style
The
GMAT
sentence
correction
test
follows
some
general
standards
to
form
effective sentences. Review the following points and keep them in
mind as you tackle the questions.
Prefer the That Clause
The GMAT sentence correction test will often present sentences with a t
hat
clause.
Review
the
following
points
to
make
sure
you
understand
the
role
of the
that clause in a sentence.
Reporting V
erbs
Reporting
verbs
are
verbs
used
to
report
a
thought,
belief,
or
quote.
Com-
mon
reporting
verbs
include
argue
,
ask,
believe,
claim,
indicate,
report,
reply,
respond,
suggest,
and
tell.
It
is
important
to
use
that after
these
verbs
when
another verb follows. Look at
the following examples.
The studies indicated
that high school standar ds have risen in the
past ten years.
She claimed
that she didn’ t see the car approaching.
Subjunctive Clauses
You have already learned about the subjunctive form in the correct expres-
sion section. It is crucial that you include t
hat when
forming
a
sentence
with
a
subjunctive
verb.
This
small
difference
may
be
the
key
distinction
between
a right answer and a wrong one on the test.
X The state r
equires the school be pr
epared for an earthquake.
1 6 3
501_03_121-234_501_master.qxd 4/29/13 12:32 PM Page 164
501 GMAT
®
Questions
The preceding sentence is incorrect because it did not include that after the
verb requires. Look at the corrected version:
The state r equires that the school be pr epared for an earthquake.
Verbs Ar e Gr eater Than Nouns
Clarity is an important concept in the GMAT sentence correction section.
Many sentences on this test will be complicated, long ideas. When possi-
ble, a t
hat clause
is
preferred
over
a
noun
formation
to
achieve
a
clearer
sen-
tence. Look at the following example:
X The
idea
about
change
being
inevitable is a key message in the campaign song.
This example is considered incorrect because it is wordy. An easy way to
correct this sentence is to change the noun clause (about change being
inevitable) to a that clause with a verb.
The idea that change is inevitable is a key message in the campaign song.
In this particular example, the number of words does not change, but the
second format is preferred because it is more clear and active. Look at a few
more examples:
X Reports about the violence spr eading in the Middle East have caused unr est at
home.
Reports that violence is spr eading in the Middle East have caused unr est at
home.
X The discover y of other planets capable of supporting life has caused a stir in the
world of science.
The discover
y
that other planets ar
e capable of supporting life
has caused a stir
in the world of science.
Avoiding W ordiness
Wordiness is a big issue on the GMAT sentence correction test. In addition
to the aforementioned
points,
the
following
hints
will
help
you
ensure
you
choose
the
most
concise
sentence
every
time.
Remember,
though,
that
these
points
are
to
be
considered
in
addition
to
any
grammar
problems.
You
may
be
tempted
to
immediately
choose
the
sentence
that
fits
the
following
criteria.
However,
you
need
to
first
make
sure
there
are
no
grammar
or
clar
-
1 6 4
501_03_121-234_501_master.qxd 4/29/13 12:32 PM Page 165
501 GMAT
®
Questions
ity problems. The GMAT may try to trick you with a sentence that fits the
following criteria, but has a grammar problem. Grammar will always be
your first concern. If a sentence isn’t grammatically correct, there is no way
it can be considered a possible answer. With that in mind, review the fol
-
lowing points and keep them in mind as you tackle the questions.
Verbs Ar
e Gr
eater Than Nouns, Part II
A
sentence
may
include
an
action
noun,
a
noun
that
expresses
an
action.
Examples
include
arrival,
eating,
and
inspiration.
When
possible,
the
verb
form is preferred over the action noun. Look at
the following examples:
X His arrival
was an inspiration
to me.
This sentence is grammatically correct, but it could be more concise. Look
at the corrected, less wordy version:
His arrival inspired me.
X Their decision was
to continue the trip despite the bad weather for
ecast.
They decided
to continue the trip despite the bad weather for
ecast.
Verbs Ar
e Gr
eater Than Adjectives
When
possible,
try
to
use
a
verb
construction
over
an
adjective
construc-
tion for a less wordy sentence.
X His arrival
was inspirational to
me.
This
sentence
is
grammatically
correct,
but
it
could
be
more
concise.
Look
at
the corrected, less wordy version:
His arrival inspired me.
X The solution is irritating to the wound.
The solution irritates the wound.
Active V oice versus Passive V oice
You have already learned the difference between the active voice and pas-
sive voice in the correct expression section of this review. Although both
f
orms
are
grammatically
correct,
the
GMAT
will
generally
prefer
the
active
voice
to
the
passive
voice.
The
passive
voice
tends
to
make
sentences
longer
1 6 5
501_03_121-234_501_master.qxd 4/29/13 12:32 PM Page 166
501 GMAT
®
Questions
and not as clear. When you notice that the passive voice form makes the
sentence more confusing, look at any active voice choices (if they exist) and
consider them as possible answers (provided they are grammatically sound).
The
passive
voice
can
be
the
correct
answer.
When
the
focus
of
the
sen-
tence
is
on
the
person
or
object
receiving
the
action,
not
the
doer
of
the
action,
then the passive voice is preferred.
Look at the following example:
The terrorist attack
was witnessed
by people all over the world.
In
this passive voice example, the focus is on the terrorist attack, not the peo-
ple all over the world. The sentence is clear and concise. Consider this point
when you encounter a question that uses the passive voice.
Avoiding Redundancy
The GMAT sentence correction test will test your ability to detect redun-
dancy in a sentence. It may, however, disguise redundancy in different parts
of speech.
Look at the following examples and keep them
in mind.
X At least 300 workers, if not mor
e, participated in the strike on Friday
.
This
sentence
is
incorrect
because
at
least and
if
not
more convey
the
same
idea.
Therefore,
one
of
those
expressions
can
be
eliminated.
Look
at
a
cor-
rected version of the sentence:
At least 300 workers participated in the strike on Friday
.
X The doctors can potentially save the girl’
s life if they find a donor in time.
This
sentence
is
incorrect
because
can
potentially is
redundant.
Both
can and
potentially indicate
possibility,
so
only
one
of
them
is
necessary.
Look
at
a
corrected version of the sentence.
The doctors can save the girl’
s life if they find a donor in time.
Also,
avoid
double
comparatives
or
double
superlatives.
Adding
the
suffix
-er or
-est to
a
modifier
and
preceding
the
modifier
with
more or
most is
redundant.
X Lindsey amazed the class with her grammatical skills; she was the most
smartest person they had ever seen.
1 6 6
501_03_121-234_501_master.qxd 4/29/13 12:32 PM Page 167
501 GMAT
®
Questions
1 6 7
This sentence is incorrect. Lindsey is already the smartest. Most also means
smartest, so the phrase most smartest is redundant.
Lindsey amazed the class with her grammatical skills; she was the smartest
person they had ever seen.
Avoid double negatives.
X Tom har dly did not feel tense whenever he approached grammar .
This sentence is incorrect because hardly and did not cancel each other out.
The sentence really reads: Tom felt tense whenever approaching grammar. The
correct sentence should read:
Tom har dly felt tense whenever he approached grammar .
Beware of Eliminating T oo Much
Now that you are very aware of wordiness and redundancy, you must also
beware of common pitfalls that
can occur when you eliminate too much.
Prepositional Phrases
A prepositional phrase is a phrase that contains a preposition and a noun.
Simple phrases
such
as
on
the
bathroom
floor or
from
the
Dead
Sea are
prepo-
sitional
phrases.
These
will
pop
up
on
the
GMAT
sentence
correction
test.
In
an
effort
to
eliminate
wordiness,
you
may
be
tempted
to
combine
these
prepositional
phrases
with
the
nouns
they
modify,
such
as
in
the
following
example:
A planter of bricks
A brick planter
Both phrases are correct. Obviously
, the second example is less wordy
.
*In
general,
it
is
okay
to
combine
a
prepositional
phrase
with
its
noun
w
hen
the
preposition
is
of,
except
when
you
have
a
time
period,
quan-
tity, or other measurement as the initial word.
501_03_121-234_501_master.qxd 4/29/13 12:32 PM Page 168
501 GMAT
®
Questions
1 6 8
However, when the prepositional phrase involves a preposition other than
of, it is best to avoid combining the phrase with its noun. Look at the fol-
lowing examples:
gold from the San Francisco ar ea instead of San Francisco ar ea gold
dust on the bathroom floor instead of bathroom floor dust
the amount of water instead of the water amount
In these instances, it is best to keep the longer phrase to avoid confusion.
Avoiding Changes in Meaning
The GMAT sentence correction test may present two grammatically cor-
rect options, but one sentence has slightly changed the meaning of the orig-
inal sentence presented as choice a. You must not change the original
meaning. Look out for these common pitfalls:
Change in Modals
Modals such as may,
might,
must,
and
should exist
to
modify
a
verb.
I
might
go is
different
from
I
must
go,
which
is
different
from
I
should
go,
and
so
on.
Be
careful
that
a
modal
has
not
changed
and
altered
the
original
message
in a sentence!
Change in V
erbs
In
an
effort
to
reduce
wordiness,
you
may
be
tempted
to
change
a
verb
and
opt
for
one
that
expresses
an
action
with
fewer
words.
Make
sure
the
mean-
ing does not
change in this process. For example:
The idea developed into a fantastic plan.
The idea is a fantastic plan.
These
two
sentences
are
not
equivalent.
The
first
one
stresses
the
process
of
the
idea
changing
into
a
plan,
while
the
second
sentence
does
not
even
mention
it.
Make
sure
you
continue
to
convey
any
expressed
process
or
change in an improved sentence.
Change in Syntax (Placement of Modifiers)
Changing
the
placement
of
a
modifier
in
a
sentence
can
greatly
change
the
meaning
of
a
sentence.
When
you
review
the
choices
for
a
question,
pay
501_03_121-234_501_master.qxd 4/29/13 12:32 PM Page 169
501 GMAT
®
Questions
particular attention to any modifiers and whether their location in the sen-
tence has changed. This change could alter the meaning and help you
quickly eliminate that choice as a possible answer. Some of these changes
may be very subtle, and some may be more obvious. Look at the following
examples:
Just Heather was selected to participate in the sleep study
.
Heather was just selected to participate in the sleep study
.
The dentist, a r
ecent graduate from USC’
s School of Dentistr
y, selected his
partner for his new practice.
The dentist selected his partner
, a r
ecent graduate from USC’
s School of
Dentistry, for his new practice.
Logical Comparisons (Logical Predication)
Comparisons
must
make
sense
and
compare
similar
things.
The
GMAT
may
try
to
trick
you
by
testing
for
a
logical
comparison
and
conciseness.
Look at
the following example:
X Unlike Mark T
wain’s
The Adventures of Huckleberry Finn
, written in
the first person, Oscar W
ilde wrote his classic
The Picture of Dorian
Gray in the thir
d person.
Upon
first
look,
this
sentence
may
appear
correct.
It
is,
however,
making
an
illogical
comparison
by
associating
Mark
Twain’s The
Adventures
of
Huck-
leberry
Finn
(a
book)
with
Oscar
Wilde (an
author).
A
book
cannot
be
log-
ically
compared
to
an
author.
An
author
can
be
compared
to
another
author,
or
a
book
can
be
compared
to
another
book.
Look
at
a
corrected
version
of
the sentence:
Unlike Mark T
wain, who wrote
The Adventures of Huckleberry Finn
in the first person, Oscar W
ilde wrote his classic
The Picture of Dorian
Gray in the thir
d person.
This
sentence
logically
compares
the
two
authors.
Such
comparisons
are
a
favorite
topic
on
the
GMAT
sentence
correction
test,
so
pay
close
attention
when comparisons are made!
1 6 9
501_03_121-234_501_master.qxd 4/29/13 12:32 PM Page 170
501 GMAT
®
Questions
1 7 0
Diction (Word Choice)
Diction is a common issue on GMAT sentence correction questions. Some
of the most popular choices are
explained and highlighted with examples.
The GMAT sentence correction portion is also known to change words for
look-alikes in the different answer choices (e.g., practical for practicable). Be
sure that you read each choice carefully to make sure the meaning has not
changed.
Study
these
issues
and
keep
them
in
mind
as
you
tackle
the
sentence
cor-
rection questions.
According to versus In accordance with
According to is used to attribute information to a source:
According to
nutritionists, protein is a cr ucial part of our diet.
In accordance with is used to indicate compliance:
Every day I eat protein in accordance with my nutritionist’s instructions.
The correct use of in accordance with is restricted to following rules, conven-
tions, or established patterns. An actual rule or convention needs to be stated.
Amount of versus Number of
Amount of refers to an uncountable noun
(a noun that cannot be counted,
such as information)
The amount of information on the Internet is overwhelming.
Number of refers to a countable noun (a
noun that can be counted, such
as websites)
The number of websites dedicated to beauty products is overwhelming.
Because of versus Due to
Because of answers the question why? It is an adverbial phrase that modi-
fies a verb.
He won because of his persistence and drive.
501_03_121-234_501_master.qxd 4/29/13 12:32 PM Page 171
501 GMAT
®
Questions
1 7 1
Due to is an adjectival phrase that modifies a noun. It commonly follows a
form of the verb to be .
His win was due to his persistence and drive.
Between versus Among
Between is used when referring to two concepts, and before the word each.
He placed second between John and Rachel.
They have 10 minutes to r est between each event.
Among is used when referring to three or more concepts.
He divided his property among his four childr en.
Farther versus Further
Farther is the comparative form of far, which is used to refer to distance.
She drove farther today than she did yester day.
Further is also the comparative form of far, but it can be used to refer to
distances and figurative distance.
She took the idea one step further than what we had anticipated.
Few versus Little
Few is used to refer to a countable noun (a noun that can be counted, such
as websites)
There ar e few beauty websites that ar e not tr ying to sell a product.
Little is used to refer to an uncountable noun (a noun that cannot be
counted, such as information)
There is little information in this pamphlet that I consider tr ustworthy.
501_03_121-234_501_master.qxd 4/29/13 12:32 PM Page 172
501 GMAT
®
Questions
1 7 2
Fewer versus Less
Fewer is the comparative form of few, used to refer to a countable noun
(such as websites)
There ar e fewer websites in Spanish than ther e ar e in English.
Less is the comparative form of little, used to refer to an uncountable noun
(such as information)
There is less information on the Inter net in Spanish than ther e is in
English.
For + Gerund versus T o + V erb (Infinitive)
The for + gerund form is used to express the function of a thing or object.
This machine is used for counting coins.
The to + verb (infinitive) form is used to express the purpose of a thing/
object, but also of a person.
This machine is used to count coins.
She went to the stor e to buy eggs.
Like versus As
Like is used to compare only nouns, and therefore should be followed by
only a noun, pronoun, or noun phrase.
Like her sister , Alice loves to paint.
As is used to compare clauses, and therefore should be followed by a clause.
As her sister does, Alice loves to paint.
Like versus Such as
Like is used to express the idea similar to .
Like soccer, football is a ver y competitive sport.
501_03_121-234_501_master.qxd 4/29/13 12:32 PM Page 173
501 GMAT
®
Questions
1 7 3
Such as is used to list examples.
There ar e many competitive sports, such as football and soccer .
Much versus Many
Much is
used
to
describe an uncountable noun (a noun that cannot be
counted, such as information)
There is too much information on the Inter net.
Many is used to describe a countable noun (a noun that can be counted,
such as sources)
Joshua backed up his r esearch with many different sources.
Native of versus Native to
Native of is used only when referring to people.
Jacob was bor n and raised in W ashington, so he is a native of Washington.
Native to is used when referring to species, crops, and so on.
Corn is native to North America.
Rather than versus Instead of
Rather than shows preference, and can be followed by a clause.
They invested in land rather than machinery.
Rather than invest in machiner y, they invested in land.
Instead of s
hows replacement, but not necessarily preference. Also, it must
be followed by a noun or noun clause because of the preposition of. It can-
not be followed by a clause like rather than .
He drank coffee instead of tea.
They invested in land
instead of
machinery.
501_03_121-234_501_master.qxd 4/29/13 12:32 PM Page 174
501 GMAT
®
Questions
That versus Which
That is used in a restrictive or essential clause to introduce information that
is necessary to a sentence. It directly follows the noun that it modifies.
She avoids animals that appear dangerous. (In this example, she only
avoids those animals that appear dangerous, not all animals.)
Which is
used
in
a
nonrestrictive
or
nonessential
clause
to
introduce
addi-
tional
information that is not necessary to the meaning of a sentence. A
comma is used between the noun it modifies and which.
She avoids animals, which appear dangerous. (In this example, she
avoids all animals because they appear dangerous.)
Set 6
Now it is time to answer 41 GMAT Sentence Correction practice questions
that have been designed to test your effective expression skills. Since you
have already been exposed to the correct expression skills and questions,
you may encounter some concepts from the Correct Expression portion.
This has been done to prepare you for the real test, which will combine
concepts from both sections. Good luck!
134. Having been accused of polluting huge areas of the Ecuadorian
Amazon from its oil operations there in the 1970s and 1980s ,
Chevron is currently involved in the biggest environmental lawsuit
the world has ever seen.
a. Having been accused of polluting huge areas of the Ecuadorian
Amazon from its oil operations there in the 1970s and 1980s
b. Being accused of polluting huge Ecuadorian Amazon areas from
its oil operations there in the 1970s and 1980s
c. Accused of polluting huge areas of the Ecuadorian Amazon
from its oil operations there in the 1970s and 1980s
d. Having been accused of polluting huge Ecuadorian Amazon
areas from their oil operations there in the 1970s and 1980s
e. Due to being accused of polluting huge areas of the Ecuadorian
Amazon from its oil operations there in the 1970s and 1980s
1 7 4
501_03_121-234_501_master.qxd 4/29/13 12:32 PM Page 175
501 GMAT
®
Questions
135. Inspired by the success of Disneyland Paris, one French politician
wants to create France’ s very own theme park devoted to Napoleon
Bonaparte, with construction slated to begin in 2014 provided
enough funds are raised.
a. Inspired by the success of Disneyland Paris, one French politi-
cian wants to create France’s very own theme park devoted to
Napoleon Bonaparte, with construction slated to begin in 2014
provided enough funds are raised.
b. Inspired by the success of Disneyland Paris with construction
slated to begin in 2014, one French politician wants to create
France’s very own theme park devoted to Napoleon Bonaparte,
provided enough funds are raised.
c. One French politician wants to create France’s very own theme
park devoted to Napoleon Bonaparte, inspired by the success of
Disneyland Paris with construction slated to begin in 2014 pro
-
vided enough funds are raised.
d. One French politician, inspired by the success of Disneyland
Paris, wants to create France’
s very own theme park devoted to
Napoleon Bonaparte, with construction slated to begin,
pro
-
vided enough funds are raised, in 2014.
e. With construction slated to begin in 2014,
provided enough
funds are raised, one French politician wants to create France’s
very own theme park inspired by the success of Disneyland
Paris devoted to Napoleon Bonaparte.
1 7 5
501_03_121-234_501_master.qxd 4/29/13 12:32 PM Page 176
501 GMAT
®
Questions
136. Yellow saddle goatfish in the Red Sea have the capability to hunt in
groups, groups with designated members who spread out to cut off
the escape routes of the prey
.
a. Yellow saddle goatfish in the Red Sea have the capability to
hunt in groups, groups with designated members who spread
out to cut off the escape routes of the prey.
b. Red Sea yellow saddle goatfish are capable of hunting in groups,
groups with designated members who spread out to cut off the
prey’s escape routes.
c. Red Sea yellow saddle goatfish have the capability to hunt in
groups with designated chosen members who spread out
to cut
off the prey’s escape routes.
d. Yellow saddle goatfish in the Red Sea are capable of hunting in
groups with designated members who spread out to cut
off the
prey’s escape routes.
e. Red Sea yellow saddle goatfish possess the capability to hunt in
groups, groups with designated chosen members who spread
out to cut off the escape routes of the prey.
137. The hypothesis about
the coevolution of species, such as a
predator and prey , adapting to each other’ s evolutionary changes, is
fascinating.
a. The hypothesis about the coevolution of species, such as a pred-
ator and prey, adapting to each other’s evolutionary changes, is
fascinating.
b. The hypothesis that species, such as a predator and prey,
coevolve to adapt to each other’s evolutionary changes is
fascinating.
c. Coevolution, where species like a predator and prey adapt to
each other’
s evolutionary changes, is a hypothesis that is fasci-
nating.
d. The hypothesis regarding coevolution, like a predator and prey
adapting to each other’
s evolutionary changes, fascinates.
e. The hypothesis that species, like a predator and prey, coevolve
to adapt to each other’s evolutionary changes is fascinating.
1 7 6
501_03_121-234_501_master.qxd 4/29/13 12:32 PM Page 177
501 GMAT
®
Questions
138. The belief that whales were terrestrial mammals that transitioned
into the ocean is supported by evidence of whale fossils with legs
and knees.
a. The belief that whales were terrestrial mammals that transi-
tioned into the ocean is supported by evidence of whale fossils
with legs and knees.
b. The belief about whales being terrestrial mammals which tran-
sitioned into the ocean was supported by whale fossils having
legs and knees.
c. The belief about whales as terrestrial mammals transitioning
into the ocean have been supported by evidence of whale fossils
with legs and knees.
d. The belief that whales were terrestrial mammals that made the
transition into the ocean is supported by evidence of whale fos-
sils with legs and knees.
e. Whale fossils with legs and knees is evidence supporting the
belief of whales as once being terrestrial mammals.
139. A new study found
that the representation of global migrants on a
world scale is overpowered by Christians, who overrepresent a
high 49%.
a. that the representation of global migrants on a world scale is
overpowered by Christians, who overrepresent a high 49%.
b. that Christians overrepresent a high 49% of the world scale of
global migrants.
c. that Christians are overrepresented by an elevated 49% on the
world scale of global migrants.
d. that Christians represent an overwhelming 49% of global
migrants on a world scale.
e. that 49% of global migrants are represented overwhelmingly
high by Christians.
1 7 7
501_03_121-234_501_master.qxd 4/29/13 12:32 PM Page 178
501 GMAT
®
Questions
140. New workforce trends suggest that a large number of working
wives will outearn their husbands within a generation.
a. New workforce trends suggest that a large number of working
wives will outearn their husbands within a generation.
b. According to new workforce trends, a majority of working wives
will be outearning their husbands with greater wages within the
next generation.
c. New workforce trends within a generation suggest an outearn-
ing of a large number of working wives by their husbands.
d. Within a generation, a large number of husbands will be out-
earned by their working wives, according to new workforce
trends.
e. New workforce trends suggest an outearning of husbands by a
majority of their working wives within the next
generation.
141. An article revealed
one language dies every 14 days because of
communities abandoning native tongues in favor of more common
ones.
a. one language dies every 14 days because of communities aban-
doning native tongues in favor of more common ones.
b. that one language dies every 14 days due to communities aban-
doning native tongues in favor of more common ones.
c. that one language dies every 14 days because communities
abandon native tongues in favor of more common ones.
d. one language dies every 14 days because communities abandon
native tongues in favor of more common ones.
e. the death of one language every 14 days because communities
are favoring more common tongues.
1 7 8
501_03_121-234_501_master.qxd 4/29/13 12:32 PM Page 179
501 GMAT
®
Questions
142. With initial deployment scheduled in the San Joaquin and
Sacramento valleys, the Picarro is the first of many analyzers that is
part of a long-term effort of subjecting carbon dioxide, methane,
and other greenhouse gas emissions
to statewide scrutiny and
investigation.
a. many analyzers that is part of a long-term effort of subjecting
carbon dioxide, methane, and other greenhouse gas emissions
b. many analyzers, which is a part of a long-term effort to subject
how carbon dioxide, methane, and other greenhouse gases emit
c. many analyzers, part
of a long-term effort of subjecting how
carbon dioxide, methane, and other greenhouse gases are
emitting
d. many analyzers that are part
of an effort in the long term that
has subjected the carbon dioxide, methane, and other green-
house gas emissions
e. many analyzers that are part of a long-term effort to subject
carbon dioxide, methane,
and other greenhouse gas emissions
143. Scientists have observed large cracks that go on for miles in
gigantic Antarctica icebergs,
which are consistent
with the
predictions of global warming.
a. gigantic Antarctica icebergs, which are consistent with the pre-
dictions of global warming.
b. gigantic icebergs in Antarctica, findings consistent with the pre-
dictions of global warming.
c. gigantic icebergs in Antarctica, consistent with the predictions
of global warming.
d. gigantic Antarctica icebergs, where the predictions of global
warming agree with these findings.
e. gigantic Antarctica icebergs, findings consistent with the global
warming predictions.
1 7 9
501_03_121-234_501_master.qxd 4/29/13 12:32 PM Page 180
501 GMAT
®
Questions
144.
However ten European nations have approved the installation of
cobblestones bearing the names of Nazi victims peppered
throughout their cities, regardless some municipalities argue that
enough
exist and the idea is inappropriate.
a. However ten European nations have approved the installation
of cobblestones bearing the names of Nazi victims peppered
throughout their cities, regardless some municipalities argue
that enough
b. In spite of ten European nations having approved installing
cobblestones bearing Nazi victims’ names peppered throughout
their cities, regardless some municipalities argue that enough
Holocaust memorials
c. T
en European nations have approved the cobblestone installa-
tion bearing the names of Nazi victims peppered throughout
their cities, but some municipalities argue that
enough
d. Although ten European nations have approved the installation
of cobblestones bearing the names of Nazi victims to be pep-
pered throughout their cities,
some municipalities argue that
enough Holocaust memorials
e. Despite ten European nations approving cobblestone installa-
tion bearing Nazi victims’ names peppered throughout their
cities, some municipalities argue that enough Holocaust
memorials
145.
An entrepreneur seeking a loan from a financial institution will
find it difficult to persuade a lender if there is a lacking concrete
business plan to prove his or her potential for success.
a. if there is a lacking concrete business plan to prove
b. unless there will be a concrete business plan to prove
c. if there is an absence of a concrete business plan to prove
d. without a concrete business plan that proves
e. in the case there is not a business plan proving
1 8 0
501_03_121-234_501_master.qxd 4/29/13 12:32 PM Page 181
501 GMAT
®
Questions
146.
Very little has the ability to grow in the Andean highlands of Peru
due to the thin air, scarce water supply, and rocky terrain.
a. has the ability to grow in the Andean highlands of Peru due to
b. is able of growing in the Andean highlands of Peru caused by
c. can grow in the Andean highlands of Peru because
d. can grow in the Andean highlands of Peru due to
e. can grow in the Andean highlands of Peru because of
147.
New shifts in the organization of Mexico’s hydrocarbon industry
are expected to build its increasing competitiveness in the market
of international energy.
a. the organization of Mexico’s hydrocarbon industry are expected
to build its increasing competitiveness in the market of interna-
tional energy.
b. Mexico’s hydrocarbon industry organization are expected to
build its competitiveness in the international energy market.
c. the organization of Mexico’s hydrocarbon industry are expected
to increase its competitiveness in the international energy
market.
d. Mexico’s hydrocarbon industry organization predict growth in
its surging competitiveness within the international energy
market.
e. the organization of Mexico’
s hydrocarbon industry are pre-
dicted to increase its surging competitiveness in the interna-
tional energy market.
148.
The mountain climber’s decision to abandon his friend on the
slope to find help would change the lives of both men.
a. mountain climber’s decision to abandon his friend on the slope
to find help would change the lives of both men.
b. decision from the mountain climber that led to him abandoning
his friend on the slope to find help would change both men’s
lives.
c. decision made by the mountain climber on the slope of aban-
doning his friend to find help would be life-changing for both
men.
d. mountain climber’s decision of abandoning his friend on the
slope to find help would change the lives of both men.
e. mountain climber’s decision to abandon his friend to find help
on the slope would change the lives of both men.
1 8 1
501_03_121-234_501_master.qxd 4/29/13 12:32 PM Page 182
501 GMAT
®
Questions
149.
The UN’s High Commission for Refugees (UNHCR) indicates
that while there was a slight decrease in the number of refugees at
the end of 2009,
the number of internally displaced people within
their own countries rose up to 27.5 million people at the end of
2010.
a. the number of internally displaced people within their own
countries rose up to 27.5 million people at the end of 2010.
b. the number of internally displaced people rose to 27.5 million
people at the end of 2010.
c. the number of internally displaced people increased to an
amplified 27.5 million people at
the end of 2010.
d. the number of displaced people within their own countries rose
to 27.5 million people at the end of 2010.
e. the number of internally displayed people increased to 27.5 mil-
lion people at the end of 2010.
150.
Unlike the Washington Monument, the structure of the San
Jacinto Memorial Monument is not comprised entirely of stone, in
spite of being taller.
a. the structure of the San Jacinto Memorial Monument is not
comprised entirely of stone, in spite of being taller.
b. the San Jacinto Memorial Monument’s structure is not com-
prised entirely of stone, despite being taller.
c. the San Jacinto Memorial Monument
is not comprised entirely
of stone, although it is taller.
d. the structure of the San Jacinto Memorial Monument is not
comprised entirely of stone, although it is taller.
e. the San Jacinto Memorial Monument is not comprised entirely
of stone, in addition to being taller.
1 8 2
501_03_121-234_501_master.qxd 4/29/13 12:32 PM Page 183
501 GMAT
®
Questions
151.
Revolutionary 3-D printers are evidence that far-fetched Star Trek
ideas have become a reality, as users are allowed to create physical
items from digital blueprints.
a. Revolutionary 3-D printers are evidence that far-fetched Star
Trek ideas have become a reality, as users are allowed to create
physical items from
digital blueprints.
b. Revolutionary 3-D printers, which allow users to create physical
items from digital blueprints, are evidence of far-fetched Star
Trek
ideas being a modern reality.
c. The idea that far-fetched Star Trek ideas have become a reality
is supported by the invention of revolutionary 3-D printers,
which allow users to create physical items from digital blue
-
prints.
d. Proving that far-fetched Star Trek ideas have become a reality,
revolutionary 3-D printers allow users to create physical items
from digital blueprints.
e. As proof of far-fetched Star Trek ideas being a reality, revolu-
tionary 3-D printers allow users to create physical items from
digital blueprints.
152.
According to the American Red Cross, approximately 300,000
homes were damaged or destroyed by Hurricane Katrina, the
worst natural disaster in United States history.
a. approximately 300,000 homes were damaged or destroyed by
Hurricane Katrina, the worst natural disaster in United States
history.
b. Hurricane Katrina, the worst natural disaster in United States
history, was responsible for the damage and destruction of
approximately 300,000 homes.
c. approximately 300,000 homes were damaged or destroyed by
Hurricane Katrina, which is considered as the worst natural dis-
aster in United States history.
d. approximately 300,000 homes were damaged and destroyed
thanks to Hurricane Katrina, considered the worst natural dis-
aster in United States history.
e. Hurricane Katrina, the worst natural disaster in United States
history
, was responsible for damaging or destroying exactly
300,000 homes.
1 8 3
501_03_121-234_501_master.qxd 4/29/13 12:32 PM Page 184
501 GMAT
®
Questions
153.
In contrast to older automobiles that primarily used steel for its
durability and cost, carmakers are now turning to aluminum to
meet new fuel efficiency standards, remaining competitive in the
ever-changing market.
a. carmakers are now turning to aluminum to meet new fuel effi-
ciency standards, remaining
b. aluminum
is now being used by carmakers to meet new fuel
efficiency standards, thereby remaining
c. newer automobiles are now using aluminum to meet
new fuel
efficiency standards and remain
d. newer automobiles are now using aluminum to meet new fuel
efficiency standards, remaining
e. new fuel efficiency standards have forced carmakers to turn to
aluminum and remain
154.
China’s population control methods, which included a one-child
policy imposed in 1979, helped slow the fast annual rate of
population growth per year, though
many denounced the policy as
an abuse of human rights.
a. which included a one-child policy imposed in 1979, helped slow
the fast annual rate of population growth per year
, though
b. which included a one-child policy imposed in 1979, helped slow
the high annual rate of population growth, though
c. which included a one-child policy imposed in 1979, helped slow
the fast rate of population growth per year, in addition
d. which included a one-child policy imposed in 1979, helped
decrease the quick annual rate of population growth, moreover
e. which included a one-child policy imposed in 1979, helped slow
the high annual rate of population growth per year; however
,
1 8 4
501_03_121-234_501_master.qxd 4/29/13 12:32 PM Page 185
501 GMAT
®
Questions
155.
Tasmanian devils, native of the island of Tasmania close to
Australia, are endangered because of a cancer called Devil Facial
Tumor Disease.
a. native of the island of Tasmania close to Australia, are endan-
gered because of a cancer called Devil Facial Tumor Disease.
b. native of the island of Tasmania close to Australia, are endan-
gered because of a Devil Facial Tumor Disease cancer.
c. native of the Tasmania Island close to Australia, are endangered
due to a cancer called Devil Facial T
umor Disease.
d. native to the island of Tasmania close to Australia, are endan-
gered due to a Devil Facial Tumor Disease cancer.
e. native to the island of T
asmania close to Australia, are endan-
gered because of a cancer called Devil Facial Tumor Disease.
156.
Instead of viewing their money as a way to alleviate poverty,
microfinance is now seen by some donors as primarily an
investment opportunity.
a. Instead of viewing their money as a way to alleviate poverty,
microfinance is now seen by some donors as primarily an
investment opportunity.
b. Instead of viewing their money as a way to alleviate poverty,
microfinance is now currently being seen by some donors as
primarily an investment opportunity.
c. Rather than view their money as a way to alleviate poverty,
some donors are now primarily seeing microfinance as an
investment opportunity in today’s world.
d. Rather than view their money as a way to alleviate poverty
,
some donors now see microfinance as primarily an investment
opportunity.
e. Instead of view their money as a way to alleviate poverty, some
donors now see microfinance as an investment opportunity in
today’s world.
1 8 5
501_03_121-234_501_master.qxd 4/29/13 12:32 PM Page 186
501 GMAT
®
Questions
157.
Experts have differences over how tarantulas make silk come out of
their feet for maintaining a grip on steep, slippery surfaces.
a. have differences over how tarantulas make silk come out of
their feet for maintaining a grip on
b. have differing opinions as to how tarantulas make silk come out
of their feet for maintaining a grip on
c. differ as to how tarantulas are able to create silk with their feet
in order to maintain a grip on
d. differ over how tarantulas emit silk from their feet in order to
maintain a grip on
e. disagree over the abilities of tarantulas to emit silk from their
feet for maintaining a grip on
158.
The federal legislation mandated the right to have their leases
honored to tenants in the event of a foreclosure of the land being
rented.
a. The federal legislation mandated the right to have their leases
honored for tenants in the event of a foreclosure of the property
being rented.
b. Tenant leases were required to be honored in the case of a fore-
closure of the property the tenant is renting, according to fed-
eral legislation.
c. The federal legislation made mandatory for tenants to have
their leases honored in the event of a foreclosed rental property
.
d. The federal legislation made mandatory that existing tenant
leases have to be honored if a rental property is foreclosed.
e. The federal legislation mandated that tenant leases be honored
in the event
of a foreclosed rental property.
1 8 6
501_03_121-234_501_master.qxd 4/29/13 12:32 PM Page 187
501 GMAT
®
Questions
159.
Enthusiastic about the upcoming reception lunch approaching,
Mrs. Brown hastily phoned her committee members to assign tasks
and delegate responsibilities.
a. Enthusiastic about the upcoming reception lunch approaching,
Mrs. Brown hastily phoned her committee members to
b. Being enthusiastic about the upcoming reception lunch
approaching, Mrs. Brown phoned her committee members in a
hastily manner to
c. Feeling enthusiastic about the upcoming reception lunch, Mrs.
Brown hastily phoned her committee members to
d. Enthusiastic with regards to the reception lunch approaching,
the committee members were hastily phoned to
e. Enthusiastic about the upcoming reception lunch, Mrs. Brown
hastily phoned her committee members in a rushed effort to
160.
In an effort to understand climate change and rising sea levels,
scientists studied and researched samples of coral and discovered
that the ocean level rose by an increase of 46 to 60 feet 14,650
years ago.
a. scientists studied and researched samples of coral and discov-
ered that the ocean level rose by an increase of 46 to 60 feet
14,650 years ago.
b. scientists studied samples of coral and discovered that the ocean
level increased 46 to 60 feet 14,650 years ago.
c. scientists studied coral samples, discovering that the growth of
the ocean level went
up 46 to 60 feet 14,650 years ago.
d. scientists studied samples of coral, thereby discovering the
rise of the ocean level by an increased 46 to 60 feet 14,650
years ago.
e. samples of coral were studied and researched by scientists, who
discovered that the ocean level rose by 46 to 60 more feet
14,650 years ago.
1 8 7
501_03_121-234_501_master.qxd 4/29/13 12:32 PM Page 188
501 GMAT
®
Questions
161.
It is possible that more than half of the babies in the wealthiest
nations of today may reach to live to 100 years, with greater
chances increasing if they’re wealthy or slim.
a. more than half of the babies in the wealthiest nations of today
may reach to live to 100 years, with greater chances increasing
if they’re wealthy or slim.
b. more than half of today’s wealthiest nations’ babies, if they’re
wealthy or slim, will live to be 100 years old.
c. more than half of the babies in today’s wealthiest nations will
live to 100 years, with greater chances if they’re wealthy or slim.
d. with greater chances increasing if he or she is wealthy or slim,
more than half of the babies who are from the wealthiest
nations of today might live to 100 years.
e. more than half of today’s wealthiest nations’ babies may live to
be 100 years old if they’re wealthy or slim.
162. Efforts to stabilize the political and social climate of Colombia
have resulted in
large-scale rescues of kidnapping victims,
internationally coordinated arrests of drug traffickers, pivotal
captures of left-wing terrorist
groups, and renewed confidence in
the nation’
s politicians.
a. large-scale rescues of kidnapping victims, internationally coordi-
nated arrests of drug traffickers, pivotal captures of left-wing ter-
rorist groups, and renewed confidence in the nation’s politicians.
b. sweeping rescuing of kidnapping victims, internationally coor-
dinated arrests of drug traffickers, pivotal capturing of left-wing
terrorist
groups, and a renewed sense of confidence in the
nation’s politicians.
c. sweeping rescuing of kidnapping victims, internationally coor-
dinated arresting of drug traffickers, pivotal capturing of left-
wing terrorist groups, and renewed feelings of confidence in the
nation’s politicians.
d. large-scale rescues of kidnapping victims, internationally coor-
dinated arrests of drug traffickers, pivotal captures of terrorist
groups who are progressive in their ideals, and renewed confi-
dence in the nation’s politicians.
e. sweeping rescues of kidnapping victims, internationally coordi-
nated arrests of drug traffickers, pivotal captures of idealistically
progressive terrorist groups, and renovated confidence in the
nation’
s politicians.
1 8 8
501_03_121-234_501_master.qxd 4/29/13 12:32 PM Page 189
501 GMAT
®
Questions
163. A judge in New Y ork recently ruled that the 613 victims of one of
the most noteworthy Ponzi schemes should be in total paid out a
sum of $6.2 million, coming from personal and business assets that
were seized after an undercover investigation that was directed by
the U.S. Postal Service and the IRS.
a. that the 613 victims of one of the most noteworthy Ponzi
schemes should be in total paid out a sum of $6.2 million, com-
ing from personal and business assets that were seized after an
undercover investigation that was directed by the U.S. Postal
Service and the IRS.
b. that the 613 victims of one of the most noteworthy Ponzi
schemes be paid out
a total of $6.2 million, which was seized in
personal and business assets following an undercover investiga
-
tion by the U.S. Postal Service and the IRS.
c. that the state must pay out $6.2 million to the 613 victims of
one of the most noteworthy Ponzi schemes, with the money
having come from the sale of personal and business assets,
which were seized after an undercover investigation that was
directed by the U.S. Postal Service and the IRS.
d. that the 613 victims of one of the most
noteworthy Ponzi
schemes ought to be paid out a total of $6.2 million, which were
from selling the personal and business assets that were seized
following an undercover investigation led by the U.S. Postal
Service and the IRS.
e. that the state must pay out to the 613 victims of one of the most
noteworthy Ponzi schemes a total of $6.2 million, derived from
the sale of personal and business assets that were seized follow
-
ing a joint U.S. Postal Service and IRS investigation.
1 8 9
501_03_121-234_501_master.qxd 4/29/13 12:32 PM Page 190
1 9 0
501 GMAT
®
Questions
164.
A harsh new tobacco law in Australia has encountered massive
resistance from big tobacco companies that argue the new rules
abuse intellectual property rights and diminish the value of their
trademarks.
a. argue the new rules abuse intellectual property rights and
diminish the value of their trademarks.
b. argue on the grounds of intellectual property rights being vio-
lated or trademark values being diminished by the new rules.
c. argue that the new rules intellectually abuse property rights and
devalue their trademarks.
d. argue that the new rules abuse intellectual property rights and
devalue their trademarks.
e. argue that the new rules abuse property rights intellectually or
devalue their trademarks.
165.
Video game enthusiasts emphasize that a handful of deaths from
extended gaming are an inadequate means of predicting long-term
effects of video gaming and underscore that experts remain divided
on whether the overall health of gamers will suffer in the long
term and what impact their compromised health would have on
society.
a. on whether the overall health of gamers will suffer in the long
term and what impact their compromised health would have on
society.
b. on whether compromised health will be suffered among gamers
in the long term and what impact society would feel.
c. if gamers’ health will suffer in the long term or not and what
impact their compromised health would have on society
.
d. over whether gamers will become not as healthy or the impact
their compromised health will have on society.
e. as to whether fewer gamers will be healthy or the impact their
health would have on society if they were.
501_03_121-234_501_master.qxd 4/29/13 12:32 PM Page 191
501 GMAT
®
Questions
166.
Alaska is feeling the impact of global warming more than its fellow
states as average temperatures in Alaska have risen greater than
twice the national average, according
to data released by the U.S.
Global Change Research Program that was compiled over the past
50 years.
a. greater than twice the national average, according to data
released by the U.S. Global Change Research Program that was
compiled over the past
50 years.
b. over twice the national average, in keeping with data compiled
during the past 50 years released by the U.S. Global Change
Research Program.
c. more than twice the national average, in accordance with data
compiled over the past 50 years and released by the U.S. Global
Change Research Program.
d. more than twice the national average, according to 50-year
-old
data released by the U.S. Global Change Research Program.
e. more than twice the national average, according to data com-
piled over the past 50 years and released by the U.S. Global
Change Research Program.
167.
One of the most pressing concerns facing lawyers who have just
graduated from law school is the amount of fellow graduates who
currently outnumber the amount of available job openings.
a. is the amount of fellow graduates who currently outnumber the
amount of available job openings.
b. is the amount of fellow graduates that currently outnumbers the
amount of available job openings.
c. is the number of fellow graduates that currently exceeds the
number of job openings.
d. is the number of fellow graduates who currently exceed the
number of available job openings.
e. is the number of fellow graduates that currently exceed a num-
ber of job openings.
1 9 1
501_03_121-234_501_master.qxd 4/29/13 12:32 PM Page 192
501 GMAT
®
Questions
168.
Since its opening in 1869, the Suez Canal has made transportation
by water among Europe and Asia easier by providing a connection
between the Mediterranean Sea and the Red Sea.
a. the Suez Canal has made transportation by water among
Europe and Asia easier by providing a connection between the
Mediterranean Sea and the Red Sea.
b. the Suez Canal has eased transportation between Europe and
Asia by water to a greater degree by providing
a connection
between the Mediterranean Sea and the Red Sea.
c. the Suez Canal has facilitated maritime transportation between
Europe and Asia by connecting the Mediterranean Sea and the
Red Sea.
d. transportation between Europe and Asia has been facilitated by
the Suez Canal, which connects the Mediterranean Sea to the
Red Sea.
e. Europe and Asia transportation has been relieved thanks to the
Suez Canal, a connection between the Mediterranean Sea and
the Red Sea.
1 9 2
501_03_121-234_501_master.qxd 4/29/13 12:32 PM Page 193
501 GMAT
®
Questions
169.
The success of sailboat racing as a television sport largely depends
on the technology in the helicopters that fly above to a great deal,
technology so advanced that it makes the insertion of a first-down
line during the broadcast of a football game seem insignificant.
a. largely depends on the technology in the helicopters that fly
above to a great deal, technology so advanced that it makes the
insertion of a first-down line during the broadcast of a football
game seem insignificant.
b. to a great
deal depends on the helicopter technology that flies
above, technology so advanced that it
makes the insertion of a
first-down line during the broadcast of a football game seem
insignificant.
c. to a great deal depends on the helicopter technology that fly
above, technology so advanced that it
makes the first-down line
during a football game seem insignificant.
d. largely depends on the technology in the helicopters that fly
above, technology so advanced that it makes the insertion of a
first-down line during
a football game broadcast seem
insignificant.
e. largely depends on the advanced technology in the helicopters
flying above that make the insertion of a first-down line during
a football game broadcast seem insignificant.
1 9 3
501_03_121-234_501_master.qxd 4/29/13 12:32 PM Page 194
501 GMAT
®
Questions
170.
Hans Krasa had only begun to emerge as a celebrated composer,
having written the short fairy tale opera Brundibar, when the grips
of Nazism took their hold and rendered his additional developing
as a composer
impossible.
a. Krasa had only begun to emerge as a celebrated composer, hav-
ing written the short fairy tale opera Brundibar, when the grips
of Nazism took their hold and rendered his additional develop-
ing as a composer
b. Krasa had only hardly begun to emerge as a celebrated com-
poser, having written a short fairy tale opera by the name of
Br
undibar
, when the grips of Nazism took their hold and ren-
dered his farther professional development
c. Krasa had not hardly begun to emerge as a celebrated com-
poser, having written a short fairy tale opera by the name of
Br
undibar
, when the grips of Nazism took their hold and ren-
dered his developing into a better composer
d. Krasa who wrote the short fairy tale opera Brundibar had not
hardly begun to emerge as a celebratory composer when he was
taken hold by the grips of Nazism and his further professional
development
was rendered
e. Krasa,
who wrote the short fairy tale opera Brundibar, had only
begun to emerge as a celebrated composer when the grips of
Nazism took their hold and rendered his further professional
development
1 9 4
501_03_121-234_501_master.qxd 4/29/13 12:32 PM Page 195
501 GMAT
®
Questions
171.
In order to transfer media across a home from a computer to a
television, three items are needed: a media source, or server, that
contains all the media like pictures, music, and videos, a receiver
that supports the DLNA standard, which new “smart” TVs often
do; and a translator, which will convert
files to a readable format.
a. that contains all the media like pictures, music, and videos, a
receiver that supports the DLNA standard, which new “smart”
TVs often do; and a translator,
which will convert files to a
readable format.
b. that contains all the media such as pictures, music, and videos; a
receiver that supports the DLNA standard, which new “smart”
TVs often do; and a translator,
which will convert files to a
readable format.
c. which contains all the media like pictures, music, and videos, a
receiver that supports the DLNA standard, which new “smart”
TVs often do; and a translator,
which will convert files to a
readable format.
d. which contains all the media like pictures, music, and videos; a
receiver that supports the DLNA standard, that new “smart”
TVs often do; and a translator,
which will convert files to a
readable format.
e. that contains all the media such as pictures, music, and videos, a
receiver that supports the DLNA standard, which new “smart”
TVs do, and a translator, which will convert
files to a readable
format.
1 9 5
501_03_121-234_501_master.qxd 4/29/13 12:32 PM Page 196
501 GMAT
®
Questions
172.
Certain flea control methods can become ineffective if used
repeatedly on an animal; one reason is suggested by evidence that
fleas, like cockroaches, adapt to their environment and become
increasingly immune to popular flea medicine with each
generation.
a. Certain flea control methods can become ineffective if used
repeatedly on an animal; one reason is suggested by evidence
that fleas, like cockroaches, adapt to their environment
and
become increasingly immune to popular flea medicine with
each generation.
b. If used repeatedly on an animal, one reason that certain flea
control methods can become ineffective is suggested by evi-
dence that fleas, like cockroaches, increasingly adapt to their
environment
and become more immune to popular flea medi-
cine with each generation.
c. If used repeatedly on an animal, one reason certain flea control
methods can become ineffective is suggested by evidence that
there are fleas, like cockroaches, that
increasingly adapt
to their
environment and become more immune to popular flea medi
-
cine with each generation.
d. Certain flea control methods can become ineffective if they are
used repeatedly on an animal;
one reason, which derives from
evidence, suggests that fleas, like cockroaches,
adapt to their
environment and become immune to increasingly popular flea
medicine with each generation.
e. The evidence that fleas, like cockroaches, adapt to their envi-
ronment and become increasingly immune to popular flea med-
icine with each generation is suggestive of one reason, if used
repeatedly on an animal, certain flea control methods can
become ineffective.
1 9 6
501_03_121-234_501_master.qxd 4/29/13 12:32 PM Page 197
501 GMAT
®
Questions
173.
Oscar Arias, who won the Nobel Peace Prize for these efforts in
1987, submitted a peace plan for a series of disputes which led to
the Esquipulas Peace Agreement among multiple nations of
Central America, namely El Salvador, Guatemala, Nicaragua,
Honduras, and Panama, at a time when various armed conflicts
were threatening the stability of the Central American region.
a. Oscar Arias, who won the Nobel Peace Prize for these efforts in
1987, submitted a peace plan for a series of disputes which led
to the Esquipulas Peace Agreement among multiple nations of
Central America, namely El Salvador,
Guatemala, Nicaragua,
Honduras, and Panama, at a time when various armed conflicts
were threatening the stability of the Central American region.
b. Oscar Arias submitted a peace plan, which was for a series of
disputes leading to the Esquipulas Peace Agreement among
multiple nations of Central America, namely El Salvador,
Guatemala, Nicaragua, Honduras, and Panama,
at a time when
various armed conflicts were threatening the stability of the
Central American region, earning him the Nobel Peace Prize in
1987.
c. Being the receptor of the Nobel Peace Prize for his efforts in
1987, Oscar Arias submitted a peace plan for a series of various
armed conflicts that were threatening the stability of the Cen
-
tral American region which led to the Esquipulas Peace Agree-
ment between El Salvador
, Guatemala, Nicaragua, Honduras,
and Panama.
d. At a time when various armed conflicts were threatening the
stability of the Central American region, Oscar Arias submitted
a peace plan which led to the Esquipulas Peace Agreement
being signed between El Salvador, Guatemala, Nicaragua,
Hon
-
duras, and Panama; his contribution earned him the Nobel
Peace Prize in 1987.
e. At a time when various armed conflicts were threatening the
stability of the Central American region, Oscar Arias submitted
a peace plan that led to the Esquipulas Peace Agreement among
El Salvador, Guatemala, Nicaragua, Honduras, and Panama; his
contribution earned him the Nobel Peace Prize in 1987.
1 9 7
501_03_121-234_501_master.qxd 4/29/13 12:32 PM Page 198
501 GMAT
®
Questions
174.
Studies over the past decade indicate people working at home or
outside the office as tending to work longer hours, contrary to
employers’ initial fears.
a. indicate people working at home or outside the office as tend-
ing to work longer hours, contrary to
b. indicate people who work at home or outside the office as tend-
ing to work longer hours, in contradiction of
c. indicate that people who work at home or outside the office
tend to work longer hours,
contrary to
d. have indicated longer hours of work by people who work at
home or outside the office, contrary to
e. indicate that home-working people tend to work longer hours,
contrary to
1 9 8
501_03_121-234_501_master.qxd 4/29/13 12:32 PM Page 199
501 GMAT
®
Questions
Answers
Set 5
93. a. The verb consume is correct in the plural form to agree with the
plural subject the most successful nations in the world. Choice b is
incorrect because it conjugates the verb as consumes, which is only
correct if the subject is singular. Choice c also incorrectly
conjugates the verb as consumes to agree with a singular subject,
and it changes the simple present verb tense are to the present
progressive are being, which is incorrect for a general statement
that uses the verb to be. Choice d changes the verb tense are to have
been, which changes the original idea and implies the most
successful nations have already been home to the world’s poorest
people for a certain amount of time. This information cannot be
assumed. Choice e is incorrect because the phrase not only is
consuming does not agree with the plural subject the most successful
nations in the world.
94. c. The past tense set maintains a parallel structure among all the
verbs in the sentence (ruled, curtailed, set) and makes choice c the
clearest sentence. Choice a is incorrect because setting is the
present progressive tense. The sentence already uses curtailed in
the simple past, so it is best to keep this verb in the simple past as
well. Remember, parallelism is preferred. Choice b is incorrect for
the same reason; having set is not the best verb tense to maintain
unity and a parallel structure among all the actions presented.
Choice d is incorrect because it represents the future tense; this
sentence refers to actions that already happened, so the future
tense is wrong. Choice e is incorrect because it is an independent
clause. An independent clause cannot be joined to another
independent clause with just a comma. This is comma splicing and
it is incorrect.
1 9 9
501_03_121-234_501_master.qxd 4/29/13 12:32 PM Page 200
501 GMAT
®
Questions
95. d. Although public is a collective noun referring to many people, it
is singular and so the pronoun it must be used. Is, not are, is
correct because it refers to one of the most important questions, which
is singular. Choice a is incorrect because it uses they to refer to the
singular noun public. Choice b is incorrect because it uses the
plural verb are for the singular subject one of the most important
questions; it also uses the plural pronoun they to represent the
singular noun public. Choice c is incorrect because it also uses the
plural pronoun they to refer to public. Choice e is incorrect because
it uses are as the verb for the singular subject one of the most
important questions.
96. e. In order to make a logical comparison, two similar ideas must be
compared. In this sentence, the views of China are compared with
the views (those) of the United States. Choice a is incorrect because
it compares China’s views with the United States. A country’s views
cannot be logically compared with a country. Choice b is incorrect
because it uses the singular that to refer to the plural subject views.
Choice c is incorrect because it also uses the singular that to refer
to views. Choice d is incorrect because the ideas compared (the
views of China compared to the views from the United States) are not
parallel.
97. b. Choice b maintains a parallel structure by keeping all the verbs
in the gerund form; this makes for an easy and clear read. Choice a
is incorrect because the work of the scientists is not presented in a
parallel form: preserving is in the gerund form, their regeneration is a
noun, and ways to extract is also a noun. Choice c is incorrect
because the three aspects are not structured in a parallel form:
working is in the gerund form, but regenerate and extract are in their
infinitive forms. Choice d is incorrect because it adds the adjective
diligent, which would be considered an altered meaning; this also is
incorrect because it modifies the verb working and needs to be in
adverb form as diligently. Also, the possessive pronoun its to define
medicinal properties is incorrect because it is singular to refer to the
plural noun the world’s most threatened plant species. Choice e is
incorrect because it uses the comparative form more instead of the
most when describing the threatened plant species. The sentence
refers to many different plant species, so the superlative form the
most must be used.
2 0 0
501_03_121-234_501_master.qxd 4/29/13 12:33 PM Page 201
501 GMAT
®
Questions
98. e. The apostrophe is correctly omitted from Jr. since the
placement of of before Martin already indicates possession. The
verbs insisted and had are correctly formed in the past tense. Choice
a is incorrect because of Martin Luther King, Jr.s is redundant: It
has both of and the apostrophe to indicate possession. Of Martin
Luther King, Jr. or Martin Luther King, Jr.’s is correct, but not a
combination of the two forms. Also, in choice a, the verb tense had
had is incorrect because it implies that Martin Luther King, Jr. had
the makings of a great leader before the boycott and before his
contemporaries insisted about it, but not during (see Correct
Expression notes on the past perfect tense). This is illogical since
he had the makings of a great leader at the time of the boycott and
when his contemporaries insisted. Choice b is incorrect because it
uses the apostrophe in Jr.’s and because it combines both the past
progressive and simple present tense to describe two events that
happened in the past. Choice c is incorrect because it incorrectly
uses the apostrophe in Jr.’s. Choice d is incorrect because has is in
the simple present form. It must be in the simple past.
99. a. The information after the comma adequately expands on the
type of information; it is an appositive. Choice b is incorrect
because the prepositional phrase with information . . . has no clear
phrase or idea to attach to. Choice c would form two independent
clauses separated by a comma. This would be a comma splice,
which is incorrect. Choice d, without the comma, implies that the
newspapers and printing sites affect society. While this may be
true, it changes the original meaning. Choice e incorrectly
employs the present perfect verb tense for a general statement that
is better expressed in the simple present tense. It is incorrect
because that has affected needs to be closer to what it modifies,
which is information. Placing it at the end of the sentence makes
the sentence unclear.
2 0 1
501_03_121-234_501_master.qxd 4/29/13 12:33 PM Page 202
501 GMAT
®
Questions
100. d. The phrase recent findings suggests the present time. The correct
idiomatic expression is as . . . as to compare two ideas or things.
Choice a is incorrect because the verb was is in the past and deadly
must be followed by as to form the correct idiomatic expression.
Choice b is incorrect because the verb should be in the present
tense, not the past (there was). Choice c is incorrect because it uses
the present perfect form (there’s been), which sounds awkward when
referring to truth, a general statement that needs to be expressed
in the simple present form. Choice e is incorrect because it uses
as . . . than, not the correct idiomatic expression as . . . as to
compare two ideas or things.
101. b. Choice b correctly uses the idiomatic expression from . . . to to
describe the range of industries within the manufacturing sector.
Choice a is incorrect because it does not finish the idiomatic
expression from . . . to with to. Choice c is incorrect because it also
fails to finish the idiomatic expression with to; also, seems that it is
sounds awkward and is not as concise as seems to be. Choice d is
also incorrect because it uses seems that it is over seems to be, a better
option. Choice e is incorrect because it does not use the complete
expression from . . . to to describe the range of industries within the
manufacturing sector; instead, it incorrectly starts with from and
finishes with in addition to.
102. d. The dependent clause (as companies grow . . .) indicates the
sentence is in the present tense. The verbs in the main clause
should also use the present tense. Having mastered in choice a is an
awkward past participle, and become (the simple present) sounds
awkward. Choice b uses verbs in the past tense, which is incorrect
since the first part of the sentence is in the present tense. Choice c
is incorrect because will be becoming is the future present
progressive. The sentence is in the present, not the future. Choice
e is incorrect because the present perfect progressive (have been
becoming) is awkward in this context that has already used the
simple present; the present progressive is preferred.
2 0 2
501_03_121-234_501_master.qxd 4/29/13 12:33 PM Page 203
501 GMAT
®
Questions
103. e. In choice e, like is correctly followed by a noun to form a
parallel comparison between the rings of both fish and trees.
Choice a is incorrect because like is followed by a clause and the
comparison is not formed with two parallel clauses to make sense.
Choice b is incorrect because like is followed by a clause, not a
simple noun. Choice c is incorrect because as should be replaced
by like. Also, the main topic is fish in all the other sentences; in
choice c, trees seem to be the focus over fish. Choice d is incorrect
because the second clause after the comma is not formed in a
parallel fashion to agree with the first clause (as trees have rings) and
form a correct comparison.
104. c. The information that killed off dinosaurs 65 million years ago is
essential to understanding and identifying the meteorite.
Therefore, the sentence must use that, not which, and omit the
comma. Choice a is incorrect because it uses a comma and which,
which are used when extra information is given that is not
necessary to understanding the sentence; that is not the case with
this sentence. Choice b is incorrect because it changes killed to the
past perfect had killed. The past perfect is used when one event
happens before another; in this example, the turtle survived the
meteorite and the dinosaurs died at the same time. The past
perfect cannot be used in this example. For that same reason,
choice d is also incorrect. Choice d also makes the mistake of
including the comma and using which instead of that. Choice e is
incorrect because the gerund killing should be changed into a that
clause, that killed.
2 0 3
501_03_121-234_501_master.qxd 4/29/13 12:33 PM Page 204
501 GMAT
®
Questions
105. b. The ideas are all correctly structured in the gerund form to
provide a clear, comprehensible sentence. Choice a is incorrect
because the verb includes should be include to agree with the plural
subject three of the most encouraging ideas. Also, the ideas are not
structured in the same way; to detach should be detaching so the
three ideas are expressed in a parallel structure. Choice c is
incorrect because to restore is presented in the infinitive form, not
the gerund form. It should be in the gerund form to agree with the
structure of the other ideas; also, by should be eliminated before
detaching. Choice d is incorrect because includes needs to be include
to agree with the plural subject three of the most encouraging ideas.
Choice e is incorrect because the second idea (using a traveling
service station that refuels them) is wordy and it is unclear what them
refers to; also, detaching working parts to attach them to new satellites
from old ones is wordy and could be phrased better. From old ones
should directly follow what it modifies: working parts.
106. d. The first part of the sentence (not subject to change) is a
modifier. What it modifies must be placed directly afterward so it
is clear to what thought to be either symbolic or accurate descriptions
refers. Choice d successfully does this with a clear, succinct clause.
Choice a is incorrect because it implies the first part of the
sentence is describing archaeologists since archaeologists is the first
word of the independent clause. This is incorrect and as a result,
the first part of the sentence becomes a dangling modifier without
any clear referent. The same mistake is made in choice b. Choice c
is incorrect because the complete sentence would also result in a
dangling modifier; the independent clause (the answer) begins with
much time, thereby implying that much time was thought to be either
symbolic or accurate descriptions. This is illogical; much time cannot
begin the independent clause. Choice e is incorrect because it says
that cave paintings have debated archaeologists. This idea is illogical;
cave paintings cannot debate.
2 0 4
501_03_121-234_501_master.qxd 4/29/13 12:33 PM Page 205
501 GMAT
®
Questions
107. c. Choice c presents the sentence in the correct conditional
sentence form. Choice a is incorrect because the if clause employs
the structure would not have intervened; this is incorrect as the would
have + past participle structure is only correct for the result clause.
Choice b is incorrect because it also uses the would not have
structure in the if clause (in this sentence, the if clause comes
second). Also, the past participle is missing the final d to make it a
past participle (intervened) and not just the verb (intervene). Choice
d is incorrect because it changes the time reference of the entire
sentence. It implies that the economic crisis could still be avoided
if government didn’t intervene. This is illogical since the economic
crisis already happened and this choice changes the original
meaning. It does not represent the best choice. Choice e is
incorrect because the past participle intervene in the if clause is
missing the final d (intervened).
108. e. Choice e eliminates any possibility of ambiguity regarding the
antecedents of the pronouns they and its. Choice a is incorrect
because they and their have ambiguous antecedents; either pronoun
could refer to Easter Islanders or statues. Choice b is incorrect for the
same reason; only the verb tense has been changed, which does not
change the underlying problem of ambiguous antecedents. Choice
c is incorrect because it is unclear what its refers to; though it
could refer to the island, the choice is wordy and not very clear.
Choice d is incorrect because are should be is, as which refers to the
singular idea that the island’s ancient, famous statues walked by
themselves.
109. b. Choice b correctly keeps release in the subjunctive form
(required because of demand + that), and his or her is used to
correctly refer to a presidential candidate. Choice a is incorrect
because their is incorrectly used to refer to a singular presidential
candidate. Choice c is incorrect because releases should be in the
subjunctive form release as required by demand + that. Also, have
been taken should be has been taken to agree with the singular
subject how much. Choice d is incorrect because their should be his
or her, and have been taken should be has been taken. Choice e is
incorrect because releases should be release.
2 0 5
501_03_121-234_501_master.qxd 4/29/13 12:33 PM Page 206
501 GMAT
®
Questions
110. a. Choice a is correct because it keeps the comparisons parallel; the
sentence compares Democrats’ views to Republicans’ views. To
keep the comparison logical, Republicans’ must be possessive like
Democrats’. Choice b is incorrect because the comparison is not
parallel with Republican; it must be phrased in the same possessive
way as Democrats’. Choice c is incorrect because the superlative
form (the most revolutionary) is used when the comparative form
must be used; only two different groups are being compared.
Choice d also incorrectly uses the superlative form instead of the
comparative form; also, Republican’s is in singular possessive form
when it should be in plural possessive form with the apostrophe
after the s. Choice e is missing the apostrophe after Republicans to
make the comparison parallel with Democrats’.
111. e. Choice e correctly uses the number of (to indicate a specific
statistic) with the singular verb was and the idiom about the same as.
Choice a is incorrect because it uses the expression a number of
instead of the number of. The use of the expression a number of
(meaning some or many) in this sentence is illogical. Choice b is
incorrect because it also uses the expression a number of, which is
incorrect in this context. Also, the subject a number of unemployed
Americans requires a plural subject; in choice b, a singular subject is
incorrectly used. Choice c is incorrect because it uses the
expression about the same than; the correct expression is about the
same as. Choice d is incorrect because it uses the plural verb were
instead of was, and the incorrect expression about the same than
instead of about the same as.
112. a. Choice a uses the correct verbs to agree with the plural subjects
market forces and movements of a stock price. Choice b is incorrect
because the plural subjects require the verbs are and are, not is and
is. Choice c is incorrect because the second verb is should be are to
agree with market forces. Choice d is incorrect because the first
verb is should be are to agree with movements of a stock price. Choice
e is incorrect because subjecting does not make sense in this
context.
2 0 6
501_03_121-234_501_master.qxd 4/29/13 12:33 PM Page 207
501 GMAT
®
Questions
113. d. Choice d is grammatically correct because it uses the
subjunctive voice be established (required by the preceding clause
suggests that) and proper verb forms and sentence structure to avoid
any ambiguous pronoun references. The subjunctive voice is used
because this sentence is implying that a potential new standard
should be established by using the verb suggest. Choice a is incorrect
because they has no logical referent. Choice b is incorrect because
is established is wrong; the subjunctive voice is needed after suggests
that. Choice c is incorrect because to be established needs to change
to be established (without to), and they has no logical referent.
Choice e is incorrect because is established needs to change to be
established, and they has no logical pronoun referent.
114. b. Choice b correctly uses the conditional tenses were and had
required after the verb wish. Choice a is incorrect because was is
used instead of the correct were, and wishes is used instead of wish
to go after the modal may and to agree with feel. Choice c is
incorrect because was is used instead of were, and will has replaced
may, which is incorrect because it changes meaning. Choice d is
incorrect because may wish has been changed to wishes, and may still
feel has been changed to still feels; both changes alter meaning.
Also, choice d uses was instead of were. Choice e is incorrect
because has should be had.
2 0 7
501_03_121-234_501_master.qxd 4/29/13 12:33 PM Page 208
501 GMAT
®
Questions
115. c. Choice c uses correct subject-verb agreement and appropriate
idiomatic expressions to form a grammatically correct sentence.
Choice a is incorrect because were should be was to agree with the
singular subject a system of underground tunnels; also, in the case a
national emergency is not a correct idiomatic expression. The
placement of the modifier with a complex communication network
after were built on an impressive scale is confusing because it’s unclear
whether the building of the system has a complex communication
network or whether the system itself does. Choice b is incorrect
because were should be was to define a system . . . ; also, the clause
which has a complex communication network is incorrect. The which
must always refer to the noun immediately before it; in this case,
that would be scale when it needs to be a system of underground
tunnels. That entire clause needs to be reworded. Also, the
dependent clause leading to . . . could be worded better. Choice d is
incorrect because occurs needs to change to occur in the clause
should a national emergency occurs. Choice e is incorrect because were
needs to change to was, and lead needs to change to leads to agree
with the singular subject.
116. a. Choice a correctly utilizes the idiomatic expression neither . . .
nor with the appropriate subject-verb agreement. Choice b is
incorrect because argue should be argues to agree with the singular
noun a majority of society, and are should be is to agree with the
expression neither . . . nor. Choice c is incorrect because argue
should be argues; also, they has no clear referent since a majority of
society is singular. Choice d is incorrect because not . . . or is not a
correct idiomatic expression; also, argue should be argues. Choice e
is incorrect because not . . . nor is not a correct idiomatic
expression. The correct expression is neither . . . nor.
2 0 8
501_03_121-234_501_master.qxd 4/29/13 12:33 PM Page 209
501 GMAT
®
Questions
117. d. Choice d represents the most logical, grammatically correct
sentence. Choice a is incorrect because the clause rather than
waiting to die should modify what directly follows it. In this case, it
is Oregon’s right-to-die law, which is illogical. Choice b makes the
same mistake. Choice a is also incorrect because the clause which
have less than six months to live is set off by commas as a nonessential
clause (also called a nonrestrictive clause). This is incorrect
because the information between the commas is essential to
describing what kind of mentally stable people. The commas imply
that mentally stable people have access to medication to accelerate
their demise, and the fact that they have less than six months to
live is extra information. This is incorrect since that information is
essential to the definition and meaning of the sentence. Choice c is
incorrect because rather than waiting to die is immediately followed
by the demise, not mentally stable people. It modifies mentally stable
people, so mentally stable people must follow the comma. Also, who
permits them access to medication should be worded as that permits
them access to medication since it refers to the law (a concept, not a
person). Choice e is incorrect because it also has the demise as the
subject, instead of mentally stable people, following the clause rather
than waiting to die. In addition, the sentence is not as clear or
succinct as choice d.
118. e. Choice e presents the three ideas in a parallel verb form that
results in a clear, grammatically correct sentence. Choice a is
incorrect because the kidnapping of children does not maintain the
same gerund verb form as the other factors presented; using the in
front of kidnapping changes its structure and it loses its parallelism
with the rest of the sentence. Also, for taking as is incorrect to express
purpose; the infinitive form (to take as) must be used in this context.
Choice b is incorrect because the first verb is in the gerund form and
the second and third verbs are in the infinitive form. This does not
maintain a parallel structure among all elements. Choice c is
incorrect because the first element (his plundering of villages) is
presented as a noun, and the rest are presented as simple gerunds.
Once again, a parallel structure among all the elements is not
achieved. Choice d is incorrect for the same lack of parallel structure;
the first element is presented in the past tense (he plundered) and the
other two elements are presented in the gerund form. Consistency
among the verbs is essential to achieve parallelism.
2 0 9
501_03_121-234_501_master.qxd 4/29/13 12:33 PM Page 210
501 GMAT
®
Questions
119. c. Choice c correctly uses the expression as . . . as and
appropriately compares the excitement of the competition between
the two parties with to inserted after the second as. Choice a is
incorrect because it fails to insert to after the second as to form a
grammatically correct comparison. Choice b is incorrect because the
correct expression as . . . as is inappropriately changed to as . . . than
and the to is also missing to form a correct comparison. Choice d is
incorrect because the second as has been changed to than. Choice e
is incorrect because then is used instead of as.
120. d. Choice d is a grammatically correct sentence that corrects all
pronoun and adverb errors. Choice a is incorrect because full veiled
should be fully veiled since veiled is an adjective and fully acts as an
adverb; also, who is incorrect to refer to Western opposition; that is
the correct word. Choice b is incorrect because full should be fully,
her should be their (twice in the sentence) to agree with the plural
subject women, and who should be that in reference to Western
opposition. Choice c is incorrect because the adjective heavy
should be heavily as it’s an adverb that describes the adjective
oppressed. Choice e is incorrect because her should be their in two
instances to agree with the plural subject women.
121. b. Choice b is the best grammatically correct sentence. Choice a is
incorrect because well should be good; good describes athletic abilities,
so it is an adjective. Also, the second clause of the sentence is
missing a final does at the end to appropriately compare DNA-based
tests to a standard performance test. Choice c is incorrect because
well must be good, and much beyond a standard performance test
should read much beyond what a standard performance test does to
appropriately compare the two types of tests. Choice d is incorrect
because well needs to be good. Choice e is incorrect because even
though the comparison is inverted in the second part of the
sentence and still maintains the original meaning, a do is missing at
the very end to appropriately compare the two types of tests.
2 1 0
501_03_121-234_501_master.qxd 4/29/13 12:33 PM Page 211
501 GMAT
®
Questions
122. e. Choice e fixes the problem of the dangling modifier resigned to
accept his disqualification from the Olympic trials by starting the
independent clause with the athlete. Choice a is incorrect because
there is no subject in the sentence to which the modifier resigned to
accept his disqualification from the Olympic trials can attach. Choice b
is incorrect for the same reason, despite having inverted the clauses
in the sentence. Choice c is incorrect because it is missing both a
subject and a verb. Choice d is incorrect because it suggests that
silence resigned to accept his disqualifications; this is illogical. A
reasonable noun needs to be inserted.
123. e. Choice e is the correct option. Choice a is incorrect because the
preposition with is unnecessary and each is singular, so it needs to
be followed by its, not their. Choice b is incorrect because each
products is illogical; each is singular so it cannot be followed by a
plural noun. The same logic can be applied to choice c. It would
also need an article before the singular product. Choice d is
incorrect because with and as are unnecessary. The resulting
expression is not idiomatic.
124. a. Choice a is grammatically correct with no ambiguous pronoun
references. Choice b is incorrect because considered as is not a
correct idiomatic expression. The as is incorrect. Also, it is unclear
whether the referent of his in his works refers to Joseph C. Hart or
William Shakespeare. Choice c is incorrect because they has no
logical referent, and considered as is not an idiomatic expression.
Explicit must be explicitly to define the verb question, and his
authorship of Shakespeare’s works is misleading to imply Hart’s
authorship of Shakespeare’s works, which is illogical. Choice d is
incorrect because considered to be is unidiomatic, and the greater
playwrights must be changed to the greatest playwrights since more
than two playwrights are being compared. The him in choice d is
also ambiguous, as it could possibly refer to Shakespeare or Joseph
C. Hart. Choice e
is incorrect because regarded to be is not a correct
idiomatic expression, and it is unclear exactly to whom him refers
at the end of the sentence.
2 1 1
501_03_121-234_501_master.qxd 4/29/13 12:33 PM Page 212
501 GMAT
®
Questions
125. c. Choice c is a grammatically correct and concise sentence that
appropriately uses the expression x and y, as well as z. Choice a is
incorrect because the preposition on is not appropriate; it could be
substituted by in or when. Also, like cannot be given to list
examples; the correct expression is such as. Choice b is incorrect
because the expression x and y and z is not idiomatic. Also, like
needs to be substituted by such as. Choice d is incorrect because
the expression x, y, as well as z is not idiomatic; x and y require and
between them. Choice e is incorrect because the verb consider
should be considers to match its singular subject the Happy Planet
Index.
126. b. Choice b conjugates all the verbs in the correct tense and uses
more quickly to describe the verb control. Choice a is incorrect
because exercised is in the past tense, which does not agree with the
first verb plans in the simple present tense, and the pronouns they
and their are used, despite the singular subject a diabetic. Quicker
(comparative adjective) is also incorrect because it defines the verb
control, which requires an adverb (more quickly). Do must be omitted
from the comparison between two types of diabetics. Choice c is
incorrect because the pronoun they is used for a singular subject,
quicker is used instead of more quickly, and does needs to be omitted
from the comparison. Choice d is incorrect because exercised, will
have regained, and stuck must be in the present tense. Choice e is
incorrect because does must be omitted from the comparison.
127. a. Choice a correctly uses the semicolon to join two independent
clauses that have similar meanings. Choice b is incorrect because
two independent clauses (the first ending with records, the second
beginning with as an example) are joined by a comma. This is
incorrect and constitutes a comma splice. Choice c is incorrect
because like must be used to compare two nouns (which does not
happen in this sentence). Choice d is incorrect because the clause
starting with which does not represent an independent clause.
Therefore, a semicolon cannot be used to separate the first clause
from the second. Also, which technically refers to records, which is
illogical. In addition, the wording is awkward. Choice e is
incorrect because such as to be helping is awkward and wordy. It
would sound more idiomatic if to be were omitted.
2 1 2
501_03_121-234_501_master.qxd 4/29/13 12:33 PM Page 213
501 GMAT
®
Questions
128. d. Choice d uses the correct verb forms to agree with its subjects.
Choice a is incorrect because as much as must be as many as to
agree with the countable noun pieces of floating plastic, is must be are
to agree with the plural subject pieces of floating plastic, and makes up
must be make up to agree with the plural subject. Choice b is
incorrect because as much as must change to as many as, is must
change to are to agree with pieces of floating plastic, and the present
perfect tense have made up must change to the simple present tense
to agree with the rest of the sentence. Choice c is incorrect
because indicate must change to indicates, is must change to are, and
is making up must be changed to the simple present make up since
this is a general statement. Choice e is incorrect because indicate
must change to indicates and as many of is not a correct idiomatic
expression. It must be changed to as many as.
129. c. Choice c is correct because both clauses the frequency of employee
turnover and servers’ treatment of their customers are parallel as
nouns. Choice a does not offer a parallel clause to the first clause
in the sentence, nor is the singular verb is correct. It should be are
to agree with the compound subject. Choice b is incorrect because
it is a clause, not a noun that is parallel to the frequency of employee
turnover. Choice d is incorrect because is must be are to agree with
the compound subject. Choice e, though phrased as a noun in a
fashion parallel to the frequency of employee turnover, does not make
sense when inserted into the sentence as customers’ reactions to their
servers. This changes the original meaning.
2 1 3
501_03_121-234_501_master.qxd 4/29/13 12:33 PM Page 214
501 GMAT
®
Questions
130. e. Choice e uses the correct idiomatic expression rivalry between x
and y and the correct positive conditional verb forms to form a
logical, grammatically correct sentence. Choice a is incorrect
because of must be between to form the idiomatic expression rivalry
between x and y, and hadn’t must be had to make logical sense.
Coca-Cola didn’t buy Pepsi. If it had bought it, the rivalry may never
have been. Choice b makes the same mistake using of instead of
between, and it also incorrectly uses hadn’t seized and hadn’t bought
when both instances must be positive to make logical sense.
Choice c is incorrect because it too uses hadn’t instead of had.
Choice d is incorrect because the result clause is not conjugated
correctly and the modal of possibility may has been changed to a
more concrete would: Would never be must be changed to would
never have been, and in order to preserve the original implied
possibility, would must be changed to may.
131. b. Choice b uses correct verb tense and subject-verb agreement to
convey a clear, sound sentence. Choice a is incorrect because had
voted needs to be voted. The past perfect would only be used if
something else in the sentence happened afterward (see
Explanation of past perfect tense in Instructional Text). Choice c is
incorrect because had voted must be voted, and the trying of is not
idiomatic; the should be eliminated to make the sentence read
more naturally in English. Choice d is incorrect because in favor of
+ infinitive is not an idiomatic expression; to repeal needs to be
r
epealing in order for the expression to be correct. Choice e is
incorrect because the for + gerund form (for repealing) needs to
change to the infinitive form (to repeal) in order to express purpose.
2 1 4
501_03_121-234_501_master.qxd 4/29/13 12:33 PM Page 215
501 GMAT
®
Questions
132.
d. Choice d successfully makes the two clauses parallel for an
effective sentence. Choice a is incorrect because the ideas are not
parallel: the perils of capitalism is expressed as a noun phrase, and
that the democracy is corrupted is expressed as a that clause with a
subject and verb. Choice b is incorrect because that is missing after
agree. It also changes the meaning by omitting any mention of
criticism. Choice c is incorrect because the notion of possibility
(originally conveyed with may) is missing, which changes the
meaning, and the expression to the extent of the perils is awkward.
Choice e is incorrect because the correct idiomatic expression is
however much, not however many to describe the extent of the
United States citizens’ criticism (an uncountable noun).
133. c. Choice c correctly uses it to agree with the singular subject the
private lending sector. Also, adding the phrase to be made helps clarify
the sentence meaning. Choice a is incorrect because it is must be
they are to agree with the plural subject payments. Choice b is
incorrect because the active voice is preferred over the passive
voice; the passive voice in this sentence makes it unclear who does
not expect payments to be made. Also, will not be expected to be paid
suggests some control over the expectations, which is awkward and
changes meaning. Choice d is incorrect because the passive voice is
less clear, and the expression payments . . . to be paid is redundant.
The future tense are going to be is awkward in this choice as well.
Choice e is incorrect because they must be it to agree with the
singular subject the private lending sector, and it is must be they are to
agree with the plural subject payments.
2 1 5
501_03_121-234_501_master.qxd 4/29/13 12:33 PM Page 216
501 GMAT
®
Questions
Set 6
134. c. Starting the sentence with Accused of is the most concise and
effective way to present the information. Choice a is not the best
answer because having been at the beginning of the sentence is not
necessary and is best omitted. Choice b is incorrect because being
accused is not the most concise way to express the idea. The being is
confusing and is best omitted. Also, combining huge areas of the
Ecuadorian Amazon to read huge Ecuadorian Amazon areas is unclear.
The phrase is best left as it originally appears with the of. Choice d
is incorrect for the same unclear huge Ecuadorian Amazon areas and
because having been at the beginning of the sentence is unneces
-
sary. Choice e
is too wordy; due to being
at the beginning of the
sentence is incorrect.
135. a. This choice provides the clearest sentence with the most
logically placed modifiers. Choice b is incorrect because the
modifier with construction slated to begin in 2014 is placed after
Disneyland Paris; this placement is confusing because it implies the
construction of Disneyland Paris is slated to begin in 2014. Choice
c is incorrect because the placement of the modifiers results in a
confusing sentence. With construction slated to begin in 2014 provided
enough funds are raised should be placed closer to France’s very own
theme park devoted to Napoleon Bonaparte, not directly after the
mention of Disneyland Paris. It is unclear whether Disneyland
Paris or France’s theme park devoted to Napoleon Bonaparte is
slated to begin construction in 2014. Choice d is not the best
option presented; it has many commas that cut off the flow and
cohesion of the sentence. Choice e is incorrect due to syntax. The
placement of devoted to Napoleon Bonaparte at the end of the
sentence after Disneyland Paris makes it seem as though Disneyland
Paris is devoted to Napoleon Bonaparte. This is incorrect.
2 1 6
501_03_121-234_501_master.qxd 4/29/13 12:33 PM Page 217
501 GMAT
®
Questions
136. d. The phrase yellow saddle goatfish in the Red Sea is best left as is
without collapsing it into a noun-adjective phrase (Red Sea yellow
saddle goatfish). Choice d effectively eliminates the comma by
combining the appositive (groups with designated members) with the
rest of the sentence. Choice a is incorrect because have the
capability to hunt in groups is understandable, but it is not the most
effective way to express the goatfish’s abilities. Choice b better
phrases this part of the sentence with are capable of hunting in
groups. However, choice b describes the goatfish as Red Sea yellow
saddle goatfish. This is incorrect because it is not known whether
the yellow saddle goatfish are exclusively from the Red Sea; they
are best left with the location placed afterward following in.
Choice c is incorrect for the same noun-adjective error (Red Sea
yellow saddle goatfish) and because have the capability to hunt in groups
is not the most effective wording. It also uses the phrase designated
chosen members, which is redundant. Choice e makes the same
mistake with the Red Sea yellow saddle goatfish combination and by
placing designated and chosen next to each other. Furthermore,
possess the capability to hunt in groups is wordy.
137. b. This sentence effectively and concisely describes the idea with a
that clause that changes the idea into an active verb. Choice a,
while understandable, is wordier because it uses more nouns than
active verbs. Choice c, while one of the shortest options presented,
incorrectly uses like where it should use such as. Choice d, like
choice a, uses more nouns than action verbs; furthermore, the verb
fascinates at the end of the sentence is awkward as it doesn’t specify
who is fascinated. The idea seems to be cut off. Choice e, while it
correctly changes the structure of the sentence to include a that
clause, it incorrectly uses like instead of such as.
2 1 7
501_03_121-234_501_master.qxd 4/29/13 12:33 PM Page 218
501 GMAT
®
Questions
138.
a. This sentence effectively uses the that clause and active verbs.
Choice b is incorrect because the belief about whales being terrestrial
mammals is wordy and confusing. Which is incorrect and should be
replaced by that; also, was supported is unclear because the evidence
still supports the belief; therefore, was supported needs to be in the
present tense as is supported. Choice c avoids the that clause, which
leads to excessive nouns and not enough action verbs.
Furthermore, have been supported is incorrectly structured in the
plural tense and does not agree with the singular subject the belief.
It should read has been supported. Choice d, while one of the best
options presented, fails in comparison to choice a because made the
transition is not as succinct as transitioned. Choice e is awkwardly
structured and does not represent the best sentence. The belief of
whales as once being terrestrial mammals can be phrased much better
as the belief that whales were once terrestrial mammals. Also, is must
change to are to agree with the plural subject whale fossils.
139. d. Choice d is the most concise sentence that doesn’t repeat any
information. Choice a is extremely wordy and redundant; the high
numbers of Christians on the scale is overemphasized with the
words overpowered, overrepresent, and high. Choice b, while less
wordy than choice a, is still redundant with overrepresent and high;
also, the original phrase a world scale has been changed to the world
scale. This change in article from a to the changes meaning and
implies there is only one scale. Choice c changes the wording to
the passive voice, a style that is generally not preferred on this test.
It is redundant with overrepresented and elevated. Like choice b, it
also changes the article a before world scale to the to imply there is
only one supposed world scale. This cannot be assumed from the
information given. Choice e is in the passive voice; generally
speaking, the active voice is preferred on this test. It also leaves out
the information on a world scale.
2 1 8
501_03_121-234_501_master.qxd 4/29/13 12:33 PM Page 219
501 GMAT
®
Questions
140. a. Choice a is the best sentence because it is in the active voice
(done using the that clause) and it is clear. Choice b is redundant
with the phrase will be outearning their husbands with greater wages.
With greater wages is superfluous since the verb outearn already
conveys this information. Also, within the next generation is wordier
than within a generation. Choice c is unclear due to the placement
of within a generation directly after new workforce trends. This
placement implies within a generation defines new workforce trends,
not when wives will outearn their husbands. The sentence is
awkward and confusing as it also seems to imply that husbands will
outearn their working wives; this is not the same idea expressed in
the original sentence. Choice d is worded in the passive voice,
which results in an awkward and wordier sentence that does not
represent the best option. Choice e is incorrect because it uses
more noun clauses instead of active verbs.
141. c. That needs to be included after revealed in this sentence. Choice
c also maintains the second part of the sentence (because
communities abandon . . .) in the active voice, which makes for the
clearest, most succinct sentence. Choice a is incorrect because that
is omitted after the reporting verb revealed, and the noun phrase
because of communities abandoning is not as clear or efficient as the
active verb choice because communities abandon in choice c. Choice b
is incorrect because it uses due to; because of would be the correct
phrase, but the sentence can be worded even better as the
explanation for choice a suggests. Choice d is missing that at the
beginning of its answer. Choice e is incorrect because the article
does not reveal the death of one language every 14 days. Such a
sentence would imply that the article spanned various 14-day
periods to reveal the death of a different language. This choice
greatly changes the information presented in the original sentence.
2 1 9
501_03_121-234_501_master.qxd 4/29/13 12:33 PM Page 220
501 GMAT
®
Questions
142. e. Choice e correctly uses the that clause followed by are since that
refers to many analyzers. Choice a is incorrect because is should be
are to agree with the plural many analyzers. Effort + infinitive (effort
to subject) is preferred over effort + of + gerund (effort of subjecting).
Choice b is incorrect because is should be the plural are after
which to agree with many analyzers. Also, the entire phrase does
not flow well with the remaining part of the sentence to statewide
scrutiny and investigation; it implies that the gases are emitting to
statewide scrutiny, which does not make sense. Choice c is
incorrect for the same nonsensical reason as choice b; it implies
that the efforts are to subject how the gases are emitting to
scrutiny, which does not make sense. Choice d is incorrect because
effort in the long term is best phrased as long-term effort; also, the
two relative clauses beginning with that right after each other
result in a confusing and wordy sentence.
143. b. This sentence is logical. Findings refers back to the observations
of the scientists, and the icebergs are best described as gigantic
icebergs in Antarctica, not gigantic Antarctica icebergs. Choice a is
incorrect because it names the icebergs as Antarctica icebergs, and
because which, in this context, refers to Antarctica icebergs (since
which must always refer to the noun immediately before it). This is
incorrect because the second clause refers to large cracks observed,
not the icebergs themselves as the structure of choice a suggests.
Choice c is incorrect because it is unclear what the phrase consistent
with the predictions of global warming refers to. Choice d is incorrect
because where cannot logically refer to Antarctica icebergs. Choice e
is incorrect because the icebergs are named Antarctica icebergs
versus the correct icebergs in Antarctica. Remember that the
shortest idea isn’t always the best!
2 2 0
501_03_121-234_501_master.qxd 4/29/13 12:33 PM Page 221
501 GMAT
®
Questions
144. d. Choice d correctly utilizes the transition word although at the
beginning of its sentence to introduce a contrasting idea. Choice a
is incorrect because it suggests the wrong transition word (however
is illogical at the beginning) and an additional transition word
regardless, which is incorrect. It also does not define the referent of
enough. Enough stands alone in the sentence and its referent is
unclear. Choice b is incorrect because the wording having approved
installing cobblestones sounds awkward. Although it is shorter than
other choices, the repeated gerund form sounds awkward. In
addition, choice b offers a superfluous transition word regardless
after the comma. Having two transition words for one sentence is
unnecessary. Choice c correctly uses but after the comma, but
there is no referent for enough and the wording cobblestone
installation bearing the names of Nazi victims implies the installation
bears the names of the victims, not the cobblestones. Choice e
commits the same mistake with the wording cobblestone
installation. . . .
145. d. Choice d effectively presents the idea in the clearest way with
the fewest words. Choice a is incorrect because it is wordy and the
phrase if there is a lacking concrete business plan is awkward; action
verbs are preferred over nouns. Choice b is incorrect because the
expression x will happen unless y happens first calls for the present
tense after unless; in this choice, the future tense is employed.
Choice c is incorrect because the clause is wordy and awkward.
Choice e is incorrect because it does not represent the clearest
option.
146. e. Choice e represents the most concise and clearest sentence of all
the options presented. Choice a is incorrect because has the ability
to grow can be phrased with fewer words and due to needs to be
replaced by because of. Choice b is incorrect because is able of
growing is not a correct expression (is able needs to be followed by
an infinitive), and caused by is inappropriate in the sentence. Choice
c is incorrect because because needs to be followed by a clause with
a verb, not just a noun (the thin air . . .) as the choice suggests.
Choice d is incorrect because due to needs to be replaced by
because of.
2 2 1
501_03_121-234_501_master.qxd 4/29/13 12:33 PM Page 222
501 GMAT
®
Questions
147. c. Choice c makes for a clear and concise sentence that represents
the best option. Choice a is incorrect because the language is
redundant (build its increasing) and the market of international energy
can be reworded to be more concise. Choice b is incorrect because
Mexico’s hydrocarbon industry organization cannot be combined
without making the idea unclear; the phrase is best left as the
organization of Mexico’s hydrocarbon industry. Choice d is incorrect
because the phrasing Mexico’s hydrocarbon industry organization is
incorrect, and it is redundant to use both growth and surging.
Choice e is incorrect because it is also redundant (increase its
surging competitiveness).
148. a. This sentence is clear and concise. Choice b is incorrect because
decision from the mountain climber is improperly worded; from
should be of. Also, that led to him abandoning his friend, while
comprehensible, is a bit wordy. Choice c is incorrect due to its
wordiness. Of abandoning his friend is better phrased to abandon his
friend, and the adjective phrase would be life-changing for is more
concise as a verb phrase would change the lives of. Choice d is not
the best choice because of abandoning his friend is better worded as
to abandon his friend. Choice e is not the best choice because the
phrase on the slope is placed after to find help, which implies that the
mountain climber went to find help on the slope and did not
abandon his friend there. This placement changes the idea
presented in the original sentence.
149. b. The sentence is concise and clear without any redundant
phrases. Choice a is incorrect because internally displaced people
within their own countries is redundant. Internally displaced already
implies they are displaced within their own countries. Choice c is
incorrect because increased to an amplified 27.5 million people is
redundant; amplified is superfluous in the sentence. Choice d,
while grammatically correct, changes the expression internally
displaced people to a wordier displaced people within their own countries.
Therefore, it is not the best option. Choice e is incorrect because
the correct adjective is displaced, not displayed.
2 2 2
501_03_121-234_501_master.qxd 4/29/13 12:33 PM Page 223
501 GMAT
®
Questions
150. c. Choice c correctly compares the Washington Monument to the
San Jacinto Memorial Monument with the correct contrast word
although followed by a dependent clause. Choice a is incorrect
because the Washington Monument is compared to the structure of
the San Jacinto Memorial Monument; this is not an equal
comparison. Either the two monuments are compared, or the
structures of the two monuments are compared, but not one of
each. Choice b is incorrect because it also wrongly compares the
Washington Monument to the San Jacinto Memorial Monument’s
structure, an unequal comparison. Choice d is incorrect for the
same unequal comparison. Choice e is incorrect because the wrong
transition word (in addition to) is used, when in fact a contrasting
transition word should be used.
151. d. Choice d represents the clearest and most concise sentence.
Choice a is not the best sentence because it uses noun clauses (are
evidence) and the passive voice (users are allowed) over active verbs
(like proving and allow users in choice d). The sentence is
comprehensible, but it is wordier and more passive than choice d.
Choice b is incorrect due to its wordiness in the last clause after
the second comma (are evidence of far-fetched Star Trek ideas being a
modern reality); furthermore, choice b changes the meaning from
have become to being a modern reality. This change in verb from
become to being changes the meaning and progression of
transformation implied in the initial sentence. Choice c is
incorrect because it is in the passive voice, which makes the
sentence wordier and not as clear. Choice e is incorrect because it
also changes have become a reality to being a reality, which is a
change in meaning. Also, the phrase as proof of far-fetched Star Trek
ideas being a reality could be more succinctly worded.
2 2 3
501_03_121-234_501_master.qxd 4/29/13 12:33 PM Page 224
501 GMAT
®
Questions
152. a. Choice a represents the best sentence for its succinctness and
clarity. Choice b is incorrect because homes cannot be damaged
and destroyed at the same time; it is either one or the other. Choice
c is incorrect because considered as is incorrect. Choice d is
incorrect because damaged and destroyed needs to be damaged or
destroyed; also, the word choice thanks to is inappropriate in this
context; thanks to is typically used for a positive situation, not one
of suffering and destruction. Choice e is incorrect because
approximately has been changed to exactly; this change in word
choice is inappropriate because it changes the meaning.
153. c. In this choice, older automobiles are clearly contrasted with newer
automobiles, and to meet is parallel with remain (the to is implied).
Choice a is incorrect because older automobiles cannot be contrasted
with carmakers; also, remaining is not structured in the infinitive
form (remain) to agree with to meet. Choice b is incorrect because
automobiles cannot be contrasted with aluminum. Choice d is
incorrect because remaining is best phrased in the infinitive form to
be parallel with to meet. Choice e is incorrect because older
automobiles cannot be contrasted with new fuel efficiency standards.
154. b. Choice b is a grammatically correct sentence that uses accurate
idiomatic expressions without being redundant. Choice a is
incorrect because a rate is high, not fast, and it is redundant to have
annual and per year in a sentence to describe the same thing.
Choice c is incorrect because fast should be replaced by high, and
in addition is not the correct word to introduce a contrasting idea.
Choice d is incorrect because a quick rate is not a good phrase
choice, and moreover is not the correct word to introduce a
contrasting idea. Choice e is incorrect because annual rate per year
is redundant.
2 2 4
501_03_121-234_501_master.qxd 4/29/13 12:33 PM Page 225
501 GMAT
®
Questions
155. e. Choice e is correct because it uses the correct expression native
to to describe the origin of a species and it correctly uses because of.
Choice a is incorrect because native of should be replaced by native
to. Choice b is incorrect because native of needs to be replaced by
native to, and a Devil Facial Tumor Disease cancer is best phrased as a
cancer called Devil Facial Tumor Disease. Choice c is incorrect
because native of needs to be replaced by native to, and due to needs
to be replaced by because of. Choice d is incorrect because due to
should be because of, and a Devil Facial Tumor Disease cancer is best
worded as a cancer called Devil Facial Tumor Disease.
156. d. Choice d is the most effective, succinct sentence. Choice a is
incorrect because the first dependent clause before the comma is a
dangling modifier with no logical referent. It implies that
microfinance views their money as a way to alleviate poverty, which is
illogical. Also, rather than would be preferred in this sentence
because rather than implies preference, whereas instead of is more
of a replacement, not a preference. Choice b is incorrect because
the first clause continues to be a dangling modifier with no logical
referent, and now currently is redundant and wordy. Also, rather
than is a better option in this sentence. Choice c is incorrect
because now . . . seeing . . . in today’s world is redundant; in today’s
world is best omitted from the sentence since the time frame of the
present is already conveyed. Also, the sentence would be more
effective if both clauses utilized the same verb tense. Choice e is
incorrect because view must be viewing if it is to follow instead of.
Also, now . . . in today’s world is redundant.
2 2 5
501_03_121-234_501_master.qxd 4/29/13 12:33 PM Page 226
501 GMAT
®
Questions
157. d. Choice d represents the most concise and effective sentence.
Choice a is incorrect because have differences is better worded as
differ, and for maintaining, when it describes a purpose, needs to be
expressed in the infinitive form (to maintain). Choice b is incorrect
because have differing opinions as to is wordy and awkward, and for
maintaining needs to be in the infinitive form (to maintain) since it
describes purpose. Choice c is incorrect because the expression
create silk with their feet is misleading and can imply something
different from the original idea of making silk come out of their feet or
emit silk from their feet; the expression is also wordier than emit silk
from their feet as suggested by choice d. Choice e is incorrect
because disagree over the abilities of is wordy, and for maintaining
needs to be in the infinitive form.
158. e. Choice e is the best option because it is succinct and clear.
Choice a is incorrect because it is wordy and awkward: mandated
the right to have their leases honored for tenants is comprehensible,
but wordy and awkward. Choice b is incorrect because it is very
wordy; also, the use of the passive voice makes the message
awkward and changes the focus. Choice c is incorrect because the
subjunctive form (made mandatory that tenants have) is preferred
over the wordier infinitive form (made mandatory for tenants to
have). Choice e successfully uses the subjunctive form. Choice d is
incorrect because it is redundant; in the phrase made mandatory
that existing tenant leases have to be honored, both made mandatory
and have to be honored convey obligation. Have to is superfluous in
the sentence. Also, made mandatory is better expressed as a single
verb mandated.
2 2 6
501_03_121-234_501_master.qxd 4/29/13 12:33 PM Page 227
501 GMAT
®
Questions
159.
c. Choice c represents the most succinct option. Choice a is
redundant because it includes both upcoming and approaching.
Upcoming already indicates the immediacy of the lunch. Choice b is
incorrect because being, approaching, and in a hastily manner are
superfluous: being can easily be omitted, approaching is redundant
after upcoming, and in a hastily manner can be reworded with hastily
before the verb phoned for a more concise sentence. Choice d is
incorrect because with regards to is incorrect; the correct expression
is with regard to or regarding. Also, the second phrase must not be
in the passive voice with the committee members beginning it as this
implies that the committee members are enthusiastic about the upcoming
reception lunch, not Mrs. Brown, as the original sentence indicates.
Mrs. Brown is not even mentioned in choice d, so this choice must
be eliminated as a possible answer. Choice e is incorrect because in
a rushed effort is redundant; hastily already conveys the hurriedness
of Mrs. Brown.
160. b. Choice b is a grammatically correct, clear sentence. Choice a is
incorrect because it is redundant: the phrase studied and researched
is unnecessary since studied and researched convey the same idea; the
same is true with rose by an increase of. Choice c is incorrect because
the growth of the ocean level went up is redundant; both went up and
growth indicate an upward movement. Also, parallel structure is
preferred on the GMAT, so the verb discovering would best be
expressed as discovered to mirror the other verb studied. The same
can be said for choice d; thereby discovering is better phrased as and
thereby discovered to maintain parallel structure. Also, rise and
increased are redundant since they both express an upward
movement; only one of those words is needed. Choice e is
incorrect because studied and researched together are redundant,
and more before feet changes the original meaning and implies the
ocean level had already risen. Also, the first clause samples of coral
were studied and researched by scientists is in the passive voice, which
is generally less preferred than the active voice.
2 2 7
501_03_121-234_501_master.qxd 4/29/13 12:33 PM Page 228
501 GMAT
®
Questions
161. c. Choice c correctly conveys the same idea as the original
sentence, yet it eliminates any redundancy and wordiness. Choice a
is incorrect because it is redundant. Both it is possible and may
convey possibility so may needs to be eliminated and replaced by
will. In choice b, today’s wealthiest nations’ babies is awkward; the
phrase is best left with the babies placed before the rest of the
expression. Choice d is incorrect because if the answer is plugged
into the rest of the sentence, it reads it is possible with greater chances
increasing; half of the sentence needs to be read before the reader
even knows what is being discussed. This results in an unclear
sentence. Also, the wrong pronouns he or she are used to refer to
the plural subject more than half of the babies. The sentence is also
wordy. In addition, the use of the modal might results in a
repetitive sentence, since both it is possible and might imply
possibility. Choice e is incorrect because the phrase today’s
wealthiest nations’ babies is awkward and best left with babies
beginning the expression to avoid confusion. Also, may must
change to will since possibility is already conveyed with it is possible.
162. a. Choice a is the most succinct and effective sentence. Choice b is
incorrect because two of the ideas (rescuing and capturing) have
been changed into the gerund form, which causes the sentence to
lose its flow and parallel structure. Furthermore, a renewed sense of
confidence is wordier than renewed confidence without adding any
more substance. Although all the results are in the same part of
speech in choice c, the gerund form (sweeping rescuing, arresting,
capturing, feelings) does not read as well or succinctly as the noun
form (rescues, arrests, captures, confidence); the sentence is wordier
and less effective. Choice d does use the more effective noun form
for its list, but the clause who are progressive in their ideals is wordier
than choice as adjective left-wing. Choice e is incorrect because
idealistically progressive is not as succinct as left-wing, and renovated
has been used instead of renewed; the latter substitution changes
meaning and renders the choice incorrect.
2 2 8
501_03_121-234_501_master.qxd 4/29/13 12:33 PM Page 229
501 GMAT
®
Questions
163. b. Choice b is the most effective and concise sentence. Choice a is
incorrect because should is an incorrect modal to describe the
ruling of a judge; should is used to describe a moral obligation, not
a judge’s ruling. Also, choice a is redundant with its wording of in
total and a sum of; both expressions convey the same meaning. The
rest of the sentence, though grammatically correct, involves two
different that clauses that result in a wordy, drawn-out sentence.
Choice c is not the best option because it is wordier without
offering any additional information. Notice the difference of with
the money having come from the sale of personal and business assets,
which were seized after an undercover investigation versus which was
seized in personal and business assets following an undercover
investigation in choice b. Choice b is much more succinct. Also,
choice c adds additional information of the state paying out the
money that is not present in the original sentence. This
information cannot be concluded from what is given in the original
sentence. Therefore, it inappropriately alters the meaning. Choice
d is incorrect because the modal ought to is inappropriate for a
judge’s ruling. Ought to suggests a moral obligation. Also, which
were from selling is awkward, and the verb were does not agree with
the singular subject a total of $6.2 million. Choice e is incorrect
because, like choice c, it suggests that the judge ruled that the state
must pay out the money; this information constitutes an
assumption, not a fact, and it is therefore inappropriate. It also
places the direct object a total of $6.2 million after the indirect
object to the 613 victims; the resulting sentence is awkward and
the direct and indirect objects are best reversed to achieve the
most clarity.
164. d. Choice d correctly uses the that clause after argue with an
effective, concise structure and word choice. Choice a is incorrect
because that is missing after argue; also, diminish the value is not as
succinct as devalue. Choice b is incorrect because the passive
structure makes the sentence wordier without adding any
additional information, and or is used instead of and, which
changes the meaning. Choice c changes the meaning from abuse
intellectual property rights to intellectually abuse property rights. Choice
e is incorrect because intellectually now functions as an adverb
instead of an adjective and or is used instead of and, both of which
change the meaning of the clause.
2 2 9
501_03_121-234_501_master.qxd 4/29/13 12:33 PM Page 230
501 GMAT
®
Questions
165.
a. Choice a is a clear, grammatically correct sentence. Choice b is
not the best choice because it is in the passive voice for the first
part (on whether compromised health will be suffered among gamers in
the long term) and in the active voice for the second (what impact
society would feel). This creates confusion and makes the sentence
not as clear. Choice c is not the best choice because if . . . or not is
redundant; whether is best (without or not). Choice d is incorrect
because the wording alters the meaning from the original sentence:
whether the overall health of gamers will suffer in the long term is more
serious than whether gamers will become not as healthy. And is
changed to or in the sentence, which changes the meaning. Also,
the modal would has been changed to will (the impact their
compromised health will have on society), which changes the meaning.
Choice e is incorrect because the wording once again changes the
meaning. Whether fewer gamers will be healthy is quite different
from whether the overall health of gamers will suffer in the long term.
Also, and has been changed to or, and the final clause or the impact
their health would have on society if they were is illogical. It would
make more sense if were changed to weren’t.
166. e. Choice e is correct because it correctly chooses more than before
twice the national average. More than is used when a quantity is
involved. Furthermore, it represents the most concise and effective
sentence. Choice a is incorrect because greater than must not be
used before twice the national average to describe the rising of
temperatures. Greater than can be used to compare numbers, but
not the rising of the numbers/temperatures. Choice b, in keeping
with, although a correct expression, is not as preferred stylistically
as according to. The sentence would also be more clear if and were
placed after years and before released. Choice c is incorrect because
in accordance with is inappropriate in this sentence. In accordance
with is correctly used with rules, conventions, or established
patterns. Data is not considered one of the aforementioned.
Choice d is incorrect because 50-year-old data is not the same as
data that was compiled over the past 50 years. The change in wording
changes the meaning entirely.
2 3 0
501_03_121-234_501_master.qxd 4/29/13 12:33 PM Page 231
501 GMAT
®
Questions
167. c. Choice c correctly uses the number of before the two countable
nouns fellow graduates and job openings without any redundancy or
wordiness. Choice a is incorrect because the amount of must be
used only before uncountable nouns. Fellow graduates and available
job openings are countable nouns. Who must be substituted by that
since the word refers to the amount of graduates, not the graduates
themselves. Because the amount of is singular, outnumber must be
outnumbers to agree with a singular subject (if the amount of were
correct, which it isn’t). Furthermore, available job openings is
redundant since the word opening already indicates availability.
Choice b is incorrect because the amount of must be replaced by the
number of in both cases, and available job openings is redundant.
Choice d is incorrect because who must be replaced by that to
agree with the subject the number of, exceed must change to exceeds
to agree with the singular subject the number of fellow graduates, and
available job openings is redundant. Choice e is incorrect because
exceed needs to be exceeds to agree with the singular subject the
number of, and a number of job openings must be the number of job
openings.
168. c. Choice c is the correct answer because it is the most succinct,
effective sentence. Choice a is incorrect because among must be
between. Choice b is incorrect because eased . . . to a greater degree is
redundant and transportation is separated from its modifier by water.
To achieve the most clarity, the modifier needs to be placed
directly next to what it defines. Choice d is incorrect because it is
in the passive voice, which results in an ambiguous dependent
clause because since its opening in 1869 doesn’t refer to anything. In
addition, any reference to water has been omitted, thereby
changing the meaning of the sentence. Choice e is incorrect
because the reference to water has also been omitted. It also poses
the same problem as choice d in that the first clause of the
sentence has no logical referent.
2 3 1
501_03_121-234_501_master.qxd 4/29/13 12:33 PM Page 232
501 GMAT
®
Questions
169. d. Choice d is a concise and clear sentence. Choice a is incorrect
because largely . . . to a great deal is redundant. Choice b is
incorrect because helicopter technology is different from technology in
the helicopters; in this case, shortening the expression changes the
meaning. Also, helicopter technology cannot fly, so the idea is
nonsensical. Choice c is incorrect because helicopter technology
cannot fly, only helicopters can. Also, key elements have been
omitted from the clause after the comma; namely, insertion and
broadcast, which serve to indicate that first-down lines in broadcast
games are being referenced, not first-down lines at regular games,
the latter of which are irrelevant to the technology being discussed.
Choice e is incorrect because makes is conjugated in the plural
make to refer to helicopters. Helicopters do not make the insertion
of a first-down line during a football game broadcast seem
insignificant; it is the technology that does. Therefore, this
sentence is incorrect.
170. e. Choice e correctly uses commas to set off the nonrestrictive
clause who wrote the short fairy tale opera Brundibar and uses
concise, efficient language. Choice a, though grammatically sound,
is incorrect because his additional developing as a composer is
awkward; developing sounds better as development. Choice b is
incorrect because only hardly is redundant and farther is not the
correct choice. Farther is used when referring to distance; further
means to a greater extent. Choice c is incorrect because not hardly is
incorrect; the two words cannot be used next to each other. Also,
his developing into a better composer is an awkward phrase; developing
must be changed to development and the selection of better implies
that Krasa may have needed improvement. Choice d is incorrect
because a comma is missing after Krasa and Brundibar to set off the
information between those two words as a nonrestrictive clause.
Also, not hardly is not a correct expression and celebratory does not
mean the same thing as celebrated. The sentence is also phrased in
the passive voice, which results in a wordier and more confusing
sentence.
2 3 2
501_03_121-234_501_master.qxd 4/29/13 12:33 PM Page 233
501 GMAT
®
Questions
171. b. Choice b correctly sets off the three items with semicolons and
uses such as before a list. Choice a is incorrect because like pictures,
music, and videos needs to be replaced with such as pictures, music,
and videos; also, a semicolon is needed before a receiver that supports
the DLNA standard, not a comma. Choice c is incorrect because
the first which must be that to begin a restrictive clause; like needs
to change to such as, and the comma needs to change to a
semicolon before a receiver that supports. Choice d is incorrect
because which needs to change to that for a restrictive clause (it is
essential that the media source or server contain all the media such as
pictures, music, and videos); also, that before new “smart” TVs needs
to change to which for a nonrestrictive clause (which new “smart”
TVs often do is extra information about the translator, not essential
information). Choice e is incorrect because only commas are used.
Semicolons need to separate the three items that are needed to
transfer media across a home; that is, semicolons are needed after
videos and before a receiver, and after do and before and a translator.
172. a. Choice a is correctly constructed to result in a clear,
comprehensible sentence. Choice b is incorrect because if used
repeatedly on an animal is placed in such a way that it defines one
reason. The syntax is wrong and the modifier needs to be moved.
Choice c commits the same error with if used repeatedly on an
animal. It also becomes wordier by adding there are fleas, like
cockroaches, that. . . . There are does not add any value to the
sentence; rather, it only makes it wordier and more confusing.
Choice d is incorrect because increasingly has been moved to
modify popular flea medicine; this is incorrect since it changes the
meaning. Choice e is incorrect because if used repeatedly on an
animal is placed so it defines one reason. The sentence structure
results in an illogical sentence.
2 3 3
501_03_121-234_501_master.qxd 4/29/13 12:33 PM Page 234
501 GMAT
®
Questions
173.
e. Choice e is a concise and clear sentence that includes a
semicolon to logically show the sequence of events in the sentence.
Choice a is incorrect because which should be that since the clause
is restrictive. Choice b is incorrect because the clause following
which should be restrictive without commas and with that instead
of which. Also, the appositive earning him the Nobel Peace Prize in
1987 seems to have been tacked on to the end of the sentence; this
confusing structure makes the sentence awkward. Choice c is
incorrect because its structure implies that Oscar Arias submitted a
peace plan because he received the Nobel Peace Prize. Also,
between needs to change to among. Choice d is incorrect because
which needs to be that, being signed is superfluous, and between
needs to be changed to among.
174. c. Choice c correctly uses the that clause to create a clear, active
sentence. Choice a is incorrect because the noun formation ( people
working) is less preferred than the active that clause formation.
Choice b is incorrect because the latter part of its suggested
variation (as tending to work longer hours, in contradiction of ) is
wordier than choice c. Choice d is incorrect because the verb tense
unnecessarily changes to have indicated, the meaning changes by
excluding tending to, and the noun clause (longer hours of work by) is
less preferred than an active verb clause. Choice e is incorrect
because home-working people is not idiomatic and the modifier
outside the office has been omitted.
2 3 4
501_04_235-298_501_master.qxd 4/29/13 12:35 PM Page 235
4
Reading
Comprehension
Reading comprehension questions on the GMAT tend to be
intimidating and even feared, but they don’t need to be with strategizing
and planning. The subject matter might include unfamiliar subjects with
complicated jargon, but knowing how to read the passages and what to look
for is critical to succeeding. This kind of reading is probably already famil-
iar in your life; it’s only a matter of accessing and employing that skill set
as you take the GMAT. Everyone skims when reading, which helps zero in
on important or necessary information. We do this constantly in life—skim-
ming articles for juicy tidbits, breezing through ingredients to find the one
that’s missing, looking at a map and zeroing in on where to be directed and
so on. It’s eliminating the stress and fear of the test while asserting these
skills that will get you through this section.
The GMAT reading comprehension passages are approximately 350
words in length and will be followed by four or so questions. The passages
vary in structure depending on the subject matter and the author’s purpose.
The questions will cover subjects in science, social science, and business.
There are four primary question types on the GMAT:
understanding words and statements
understanding the logical relationships between significant
points and concepts
501_04_235-298_501_master.qxd 4/29/13 12:35 PM Page 236
501 GMAT
®
Questions
236
drawing inferences from facts and statements
understanding quantitative concepts in verbal material
There is no telling which question type will be on your test, but identify-
ing each question type will help you to know what to look for.
Reading Strategies
Developing a strategy on how you will read these passages is your best
defense against this GMAT section. This will help you to avoid wasting
time looking for the answers after you’ve read the passage.
Skim the passage quickly to get an overall sense of it. Is it objective or
subjective? Is the author presenting an argument or hypothesis or relaying
knowledge on a subject? Is there a specific tone of the passage, for instance,
is it criticizing? Or is it imploring the reader to change an opinion? What
is the overall subject matter and what is being addressed? Briefly assessing
these things will help to generally understand the passage.
Next, read the first question and then read the passage again, this time
reading more closely. Look for transitional words and phrases, such as how-
ever, although, or that being said. Many times these phrases introduce the
main idea or present the argument of the passage. By reading the first ques-
tion before getting started on your closer reading, you are multitasking—
you are looking closely at the passage and hunting for the answer to the first
question at the same time.
Understanding Words and Statements
The ability to determine the meaning of unfamiliar words from their con-
text is an essential skill for reading comprehension. Sometimes, there will
be unfamiliar words whose meaning you can’t determine without a dic-
tionary. But more often than not, a careful look at the context will give you
enough clues to meaning. Larger words can be broken down to their roots,
or can be deciphered by looking at the prefix or suffix.
The general tone or theme of the text can also help you figure out the
meaning of an unfamiliar word. Titles can also provide clues about the tone
of a story and the type of vocabulary words that are likely to be found in the
501_04_235-298_501_master.qxd 4/29/13 12:35 PM Page 237
237
text. Tone questions are prevalent on the GMAT; a question might ask you
to identify the slant of the author or the motivation for writing the passage.
If you are unfamiliar with a particular word, use context clues to try to
figure out its meaning. Draw on the important clues in the sentences that
appear directly before and after the unfamiliar word or passage.
Punctuation can also help you decipher unfamiliar words. Parentheses
are often used to highlight or explain words or phrases and elaborate on the
words that precede them.
Understanding the Structure between Significant
Points and Concepts
On the GMAT, some questions require you to understand the different
points and concepts being presented in a passage. Identifying the author’s
main idea and the secondary, or supporting, ideas is key to answering these
types of questions.
Main idea questions are prevalent on the GMAT. They test your abil-
ity to assess the passage as a whole and identify a summation of it. The
trick is to know the difference between supporting ideas that support the
main idea and the main idea itself. Many times, in a main idea question, the
answer choices are filled with supporting ideas that are true, but they are
not the central theme. Identifying the difference early will help you weed
out incorrect answer choices. Main idea questions are often presented in
the first paragraph of a passage. Sometimes, the main idea is reiterated in
the last paragraph. This is where to best look for your answer to a main
idea question.
Supporting idea questions are also prevalent on the GMAT. The answers
always draw on facts given in the passage, not on assumptions you can make
after reading the passage. The answers are often an exact paraphrase of the
facts given in the passage.
There may be a few questions that require you to apply information.
These questions are a blend of structure and main idea/secondary idea
questions. You may be asked to identify basic passage structure and signifi-
cant relationships or scenarios, and then asked to apply that information to
a completely different set of circumstances. You are looking for the simi-
larities of the structure, rather than the similarities of the subject, when
answering these questions.
501 GMAT
®
Questions
501_04_235-298_501_master.qxd 4/29/13 12:35 PM Page 238
Drawing Inference from Facts and Statements
There will be several inference questions on the GMAT. In these questions,
information is not simply stated; however, specific clues, structure, and lan-
guage will lead to a conclusion. Identifying the inference requires the appli-
cation of all the facts of the passage, then drawing a reasonable assumption.
Some questions may ask you to determine an implied main idea. Find-
ing an implied main idea is a lot like finding the stated main idea. A main
idea is defined as an assertion about the subject that controls or holds
together all the ideas in the passage. Therefore, the main idea must be gen-
eral enough to encompass all the ideas in the passage. Much like a net, it
holds everything in the passage together. Implied main ideas are the same;
they are just an invisible net.
GMAT questions are not likely to make wild inferences. They are gen-
erally modest and logical, with very little emotion. Eliminate your personal
opinion at all times; it will lead you in the wrong direction. Utilize logic
instead. Keep the tone of the passage in mind when you’re answering these
questions, as well. Tone and inference cannot contradict one another, so any
answer choices that are not parallel with the tone can be eliminated.
Physical or Biological Science Passages
Physical or biological sciences are one of three different general subjects
explored on the GMAT. Generally, these passages will deal with fieldwork
or a study in progress and its subsequent report. That being said, an angle
or subjectivity can often be found in these passages, either refuting or sup-
porting the main idea. Focus on finding the central idea at once; it’s likely
a hypothesis of some kind.
The topics will range a wide variety of scientific areas, but will generally
stay current. A scientific breakthrough in 1987 isn’t relevant unless the find-
ings support a study in progress today.
Set 7
Now it is time to answer GMAT reading comprehension practice questions
that have been designed to test your physical or biological science skills.
Read the passages and answer the questions that follow. Good luck!
238
501 GMAT
®
Questions
239
Antibiotics and similar drugs, together called antimicrobial agents,
have been used for the last 70 years to treat patients who have infec-
tious diseases. Since the 1940s, these drugs have greatly reduced illness
and death from infectious diseases. Antibiotics have been beneficial
and, when prescribed and taken correctly, their value in patient care is
enormous. However, these drugs have been used so widely and for so
long that the infectious organisms the antibiotics are designed to kill
have adapted to them, making the drugs less effective. Many fungi,
viruses, and parasites have done the same. Some microorganisms may
develop resistance to a single antimicrobial agent (or related class of
agent), while others develop resistance to several antimicrobial agents
or classes. These organisms are often referred to as multidrug-resistant
or MDR strains. In some cases, the microorganisms have become so
resistant that no available antibiotics are effective against them.
Antimicrobial drug resistance occurs everywhere in the world and is
not limited to industrialized nations. Hospitals and other healthcare
settings are battling drug-resistant organisms that spread inside these
institutions. Drug-resistant infections also spread in the community at
large. Examples include drug-resistant pneumonias, sexually transmit-
ted diseases (STDs), and skin and soft tissue infections.
People infected with drug-resistant organisms are more likely to
have longer and more expensive hospital stays, and may be more likely
to die as a result of the infection. When the drug of choice for treating
their infection doesn’t work, they require treatment with second- or
third-choice drugs that may be less effective, more toxic, and more
expensive. This means that patients with an antimicrobial-resistant
infection may suffer more and pay more for treatment.
175.
Which of the following would the author be most likely to
recommend regarding this passage?
a.
A stronger focus in research should be implemented to combat
multidrug-resistant strains.
b.
An increase in funding should be given to third world countries
in an effort to both treat and research new MDR strains.
c.
Staph and MRSA should be listed as deadly diseases.
d.
Safe sex education should be reprioritized as paramount, as it’s a
controllable variable whereas the spreading of soft tissue infec-
tions isn’t.
e.
Healthcare professionals should lower their fees for people who
suffer from MDR strains.
501 GMAT
®
Questions
501_04_235-298_501_master.qxd 4/29/13 12:35 PM Page 239
501_04_235-298_501_master.qxd 4/29/13 12:35 PM Page 240
240
501 GMAT
®
Questions
176. Which of the following best expresses the main idea of the
passage?
a. Drug-resistant infections are causing an increase in hospital and
medical fees.
b. Drug-resistant pneumonia and fierce STDs are on their way to
becoming a global epidemic unless preparedness action is
aggressively taken.
c. The longtime use of certain antibiotics has rendered them inef-
fective against the ever-adapting viruses they combat.
d. The value of antibiotics has been underestimated; hence, its
continuing development has been neglected.
e. Antibiotics are frequently not taken correctly, weakening the
immune system against MDR strains.
177. Of the following situations, which most resembles the information
regarding antibiotics in the passage?
a. a superhero who has lost his invisibility powers
b. a cucumber in a salt solution halfway on a course to being a
pickle
c. a sixteenth-century castle surrounded by a giant moat hosting a
kingdom that is being attacked by a rival empire that has canoes
d. two apple pies entered in a bake-off, one made with Granny
Smith apples and one made with Red Delicious
e. Buying tickets to a sold-out show on the street outside the
venue rather than on the Internet through an overinflated
third-party source
178. What is most likely the structural purpose of the fourth sentence?
a. to report more data regarding the author’s interest in the study
b. to begin a comparison between two different ideas
c. to introduce a rebuttal to a widely accepted opinion
d. to vehemently denounce an institution’s stance
e. to appeal to the reader’s sense of compassion
501_04_235-298_501_master.qxd 4/29/13 12:35 PM Page 241
241
179. Based on the information in the passage, which of these statements
is true?
a. As infectious organisms evolve, less effective and more expen-
sive drugs will have to be used for treatment.
b. Hospitals are the only institutions that benefit from the ineffi-
cacy of antibiotics since they’re likely to reap more capital with
increased hospital stays.
c. People who live in industrialized areas are more prone to con-
tracting an MDR strain.
d. If antibiotics were taken as prescribed more often, the viruses
being fought would never have had the chance to adapt to
them.
e. We can expect a rise in sexually transmitted diseases in our soci-
ety since we’ve lost the power to treat them.
180. The tone of this passage could be described as:
a. critical
b. sarcastic
c. admonishing
d. infuriated
e. resentful
Biotechnology can be broadly defined as “using living organisms or
their products for commercial purposes.” As such, biotechnology has
been practiced since the beginning of recorded history in such activi-
ties as baking bread, brewing alcoholic beverages, or breeding food
crops or domestic animals.
A narrower and more specific definition of biotechnology is “the
commercial application of living organisms or their products, which
involves the deliberate manipulation of their DNA molecules.” This
definition implies a set of laboratory techniques developed within the
last 20 years that have been responsible for the tremendous scientific
and commercial interest in biotechnology, the founding of many new
companies, and the redirection of research efforts and financial
resources among established companies and universities.
Combining DNA from different existing organisms (plants, animals,
insects, bacteria, etc.) results in modified organisms with a combina-
tion of traits from the parents. The sharing of DNA information takes
501 GMAT
®
Questions
501_04_235-298_501_master.qxd 4/29/13 12:35 PM Page 242
242
501 GMAT
®
Questions
place naturally through sexual reproduction and has been exploited in
plant and animal breeding programs for many years.
However, sexual reproduction can occur only between plants and
animals of the same species. A Holstein cow can be mated with a
Hereford bull because the two animals are different breeds of the same
species, cattle. But trying to mate a cow with a horse, a different
species of animal, would not be successful.
What’s new since 1972 is that scientists have been able to identify
the specific DNA genes for many desirable traits and transfer only
those genes, usually carried on a plasmid or virus, into another organ-
ism. This provides a method to transfer DNA between any living cells
(plant, animal, insect, bacterial, etc.). Virtually any desirable trait found
in nature can, in principle, be transferred into any chosen organism.
An organism modified by genetic engineering is called transgenic.
Specific applications of genetic engineering are abundant and
increasing rapidly in number. Genetic engineering is being used in the
production of pharmaceuticals, gene therapy, and the development of
transgenic plants and animals.
181. Which of the following best describes the tone of the passage?
a. ambivalent
b. critical
c. defensive
d. enthusiastic
e. objective
182. Based on what’s been said thus far, what is the next paragraph most
likely to discuss?
a. the effects of what would happen if a cow and horse were mated
b. the specific examples of biotechnology and its benefits to
science
c. what happened in 1972 and which scientists were involved
d. who opposes the use of biotechnology
e. how the production of beer is not so different from the produc-
tion of pharmaceuticals
501_04_235-298_501_master.qxd 4/29/13 12:35 PM Page 243
243
501 GMAT
®
Questions
183. The outcome of the mating of a Holstein cow and a Hereford bull
would be called:
a. an abomination
b. transgenic
c. improbable
d. manipulation
e. transference
184. The relationship between transgenic plants and biotechnology is
most like the relationship between:
a. a graham cracker crust and apple pie
b. hardwood floors and a shag carpet
c. an airline pilot and a jet plane
d. opiate abuse and addiction
e. sheet music and a grand piano
185. According to the passage, all the following are products of
biotechnology EXCEPT:
a. new lines of pharmaceuticals
b. the breeding of transgenic animals
c. the production of alcoholic beverages
d. the development of viral warfare
e. the transferring of DNA between different organisms
186. Which of the following best summarizes the main idea of the
passage?
a. The sweeping changes in the use of biotechnology since 1972
have redefined its necessity to mankind.
b. Although still in its fledgling stages, biotechnology shows
promise to help benefit our society one day.
c. As the science of biotechnology continues to broaden and
strengthen, the fruits of its applications become more powerful
and abundant.
d. The swapping of DNA could sometimes yield beneficial results,
but on the whole is too risky to undertake.
e. The development of transgenic plants is paramount to the
development of better pharmaceuticals.
501_04_235-298_501_master.qxd 4/29/13 12:35 PM Page 244
244
501 GMAT
®
Questions
Men with high risk for heart disease had lower blood pressure after
drinking nonalcoholic red wine every day for 4 weeks, according to a
new study in the American Heart Association journal Circulation
Research.
Nonalcoholic red wine increased participants’ levels of nitric oxide,
which helped decrease both systolic and diastolic blood pressure,
researchers said. Nitric oxide is a molecule in the body that helps
blood vessels relax and allows more blood to reach your heart and
organs.
Researchers studied 67 men with diabetes or three or more cardio-
vascular risk factors, who ate a common diet plus one of the following
drinks: about 10 ounces of red wine, nonalcoholic red wine, or about 3
ounces of gin. All the men tried each diet/beverage combination for 4
weeks.
The red wine and nonalcoholic wine contained equal amounts of
polyphenols, an antioxidant that decreases blood pressure.
During the red wine phase, the men had very little reduction in
blood pressure and there was no change while drinking gin. However,
after drinking nonalcoholic red wine, blood pressure decreased by
about 6 mmHg in systolic and 2 mmHg in diastolic blood pressure—
possibly reducing the risk of heart disease by 14% and stroke by as
much as 20%.
Researchers concluded that the alcohol in red wine weakens its abil-
ity to lower blood pressure. But polyphenols—still present after alco-
hol is removed from wine—are likely the beneficial element in wine.
187. The study in the passage would best conclude that
a. alcohol has no effect on blood pressure.
b. alcoholics are the most likely to suffer strokes.
c. the antioxidants present in red wine that are beneficial to health
concerns are negated by alcohol.
d. polyphenols are detrimental to health.
e. drinking gin is a useless endeavor.
501_04_235-298_501_master.qxd 4/29/13 12:35 PM Page 245
245
501 GMAT
®
Questions
188. Based on the information in the passage, nonalcoholic red wine
will have a better impact on combating
a. neither heart disease nor stroke.
b. stroke.
c. heart disease.
d. stroke and heart disease.
e. nitric oxide.
189. It can be inferred from the passage that diabetic men
a. have a propensity for eating and drinking beyond the point of
average human consumption.
b. are susceptible to having a negative reaction to nitric oxide.
c. are at higher risk than the average male for developing heart
disease.
d. have a physiological reaction to red wine.
e. have blood pressure that’s anywhere from 14% to 20% higher
than the average male’s.
190. According to the information in the passage, which of the
following best expresses the main idea?
a. The consumption of alcoholic beverages is constantly proving
to be detrimental to health.
b. Polyphenols are of great value to the human body.
c. The alcohol in red wine negates the positive effects the con-
sumption of polyphenols can achieve.
d. Nitric oxide helps blood vessels to relax, allowing more blood
flow to the heart and brain.
e. The risk of heart disease can be reduced by drinking red wine
191. The purpose of paragraph 5 is to
a. introduce the variables that will support a thesis.
b. present the counterpoint to the passage.
c. include the support of additional peripheral expertise.
d. present the findings for the conclusion of a study.
e. defend an opinion.
501_04_235-298_501_master.qxd 4/29/13 12:35 PM Page 246
246
501 GMAT
®
Questions
192. The two different wines mentioned in the passage most resemble:
a. a Labrador retriever and a Labradoodle
b. whole milk and lactose-free milk
c. Granny Smith apples and Fuji apples
d. a gas oven and a microwave oven
e. a Big Wheel and a bicycle
Widespread media coverage tells us today of a new drug that “halts”
Alzheimer’s symptoms “for three years.” The news is based on a press
release issued yesterday that highlighted positive early results of
research into the use of intravenous immunoglobulin to treat
Alzheimer’s disease.
Intravenous immunoglobulin (IVIG) is a medication made by har-
vesting antibodies from donated blood. It is currently used to treat
severe forms of infection and a number of autoimmune conditions
(where the immune system attacks healthy tissue).
The idea behind using IVIG to treat Alzheimer’s disease is that it
could encourage the immune system to “attack” abnormal clumps of
protein (amyloid plaques) that can develop in the brains of people with
Alzheimer’s disease.
Some media coverage of the press release was inaccurate. The Daily
Express tells us there is a “pill to beat Alzheimer’s” when IVIG is actu-
ally given by injection into a blood vessel. The Daily Mail describes it
as a “new vaccine,” which is technically incorrect as it implies only one
injection needs to be given when in fact IVIG was injected every two
weeks.
Once past the somewhat misleading headlines, most coverage does
mention that it may be 10 years before this drug can be available, if it
passes further scrutiny. IVIG can also be very expensive to manufac-
ture, so this may limit its availability through the national health
service.
Limited conclusions can be drawn from this research as it is in its
early stage, was conducted on a small number of people, and was not
peer-reviewed. Larger studies that compare IVIG to other existing
treatments for Alzheimer’s disease are required to determine how safe
and effective the drug is.
501_04_235-298_501_master.qxd 4/29/13 12:35 PM Page 247
247
193. Which of the following would be the most fitting title to this
passage?
a. New Drug to Combat Alzheimer’s to Be Released on the
Market
b. Alzheimer’s Cure a Complete Fabrication
c. Miracle Pill for Alzheimer’s Has Patients Hopeful
d. Weakened Immune Systems of Patients to Blame for
Alzheimer’s
e. Reports of Potential Breakthrough Alzheimer’s Treatment Both
Premature and Misinformed
194. The author’s attitude about the medication being studied to treat
Alzheimer’s would be best described as:
a. skeptical
b. ambivalent
c. stoic
d. enthusiastic
e. cantankerous
195. According to the passage, all of the following are true about IVIG
EXCEPT:
a. IVIG will be administered by injection.
b. IVIG will encourage the immune system to fight proteins in the
brain.
c. IVIG can be defined as a vaccine.
d. IVIG may not be available for 10 years or more.
e. IVIG may not be cost-efficient to produce.
196. Which of the following would best describe the purpose of the
fourth paragraph?
a. It refutes an argument presented by others in the medical field.
b. It provides an objective approach that has been expounded
upon by many.
c. It serves to introduce the author’s misgivings about the media.
d. It denounces the accuracy of a study.
e. It combines the background information with the opinions of
different medical professionals.
501 GMAT
®
Questions
501_04_235-298_501_master.qxd 4/29/13 12:35 PM Page 248
248
501 GMAT
®
Questions
197. Which of the following statements is the author most likely to
agree with?
a. Exciting new medical breakthroughs regarding the treatment of
Alzheimer’s could be around the corner, but it is unknown how
long the road ahead is.
b. The use of IVIG should be introduced to senile mice before it
is used to treat people.
c. Research dedicated to a drug that doesn’t attack abnormal
clumps but rather coaxes them should be paramount for any sci-
entists involved in this study.
d. The media have a tendency to deliver misleading information
to the public because sensationalism sells.
e. A national healthcare system could introduce medications to
people with Alzheimer’s who couldn’t afford it otherwise.
Even the smallest quantity of salmonella may, in the future, be easily
detected with a technology known as SERS, short for “surface-
enhanced Raman scattering.” U.S. Department of Agriculture (USDA)
scientist Bosoon Park at Athens, GA, is leading exploratory studies of
this analytical technique’s potential for quick, easy, and reliable detec-
tion of salmonella and other foodborne pathogens.
According to the U.S. Centers for Disease Control and Prevention,
salmonella causes more than one million cases of illness in the United
States every year.
If SERS proves successful for cornering salmonella, the technique
might be used at public health laboratories around the nation to rapidly
identify this or other pathogens responsible for outbreaks of foodborne
illness, according to Park, an agricultural engineer. What’s more, tomor-
row’s food makers might use SERS at their in-house quality control labs.
The Agricultural Research Service is the USDA’s chief intramural
scientific research agency. Park’s research supports the USDA priority
of enhancing food safety.
In a SERS analysis, a specimen is placed on a surface, such as a
stainless steel plate, that has been “enhanced,” or changed from
smooth to rough. For some of their research, Park’s team enhanced the
surface of stainless steel plates by coating them with tiny spheres, made
up of a biopolymer encapsulated with nanoparticles of silver.
Rough surfaces, and colloidal metals such as silver, can enhance the
scattering of light that occurs when a specimen, placed on this
nanosubstrate, is scanned with the Raman spectrometer’s laser beam.
501_04_235-298_501_master.qxd 4/29/13 12:35 PM Page 249
249
501 GMAT
®
Questions
The scattered light that comes back to the spectroscope forms a dis-
tinct spectral pattern known as a Raman spectral signature, or Raman
scattered signal. Researchers expect to prove the concept that all mole-
cules, such as those that make up salmonella, have their own unique
Raman spectral signature.
The idea of using a substrate of silver nanoparticles for Raman spec-
troscopy is not new. But in SERS studies to detect foodborne
pathogens, the use of a surface enhanced with biopolymers coated with
silver nanoparticles is apparently novel.
In work with comparatively large concentrations of two different
kinds, or serotypes, of salmonella enterica—enteritidis and
typhimurium—Park’s tests showed, apparently for the first time, that
SERS can differentiate these two serotypes. With further research,
SERS may prove superior for finding very small quantities of bacteria
in a complex, real-world background, such as a food or beverage sam-
ple, Park notes.
198. Which of the following is NOT like the rest of the answer
choices?
a. foodborne pathogens
b. salmonella
c. typhimurium
d. clemteritidis
e. biopolymers
199. Which of the following most accurately expresses the main idea of
the passage?
a. Colloidal metals such as silver can enhance the scattering of
light that occurs when a specimen is placed under a Raman
laser.
b. Scientists are close to finding a way to detect salmonella and
other foodborne illnesses with the development of a surface-
enhanced Raman scattering laser.
c. The U.S. Department of Agriculture has been diligently look-
ing for an effective way to detect foodborne illness earlier.
d. The idea of using a substrate of silver nanoparticles is revolu-
tionary and SERS is one way of illustrating that.
e. According to the Centers for Disease Control and Prevention,
salmonella causes a great number of health problems that the
general public is not aware of.
501_04_235-298_501_master.qxd 4/29/13 12:35 PM Page 250
250
501 GMAT
®
Questions
200. According to the passage, a nanosubstrate is
a. a laser beam that can detect salmonella.
b. tiny spheres made of a colloidal metal.
c. an enhanced surface that the specimen being tested will be
placed on.
d. the scattered light that comes back to the spectroscope after
using SERS.
e. a foodborne pathogen.
201. The enhanced surface described in the passage most resembles
which of the following?
a. a stripped screw that no drill bit can reckon with
b. a peach’s skin as opposed to an apple’s skin
c. a skinned knee after a few days
d. a cat’s reaction to a red laser beam light
e. an old tire’s tread
202. The author’s inclusion of the information in the second paragraph
is most likely intended to
a. criticize the work of the United States Centers for Disease
Control.
b. support the research of Park for the SERS project.
c. alienate the specific problems salmonella can produce.
d. emphasize the importance of the research being conducted.
e. reiterate the main idea of the article.
Social Science Passages
The second type of passage found on the GMAT deals with the social sci-
ences. These passages might touch on history, geography, and politics.
They may cover current social topics. These passages are probably the eas-
iest to read; they are unlikely to contain scientific jargon and business lan-
guage. They are, however, most likely to be accompanied by inference
questions, so read carefully. Try not to jump to conclusions based on your
opinions and judgments.
501_04_235-298_501_master.qxd 4/29/13 12:35 PM Page 251
251
501 GMAT
®
Questions
Set 8
Now it is time to answer GMAT reading comprehension practice questions
that have been designed to test your social science skills. Read the passages
and answer the questions that follow. Good luck!
In 1891, the federal government assumed responsibility from the states
for regulating immigration through the Immigration Act of 1891,
which established the Office of Immigration (later the Bureau of
Immigration) to administer immigration affairs. The government also
appropriated money to build a new immigrant inspection station on
Ellis Island. The Immigration Act assigned the Marine Hospital Service
(later the Public Health Service) the responsibility of examining the
health of immigrants entering the United States.
Before construction of Ellis Island’s first immigration depot began,
the island was doubled in size with landfill. A ferry slip was dredged
and a dock installed next to the main building site. A number of older
buildings from the island’s time as a military post were adapted for
reuse. Ellis Island’s first immigration building, constructed of Georgia
pine, opened on January 1, 1892.
Due to the economic depression at the time, immigration was light
and Ellis Island inspectors had no difficulty processing the fewer than
20,000 immigrants who arrived annually. On June 15, 1897, a fire
destroyed the complex of wooden buildings. Although 140 immigrants
and numerous employees were on the island, no one was killed.
The government announced almost immediately that Ellis Island
would be rebuilt with fireproof buildings. The New York architectural
firm of Boring and Tilton was awarded the contract after a competi-
tion entered by five outstanding firms, including McKim, Mead &
White.
The first building to be built was the new main immigration build-
ing, which opened on December 17, 1900. Following its completion,
the kitchen and laundry and powerhouse buildings were erected in
1901 and the island was enlarged by landfill to make room for a hospi-
tal complex. In March of 1902, the main hospital building opened.
The hospital had the space and equipment to care for 125 patients but
it was still not enough—the hospital was overwhelmed with patients
diagnosed with trachoma, favus, and other contagious illnesses that
501_04_235-298_501_master.qxd 4/29/13 12:35 PM Page 252
252
501 GMAT
®
Questions
warranted exclusion. Over the next seven years, additional buildings
were added to the hospital complex, including the Hospital Addition /
Administration building, the New Hospital Extension, and the Psy-
chopathic Ward. The island was also enlarged once more using land-
fill, which allowed for the construction of a contagious disease hospital
and isolation wards, as well as additional support buildings.
203. Which of the following statements best summarizes the passage?
a. Architecture on Ellis Island set a precedent on how immigrants
were treated, and was used as a means of segregation.
b. Immigrants were forced to stay in hospitals if they were not
well enough to help build new buildings on the island before
they were admitted to the United States.
c. Ellis Island had a health epidemic due to disease and illness
brought by immigrants that led the government to build large
extensions on the existing hospital and surrounding grounds.
d. The island had to be enlarged through the use of landfill to add
many isolation wards for the mentally ill who were to be
deported.
e. Due to a large fire, thousands of immigrants were deported
until the island was completely demolished and rebuilt after the
Great Depression.
204. What role did the federal government play in regulating
immigration on Ellis Island?
a. The government consulted an award-winning architecture firm
that was hired to design buildings that had only small capacity,
thus forcing overflow to be deported.
b. They oversaw the employment of every physically able immi-
grant to work on construction sites before they were accepted
into the United States.
c. The government set up mandates such as one requiring that all
immigrants be tested for literacy upon entering the United
States.
d. Certain age groups were not allowed to enter the United States.
e. The government oversaw the physical and mental inspection of
all people trying to enter the United States.
501_04_235-298_501_master.qxd 4/29/13 12:35 PM Page 253
253
501 GMAT
®
Questions
205. Which of the following statements presented in paragraph 2 helps
to further the passage the most?
a. The island was doubled in size with landfill.
b. Immigration was light due to an economic depression.
c. Older buildings already on the island were repurposed to func-
tion for immigration.
d. The buildings were constructed of Georgia pine.
e. The first immigration building was opened on January 1, 1892.
206. Based on the information provided, the author of the passage
would most likely agree with which of the following statements?
a. If the buildings had been made with fire-safe materials the first
time around, a lot of energy could have been spared rebuilding
the island.
b. It would have been advantageous to have built the hospital
before any of the other buildings on the island.
c. An economic depression spared the island from being over-
whelmed much sooner than it was.
d. The island was enlarged by landfill to accommodate people as
more responsibilities for them presented themselves.
e. The creation of competition to erect the buildings on Ellis
Island helped to keep the architects at their very best when
drafting plans for expansion.
207. What was constructed after the completion of the third landfill?
a. the main house constructed of Georgia pine
b. the laundry room
c. a psychopathic ward
d. a contagious disease ward
e. the main hospital building
Almost 12 million immigrants were processed through the immigra-
tion station on Ellis Island between 1892 and 1954 when the station
closed. By 1924, however, the number of immigrants being processed
at Ellis Island had been significantly reduced by anti-immigration leg-
islation designed to establish quotas by nationality. This legislation
dramatically reduced the number of immigrants allowed to enter the
United States.
501_04_235-298_501_master.qxd 4/29/13 12:35 PM Page 254
254
501 GMAT
®
Questions
The Emergency Quota Act, passed in 1921, ended the United
States’ open door immigration policy. The law significantly reduced
the number of admissions by setting quotas according to nationality.
The number of people of each nationality that could be admitted to
the United States was limited to 3% of that nationality’s representation
in the U.S. census of 1910. The law created havoc for those on Ellis
Island and thousands of immigrants were stranded on the island await-
ing deportation. The island sometimes became so overcrowded that
officials had to admit excess-quota immigrants.
The First Quota Act was replaced with the even more restrictive
Immigration Act of 1924. This act further limited admissions of each
nationality to the United States to 2% of that nationality’s representa-
tion in the 1890 census. The act sought not only to limit admissions to
the United States, but also to curtail immigration of southern and east-
ern Europeans, who by the 1900s comprised over 50% of the immi-
grant flow. Additionally, the Immigration Act of 1924 allowed
prospective immigrants to undergo inspection before they left their
homeland, making the trip to Ellis Island unnecessary.
Anti-immigration legislation passed in the 1920s, as well as the
Great Depression, kept immigration at an all-time low. For the first
time in Ellis Island’s history, deportation far outnumbered admissions.
In view of this situation, the Ellis Island Advisory Committee (a com-
mittee appointed by the Department of Labor under Franklin D.
Roosevelt’s New Deal program) advised that new buildings be erected
for detained immigrants to separate them from deportees, who were
often criminals. This final surge of construction included the new
immigration building, the new ferry house, and the new recreation
building and recreation shelters.
208. According to the passage, what was the main reason that
immigration numbers fell during the 1920s?
a. the enactment of anti-immigration legislation
b. the Great Depression
c. overcrowding at Ellis Island
d. overwhelming disease
e. None of these are accurate.
501_04_235-298_501_master.qxd 4/29/13 12:35 PM Page 255
501 GMAT
®
Questions
255
209. Which best represents the main idea of the passage?
a. Quota legislation was designed to control the number of immi-
grants based on their nationality.
b. Due to criminality, a series of quotas were designed and imple-
mented.
c. The Emergency Quota was enacted to ostensibly shut the door
on the United States’ open door policy until after the Great
Depression.
d. The enactment of quotas only allowed 1% of one nationality
that was currently represented in the United States in at a time.
e. The threat of deportation caused widespread panic on Ellis
Island during the 1920s.
210. The tone of the passage would be best described as:
a. restrained
b. nostalgic
c. inflammatory
d. informative
e. contemptuous
211. The Immigration Act of 1924 would most likely negatively affect
which of the following groups of people?
a. the Ellis Island Advisory Committee
b. citizens of Greece wishing to emigrate
c. citizens of Great Britain wishing to emigrate
d. the Lower East Side of New York City
e. citizens of Italy and Great Britain wishing to emigrate
212. What is the purpose of the fourth paragraph?
a. to support a theory previously expanded on
b. to paraphrase the information already given and reach a general
conclusion
c. to refute the facts provided
d. to pinpoint specific problems with the outcome of previous
paragraphs
e. to criticize an institution
501_04_235-298_501_master.qxd 4/29/13 12:35 PM Page 256
256
501 GMAT
®
Questions
The intense preparation required for the law school admission test
(LSAT) changes the structure of the brain, resulting in stronger con-
nections between areas of the brain that play an important role in
reasoning.
That’s the finding of University of California, Berkeley, neuroscien-
tists who used diffusion tensor imaging to analyze the brains of 24 col-
lege students or recent graduates before and after 100 hours of LSAT
training over three months.
The findings suggest that training people in reasoning skills can
reinforce brain circuits involved in thinking and reasoning and might
even help increase a person’s IQ scores, the researchers said.
“The fact that performance on the LSAT can be improved with
practice is not new. People know that they can do better on the LSAT,
which is why preparation courses exist,” study leader Allyson Mackey,
a graduate student in UC Berkeley’s Helen Wills Neuroscience Insti-
tute, said in a university news release.
“What we were interested in is whether and how the brain changes
as a result of LSAT preparation, which we think is, fundamentally, rea-
soning training. We wanted to show that the ability to reason is mal-
leable in adults,” she explained.
The U.S. National Institute of Neurological Disorders and Stroke
funded the study, along with Blueprint Test Preparation, the release
noted.
The study was published recently in the journal Frontiers in Neu-
roanatomy.
“A lot of people still believe that you are either smart or you are not,
and sure, you can practice for a test, but you are not fundamentally
changing your brain,” senior author Silvia Bunge, an associate profes-
sor in the UC Berkeley department of psychology and the Helen Wills
Neuroscience Institute, said in the news release.
“Our research provides a more positive message. How you perform
on one of these tests is not necessarily predictive of your future suc-
cess, it merely reflects your prior history of cognitive engagement, and
potentially how prepared you are at this time to enter a graduate pro-
gram or a law school, as opposed to how prepared you could ever be,”
Bunge noted.
Another expert, John Gabrieli, a professor of cognitive neuroscience
at the Massachusetts Institute of Technology, agreed.
501_04_235-298_501_master.qxd 4/29/13 12:35 PM Page 257
257
501 GMAT
®
Questions
“I think this is an exciting discovery,” Gabrieli, who was not
involved in the study, said in the news release. “It shows, with rigorous
analysis, that brain pathways important for thinking and reasoning
remain plastic in adulthood, and that intensive, real-life educational
experience that trains reasoning also alters the brain pathways that
support reasoning ability,” he explained.
213. Which of the following best describes the tone of this passage?
a. passionate
b. despondent
c. articulate
d. opinionated
e. arrogant
214. All the following institutions were included in the study EXCEPT:
a. University of California at Berkeley
b. Helen Wills Neuroscience Institute
c. Massachusetts Institute of Technology
d. The U.S. National Institute of Neurological Disorders and
Stroke
e. Blueprint Test Preparation
215. The purpose of paragraph 3 is best described by which of the
following?
a. to present more cumulative data
b. to introduce an argument that defies a theory
c. to introduce the comparative study of two different theories
d. to paraphrase the hypothesis
e. none of the above
216. This passage suggests:
a. insight to how test takers will score in the future
b. humans have pliable minds capable of change despite how smart
we are.
c. people are born with little or no ability to reason.
d. lawyers are more likely to have heightened reasoning skills.
e. 100 hours is enough time to affect a person’s ability to reason.
501_04_235-298_501_master.qxd 4/29/13 12:35 PM Page 258
258
501 GMAT
®
Questions
217. Which of the following best describes the main idea of the
passage?
a. An increase in IQ is possible through training that will
strengthen brain circuits used for reasoning and thinking.
b. The U.S. National Institute of Neurological Disorders and
Stroke funded the study because of the inferred link between a
brain in atrophy and stroke.
c. Merely practicing for a test can make your brain fundamentally
change chemically.
d. Preparedness can predict your future success and reflect your
prior cognitive engagement.
e. Preparing for law school will inadvertently shift an individual’s
IQ upward.
218. According to the passage, all of the following are true EXCEPT:
a. Possible improvement on the LSAT with training and prepara-
tion is old news.
b. Even after preparation, a score of a test is only predictive of
your brain function at the time of taking the test and not of how
you could score in the future.
c. Brain function specific to reasoning is malleable through adult-
hood.
d. Intensive, real-life educational experience that trains reasoning
cannot alter the brain pathways that support reasoning ability.
e. The LSAT fundamentally amounts to reasoning training.
Thirty percent of health spending in the United States in 2009—about
$750 billion—was wasted on unnecessary services, excessive adminis-
tration costs, fraud, and other problems, a government advisory panel
said Thursday.
The report from the Institute of Medicine urges that changes be
made to the United States healthcare system to reduce costs and
improve care.
Institute of Medicine experts added, however, that inefficiency, a
vast amount of data, and other economic and quality issues obstruct
efforts to improve health and threaten the nation’s economic stability
and global competitiveness, the document warned.
Numerous inefficiencies caused needless suffering. One estimate
indicates that about 75,000 deaths might have been prevented in 2005
501_04_235-298_501_master.qxd 4/29/13 12:35 PM Page 259
259
501 GMAT
®
Questions
if every state had delivered healthcare at the level of the best-
performing state.
Gradual upgrades and changes by individual hospitals or healthcare
providers are inadequate to solve the problems, the report committee
said.
“Achieving higher-quality care at lower cost will require an across-
the-board commitment to transform the U.S. health system into a
‘learning’ system that continuously improves by systematically captur-
ing and broadly disseminating lessons from every care experience and
new research discovery,” according to an Institute of Medicine news
release.
Solutions include greater use of electronic health records, promot-
ing patient and family involvement in healthcare decision making, and
quicker adoption of medical breakthroughs.
“It will necessitate embracing new technologies to collect and tap
clinical data at the point of care, engaging patients and their families as
partners, and establishing greater teamwork and transparency within
healthcare organizations,” according to the news release. “Also, incen-
tives and payment systems should emphasize the value and outcomes
of care.”
“The nation has the knowledge and tools to improve the health sys-
tem so it can provide better quality care at lower cost,” the report
authors said.
“The threats to Americans’ health and economic security are clear
and compelling, and it’s time to get all hands on deck,” report commit-
tee chairman Mark Smith, president and CEO of California Health-
Care Foundation, said in the news release.
219. This passage is primarily concerned with
a. the advancement of technology to better aid the healthcare
system.
b. the pain and suffering current patients are living with due to an
incompetent system.
c. the paramount necessity to lower spending while boosting care
quality within the medical field.
d. the ill effects of poorly appropriated budget and its inadvertent
consequences.
e. the inevitable decline in our healthcare due to fraud, theft, and
shoddy products.
501_04_235-298_501_master.qxd 4/29/13 12:35 PM Page 260
260
501 GMAT
®
Questions
220. Which of the following is NOT being suggested to change our
healthcare system?
a. gradual upgrades and changes by individual hospitals
b. the implementation of an ever-evolving healthcare system
c. encouraging families to get involved with sick relatives’ progno-
sis and treatment
d. using the knowledge and resources already available to make
substantial headway
e. digitalizing our record system
221. The author of the passage would be most likely to agree with
which of the following statements?
a. Obamacare should be implemented at once.
b. The U.S. healthcare system has the wherewithal to be much
better than it is, but due to human error it remains redundant
and ineffective.
c. This country is in desperate need of a universal healthcare sys-
tem.
d. In spite of its ineffectiveness, the United States still has one of
the strongest healthcare systems.
e. Insurance companies that do not process individuals’ claims
because of legal loopholes should be disenfranchised.
222. The author’s attitude toward the U.S. healthcare system could be
best described as:
a. irrational
b. critical
c. impartial
d. moralistic
e. sardonic
223. The report being discussed in the passage would be of interest to
which of the following individuals?
a. healthcare professionals
b. people without insurance
c. insurance companies
d. data entry specialists
e. Americans
501_04_235-298_501_master.qxd 4/29/13 12:35 PM Page 261
261
501 GMAT
®
Questions
224. The basic structure of this passage is
a. to present a point and its counterpoint.
b. to state a hypothesis and evidence to support said theory.
c. to deliver data supporting an idea and a recommended agenda.
d. to contrast two opposing theories.
e. to attack an opposing theory.
In 1977, an international tobacco industry trade group, including rep-
resentatives of all major cigarette companies, noted in a document on
the social acceptability of smoking that “cigarette smoking is becoming
a downscale social activity.” As the disease effects of smoking became
better understood, more affluent and educated people were the most
likely to quit. Cigarette companies thus increasingly marketed toward
lower-income, less educated, and minority segments of the U.S. popu-
lation. A marketing study done for R.J. Reynolds (RJR) noted in a
downscale market profile that this demographic was “more impres-
sionable to marketing/advertising . . . they’re more susceptible.
They’re less formed intellectually . . . more malleable.”
In addition to general marketing efforts directed toward downscale
consumers, by the early 1990s the homeless had become one of the
sub-populations specifically targeted by major cigarette companies.
For example, RJR included direct targeting to the homeless as part of
an urban marketing plan in the 1990s, focused on the advertising of
“value” brands to “street people.” In 1990, American Tobacco paid for
product placement and supplied cigarettes for the movie Robocop 3,
which showed homeless activists smoking Pall Malls and Lucky
Strikes. In addition to advertising, tobacco companies gave away ciga-
rette brand logo products to the homeless; for example, in 1994 Philip
Morris (PM) apparently distributed 7,000 Merit cigarette brand
labeled blankets to New York homeless shelters and homeless individ-
uals. By the late 1990s, the ties between homelessness and smoking
had grown so overt that a major marketing periodical characterized
the target market of Brown and Williamson’s GPC brand as “Home-
less Man.”
Offering free samples is a well-established strategy by which
tobacco companies recruit new smokers, and cigarette samples were
distributed to homeless shelters, mental hospitals, and homeless service
501_04_235-298_501_master.qxd 4/29/13 12:35 PM Page 262
262
501 GMAT
®
Questions
organizations. In 1988 alone, Lorillard Tobacco Company spent over
$570,000 on cigarette donations, though not all of these were distrib-
uted to marginalized populations. Internal company documents show
that in a single month in 1990, however, Lorillard distributed over 100
sample packs apiece to a homeless shelter, a soup kitchen, and a mental
health association. Similar donations were logged regularly from 1983
to at least 1993.
225. According to the passage, what other demographics will likely be
targeted by the tobacco industry?
a. senior citizens
b. adolescent children
c. the homeless
d. the affluent
e. actors
226. The author implies that the motivation of RJR to distribute Merit
cigarette brand labeled blankets to homeless shelters in New York
City was to
a. elevate public opinion of the tobacco industry.
b. give back to the community it was receiving business from.
c. illustrate compassion for humanitarian concerns in the face of
scrutiny from the medical field.
d. target homeless people as a marketing strategy.
e. achieve recognition as a charitable organization.
227. Taken in context with the passage, malleable most nearly means:
a. breakable
b. refractory
c. pliable
d. malicious
e. intractable
501_04_235-298_501_master.qxd 4/29/13 12:35 PM Page 263
263
501 GMAT
®
Questions
228. Which of the following best represents the main idea of the
passage?
a. The tobacco industry gives out free cigarettes to the less fortu-
nate population in an effort to stimulate more popularity among
them.
b. As the awareness of the ill effects of cigarettes increases, the
affluent and more educated start stubbing out their cigarettes.
c. The homeless and other downscale consumers became a main
target for tobacco marketing.
d. Merit, Pall Mall, and Lucky Strike are the most popular ciga-
rettes within the homeless demographic of cigarette smokers.
e. R.J. Reynolds is a despicable company that lacks both tact and
shame in its approach to selling cigarettes.
229. Which of the following best describes the author’s tone?
a. poignant objectivity
b. critical and informative
c. impartial and somber
d. restrained scorn
e. cold anger
230. Based on the information in the passage, which of the following is
the author most likely to be affiliated with?
a. R.J. Reynolds
b. New York City homeless shelters
c. an antismoking campaign
d. the board of health
e. a homeless activist group
Business Passages
The final type of GMAT passage involves business-related topics. Business
passages can be challenging to read; they will contain unique vocabulary
and “business-speak.” Read slowly and try to identify root words if you
encounter an unfamiliar term. It is also helpful to identify the author of
these types of passages or determine whether a passage deals with a specific
business or industry. This will help you classify the tone, specifically
whether a passage will be subjective or objective in nature.
501_04_235-298_501_master.qxd 4/29/13 12:35 PM Page 264
264
501 GMAT
®
Questions
Set 9
Now it is time to answer GMAT Reading Comprehension practice ques-
tions that have been designed to test your business skills. Read the passages
and answer the questions that follow. Good luck!
A gene that keeps switchgrass forever young could have far-reaching
implications for the development of the plant as a biofuel crop, accord-
ing to U.S. Department of Agriculture (USDA) scientists.
Inserting a specific gene called corngrass from corn into switchgrass
essentially keeps the perennial grass in its juvenile form—a plant that
doesn’t flower, doesn’t produce seeds, and doesn’t have a dormant
growth phase. Because of these changes, the sugars making up the
plant starch are more readily available for conversion into cellulosic
ethanol.
According to Agricultural Research Service (ARS) geneticist Sarah
Hake, the starch in these transgenic plants stays inside the stem
because it isn’t needed elsewhere for nourishing flower buds and blos-
soms. As a result, starch levels can increase as much as 250%, which
increases the sugars that can be fermented into ethanol.
Hake, director of the ARS Plant Gene Expression Center in Albany,
CA, teamed with University of California–Berkeley plant geneticist
George Chuck to conduct this investigation. ARS is the USDA’s chief
intramural scientific research agency, and this work supports the
USDA priority of developing new sources of bioenergy.
The scientists observed that the leaves in the transgenic switchgrass
are not nearly as stiff as leaves in switchgrass cultivars that haven’t been
modified. In addition, they determined that leaf lignin is slightly dif-
ferent in the transgenic switchgrass than leaf lignin in other plants.
This could lead to new findings on how to break down the sturdy
lignin and release sugars for fermentation, a development that will be
essential to the commercial production of cellulosic ethanol.
The researchers are now introducing DNA segments called genetic
promoters that would “turn on” the expression of the corngrass gene
in aboveground switchgrass shoots. This could help increase root mass
development that otherwise would be inhibited by the gene. Hake and
Chuck also suggest that developing nonflowering switchgrass varieties
would eliminate the possibility of cross-pollination between transgenic
switchgrass cultivars and other switchgrass cultivars.
501_04_235-298_501_master.qxd 4/29/13 12:35 PM Page 265
265
501 GMAT
®
Questions
231. The following are all side effects of the modified switchgrass
EXCEPT:
a. stiffer leaves
b. infertility
c. dormant growth
d. inability to flower
e. higher sugar levels
232. The passage suggests that the development of transgenic
switchgrass will
a. eliminate the need to farm corn.
b. support the USDAs priority of finding alternative fuel sources.
c. strengthen the plant lignin.
d. introduce new DNA segments.
e. pit the Department of Agriculture against the Plant Gene
Expression Center.
233. The passage’s primary function is to
a. criticize the work of an institution.
b. compare two possible solutions when only one is needed.
c. provide the outcome of a finished study.
d. optimistically deliver the findings of a study in progress.
e. eliminate assumptions that are not critical to the findings.
234. Geneticist Sarah Hake heads the
a. U.S. Department of Agriculture.
b. University of California–Berkeley.
c. corngrass study.
d. Plant Gene Expression Center.
e. Agricultural Research Center.
235. Which of the following would be most interested in the study?
a. corn farmers
b. Orville Redenbacher
c. alternative energy car manufacturers
d. the president of the United States
e. DNA specialists
501_04_235-298_501_master.qxd 4/29/13 12:35 PM Page 266
266
501 GMAT
®
Questions
236. Which of the following inferences can be drawn based on the
information provided in the passage?
a. The USDA has previously neglected its duty to prioritize the
development of biofuels.
b. Our society is becoming more open to the idea of widespread
use of genetically engineered products.
c. We will inevitably drain all our fuel sources, forcing the priori-
tization of the USDA to produce more.
d. The development of switchgrass can lead to a lot of promising
uses from different avenues.
e. The cross-pollination of different variants of switchgrass is a lit-
tle problem that could have devastating effects.
Natural gas use for power generation rose this summer because of hot
weather-driven electricity demand for air conditioning coupled with
low natural gas prices. According to Bentek Energy, estimated daily
natural gas use to produce electric power (also called power burn)
averaged 26.3 billion cubic feet per day (Bcf/d) so far in 2012
( Jan 1–Aug 15), up 24% compared to the same period for 2011.
Bentek Energy, which has been estimating power burn since January
2005, said that 17 of the 25 highest days of power burn since 2005
occurred this summer between June 28 and August 9. The two main
drivers of the increased use of natural gas at power plants this year are
weather and a structural shift toward generating more electricity from
natural gas-fired power plants.
The National Oceanic and Atmospheric Administration reported
the warmest first half of the year since 1895 in 28 states, and that heat
continued in July and August. U.S. population-weighted cooling
degree days (CDDs), a measure of cooling requirements, averaged
26% higher than the 30-year average from January 1 through August
15, and has been consistently above average for most of the year.
Regionally, CDDs in the Midwest, where hot, dry weather was partic-
ularly severe, were 59% above their 30-year average, with the North-
east, South, and West at 43%, 18%, and 14%, respectively, above their
corresponding averages.
501_04_235-298_501_master.qxd 4/29/13 12:35 PM Page 267
267
501 GMAT
®
Questions
In April 2012, the Energy Information Administration (EIA) reported
that monthly shares of coal- and natural gas-fired generation were
equal for the first time. This is a result of several factors, including:
lower natural gas prices, the result of new drilling technologies,
growing production, a large increase in proved reserves, and robust
natural gas infrastructure additions over the last several years
power plant efficiencies, with newer natural gas units more efficient
than older coal units and rising capacity factors of natural gas-fired
units
Coal unit retirements, expecting almost 9,000 MW of coal-fired
capacity to be retired in 2012, with additional retirements in subse-
quent years
237. Which of the following most accurately summarizes the main
point of the passage?
a. It is clear by how much energy we are burning that the earth’s
weather patterns are becoming increasingly warmer.
b. Coal units are not as efficient or cost-effective as natural gas
sources.
c. The Northeast and Midwest are going to become warmer and
warmer each year.
d. Due to weather and lower natural gas prices, power plants are
using natural gas more than ever as an energy source.
e. The earth is running out of efficient ways to produce electricity
as it gets warmer and warmer, creating a precarious predica-
ment.
238. According to the passage, Bentek Energy is responsible for
a. an alliance with National Oceanic and Atmospheric Administra-
tion to strategize what to do about the energy crisis.
b. gathering and collecting data that will assess the use of natural
gases.
c. newer natural gas units that are more efficient than older coal
units.
d. the leveling of the playing field between coal and natural gas.
e. bringing up public consciousness about the amount of power
being used.
501_04_235-298_501_master.qxd 4/29/13 12:35 PM Page 268
268
501 GMAT
®
Questions
239. Based on the passage, which of the following statements can be
inferred regarding the earth’s future weather patterns?
a. Eventually all life will either adapt to hotter conditions or
become extinct.
b. There will not be enough energy left on the planet to accom-
modate the drastic differences in weather.
c. If it continues to grow warmer, a parallel rise in energy use will
occur, accounted for by air conditioning.
d. The West is the best region to live in for moderate weather
conditions.
e. Hurricanes and typhoons will become more devastating and
increase in occurrence.
240. The purpose of the last sentence of the first paragraph is to
a. refute the facts that Bentek Energy keeps delivering.
b. emphasize the central idea of the passage.
c. present a new point of view to be discussed.
d. segue into a new discussion of more interest to the author.
e. deliver a hopeful message regarding the increased use of power.
241. Based on the information provided by the passage, which would
most likely reflect the author’s attitude regarding natural gas
usage?
a. dreamy
b. objective
c. hollow
d. complimentary
e. elegiac
242. Which of the following pairs most resembles the relationship
between coal and natural gas as presented in the passage?
a. a hard-wired chandelier and a table lamp
b. a kick drum and a drum machine
c. a video rental store and a streaming video system
d. an electrical outlet and a generator
e. a bicycle and a skateboard
501_04_235-298_501_master.qxd 4/29/13 12:35 PM Page 269
269
501 GMAT
®
Questions
Our flagship market is focused on building strong brands, translating
brand value into customer value, and strengthening system capabilities
to sustain and repeat success. We delivered solid results in a challeng-
ing environment, including 2% organic volume growth, as we contin-
ued our integration efforts following the largest acquisition in our
company’s history, creating synergy savings to reinvest in our brands
and capabilities. Strong consumer and customer programs included
ABC Soda, which generated more than 2.1 billion consumer impres-
sions, and continued successful marketing partnerships between ABC
Soda and XYZ Drinks. Effective execution of our occasion-based,
brand, package, price, and channel strategies delivered volume and
value share gains across beverage categories. ABC Soda achieved its
sixth consecutive year of double-digit volume growth; XYZ Drinks all
grew triple digits.
The Europe Group overcame an uncertain economic environment
to deliver 5% operating income growth and 4% unit case volume
growth. The 130th anniversary of ABC Soda was activated with pas-
sion and creativity, driving brand love on a massive scale. Other mar-
keting highlights included ABC Soda & Meals, driving sales with
imaginative partnerships and cross promotions. We scored with sum-
mer music campaigns and built momentum for the London 2012
Olympic Games. A key strategic success was strengthening ties with
bottling partners and driving growth with key customers. We spent
more time in the marketplace, using the insights to act with greater
flexibility and get closer to consumers with an adapted brand, package,
and price architecture.
In 2011, we continued to execute the strategies of the Bottling
Investments Group. Our core focus on top-line growth and aggressive
cost management, combined with marketplace execution, operational
excellence, and productivity, generated strong performance. We grew
unit case volume 3% on a comparable basis after adjusting for the
impact of the sale of our Norway and Sweden bottling operations.
However, on a reported basis, unit case volume was even with the prior
year. We continued to focus on prudent capital planning to ensure we
have the capacity to meet sales growth. Our focus on improving envi-
ronmental metrics has resulted in significant positive changes, espe-
cially in energy and water usage. In addition, we opened more than
501_04_235-298_501_master.qxd 4/29/13 12:35 PM Page 270
270
501 GMAT
®
Questions
450,000 new outlets, placed an incremental 150,000 new coolers, and
continued building market segmentation capabilities to ensure con-
sumers continue to have access to our brands for all occasions, in the
right packages, at the right price. We remained focused on the imple-
mentation of QRS Colas, our end-to-end bottler operating system that
enables the development of standard tools, data, and systems geared
toward enhancing sales force effectiveness.
243. What business strategies led to the generation of double-digit
volume growth?
a. brand value lead to repeat success while using “old school door-
to-door” advertising
b. longer time spent asking the opinions of longtime consumers
c. strengthening ties with bottling partners and driving growth
with key customers
d. targeting larger areas and other countries
e. aggressive media marketing
244. Based on the information provided, what is the tone of this
passage?
a. factual ambivalence
b. enthusiastic pride
c. objective reporting
d. inconclusiveness
e. critical
245. Which of these situations most resembles the situation described
in this passage?
a. a mutant strain of super ants that recolonizes so efficiently that
they become the strongest ants in their habitat, nearly extermi-
nating other species
b. a blockbuster film that becomes a miniseries on TV
c. a car dealership that changes its business plan to exclude
imported cars
d. a lamp company formerly specializing in all lighting limiting
itself to chandeliers
e. None of these are true.
501_04_235-298_501_master.qxd 4/29/13 12:35 PM Page 271
271
501 GMAT
®
Questions
246. Which best describes the basic structure of this passage?
a. comparing different strategies
b. accumulating data to support a conclusion
c. different theories leading to the same conclusion
d. disproving an existing hypothesis
e. compiling facts to sway an opinion
A hedge fund is an investment fund that can undertake a wider range
of investment and trading activities than other funds, but is generally
only open to certain types of investors specified by regulators. These
investors are typically institutions, such as pension funds, university
endowments and foundations, or high-net-worth individuals who are
considered to have the knowledge or resources to understand the
nature of the funds. As a class, hedge funds invest in a diverse range of
assets, but they most commonly trade liquid securities on public mar-
kets. They also employ a wide variety of investment strategies, and
make use of techniques such as short selling and leverage.
Hedge funds are typically open-ended, meaning that investors can
invest and withdraw money at regular, specified intervals. The value of
an investment in a hedge fund is calculated as a share of the fund’s net
asset value, meaning that increases and decreases in the value of the
fund’s investment assets (and fund expenses) are directly reflected in
the amount an investor can later withdraw.
Most hedge fund investment strategies aim to achieve a positive
return on investment whether markets are rising or falling. Hedge
fund managers typically invest their own money in the fund they man-
age, which serves to align their interests with those of investors in the
fund. A hedge fund typically pays its investment manager a manage-
ment fee that is a percentage of the assets of the fund, and a performance
fee if the fund’s net asset value increases during the year. Some hedge
funds have a net asset value of several billion dollars. As of 2009, hedge
funds represented 1.1% of the total funds and assets held by financial
institutions. As of April 2012, the estimated size of the global hedge
fund industry was US$2.13 trillion.
Because hedge funds are not sold to the public or retail investors,
the funds and their managers have historically not been subject to the
same restrictions that govern other funds and investment fund
managers with regard to how the fund may be structured and how
501_04_235-298_501_master.qxd 4/29/13 12:35 PM Page 272
272
501 GMAT
®
Questions
strategies and techniques are employed. Regulations passed in the
United States and Europe after the 2008 credit crisis are intended to
increase government oversight of hedge funds and eliminate certain
regulatory gaps.
247. According to the passage, the relationship between Bank of
America and a hedge fund is most like:
a. an iPad and a plasma television
b. Prospect Park and a private Long Island country club
c. a Mack truck and a motorcycle
d. a state hospital and a chiropractor’s clinic
e. Domino’s Pizza and a local pizzeria
248. According to the information in the passage, which of the
following is NOT true of hedge funds?
a. Withdrawal from hedge funds can be made based on the value
of the fund at the time.
b. The people who manage hedge funds typically invest their own
money, helping to form solidarity with the original investors’
interests.
c. High-net-worth individuals can invest in hedge funds.
d. Hedge funds typically maintain a positive return, unless the
market is doing considerably poorly.
e. The 2008 credit crisis affected the strategies previously
employed by hedge fund specialists.
249. Based on the information in the passage, “liquid securities” most
likely refers to:
a. difficult to access funds, swimming in a sea of paperwork before
they can be accessed
b. stock in alcoholic beverages
c. funds that are easy to trade and don’t change value while chang-
ing hands
d. savings bonds
e. the stock market’s private cache
501_04_235-298_501_master.qxd 4/29/13 12:35 PM Page 273
273
501 GMAT
®
Questions
250. This passage would best be described as:
a. didactic
b. sententious
c. indignant
d. irreverent
e. detached
251. Based on the information provided in the last paragraph, which of
the following can be reasonably inferred regarding hedge funds?
a. Hedge funds relentlessly manipulate money to their advantage
even when an economic crisis ensues.
b. If hedge funds were made available to the general public, they
wouldn’t be as lucrative based on inexperience.
c. While hedge funds may represent trillions of dollars, other
institutions have a far greater percentage of the world’s capital.
d. Prior to the credit crisis of 2008, hedge funds were likely to be
circumventing regulatory conditions that most others are sub-
ject to.
e. A hedge fund is an unscrupulous means of creating money
among the wealthiest citizens.
The Penelope Royalty Trust is a United States oil and natural gas
royalty trust based in Waco, Texas. With a market capitalization of
US $620,040,000, and an average daily trading volume of about
237,000 shares at the end of 2007, it is one of the largest royalty trusts
in the United States. Its source of revenue is oil and gas pumped from
the geologic formation at the Land of Penelope, as well as a few loca-
tions in other parts of the country.
Most of the trust’s properties are on Jeepers Ranch in Curr County,
Texas, where it owns a 75% net overriding royalty interest in the fee
mineral interests (in this case, oil and natural gas). Other properties of
the trust are in 32 other Texas counties, most of which are in the west-
ern portion of the state, on the High Plains; the trust owns a 95% net
overriding royalty interest in all its properties outside of the Jeepers
Ranch.
The principal productive zones for oil on Jeepers Ranch are in two
geologic units, the Grayburg and the San Andreas, at a depth of from
2,800 to 3,400 feet (1,000 m) below ground surface; however there are
a total of 12 producing zones on the ranch, including one at a depth of
501_04_235-298_501_master.qxd 4/29/13 12:35 PM Page 274
274
501 GMAT
®
Questions
10,600 feet (3,200 m). As of the end of 2006, there were a total of 620
operational and productive oil wells and 142 natural gas wells on the
Jeepers Ranch in the trust. On December 31, 2006, the trust claimed a
lifetime of approximately 8.3 years for all mineral reserves of the trust.
Penelope Royalty Trust came into being in November 1980, with an
agreement between Brickabrack Royalty Company and the Nations
Star Bank of Fort Worth. As is the case with U.S. royalty trusts, the
trust cannot function as a business, and has no employees; all opera-
tions and maintenance are carried out by the trustee and its subcon-
tractors. Currently, the assets of the trust are managed by Barnaby and
Boom, Inc., which acquired Melenial oil, the previous operator.
The trust pays a relatively high dividend, yielding an annual rate of
12.4% in early 2008; in addition, it pays out monthly, a relative rarity
for U.S. stocks. However, its distribution is dependent on the prices of
oil and gas; thus, unlike traditional stocks (that, when declaring a divi-
dend, usually maintain it at the same amount for each quarter of the
year), the dividend payout will differ each month.
Since the trust’s assets are considered a depletable resource, its divi-
dend payments are not taxed at the regular dividend rate, but rather as
return of capital instead of return on investment; this is an additional
tax advantage in the United States, and applies to all royalty trusts.
252. Which of the following best expresses the main idea of the
passage?
a. The principal production zones for oil of Penelope Royalty
Trust are in two different zones.
b. The Texas-based gas trust Penelope Royalty has grown consid-
erably since its 1980 beginning.
c. The Penelope Royalty Trust’s dealings with oil give it the
advantage of different tax laws because its product is considered
a depletable resource.
d. The Jeepers Ranch has turned out to be a very lucrative place
for Penelope Royalty Trust.
e. The land of Penelope is rich with oil and inspired a trust to be
developed; thus, it bears its name.
501_04_235-298_501_master.qxd 4/29/13 12:35 PM Page 275
275
501 GMAT
®
Questions
253. According to the passage, who currently controls the assets of the
Penelope Royalty Trust?
a. Penelope Royalty Trust
b. Melenial Oil
c. Barnaby and Boom, Inc.
d. Jeepers Ranch
e. Brickabrack Royalty Company and Nations Star Bank
254. What is the author of this passage primarily concerned with?
a. the Penelope Royalty Trust’s contribution to the gas industry
and its proposal to preserve its resources for the future
b. the Penelope Royalty Trust’s history and its current standing in
the economic divisions of energy and finance
c. the growing crisis of oil depletion, with a renewed sense of hope
because of domestic oil discoveries, specifically those of the
Penelope Trust
d. the unfair advantage oil producers have regarding tax breaks
e. a depiction of how many companies can be involved in one pri-
mary interest
255. Where does most of the production of oil take place for the
Penelope Royalty Trust?
a. in the High Plains
b. in the western part of Texas
c. at a depth of 10,600 feet
d. at the Grayburg and the San Andreas on Jeepers Ranch
e. at 142 natural gas wells
256. For what purpose does the author include the fifth paragraph?
a. to cite how and why oil and gas differ from other stocks
b. to show the unfair advantages oil companies receive
c. to negate any disdainful opinion of oil companies
d. to explain dividends and returns to laymen
e. to show how odd the U.S. economic system can be
501_04_235-298_501_master.qxd 4/29/13 12:35 PM Page 276
276
501 GMAT
®
Questions
Answers
Set 7
175. a. The author’s concern is clearly stated: Although antibiotics have
been useful to date, new illnesses have developed that will not be
combated by this treatment approach. The author never implies
that funding be given to third world countries, therefore the
answer is not choice b. It is never implied that staph and MRSA
should be listed as deadly diseases, therefore choice c is incorrect.
Sexually transmitted diseases are not the main concern of the
author, so the answer is not choice d. It is not implied that fees will
be lowered for patients in spite of having to pay more, so the
answer is not choice e.
176. c. The main idea of the passage is stated after the word however, a
usually telling transitional word. “However, these drugs have been
used so widely and for so long that the infectious organisms the
antibiotics are designed to kill have adapted to them, making the
drugs less effective.” The fact that drug-resistant infections are
becoming more problematic is a supporting idea, so choice a is
incorrect. There is no implication that an epidemic will ensue, so
the answer is not choice b. The development of antibiotics has not
been neglected, so choice d is incorrect. Choice e is not true
according to the passage.
177. c. The antibiotics used to work to combat illness, but have lost
their efficiency as the illnesses have adapted to their defenses, just
as a moat may have been effective to protect a kingdom until
enemies found a way to get across the water. A superhero losing
any power wouldn’t speak to the evil attacking it or why it needed
to be invisible to begin with, so choice a is incorrect. Choice b is
incorrect because a cucumber in a salt solution becoming a pickle
is similar to viruses changing, but doesn’t resemble this situation as
there is no parallel to the antibiotics stagnation. Two pies of
different flavors in a bake-off isn’t anything like the antibiotics
situation, and neither is buying tickets to a show anywhere, so
choices d and e are incorrect.
501_04_235-298_501_master.qxd 4/29/13 12:35 PM Page 277
277
501 GMAT
®
Questions
178. c. The first three lines of the passage state already accepted,
studied, and proven information. The fourth line introduces new
findings on the subject, suggesting a rebuttal of the previously
widely believed information. The author isn’t reporting data that
matches what has already been stated, therefore choice a is
incorrect. Choice b is incorrect because the passage does not
compare two different ideas. The initial information given is not
an idea but proven material. There is nothing so drastic as
vehemence suggested in the passage, and the author doesn’t seem
to be begging for the reader’s compassion, so choices d and e are
incorrect.
179. a. As infectious organisms evolve, a wider variety of drugs will have
to be used if the first treatment prescribed isn’t effective. This can
be costly, and second choice drugs will have to be used as an option
as well, as stated in the last paragraph. Hospitals may incur higher
fees, but it is never suggested there is anything beneficial about this
study, so choice b is incorrect. The passage states that not only
industrialized nations will feel the brunt of this development,
contrary to choice c. It is never suggested that antibiotics weren’t
taken as prescribed, so choice d is incorrect. It is never stated that
an increase in STDs is expected, just that they are in fact drug-
resistant infections that could spread in the community at large, so
choice e is incorrect.
180. c. Admonishing is defined as warning or cautioning, which is
suggestive of the author’s intent. The drug’s previous successes are
mentioned, but the passage suggests that further study is needed or
there could be potentially hazardous consequences such as a rise in
illness and the upset of inflated drugs. The author is somewhat
critical (choice a) of the current usefulness of antibiotics, but isn’t
critical as a whole against any specific institution, only suggestive
that further action is required. The author’s tone is not sarcastic
(choice b) at all. The adjectives in choices d and e suggest
emotional response, which the author does not imply.
501_04_235-298_501_master.qxd 4/29/13 12:35 PM Page 278
278
501 GMAT
®
Questions
181. d. The author cites the benefits and leaping bounds science has
been able to make since the discovery. The tone of the article is
one of excitement, using adjectives such as “tremendous” and
citing examples of its efficacy. Ambivalence is contrary to the tone,
so choice a is incorrect. The answer is not choice b because the
author doesn’t criticize anything but rather supports the findings.
The answer is not choice c; although the author seems to be
defending the legitimacy of the use of biotechnology, it is never
suggested that anyone else has attacked it. Objectivity (choice e)
would imply that no emotion is involved in the findings, and the
author has a clear interest in the further development of
biotechnology.
182. b. The last line in the last paragraph cites nonspecific examples of
where genetic engineering has been used with benefit. It is implied
that the author will continue to articulate the strides made with the
use of genetic engineering by more specifically citing its
contribution. The answer is not choice a. The reference made to
mating a cow and a horse was made to cite an example of what
couldn’t work prior to new scientific findings. There is never
implication that a cow and a horse should swap DNA for any
reason. Although reference can be made eventually to the
advancements made in 1972, this segue would be clumsy if made
here, and therefore choice c is incorrect. The answer is not choice
d. There is never any reference to opponents of biotechnology.
The answer is not choice e. There is never any additional implied
link between beer and pharmaceuticals.
183. b. The end of the fifth paragraph states, “An organism modified by
genetic engineering is called transgenic.” Although any individual
may have the opinion that DNA swapping is an abomination, it’s
not called that technically, so choice a is incorrect. The answer is
not choice c. It is stated that manipulating the DNA between these
two animals of the same breed has been possible for years.
Although it is a manipulation, this is not what it is definitively
called, so choice d is incorrect. This is not what transference
means, so choice e is incorrect.
501_04_235-298_501_master.qxd 4/29/13 12:35 PM Page 279
279
501 GMAT
®
Questions
184. d. Transgenic plants are both a product of and an example of
biotechnology just as opiate abuse could be the product of
addiction and an example of a specific addiction. None of the other
relationships between the things listed share the same kind of
relationship.
185. d. The passage references transmitting DNA through viruses, yet
never suggests using the viruses in an act of war or as a weapon. All
of the rest of the answers are mentioned in the passage as examples
of biotechnology.
186. c. The passage explores the uses of biotechnology since inception,
which predates its study, up until its present, and how more recent
findings continue to broaden the horizon of its advantages. Choice
a is certainly a supporting idea of the passage, but would not
qualify as the main idea. The answer is not choice b since the
benefits of biotechnology are documented throughout the passage.
The author clearly supports the continuance of this study, and
never mentions risks or concerns, therefore choice d is incorrect.
Choice e could also be inferred as true, but is a less significant
point of the passage.
187. c. This statement would best describe the main idea of this
passage. Alcohol negates the benefits of the polyphenols that
decrease blood pressure. Its other effects aren’t discussed;
therefore, choice a is incorrect. There is no correlation made
between alcoholics and strokes, so choice b is incorrect.
Polyphenols are the antioxidant known to decrease blood pressure,
so choice d is incorrect. Gin may not decrease blood pressure, but
whether drinking it is useless is subjective, so choice e is incorrect.
188. d. The passage states that drinking nonalcoholic red wine can
potentially reduce the risk of heart disease by 14% and stroke by as
much as 20%. All of the other answer choices are contrary to the
facts given in the passage.
501_04_235-298_501_master.qxd 4/29/13 12:35 PM Page 280
280
501 GMAT
®
Questions
189. c. It is implied that men with diabetes are at higher risk to develop
heart disease than the average male. Choice a is a complete
fabrication, so it is therefore incorrect. It is never stated or
understated that men will respond poorly to nitric oxide, so choice
b is incorrect. No physiological reaction is ever reported, so choice
d is incorrect. There is no evidence in the passage to support
choice e.
190. c. The study discussed in the passage was based on proving that
the removal of alcohol from red wine would yield greater results.
Neither choice a or b was implied at any point in the passage.
Choices d and e were both stated at one point, but could not be
concluded as the main idea as they weren’t discussed enough to
qualify as main ideas.
191. d. The offered data supports the conclusion of the study, unlike
choice a, which states that data is being introduced. This is
incorrect because the study has already been described and
therefore isn’t being introduced. Choice b is incorrect because
there isn’t a counterpoint. Choice c is incorrect because there
aren’t any outside sources being accessed. The paragraph produces
facts from a study, not opinions, making choice e incorrect.
192. b. Lactose-free milk will have started out as whole milk before the
lactose was removed from it, just like nonalcoholic red wine would
have alcohol in it until removed. A Labradoodle is genetically
engineered from a Labrador retriever’s DNA, but the dog can’t be
a Labrador first and then a Labradoodle. Granny Smith and Fuji
apples are two different kinds, but they are never the same
product. A gas oven and a microwave are just two different kinds of
ovens, not at all the same relationship, and the same goes for the
Big Wheel and a bicycle.
501_04_235-298_501_master.qxd 4/29/13 12:35 PM Page 281
281
501 GMAT
®
Questions
193. e. The author concedes that the treatment looks positive and could
make incredible strides for the treatment of Alzheimer’s. However,
this article points out inconsistencies in the findings of the report
versus the articles being printed about it. The article points out
that the treatment may not be available for 10 years, so choice a is
incorrect. While the author remains skeptical about the
advancement, there have been legitimate findings that would
render the drug credible. Therefore, choice b is incorrect. The
answer is not choice c since the drug is to be administered
intravenously. The title in choice d does not match any of the
information in the article, so it is incorrect.
194. a. The author reports that although the drug is being researched,
the research is in its early stages, and since it hasn’t been peer
reviewed, it can hardly be justified. The answer is not choice b.
The author has too much of an opinion to be considered
ambivalent or choice c, stoic. Enthusiasm generally denotes
positivity, and if the author is enthusiastic at all it is to criticize, so
choice d is incorrect. Whether the author seems cantankerous has
no bearing on the passage, so choice e is incorrect.
195. c. In the fourth paragraph the author states that although a news
source called it one, a vaccine would technically only be
administered once, and this drug will need to be delivered every
two weeks. The drug will be administered by injection, so choice a
is incorrect. The answer is not choice b. The author reports that
IVIG will encourage the immune system to fight proteins in the
brain of Alzheimer’s patients. The answer is not choice d. The
author reports that IVIG may not be available for 10 or more
years. The answer is not choice e. The author states that IVIG
may be expensive to manufacture.
501_04_235-298_501_master.qxd 4/29/13 12:35 PM Page 282
282
501 GMAT
®
Questions
196. c. The first two paragraphs present the background information
about the drug, but the fourth paragraph introduces the point the
author was trying to make by writing it. Until now there has been
no mention of misleading information. Yet from now until the end
of the article, the author cites ways in which the media presented
false or misleading information regarding the drug’s legitimacy.
The author argues that the media acted with haste to publish the
medical field’s findings, but is not arguing a point presented by
anyone who made such findings, so choice a is incorrect. The
fourth paragraph is contrary to objectivity, so choice b is incorrect.
The study isn’t being denounced, it is only suggested that it needs
more research before a conclusion can be reached, so choice d is
incorrect. This paragraph does not compare the opinions of
medical professionals, so choice e is incorrect.
197. a. The author acknowledges that medical advances have been
made regarding the treatment of Alzheimer’s, but shows disdain
for the media’s inaccurate and premature conclusions. The
advancements are surely exciting, but when IVIG will be available
is unknown. Although the author states that more research is
needed regarding IVIG, there is never any mention of testing on
mice, so choice b is incorrect. The answer is not choice c. There is
never mention of coercing abnormal clumps rather than attacking
them. The answer is not choice d. The author never suggests why
the media presented misleading information, but only that it was
done. The author hasn’t made reference to a national healthcare
system, so choice e is incorrect.
198. e. All the other choices are foodborne illnesses or conditions that
will lead to a foodborne illness.
199. b. The passage is about the development and implementation of
SERS, how it works, where it will work, and how that will affect
tracking foodborne illness. This is clearly the main idea. The facts
regarding colloidal metals is a supporting idea of this passage, not
the main idea, rendering choice a incorrect. Choices c and d are
both supporting ideas as well, so they are both incorrect. There is
no evidence to back choice e, so it is incorrect.
501_04_235-298_501_master.qxd 4/29/13 12:35 PM Page 283
283
501 GMAT
®
Questions
200. c. The sixth paragraph states that a specimen will be placed on this
enhanced surface, a nanosubstrate. According to the passage, a
nanosubstrate is not a laser beam, so choice a is incorrect. The
surface can be enhanced by tiny spheres of colloidal metal, but
that’s not a defining characteristic, making choice b incorrect. A
nanosubstrate is not described as the scattering of light, so choice
d is incorrect. A nanosubstrate is not a foodborne pathogen, so
choice e is incorrect.
201. c. The surface in the passage is described as changing from smooth
to rough, just as the surface of a knee would change from smooth
to rough while it was scabbing over and healing. A stripped screw
would be an example of a surface changing its shape, but not from
smooth to rough, so choice a is incorrect. Peaches and apples are
different fruits with different surfaces, unlike the one surface that
changes, making choice b incorrect. A cat’s reaction to a laser has
nothing to do with the enhanced surface, so choice d is incorrect.
An old tire’s tread would go from grooved to smooth, so choice e
is incorrect.
202. d. Including how many people are affected by salmonella in the
United States yearly further emphasizes how useful the
development of the SERS will be after its completion. The author
never criticizes the work of any institution, so choice a is incorrect.
The author clearly supports the SERS project, but the second
paragraph does not mention Parks’ research, so choice b is
incorrect. The second paragraph does call attention to the
problems salmonella creates, but its purpose is beneath the surface
of the obvious, so choice c is incorrect. The second paragraph does
not contain the main idea of the article, so choice e is incorrect.
501_04_235-298_501_master.qxd 4/29/13 12:35 PM Page 284
284
501 GMAT
®
Questions
Set 8
203. c. The government and affiliates of the island wanted to keep the
number of health epidemics low by expanding the hospital grounds
to accommodate ailing immigrants, as well as those scheduled for
deportation. There was no mention of segregation in the passage,
so choice a is incorrect. Immigrants were not forced to build any
new buildings in order to be admitted into the United States, so
choice b is incorrect. There was no specific evidence in the passage
to support the statement of choice d. The fire did not do enough
damage to warrant the demolition to the entire island, so choice e
is incorrect.
204. e. When the government assumed responsibility of the island, the
welfare of the people entering the country also became their
charge. The architecture firm was hired to build because of the
overflow, not for aesthetics, so choice a is incorrect. There was no
mention of anyone being forced to work for entry, so choice b is
incorrect. Neither literacy nor age discrimination was mentioned
in the passage, so choices c and d are also incorrect.
205. d. The fact that houses were constructed of Georgia pine becomes
relevant when the information is presented that a fire destroyed all
the buildings, and the subsequent necessity to rebuild with fire-safe
materials. Choice a does not help to further the passage
structurally, as relevant as the information is. Inclusion of the
economy’s impact on immigration isn’t brought up until paragraph
3, so choice b is incorrect. Choices c and e are both true and a part
of paragraph 2, but the facts stand alone.
206. d. This passage is told completely from a historical point of view
free of opinion and bias. Choice d is the only choice that
eliminates opinion and is true. The author never insinuates that
the buildings were built with shoddy materials, so choice a is
incorrect. The author never inserts advice on what would have
been better, so choice b is incorrect. The depression is never given
credit for the island’s initial emptiness, so choice c is incorrect.
The author never implies that the spirit of competition aided the
island’s structural integrity, so choice e is incorrect.
501_04_235-298_501_master.qxd 4/29/13 12:35 PM Page 285
285
501 GMAT
®
Questions
207. d. The third landfill would be the last landfill mentioned, and
directly following it the contagious disease ward was constructed.
All the other choices were constructed before the last landfill was
completed.
208. e. The answer includes both the enactment of anti-immigration
legislation and the Great Depression, so choice e, none of these
are accurate, is correct. Choices c and d were not mentioned as
factors for the decreased numbers.
209. a. Limited percentages of each nationality were allowed in order to
curb the flow of immigration to the United States. Criminal
activity was never linked to the enforcement of quota, so choice b
is incorrect. The numbers were reduced, but not completely
stopped during the Great Depression; choice c is incorrect. The
percentages were either 2% or 3%, not 1%, so choice d is
incorrect. There was never any mention of panic on the island as
choice e would indicate.
210. d. Informative best matches the author’s tone. This passage
presents from a historical vantage point, refraining from
interjection of opinion or emotion. Restrained would insinuate the
holding back of emotion, which was not found. Nostalgic also
implies an emotional response to the past, which is not present.
Inflammatory does not describe the tone; there is no presence of
anger or opinion. Contemptuous is incorrect as well, as it too
suggests the show of emotion.
211. b. The act would restrict immigrants to 2% of their nationality
currently living in the United States. The passage continues to
inform that more than 50% of the immigrants were from southern
and eastern Europe; Greece is in Southeast Europe. The Ellis
Island Advisory Committee would not be gravely affected by the
1924 act. Citizens of Great Britain are in northern Europe, so they
would have had a greater chance to immigrate. The Lower East
Side of New York City is never mentioned in the passage.
501_04_235-298_501_master.qxd 4/29/13 12:35 PM Page 286
286
501 GMAT
®
Questions
212. b. The fourth paragraph provides a summation of the other three
paragraphs and includes some outcomes of said information, so
choice b is the most logical answer. There is no theorizing evident,
so choice a is incorrect. No argument or counterpoint is ever
addressed, making choices c, d, and e incorrect.
213. a. Although the author never states an opinion, the enthusiasm
about the scientists’ findings can only be regarded as passionate.
Nowhere in the passage does the tone reflect despondency, making
choice b incorrect. Articulation is not an adjective describing tone,
making choice c incorrect. The article is not opinionated and the
tone is not arrogant, making choices d and e incorrect.
214. c. The Massachusetts Institute of Technology commented on the
study, but was not directly involved in it. The study was conducted
at the Helen Wills Neuroscience Institute located at the University
of California at Berkeley, making choices a and b incorrect. Both
the U.S. Institute of Neurological Disorders and Stroke and
Blueprint Test Preparation funded the study, making choices d and
e incorrect.
215. d. The third paragraph is a good paraphrase of the first paragraph,
both of which could be seen as the hypothesis, so the answer
cannot be choice e. Choice a is incorrect because no cumulative
data is presented in paragraph 3. The third paragraph does not
present an argument, making choice b incorrect. Two different
theories are not being presented here, only one, making choice c
incorrect.
216. b. This point is made in two different statements that can be
joined. Choice a is not the answer because there is no such insight.
The passage states that people are born with reason and that it can
be developed, so choice c is incorrect. The test in question is the
LSAT, but is never mentioned that only lawyers can benefit from
reasoning training, so choice d is incorrect. Choice e is just not
true.
501_04_235-298_501_master.qxd 4/29/13 12:35 PM Page 287
287
501 GMAT
®
Questions
217. a. The main idea of the passage is mentioned in the first paragraph,
and then reiterated and paraphrased in the third paragraph, “that
training people in reasoning skills can reinforce brain circuits
involved in thinking and reasoning and might even help increase a
person’s IQ scores.” There is no mention of why the U.S. National
Institute of Neurological Disorders and Stroke funded the study,
rendering choice b incorrect. Although practicing for the LSAT is
stated to change the brain, it never says it will do so chemically,
making choice c false. The passage states that preparedness can
NOT predict your future success, making choice d incorrect. The
statement in choice e may be true, but it is sure to be subjective for
each individual and would not be labeled as a main idea.
218. d. The last paragraph states “that intensive, real-life educational
experience that trains reasoning also alters the brain pathways that
support reasoning ability.” It is stated that improving on the LSAT
after preparation is not a new idea, making choice a true. The
statement of choice b is mentioned in the third to last paragraph.
The passage states that brain function specific to reasoning is
plastic, or malleable, through adulthood, so choice c is true. It is
stated in the passage that among other things, the LSAT is
reasoning training, making choice e true.
219. c. The first line of the second paragraph states that the report the
passage concerns urges changes be made to our healthcare system
to lower cost while improving healthcare quality. The
advancement of technology is a supporting idea, but isn’t
mentioned enough to be the primary concern. An increase in pain
and suffering is an outcome of the main idea. A poorly
appropriated budget is also a supporting idea, as is the inevitable
decline of healthcare for various reasons.
220. a. Gradual upgrades and changes are cited as inadequate and part
of the problem. Choice b is incorrect; it is suggested that a
learning system be implemented. The seventh paragraph
references encouraging family involvement in patient care, so
choice c is incorrect. The second-to-last paragraph suggests that
we already have the know-how and technology, so choice d is
incorrect. The eighth paragraph states the intent of electronic
health records, so choice e is incorrect.
501_04_235-298_501_master.qxd 4/29/13 12:35 PM Page 288
288
501 GMAT
®
Questions
221. b. The passage provides examples of a number of human errors
involved in the healthcare system, such as fraud, excessive cost, and
unnecessary spending. A diligent recommitment by people
involved is recommended to fix the healthcare system. Neither
Obamacare nor a universal healthcare system is ever mentioned,
rendering choices a and c incorrect. The United States ranking
among worldwide healthcare systems is never mentioned, so
choice d is incorrect. Insurance companies are arguably part of the
problem, but the conclusion in choice e cannot be drawn with no
reference to them.
222. b. The author cites what is wrong with the state of the current
healthcare system and what could be done to fix it. Whether the
author is irrational would be an opinion of the material presented,
but not the author’s voice. The passage draws too many references
to fault, rendering it very partial. There is no information to
support a moral platform. The passage refrains from both mocking
sneering, never intonating sardonicism.
223. e. All Americans are affected by the healthcare system provided
they live in the country. Therefore, it could not be limited to any
of the other choices, as it would be all-inclusive.
224. c. The author reports the Institute of Medicine findings after
much research in the first half of the passage and then delivers the
possible solutions in the second half. There is no counterpoint, so
the answer is not choice a. The Institute of Medicine isn’t
presenting a hypothesis, making choice b incorrect. There isn’t a
comparison being drawn, so choice d is incorrect. Attacking
denotes violence, an emotional response far more drastic than this
passage ever suggests, rendering choice e incorrect.
225. c. The passage reports that smoking is becoming a “downscale
social activity.” The homeless are the only answer here that would
be specific to downscale and social. Neither senior citizens nor
adolescent children were mentioned as a target. The affluent were
mentioned as a group likely to quit smoking right now, and actors
weren’t mentioned at all.
501_04_235-298_501_master.qxd 4/29/13 12:35 PM Page 289
289
501 GMAT
®
Questions
226. d. The passage states “Cigarette companies thus increasingly
marketed toward lower-income, less educated, and minority
segments of the U.S. population.” Distribution of blankets is one
of the marketing tools employed. The tobacco industry did not
seek better public opinion, so choice a is incorrect. Although the
industry may have been thanking the homeless for doing their part
to keep them in business, this wasn’t the motivation, so choice b is
incorrect. There is no mention of seeking compassion from
anyone, so choice c is incorrect. The author does not imply that
seeking to be seen as charitable is any sort of motivation, so choice
e is incorrect.
227. c. A tobacco industry representative stated that this demographic
would be more susceptible to marketing and therefore more easily
moldable or pliable to suit the needs of the company. The tobacco
industry wouldn’t benefit from a breakable new demographic; a
stubborn, or refractory, demographic wouldn’t be ideal so choices a
and b are incorrect. The answer is not choice d because it would
not make any sense. The answer is not choice e. Intractable is
synonymous with stubborn, so again, this wouldn’t be ideal.
228. c. The passage cites several examples of the ways in which the
tobacco industry has targeted homeless people as its most likely
demographic to continue smoking. The answer is not choice a
because giving out cigarettes is just one example of how the
marketing is done, and not the central point of the passage. Choice
b is also a supporting idea, as the affluent are only mentioned once
in the beginning of the passage. There is no evidence to support
the insinuation of choice d. Choice e is incorrect because it is an
opinion, and although this may be the opinion of the author, it is
not stated.
501_04_235-298_501_master.qxd 4/29/13 12:35 PM Page 290
290
501 GMAT
®
Questions
229. b. The passage delivers the information, but there is definitely a
critical attitude toward the tobacco industry. Pointing out all the
industry’s dirty tricks without using angry adjectives keeps the tone
at critical rather than angry. The answer is not choice a because
the author is not objective, nor is it choice c because although it is
somber it is not impartial. There is never an indication that the
author is holding back anything, so choice d is incorrect. Cold
anger (choice e) would be too harsh a description for this article.
230. c. The author is bringing to light the unethical tactics of the
tobacco industry, making an anti-tobacco movement the most
likely of all the answers. The answer would not be choice a as R.J.
Reynolds is a tobacco company, and such crude evidence wouldn’t
be offered from within. Choice b is a completely arbitrary choice.
Although the author could be involved with the board of health,
this passage concentrates on smoking rather than a full spectrum of
health issues, making c rather than d the better choice. It is never
implied that the author is in league with a homeless activist group,
so choice e is incorrect.
Set 9
231. a. The passage states that the scientists observed that the leaves in
the transgenic switchgrass are not nearly as stiff. The answer is not
choice b because it is noted that the modified plant will not
produce seeds. It is noted that the plants will not have a dormant
growth phase, so choice c is incorrect. In the second paragraph, it
is said that the modified plant will not be able to flower, so choice
d is incorrect. The answer is not choice e. In the third paragraph,
it is said the modified plant will have 250% more starch, and as a
result will yield more sugar.
501_04_235-298_501_master.qxd 4/29/13 12:35 PM Page 291
291
501 GMAT
®
Questions
232. b. The end of the fourth paragraph states that “the work supports
the USDA priority of developing new sources of bioenergy.”
Choice a is incorrect because the passage never suggests anything
about the cessation of corn production. Choice c is incorrect; it is
stated only that the plant lignin will be different; if anything, it can
be inferred that it may be weaker. Choice d is incorrect because
researchers are introducing new DNA segments, not the plant.
Nowhere in the passage is any kind of dispute mentioned between
the Department of Agriculture and the Plant Gene Expression
Center, so choice e is incorrect.
233. d. The passage delivers the facts of the study thus far and
highlights the hopes of the scientists involved. Choice a is
incorrect because there is no criticism to be found in this passage.
There is no comparison in this passage, so choice b is incorrect.
The passage reports the finding of a study in progress, so choice c
is incorrect. Coice e is incorrect because nothing is eliminated.
234. d. The first line of the fourth paragraph states that Hake is the
director of the Plant Gene Expression Center. The center is a part
of the ARS, but it never stated that she is the director of it. She
may be leading the corngrass study as well, but this is also never
stated. It is never mentioned that she has anything to do with the
Department of Agriculture.
235. c. The manufacturers of cars that run on alternative energy would
be the most interested to hear the findings of the study since their
product relies on the continual growth of biofuels. Corn farmers
will probably not be planting very many genetically engineered
plants, and will likely stick to corn. Switchgrass and popcorn have
no relationship to each other, therefore Orville Redenbacher will
probably not have very much interest. Although the president may
find the data fascinating, its results certainly won’t further any of
his more pressing agendas. DNA specialists will also be interested
in the findings, but not as much as people dependent on
switchgrass production for their products to be useful.
501_04_235-298_501_master.qxd 4/29/13 12:35 PM Page 292
292
501 GMAT
®
Questions
236. d. The researchers express enthusiasm for the development of the
study because its production could have a multitude of positive
effects for developing products. There is no mention in the
passage of the USDA dropping the ball on the development of
biofuels, making choice a incorrect. Nowhere in the passage is the
attitude of American society toward genetically engineered
products implied. Choices b and c can be tricky since they are both
ideas taken from pop culture, but not from the information
provided in the passage. Choice e is incorrect because there is no
evidence that the cross-pollination of switchgrass could lead to
devastating effects.
237. d. This statement is made once in the first paragraph, as the first
line of the passage, and then again in the second paragraph.
Although data was collected to suggest the earth’s weather patterns
are getting warmer, it is not the central idea of the passage, so
choice a is incorrect. There is no specific citation of the efficiency
of coal versus natural gas, so choice b is incorrect. The Northeast
and Midwest are only mentioned once in the passage, so choice c
is incorrect. There is no reference to the depletion of energy
sources, so choice e is incorrect.
238. b. The author states in the first paragraph that Bentek Energy has
been estimating power burn since January 2005. There is no
alliance mentioned between the National Oceanic and
Atmospheric Administration and Bentek Energy, so choice a is
incorrect. It is never implied that Bentek Energy created new gas
units as choice c would indicate, so it’s incorrect. The answer is not
choice d because no playing field is mentioned. There is also no
mention of an agenda to raise public awareness, so choice e is
incorrect.
501_04_235-298_501_master.qxd 4/29/13 12:35 PM Page 293
293
501 GMAT
®
Questions
239. c. The two points made regarding the use of natural gas are
symbiotic. It can be reasonably inferred that if the weather
continues to get warmer, the use of energy-gobbling air
conditioning will continue to rise. There is no tone of impending
doom with looming extinction articulated, so choice a is incorrect.
There is no reference to a dwindling gas resource, so there is no
need to assume how much energy there is from the facts in this
article, so choice b is incorrect. Although the West’s weather
conditions are mentioned to have changed the least, this cannot be
inferred as being favorable since this is a subjective matter, so
choice d is incorrect. Based on the information in the passage,
there is no reason to conclude that there will be an increase in
devastating storms, so choice e is incorrect.
240. b. This sentence reiterates the central idea of the passage. The
passage never presents an argument, so choice a is incorrect. A
new point of view is never presented, so choice c is incorrect.
Choice d is incorrect since the author writes with objectivity,
presenting data that’s been collected. There is no way to tell what
would be of most interest to him. There is never an intimation of
hope in the passage, so choice e is incorrect.
241. d. The author cites from a complimentary standpoint the effects of
weather and other variables on the use of natural gas, and noting
that old, less effective agents are being replaced implies the author
views natural gas in a favorable light. Whereas the author seems to
be positive, he never comes off as dreamy, so choice a is incorrect.
The author’s tone is slightly biased, ruling out objective (choice b)
as the correct answer. The passage provides facts to back the
findings, so the tone cannot be hollow (choice c). The passage is far
from elegiac, which would be lamenting or sad, so choice e is
incorrect.
501_04_235-298_501_master.qxd 4/29/13 12:35 PM Page 294
294
501 GMAT
®
Questions
242. c. The movie rental store and streaming video are both vessels that
supply the consumer with access to movies. Renting movies was a
primary way of watching releases at home for a long time. Like
natural gases, as streaming video gained popularity, numbers of
movie rental stores began to dwindle, with huge franchises going
out of business due to a lack of demand, just as coal will. A
chandelier and a table lamp are both sources of light, but the
relationship doesn’t resemble that between coal and natural gas. A
kick drum and a drum machine are two forms of percussion, and
one is new and the other is old, but the drum machine will not
replace the kick drum. An electrical outlet and a generator are two
different sources of power. A bicycle and a skateboard are two
modes of transportation, but one is not likely to replace the other.
243. c. Choice a is incorrect because it was not only brand value, but
the translation into customer value that led to strengthening the
system capabilities leading to repeat success. Although consumers
were targeted through promotions and other programs, the
passage does not mention formal opinion surveys of consumers,
making choice b incorrect. Choice c is the correct answer, making
choices d and e incorrect.
244. b. This passage is written from the first-person perspective
boasting the company’s successes and ambitions. Choice b is the
only available option. Choice a is incorrect because although this
claims to be a factual report, it is by no means ambivalent. Choice
c is incorrect because a first-person account is hardly ever anything
but subjective. There is no hypothesis present and nothing to
conclude, so choice d is incorrect. Choice e is incorrect because
there is no trace of criticism.
245. a. Just as XYZ Cola did, the mutant ants become the strongest
most capable ants in their species. XYZ leveled the competition.
Choice b is incorrect because a blockbuster film is much larger
than a miniseries, which is generally less successful. Choice c is
incorrect because we read that ABC Soda has expanded to include
other types of beverages; choice d is incorrect for the same reason.
501_04_235-298_501_master.qxd 4/29/13 12:35 PM Page 295
295
501 GMAT
®
Questions
246. b. The passage seeks to show the accomplishments of the company
over time, relating all its feats. There were no comparisons noted,
so choice a is incorrect. There weren’t different theories presented,
so choice c is incorrect. No hypothesis is ever noted, so choice d is
incorrect. The compiling of facts is presented factually without an
intonation of persuasion, so choice e is incorrect.
247. b. Prospect Park is a large public park and Bank of America is a
large bank open to anyone who qualifies; a private country club is
also a park but only members are allowed in to use it, just as a
hedge fund is a form of a bank but is not open to the general
public. An iPad and a plasma television have no relationship to
each other aside from both having the capability to play motion
pictures, so choice a is incorrect. A motorcycle and a Mack truck
are two forms of transportation, but their relationship does not
resemble the one in question, so choice c is incorrect. A state
hospital and a chiropractor’s clinic would both treat any patient, so
choice d is incorrect. The two pizza places have no relationship to
each other except for making pizza, so choice e is incorrect.
248. d. The first line of the third paragraph states that hedge funds will
typically always maintain a positive return regardless of how the
market is doing. The second paragraph justifies choice a. The
second line of paragraph three proves choice b to be true. The first
paragraph states that high net individuals can invest in hedge
funds, so choice c is true. The last paragraph states that the credit
crisis of 2008 affected hedge fund strategies, so choice e is
incorrect.
249. c. Based on the information provided in the passage, it can be
easily inferred that hedge funds are designed by capitalists and
economists to benefit those at the top of the finance food chain,
insinuating that those involved would reap the most benefit. If the
hedge funds lost money per transaction and were difficult to
access, this wouldn’t benefit those involved with the fund, and
high-net-worth people and institutions would not have so much of
an interest in being involved with it, making choice a incorrect.
There is never any mention of alcohol, so choice b is incorrect.
There is never any mention of savings bonds, so choice d is
incorrect. There is no evidence to support choice e.
501_04_235-298_501_master.qxd 4/29/13 12:35 PM Page 296
296
501 GMAT
®
Questions
250. a. Didactic is defined as designed or intended to teach. Choice b,
sententious, would also regard teaching, but from a moralistic
standpoint, which this passage lacks. The author speaks from an
objective standpoint, so none of the other answer choices would be
possible, including detached, because in order for one to detach,
there is an implication of a previous attachment.
251. d. The last paragraph states that after the crisis of 2008, hedge
funds were regulated more, implying that prior to that, without
supervision, liberties were taken. Choice a is a similar statement
with far too negative a connotation. There is no evidence to
support choice b. It is stated that hedge funds represent 1.1% of
the total funds, but it is not implied who has the rest of the funds,
so choice c is incorrect. Choice e is slanted and opinionated, so it
is incorrect.
252. b. This is the only answer that supports all the other ideas in the
passage. All the other answer choices are true, but they cannot
envelop the other points mentioned in the passage.
253. c. The fourth paragraph states that Barnaby and Boom, Inc.
currently controls the company’s assets. All the other options are
incorrect based on this statement.
254. b. The passage is written objectively and, over the course of the
material, details the facts surrounding the business of the Penelope
Royalty Trust. The passage never mentions the preservation of
resources, so choice a is incorrect. There is no evidence to support
an interest in possible oil depletion, so choice c is incorrect. Tax
breaks are mentioned without an indication of subjectivity, so
choice d is incorrect. The number of companies involved with
Penelope Royalty Trust is an arbitrary fact, so choice e is incorrect.
255. d. The first line of the third paragraph states that the principal
productive wells on Jeepers Ranch are at these two geologic units,
and it is previously revealed that Jeepers Ranch is the chief
producer for oil for the trust. This makes all the other choices
incorrect.
297
501 GMAT
®
Questions
501_04_235-298_501_master.qxd 4/29/13 12:35 PM Page 297
256. a. The fifth paragraph breaks down why oil and gas stocks differ
from others and why they would pay out at a different rate. Choice
b is incorrect because there is no hint of criticism noted in the
author’s tone. Choice c is incorrect for the same reason. Choice d
is incorrect because it isn’t implied that the author doesn’t already
know what the stock differences are. Choice e is incorrect because
this opinion is not stated.
501_04_235-298_501_master.qxd 4/29/13 12:35 PM Page 298501_05_299-330_501_master.qxd 4/29/13 12:36 PM Page 299
B
lank Page
SECTION 3
GMAT Integrated
Reasoning Section
In today’s business world, you will often be tasked with making
sound decisions, determining patterns, and using both verbal and quanti-
tative skills to solve problems. The GMAT integrated reasoning section
measures these skills.
The integrated reasoning section is composed of four different question
formats. These questions are intended to test your ability to problem solve
when given an array of different types of data—a skill necessary in today’s
business world.
501_05_299-330_501_master.qxd 4/29/13 12:36 PM Page 300
Blank Page
501_05_299-330_501_master.qxd 4/29/13 12:36 PM Page 301
5
Graphics
Interpretation
Graphics interpretation questions are one question type you’ll
see in the integrated reasoning section of the GMAT. With graphics inter-
pretation questions, you will be asked to analyze a graphic and use the
graphic to make well-reasoned deductions. Some of the graphics you will
find are Venn diagrams, line graphs, bar graphs, scatterplots, and scien-
tific diagrams. With each graphics interpretation question, you will be
asked to correctly fill in the blank of two statements using drop-down
menus. The correct answers are deductions best supported by the details
in the graphics.
Test-Taking Tips
When working on the graphics interpretation questions of the GMAT,
there are a few techniques to keep in mind that can help you improve your
score. Practice using the following techniques when answering the graph-
ics interpretation questions in this book.
501_05_299-330_501_master.qxd 4/29/13 12:36 PM Page 302
302
501 GMAT
®
Questions
How Should I Work on Graphics Interpretation Questions?
Begin by reading the question and the graphic carefully.
Be sure to read both of the statements you will need to
complete—this will help you know what information to look
for in the graphic.
Read all the answer choices in the drop-down menus.
Reading the answer choices will help you know the type of
information you need to find in the graphic. Additionally,
reading the answer choices will help you know the degree of
specificity the answers need to be in. For example, a question
may ask you about the slope of a graphed line. Yet the question
is not asking you to calculate the slope; the question may
simply be asking you to determine whether the slope is positive
or negative.
Eliminate any clearly incorrect answer choices.
You may be able to spot a clearly incorrect answer immediately
once you’ve seen the answer choices provided. If so, focus on
the remaining answer choices.
What Do I Do If I Don’t Know the Answer?
Try to make an educated guess.
Use the information you know and understand to choose the
best answer that you can.
Do not skip the question unless you are sure you do not know
the answer.
With the computerized GMAT, you will be unable to return to
the question later.
Make sure you understand the question and the graphic.
Take a moment to reread the statements and to review the
graphic. You may have misunderstood the question or missed a
key word (such as “not” or “only”). Once you are sure you
clearly understand what the question is asking, try to answer it
again.
Give yourself another minute or two to answer the question,
but do not take too long.
You want to be sure to answer the question correctly. But you
also want to have enough time to answer all the questions on
the test. So be sure to keep a calm, even pace as you work on
the question. And try to avoid wasting time
501_05_299-330_501_master.qxd 4/29/13 12:36 PM Page 303
303
501 GMAT
®
Questions
General GMAT Tips and Techniques
Use scrap paper.
Scrap paper is a key tool on the GMAT. You can write down
answer choices that you’ve eliminated. You can also write down
key pieces of information from the questions and the graphics.
Also, be sure to do any calculations on graph paper instead of
in your head in order to avoid careless mistakes.
Pace yourself.
Give yourself enough time to read and comprehend the
question, but be conscientious about your time. Managing
your time wisely will help you have enough time to finish the
test—or complete as many questions as possible.
Be sure you have everything you need before you begin the
test.
Be sure you have pencils, paper, and anything else you need
before beginning the test.
Try to answer all the questions on the test.
Answering all the questions on the test is a way to shoot for a
higher score. Of course, you may encounter questions you do
not know how to answer. While you do not want to wildly
guess at the answers, you do want to try to make educated
guesses. Even if you can eliminate one answer choice as
incorrect, you have improved your chances of picking the
correct answer for that question. So try to answer all the
questions, or as many questions as possible.
Set 10
Now it is time to answer GMAT graphics interpretation practice questions
that have been designed to test your integrated reasoning skills. Good luck!
501_05_299-330_501_master.qxd 4/29/13 12:36 PM Page 304
257.
Earns $50,000
No college
a year or more
degree
= 10 adults
304
501 GMAT
®
Questions
Refer to the pictograph of a survey of adults. Each symbol
represents 10 adults in a survey of 250 total adults. Complete each
statement according to the information presented in the diagram.
If one adult is randomly selected from the 250 surveyed, the
chance that the adult will make less than $50,000 a year or have a
college degree or both is:
a. 1 out of 5
b. 1 out of 2
c. 1 out of 10
d. 4 out of 5
If one adult is randomly selected from the 250 surveyed, the
chance that the adult will both make less than $50,000 a year and
have a college degree is:
a. 4 out of 25
b. 1 out of 2
c. 1 out of 10
d. 4 out of 5
501_05_299-330_501_master.qxd 4/29/13 12:36 PM Page 305
305
258.
Male Infants Mass (kg) by Age (Weeks)
5.5
Z
5.0
4.5
Y
4.0
Mass (kg)
X
3.5
3.0
2.5
2.0
1.5
1.0
0.5
0
0 1 2 3 4 5 6 7 8
Age (weeks)
501 GMAT
®
Questions
The line graph models the mass, in kilograms, of an average male
infant up to 8 weeks of age. Points X, Y, and Z represent the
masses for the male infant at ages 0.5, 2, and 8 weeks, respectively,
according to the model. For each question, select the option that
creates the most accurate statement based on the information
provided.
For integer values of the age from 0.5 weeks to 2 weeks, the
average (arithmetic mean) mass falls approximately between
_____ kilograms.
a. 3 and 3.5
b. 3.5 and 3.8
c. 3.5 and 4.2
The change in mass from age 0.5 weeks to 2 weeks is approximately
_____ the change in mass from age 2 weeks to 8 weeks.
a. equal to
b. one third
c. one eighth
501_05_299-330_501_master.qxd 4/29/13 12:36 PM Page 306
259.
More Moisture
Soil Moisture Level
Less Moisture
0 25 50 75 100
Temperature, °C
306
501 GMAT
®
Questions
The graph shown is a scatterplot with 20 points. Each line
represents soil samples taken from a specific area of land. Each
point represents a sample of soil. Each sample of soil was tested for
moisture level. Fill in the blanks in each of the following
statements based on the information given by the graph.
The relationship between the temperature of the soil and the
moisture level is:
a. positive
b. negative
c. zero
The slope of the solid line is ___ the slope of the dashed line.
a. less than
b. greater than
c. equal to
501_05_299-330_501_master.qxd 4/29/13 12:36 PM Page 307
307
260.
20 years of
No health
age or older insurance
= 20 people
501 GMAT
®
Questions
Refer to the Venn diagram of a survey. Each symbol represents 20
people in a sample of 200. Complete each statement according to
the information presented in the diagram.
If one person is selected at random from the 200 surveyed, the
chance that the person will be under 20 or have health insurance
or both is:
a. 1 out of 10
b. 2 out of 10
c. 1 out of 5
d. 9 out of 10
If one person is selected at random from the 200 surveyed, the
chance that the person will be both under 20 and have health
insurance is:
a. 1 out of 10
b. 2 out of 10
c. 1 out of 5
d. 4 out of 5
501_05_299-330_501_master.qxd 4/29/13 12:36 PM Page 308
261.
Took SAT Born before
tests 1975
No student
loans
= 10 students
308
501 GMAT
®
Questions
Refer to the pictograph of a survey. Each symbol represents 10
students in a sample of 400. Complete each statement according to
the information presented in the diagram.
If one student is selected at random from the 400 surveyed, the
chance that the student will have not taken the SAT tests, received
student loans, be born after 1975, or all of these is:
a. 1 out of 8
b. 1 out of 5
c. 3 out of 10
d. 7 out of 8
If one student is selected at random from the 400 surveyed, the
chance that the student will have not taken the SAT tests, received
student loans, and be born after 1975 is:
a. 1 out of 20
b. 1 out of 4
c. 3 out of 10
d. 7 out of 8
501_05_299-330_501_master.qxd 4/29/13 12:36 PM Page 309
309
262.
60 years of age No driver’s
or older license
= 5 people
501 GMAT
®
Questions
Refer to the pictograph of a survey. Each symbol represents 5
people in a sample of 200. Complete each statement according to
the information presented in the diagram.
If one person is selected at random from the 200 surveyed, the
chance that the person will be under 60 or have a driver’s license or
both is:
a. 1 out of 8
b. 3 out of 20
c. 7 out of 8
d. 9 out of 10
If one person is selected at random from the 200 surveyed, the
chance that the person will both be under 60 and have a driver’s
license is:
a. 1 out of 8
b. 2 out of 5
c. 9 out of 40
d. 5 out of 8
501_05_299-330_501_master.qxd 4/29/13 12:36 PM Page 310
263.
Owns two or Does not live
more cars in a city
Unmarried
= 20 people
310
501 GMAT
®
Questions
Refer to the pictograph of a survey. Each symbol represents 20
people in a sample of 700. Complete each statement according to
the information presented in the diagram.
If one person is selected at random from the 700 surveyed, the
chance that the person will own less than two cars, live in a city, be
married, or all of these is:
a. 1 out of 14
b. 2 out of 7
c. 3 out of 7
d. 6 out of 7
If one person is selected at random from the 700 surveyed, the
chance that the person will own less than two cars, live in a city,
and be married is:
a. 1 out of 14
b. 1 out of 7
c. 3 out of 7
d. 5 out of 7
501_05_299-330_501_master.qxd 4/29/13 12:36 PM Page 311
311
264.
Mass Garbage Produced by City over Time
501 GMAT
®
Questions
5
C
Mass of Garbage (millions of tons)
4
3
B
2
A
1
0
2000 2002 2004 2006 2008 2010
Time (years)
The graph models the mass, in millions of tons, of the garbage
produced annually by a city over a decade. Points A, B, and C
represent the masses of garbage produced in 2003, 2005, and 2008,
respectively, according to the model. Select the option that creates
the most accurate statement based on the information provided.
For values from years 2003 to 2010, the average (arithmetic mean)
mass of garbage falls approximately between
a. 1 million tons and 1.5 million tons.
b. 2 million tons and 2.5 million tons.
c. 3 million tons and 4 million tons.
The change in mass of garbage from 2003 to 2005 is approximately
___ the change in mass of garbage from 2005 to 2008.
a. less than
b. equal to
c. greater than
501_05_299-330_501_master.qxd 4/29/13 12:36 PM Page 312
265.
Customer Product Reviews Posted Online over Time
Number of Customer Product Reviews
100
75
50
Z
Y
25
X
0
0 5 10 15 20 25
Time (weeks)
312
501 GMAT
®
Questions
The line graph models the number of product reviews customers
wrote online regarding a product up to 25 weeks after the product
was posted online. Points X, Y, and Z represent the number of
product reviews posted at 5 weeks, 13 weeks, and 20 weeks,
respectively, according to the model. Select the option that creates
the most accurate statement based on the information provided.
For values from 5 weeks to 25 weeks, the average (arithmetic
mean) number of product reviews falls approximately between
a. 5 and 10 reviews.
b. 10 and 20 reviews.
c. 30 and 55 reviews.
The change in number of product reviews from 5 weeks to 13
weeks is approximately ______ the change in number of product
reviews from 13 weeks to 20 weeks.
a. equal to
b. 2 times
c. 4 times
501_05_299-330_501_master.qxd 4/29/13 12:36 PM Page 313
313
266.
Sales of VHS Players and VHS Tapes
5,000
C
Numbers Sold (in thousands)
4,000
VHS tapes
3,000
D
2,000
VHS players
A
1,000
B
0
1990 1995 2000 2005 2010
Time (years)
501 GMAT
®
Questions
The line graph models the number of VHS players and VHS tapes
sold at a given electronics store over time. Points A and B
represent the number of VHS players sold in 1995 and 2005,
respectively. Points C and D represent the number of VHS tapes
sold in 1995 and 2000, respectively. Select the option that creates
the most accurate statement based on the information provided.
From 1995 to 2005 the average (arithmetic mean) number of VHS
players sold falls approximately between
a. 1300 and 1200.
b. 800 and 600.
c. 400 and 200.
The percent change in the number of VHS player sales from 1995
to 2005 is approximately __________ the percent change in the
number of VHS tape sales from 1995 to 2000.
a. equal to
b. half of
c. two-thirds of
501_05_299-330_501_master.qxd 4/29/13 12:36 PM Page 314
267.
E-Readers and Tablets Scored
by Product Testers
5
4
Scores
3
2
1
0
Scored by 120 testers
Scored by 35 testers
Scored by 58 testers
Scored by 80 testers
A B C D
Brands of E-Readers and Tablets
314
501 GMAT
®
Questions
The graph portrays the results of product testers regarding
competing brands of e-readers and tablets. Bars A, B, C, and D
represent different brands of e-readers and tablets, according to
the model. Select the option that creates the most accurate
statement based on the information provided.
The score that resulted in the statistical mode was achieved
by ___________.
a. Product A
b. Product B
c. Product C
d. Product D
The average (arithmetic mean) review on all e-readers and tablets
falls approximately between
a. 1 and 2.
b. 2 and 3.
c. 3 and 4.
d. 4 and 5.
501_05_299-330_501_master.qxd 4/29/13 12:36 PM Page 315
315
268.
Bacteria Level in Greenville Lake
50°C
Water Temperature (°C)
40°C
200
30°C
150
20°C
100
10°C
50
Bacteria Level (cfu/100 mL)
0°C
0
0 5 10 15 20 25 30
Time (Days)
501 GMAT
®
Questions
The graph depicts measurements of water temperature and
bacteria levels of Greenville Lake. Both the water temperature and
bacteria levels were measured every day at noon for 30 days in a
row. The solid line represents the change in water temperature
over time, and the dashed line represents the change in bacteria
levels over time. Select the option that creates the most accurate
statement based on the information given by the graph.
The relationship between the water temperature of the lake and
the bacteria level is
a. positive.
b. negative.
c. zero.
The slope of the line representing the change in water temperature
is _______ the slope of the line representing the change in bacteria
levels.
a. less than
b. greater than
c. equal to
501_05_299-330_501_master.qxd 4/29/13 12:36 PM Page 316
269.
10
0
/g
Bacteria Levels in Ground Beef
10
2
/g
10
4
/g
10
6
/g
0
0°C 50°C 100°C 150°C 200°C
Temperature °C
316
501 GMAT
®
Questions
The graph is a scatterplot with 25 points, each representing a beef
tested for bacteria levels. Each beef sample was heated to a certain
temperature, and the bacterial level in the sample was then
measured. The solid line is the regression line and the dashed line
is the line through points (10
8
/g, 0°C) and (10
0
/g, 200°C). Fill in
the blanks in each of the following statements based on the
information given by the graph.
The relationship between the temperature of the beef sample and
the bacterial level is
a. positive.
b. negative.
c. zero.
The slope of the regression line is _________ the slope of the
dashed line.
a. less negative than
b. more negative than
c. equal to
501_05_299-330_501_master.qxd 4/29/13 12:36 PM Page 317
317
270.
60
Age of Tires on Vehicle (months)
48
36
24
12
0
0 5 10 15 20
Rate of Deceleration in Feet per Seconds (fps)
501 GMAT
®
Questions
The graph is a scatterplot with 40 points, each representing the
results of a test conducted on a vehicle. During each test, the same
vehicle and same set of tires was driven to a speed of 60 mph, and
then the vehicle decelerated to a stop. The rate of deceleration for
each test was measured. The solid line is the regression line and
the dashed line is the line through points (0/0 fps) to point (60/20
fps). Fill in the blanks in each of the following statements based on
the information given by the graph.
The relationship between the age of the tires on the vehicle and
the rate of deceleration is
a. positive.
b. negative.
c. zero.
The slope of the regression line is ______ the slope of the dashed
line.
a. less than
b. greater than
c. equal to
501_05_299-330_501_master.qxd 4/29/13 12:36 PM Page 318
271.
20
D
Lactic Acid in Millimoles per Liter (mmol/L)
18
16
14
C
12
8
B
6
A
4
2
0
0 2.5 5 7.5 10 12.5 15
Speed (km/hour)
318
501 GMAT
®
Questions
The graph is a line graph depicting the lactic acid levels in the
muscles of a jogger on a laboratory treadmill. The speed of the
jogger was recorded along with the lactic acid levels every five
minutes. Fill in the blanks in each of the following statements
based on the information given by the graph.
The relationship between the running speed and the lactic acid
levels is
a. positive.
b. negative.
c. zero.
The slope of the line between Point A and Point B is _________
the slope of the line between Point C and Point D.
a. less than
b. greater than
c. equal to
501_05_299-330_501_master.qxd 4/29/13 12:36 PM Page 319
319
272.
Archeological Dig Site
Big Falls National Park
A B C
D E F
G H I
Cretaceous fossil
Jurassic fossil
501 GMAT
®
Questions
The map shows an archeological dig site occurring in a section of
Big Falls National Park, along with markers for the locations of
fossils found. Fill in each blank using the drop-down menus to
create the most accurate statement on the basis of the information
provided.
Cretaceous fossils make up approximately ______ of the fossils
found in the archeological dig.
a. 20%
b. 25%
c. 80%
According to the map, ____________ of all Jurassic fossils were
found in Section I.
a. about one quarter
b. about one half
c. about three quarters
273.
320
501 GMAT
®
Questions
501_05_299-330_501_master.qxd 4/29/13 12:36 PM Page 320
The diagram shows, in three columns, different sections of the
electromagnetic spectrum. The far left column shows the radio
and television waves portion of the electromagnetic spectrum. The
central column shows all the types of waves in the electromagnetic
spectrum. The far right column shows the visible portion of the
electromagnetic spectrum.
Fill in each blank using the drop-down menus to create the most
accurate statement on the basis of the information provided.
The color green spans closest to ______ of the visible light
spectrum.
a. 5%
b. 25%
c. 45%
According to the diagram, the color ______ marks the beginning
of light with a 10
15
Hz frequency and the beginning of the visible
spectrum.
a. violet
b. green
c. white
d. red
501_05_299-330_501_master.qxd 4/29/13 12:36 PM Page 321
321
274.
Cross Section of Native Plants
5
Common
Ninebark
4
Black-Eyed
3
Susan
2
Buffalo
Grass
1
Length (in feet)
2
4
6
8
10
12
14
16
18
20
501 GMAT
®
Questions
The diagram shows a cross section of three different plants,
providing information regarding the plants’ height and length of
roots. Fill in each blank using the drop-down menus to create the
most accurate statement on the basis of the information provided.
The roots of the buffalo grass are approximately ______ the length
of the roots of the common ninebark.
a. 20%
b. 30%
c. 50%
According to the diagram, the roots of the Black-Eyed Susan are
approximately _________ of the total length of the entire common
ninebark plant.
a. one tenth
b. one third
c. one half
501_05_299-330_501_master.qxd 4/29/13 12:36 PM Page 322
322
501 GMAT
®
Questions
Answers—Set 10
257. d. The question asks about adults who make less than $50,000 a
year or have a college degree or both, so this involves all the adults
except for the ones represented by the overlapping space in the
Venn diagram. Choice a is incorrect because this ratio represents
adults who earn more than $50,000 a year and do not have a
college degree. Choice b is incorrect because 200 adults out of 250
is not 1 out of 2. Choice c is incorrect because 200 adults out of
250 is not 1 out of 10.
a. The question asks about adults who make less than $50,000 a
year and have a college degree, so this involves all the adults
represented outside the Venn diagram. Choice b is incorrect
because 40 adults out of 250 is not 1 out of 2. Choice c is incorrect
because 40 adults out of 250 is not 1 out of 10. Choice d is
incorrect because 40 adults out of 250 is not 4 out of 5.
258. b. The question asks about the average (arithmetic mean) mass for
values between ages 0.5 weeks and 2 weeks. These are the values
that fall between points X and Y. So the average mass must be
between 3.5 and 3.8 kg. Choice a is incorrect because these are
values before 0.5 weeks. Choice c is incorrect because these are
values between 0.5 weeks and about 3 weeks.
b. The question compares the change in mass between points X
and Y with the change in mass between points Y and Z. The mass
changes less than 0.5 kilograms between points X and Y and
changes about 1.5 kilograms between points Y and Z, so the
correct answer is choice b. Choice a is incorrect because the mass
changes about three times as much between points Y and Z as it
changes between points X and Y. Choice c is incorrect because the
mass changes about three times as much between points Y and Z as
it changes between points X and Y.
501_05_299-330_501_master.qxd 4/29/13 12:36 PM Page 323
323
501 GMAT
®
Questions
259. b. The question asks about the relationship between the
temperature of the soil sample and the moisture level. According
to the scatterplot, moisture levels decrease as the temperature
increases. Therefore, this is a negative relationship. Choice a is
incorrect because both factors would have to increase in order to
have a positive relationship. Choice c is incorrect because there is a
correlation between soil temperature and moisture levels.
c. The question compares the slope of the lines. Both lines have
the same slope. Therefore, choice a is incorrect because the slope
of the solid line is not less than the slope of the dashed line. Choice
b is incorrect because the slop of the solid line is not greater than
the slope of the dashed line.
260. d. The question asks about people who are under 20 years old or
have health insurance or both, so this involves all the people except
for the ones represented by the overlapping space in the Venn
diagram. Therefore, this means nine stick figures, or 180 people
out of a total of 200 people surveyed. Choice a is incorrect because
180 people out of a total of 200 people is not 1 out of 10. Choice b
is incorrect because 180 people out of a total of 200 people is not 2
out of 10. Choice c is incorrect because 180 people out of a total of
200 people is not 1 out of 5.
c. The question asks about people who are under 20 years old and
have health insurance, so this involves all the people represented
outside the Venn diagram. Therefore, this means two stick figures,
or 40 people out of a total of 200 people surveyed (or 1 out of 5).
Choice a is incorrect because 40 people out of 200 is not 1 out of
10. Choice b is incorrect because 40 people out of 200 is not 2 out
of 10. Choice d is incorrect because 40 people out of 200 is not 4
out of 5.
501_05_299-330_501_master.qxd 4/29/13 12:36 PM Page 324
324
501 GMAT
®
Questions
261. d. The question asks about students who have not taken the SAT
tests or received student loans or were born after 1975, or all these
factors. So this involves all the students except for the ones
represented by the overlapping space in the Venn diagram.
Therefore, this number is represented by 35 symbols out of 40
symbols in the Venn diagram, or 7 out of 8. Choice a is incorrect
because 350 students out of 400 is not 1 out of 8. Choice b is
incorrect because 350 students out of 400 is not 1 out of 5. Choice
c is incorrect because 350 students out of 400 is not 3 out of 10.
b. The question asks about students who have not taken the SAT
tests, received student loans, and were born after 1975. So this
involves all the students represented outside the Venn diagram, or
100 students out of 400 students (or 1 out of 4). Choice a is
incorrect because 100 students out of 400 is not 1 out of 20.
Choice c is incorrect because 100 students out of 400 is not 3 out
of 10. Choice d is incorrect because 100 students out of 400 is not
7 out of 8.
262. c. The question asks about people who are under 60 or have a
driver’s license, or both. So this involves all the people represented
by the Venn diagram, except for the ones represented by the
overlapping space. Therefore, the correct answer is 175 people out
of 200, or 7 out of 8. Choice a is incorrect because 175 people out
of 200 is not 1 out of 8. Choice b is incorrect because 175 people
out of 200 is not 3 out of 20. Choice d is incorrect because 175
people out of 200 is not 9 out of 10.
b. The question asks about people who are under 60 and have a
driver’s license. So this involves all the people represented outside
the Venn diagram, or 80 out of 200 (or 2 out of 5). Choice a is
incorrect because 80 people out of 200 is not 1 out of 8. Choice c
is incorrect because 80 people out of 200 is not 9 out of 40. Choice
d is incorrect because 80 people out of 200 is not 5 out of 8.
501_05_299-330_501_master.qxd 4/29/13 12:36 PM Page 325
325
501 GMAT
®
Questions
263. d. The question asks about people who own less than two cars or
live in a city or are married, or all these factors. So this involves all
the people represented in the Venn diagram, except for the ones
represented by the overlapping space. Therefore, the correct
answer is 600 people out of 700, or 6 out of 7. Choice a is
incorrect because 600 people out of 700 is not 1 out of 14. Choice
b is incorrect because 600 people out of 700 is not 2 out of 7.
Choice c is incorrect because 600 people out of 700 is not 3 out
of 7.
b. The question asks about people who own less than two cars, live
in a city, and are married. So this involves all the people
represented outside the Venn diagram, or 100 people out of 700
(1 out of 7). Choice a is incorrect because 100 people out of 700 is
not 1 out of 14. Choice c is incorrect because 100 people out of
700 is not 3 out of 7. Choice d is incorrect because 100 people out
of 700 is not 5 out of 7.
264. c. The question asks about the average (arithmetic mean) mass for
values between 2003 and 2010. These are the values that fall
between point A and the end of the graphed line. So the average
mass must be between 1.5 and 5.0 millions of tons. Choice a is
incorrect because these are values before 2003. Choice b is
incorrect because most of the values represented by the line are
greater than 2.5 million tons, so this can’t be the average.
a. The question compares the change in mass between points A
and B with the change in mass between points B and C. The mass
changes about 1 million tons between points A and B and changes
about 2 million tons between points B and C, so the correct answer
is choice a. Choices b and c are incorrect because the mass
changes less between points A and B than it changes between
points B and C.
501_05_299-330_501_master.qxd 4/29/13 12:36 PM Page 326
326
501 GMAT
®
Questions
265. c. The question asks about the average (arithmetic mean) number
of product reviews for values between 5 weeks and 25 weeks.
These are the values that fall between points X and Z on the
graphed line. The correct answer is choice c because most of the
values graphed by the line fall between 30 and 55 reviews. Choice
a is incorrect because these are the very values represented by the
line. Choice b is incorrect because these are the very values
represented by the line and most of the values represented by the
line are for 20 reviews or more.
b. The question compares the change in the number of product
reviews between points X and Y with the change in the number of
product reviews between points Y and Z. The change in the
number of product reviews between points X and Y is about 35
reviews. The change in the number of product reviews between
points Y and Z is about 15 reviews. So the correct answer is choice
b. Choice a is incorrect because the change is not equal. Choice c
is incorrect because the change between points X and Y is not four
times the change between points Y and Z.
266. b. The question asks about the average (arithmetic mean) number
of VHS players sold between 1995 and 2005. These are the values
that fall between points A and B, so the average must be between
200 and 1200 VHS players sold. Choice a is incorrect because
these values do not fall between 1995 and 2005. Choice c is
incorrect because most of the values graphed between points A and
B are greater than 400, so the average is likely greater than 400.
c. The question compares the percent change in the number of
VHS player sales from 1995 to 2005 with the percent change in the
number of VHS tape sales from 1995 to 2000. There was a change of
1,000 in the number of VHS players sold over 10 years (or 200 sales
per year) and a change of 1,500 in the number of VHS tapes sold
over 5 years (or 300 sales per year). So, the percent change in VHS
player sales is about two-thirds the percent change in VHS tape sales.
Choice a is incorrect because the percent change in the number of
VHS player sales from 1995 to 2005 and the percent change in the
number of VHS tape sales from 1995 to 2000 are not equal. Choice
b is incorrect because the percent change in the number of VHS
player sales from 1995 to 2005 is more than half of the percent
change in the number of VHS tape sales from 1995 to 2000.
501_05_299-330_501_master.qxd 4/29/13 12:36 PM Page 327
327
501 GMAT
®
Questions
267. a. The question asks about the product with the score that is the
statistical mode. Product A had the most testers give it the same
score (a score of 5), therefore this is the mode. Choice b is
incorrect because only 35 testers gave Product B a score of 2.
Choice c is incorrect because only 58 testers gave Product C a
score of 3. Choice d is incorrect because only 80 testers gave
Product D a score of 4.
d. The question asks for the average score on all e-readers and
tablets. Because a majority of the scores were 4 and 5, the average
is most likely between 4 and 5. Choice a is incorrect because no
products scored a 1 and only Product B scored a 2 thirty-five
times. Choice b is incorrect; less than half of the scores were in
this range. Choice c is incorrect; less than half of the scores were
in this range.
268. a. The question asks about the relationship between the water
temperature of the lake and the bacteria level. The graph shows
that as the water temperature increases, the bacteria level
increases, which is a positive relationship. Choice b is incorrect
because one factor would have to increase while the other factor
decreases in order to have a negative relationship. Choice c is
incorrect because there is a correlation between water temperature
values and bacteria levels.
b. The question compares the slope of the line representing the
change in water temperature with the slope of the line
representing the change in bacteria levels. The line graphed to
represent the change in water temperature has a positive slope, and
the line graphed to represent the change in bacteria levels has a
less positive slope. Choice a is incorrect because the slope of the
line representing the change in water temperatures if greater than
the slope of the line representing bacteria levels. Choice c is
incorrect because the slopes of the lines are not equal.
501_05_299-330_501_master.qxd 4/29/13 12:36 PM Page 328
328
501 GMAT
®
Questions
269. b. The question asks about the relationship between the
temperature of the beef sample and the bacterial level. According
to the scatterplot, bacteria levels decrease as the temperature
increases. Therefore, this is a negative relationship. Choice a is
incorrect because both factors would have to increase in order to
have a positive relationship. Choice c is incorrect because there is a
correlation between beef temperature and bacteria levels.
a. The question compares the slope of the lines. The regression
line has a negative slope, and the dashed line has a very negative
slope. Therefore, choice b is incorrect because the regression line
has a less negative slope than the dashed line. Choice c is incorrect
because the slopes of the lines are not equal.
270. a. The question asks about the relationship between the age of the
tires on a vehicle and the rate of deceleration. According to the
scatterplot, the rate of deceleration increases as the tires increase in
age. Therefore, this is a positive relationship. Choice b is incorrect
because one factor would have to increase while the other factor
decreases in order to have a negative relationship. Choice c is
incorrect because there is a correlation between the age of the tires
on a vehicle and the rate of deceleration.
a. The question compares the slope of the lines. The regression
line has a positive slope, and the dashed line has a very positive
slope. Choice b is incorrect because the slope of the regression line
is not greater than the slope of the dashed line. Choice c is
incorrect because the slopes of the lines are not equal.
501_05_299-330_501_master.qxd 4/29/13 12:36 PM Page 329
329
501 GMAT
®
Questions
271. a. The question asks about the relationship between the running
speed and the lactic acid levels. According to the graph, as the
speed of the runner increases, the lactic acid levels in the runner
increase. Therefore, the relationship is positive. Choice b is
incorrect because one factor would have to increase while the
other decreases in order to have a negative relationship. Choice c
is incorrect because there is a correlation between the running
speed and the lactic acid levels.
a. The question compares the slope of the line between Point A
and Point B with the slope of the line between Point C and Point
D. The slope of the line between Point A and Point B is 2/5,
whereas the slope of the line between Point C and Point D is 8/5.
Therefore, the slope of the line between Point A and Point B is
less than the slope of the line between Point C and Point D.
Choice b is incorrect because the slope of the line between Point A
and Point B is not greater than the slope of the line between Point
C and Point D. Choice c is incorrect because the slopes of the lines
are not equal.
272. a. The question asks about percentage of the fossils that are
Cretaceous. According to the map, there are 5 Cretaceous fossils
and 20 Jurassic fossils found in the dig. Therefore, there are 5
Cretaceous fossils out of a total of 25 fossils, so 20% of the fossils
found are Cretaceous fossils. Choice b is incorrect because there
are 5 Cretaceous fossils out of a total of 25 fossils, not 5
Cretaceous fossils out of a total of 20 fossils. Choice c is incorrect
because there are 5 Cretaceous fossils out of a total of 25 fossils,
not 20 Cretaceous fossils out of a total of 25 fossils.
b. The question asks about the portion of all Jurassic fossils that
were found in Section I of the map. According to the map, a total
of 20 Jurassic fossils were found, and 8 of them were found in
Section I. Therefore, 8 out of 20, or about one half, of all Jurassic
fossils were found in Section I. Choice a is incorrect because more
than 5 Jurassic fossils were found in Section I. Choice c is
incorrect because 8 out of 20 of all Jurassic fossils were found in
Section I, not 15 out of 20.
501_05_299-330_501_master.qxd 4/29/13 12:36 PM Page 330
330
501 GMAT
®
Questions
273. b. The question asks the percent that the color green takes up in
the spectrum of visible light. According to the diagram, green
takes up roughly one quarter, or 25%, of the visible light spectrum.
Choice a is incorrect because green takes up about 25% of the
visible light spectrum; violet takes up about 5% of the visible light
spectrum. Choice c is incorrect because no colors take up nearly
half of the visible light spectrum.
a. The question asks about the color that both begins the visible
light spectrum and additionally begins the light in the spectrum
that has a 10
15
Hz frequency. According to the diagram, the visible
light spectrum begins at 10
15
Hz frequency, and the color that
begins at this frequency is violet. Choice b is incorrect because
green has a lower frequency than 10
15
Hz frequency. Choice c is
incorrect because white is not a color in the visible light spectrum
on the diagram. Choice d is incorrect because red does not begin
the visible light spectrum and has a frequency closer to 10
14.5
Hz.
274. c. The question asks the percent of the length of the roots of the
common ninebark that are taken up by the roots of the buffalo
grass. According to the diagram, the roots of the buffalo grass are
8 feet long, and the roots of the common ninebark are 15 feet
long. Therefore, the roots of the buffalo grass are about 50% the
length of the roots of the common ninebark. Choice a is incorrect
because the roots of the buffalo grass are 8 feet long, not 3 feet
long. Choice b is incorrect because the roots of the buffalo grass
are 8 feet long, not 5 feet long.
b. The question asks the fraction of the length of the entire
common ninebark plant that are taken up by the roots of the
Black-Eyed Susan. According to the diagram, the common
ninebark plant is 4 feet tall with 15-foot roots. So its total length is
19 feet. The roots of the Black-Eyed Susan are 6 feet long.
Therefore, the roots of the Black-Eyed Susan are about a third of
the length of the entire common ninebark plant. Choice a is
incorrect because the roots of the Black-Eyed Susan are 6 feet
long, not 1.9 feet long. Choice c is incorrect because the roots of
the Black-Eyed Susan are 6 feet long, not 9.5 feet long.
501_06_331-370_501_master.qxd 4/29/13 12:37 PM Page 331
6
Two-Part Analysis
Two-part analysis questions are one question type you’ll see in the
integrated reasoning section of the GMAT. With two-part analysis ques-
tions, you will be asked questions that have two-part solutions. The solu-
tions will be provided in a table with several columns. In order to answer
these questions, you must choose correct answers from column 3. The cor-
rect answers are deductions best supported by the details in the text or by
provided formulas and information.
Test-Taking Tips
When working on the two-part analysis questions of the GMAT, there are
a few techniques to keep in mind that can help you improve your score.
Practice using the following techniques when answering the two-part analy-
sis questions in this book.
501_06_331-370_501_master.qxd 4/29/13 12:37 PM Page 332
332
501 GMAT
®
Questions
How Should I Work on Two-Part Analysis Questions?
Begin by reading the question carefully.
Be sure to read the entire question, including any text and
formulas provided.
Read all the answer choices in the columns.
The answer choices often provide important information that
you will need to solve the problem. So, read all the answer
choices carefully. For example, a question may ask you to
determine which answer choice fits a series of requirements.
To answer the question correctly, you will need to assess each
of the answer choices.
Eliminate any clearly incorrect answer choices.
You may be able to spot a clearly incorrect answer immediately
once you’ve seen the answer choices provided. If so, focus on
the remaining answer choices.
What Do I Do if I Don’t Know the Answer?
Try to make an educated guess.
Use the information you know and understand to choose the
best answer that you can.
Do not skip the question unless you are sure you do not know
the answer.
With the computerized GMAT, you will be unable to return to
the question later.
Make sure you understand the question and the answer choices.
Take a moment to reread the statements and to review the
answer choices. You may have misunderstood the question or
missed a key word (such as “not” or “only”). Once you are sure
you clearly understand what the question is asking, try to
answer it again.
Give yourself another minute or two to answer the question,
but do not take too long.
You want to be sure to answer the question correctly, but you
also want to have enough time to answer all the questions on
the test. Be sure to maintain a calm, even pace as you work on
the question, and try to avoid wasting time
501_06_331-370_501_master.qxd 4/29/13 12:37 PM Page 333
333
501 GMAT
®
Questions
General GMAT Tips and Techniques
Use scrap paper.
Scrap paper is a key tool on the GMAT. You can write down
answer choices that you’ve eliminated. You can also write down
key pieces of information from the questions and the answer
choices.
Pace yourself.
Give yourself enough time to read and comprehend the
question, but be conscientious about your time. Managing
your time wisely will help you have enough time to finish the
test—or complete as many questions as possible.
Be sure you have everything you need before you begin the test.
Be sure you have pencils, paper, and anything else you need
before beginning the test.
Try to answer all the questions on the test.
Answering all the questions on the test is a way to shoot for a
higher score. Of course, you may encounter questions you do
not know how to answer. While you do not want to wildly
guess at the answers, you do want to try to make educated
guesses. Even if you can eliminate one answer choice as
incorrect, you have improved your chances of picking the
correct answer for that question. So try to answer all the
questions, or as many questions as possible.
501_06_331-370_501_master.qxd 4/29/13 12:37 PM Page 334
334
501 GMAT
®
Questions
Set 11
Now it is time to answer GMAT Two-Part Analysis practice questions that
have been designed to test your integrated reasoning skills. Good luck!
275. The XP Racer is a new electric scooter. When driving on smooth
city roads, the XP Racer’s energy efficiency is X kilometers per joule
(X km/J) when traveling at a constant speed of Y kilometers per hour
(Y km/h).
In terms of the variables X and Y, select the expression that best
represents the number of joules of electricity used in one hour of
traveling on smooth city roads at a constant speed of Y. Then, select
the expression that represents the number of joules of electricity
used in a 100 km drive traveling on smooth city roads at a constant
speed of Y. Make only two selections, one in each column.
Joules of electricity
in 1 hour
Joules of electricity
in 100 kilometers
Y/X
Blank
Blank
X/Y
Blank
Blank
100/X
Blank
Blank
100/Y
Blank
Blank
X/100
Blank
Blank
Y/100
501_06_331-370_501_master.qxd 4/29/13 12:37 PM Page 335
335
501 GMAT
®
Questions
276. At a hot dog eating contest, one contestant ate at a constant rate of
H hot dogs per minute (H hot dogs/min.). The contestant had
burned calories from activity during the contest and taken in
calories from eating the hot dogs for an overall caloric intake of C
calories per hot dog (C calories/hot dog).
In terms of the variables H and C, select the expression that best
represents the number of calories taken in during one minute of
eating at a constant rate. Then, select the expression that
represents the number of calories taken in if the contestant had
eaten 100 hot dogs at a constant rate of H. Make only two
selections, one in each column.
Calories taken in
during 1 minute
Calories taken in
if the contestant
ate 100 hot dogs
C/H
Blank Blank
H/C
Blank Blank
C
2
Blank Blank
H
2
Blank Blank
(C)(H)
Blank Blank
(C)(100)
501_06_331-370_501_master.qxd 4/29/13 12:37 PM Page 336
336
501 GMAT
®
Questions
277. A survival instructor teaches a class how to survive when lost in the
wild. The survival instructor takes the class hiking through the
woods and makes sure to keep his water loss through sweat at
around W meters per liter of water lost (W m/L) when hiking at a
constant speed of S meters per hour (S m/h).
In terms of the variables W and S, select the expression that best
represents the number of liters of water lost in one hour of hiking
in the woods at a constant speed of S. Then, select the expression
that represents the number of liters of water lost in a 1,000 m hike
traveling at a constant speed of S. Make only two selections, one in
each column.
Liters of water in
1 hour
Liters of water in
1,000 meters
W/S
Blank
Blank
S/W
Blank
Blank
1,000/W
Blank
Blank
1,000/S
Blank
Blank
W/1,000
Blank
Blank
S/1,000
501_06_331-370_501_master.qxd 4/29/13 12:37 PM Page 337
337
501 GMAT
®
Questions
278. A marathon runner’s energy efficiency is C meters per calorie
(C m/cal) when running at a rate of M meters per second (M m/s).
In terms of the variables C and M, select the expression that best
represents the number of calories used in one hour of running at a
constant rate of M. Then, select the expression that represents the
number of calories used in a 500-meter race while running at a
constant rate of M. Make only two selections, one in each column.
Calories used in
1 hour
Calories used in a
500-meter race 500/C
Blank
Blank
500/M
Blank
Blank
C/500
Blank
Blank
M/500
Blank
Blank
C/M
Blank
Blank
M/C
501_06_331-370_501_master.qxd 4/29/13 12:37 PM Page 338
501 GMAT
®
Questions
279.
The following excerpt from a fictitious science news report
discusses a fictitious type of pollutant called tritans.
For years, scientists have studied the effects of tritans on
birds’ eggs, but recently scientists have also discovered that
tritans affect other egg-laying species. Tritans were known
to cause birds to lay their eggs too early and also weaken the
shell of eggs. As a result, embryos in the eggs have less of a
chance to survive. Fewer eggs resulted in live offspring, and
the numbers of these species of birds diminished. Now,
scientists recently discovered that the decreasing numbers
of certain species of lizards has also been caused by tritans.
These species of lizards tend to bury their eggs in sand
nests, whereas the birds had nests in trees. Scientists are
now researching the source of the tritans in order to limit
tritans pollution in the environment.
Based on the definition of the imaginary word tritans that can be
inferred from the previous paragraph, which of the following traits
of a pollutant must be true for that pollutant to be tritans and
which must NOT be true for that pollutant to be tritans? Make
only two selections, one in each column.
Must be true Must not be true
Traits of a pollutant
Blank
Blank
Decrease the number
of eggs laid
Blank
Blank
Comes from the
water supply
Blank
Blank
Increases the number
of embryos that die
Blank
Blank
Builds up in animals’
bodies
Blank
Blank
Thickens the shell of
eggs
338
501_06_331-370_501_master.qxd 4/29/13 12:37 PM Page 339
501 GMAT
®
Questions
280.
The following excerpt from a fictitious economics news article
focuses on a type of financial exchange called an M.T.B.
After the recent banking collapse, new regulations have
forced larger banks to acknowledge all M.T.B. exchanges
that exceed $250,000. Banks that do not disclose this
information to the government will risk losing all federal
bailout monies. In addition, all M.T.B.s involving IPOs
must be disclosed to all shareholders on a quarterly basis.
While the long-term effects of this new regulation cannot
be predicted, the goal of this regulation is to limit the
number of annual M.T.B. exchanges that larger banks
conduct.
Based on the definition of the imaginary word M.T.B. that can be
inferred from the previous paragraph, which of the following traits
of a financial exchange must be true for that financial exchange to
be an M.T.B. and which must NOT be true for that financial
exchange to be an M.T.B.? Make only two selections, one in each
column.
Must be true to
be an M.T.B.
Must not be true
to be an M.T.B.
Traits of a financial
exchange
Blank
Blank
Must be disclosed to
shareholders of IPOs
Blank
Blank
Not regulated by the
government
Blank
Blank
Involving more than
$250,000
Blank
Blank
Occurring every
three months
Blank
Blank
Happens at all
banks
339
501_06_331-370_501_master.qxd 4/29/13 12:37 PM Page 340
501 GMAT
®
Questions
340
281.
The following excerpt from a fictitious veterinary textbook
discusses a fictitious type of disease called spurnit.
The diagnosis of spurnit in dogs can be difficult as
symptoms do not always manifest until the disease has
progressed. Unlike other illnesses that cause an increase in
swelling in the glands, spurnit does not, making early
detection difficult. Nonetheless, early symptoms include a
slightly swollen tongue and/or an elevated level of white
blood cells. Early detection is key because the treatment of
spurnit varies depending on the duration the dog has
carried the disease. Early treatments can involve a simple
course of antibiotics and then a year-long daily regimen of
probiotics. Later stages of the disease often tend to resist all
treatments, but daily blood transfusions have been shown to
ease symptoms in certain cases.
Based on the definition of the imaginary word spurnit that can be
inferred from the previous paragraph, which of the following are
definitely traits of spurnit and which of the following are definitely
NOT spurnit? Make only two selections, one in each column.
Trait of spurnit NOT trait of spurnit
Blank
Blank
Only affects dogs
Blank
Blank
Causes glands to
swell
Blank
Blank
Can be detected
in a basic exam
Blank
Blank
Causes more
white blood cells
Blank
Blank
Transmitted by
the mouth
Blank
501_06_331-370_501_master.qxd 4/29/13 12:37 PM Page 341
501 GMAT
®
Questions
282.
The following excerpt from a fictitious camping guidebook focuses
on a fictitious type of plant called poison senity.
Poison senity thrives only in a limited region of the Pacific
Northwest. The oily coating and sap from the poison senity,
known as urushiol, can cause itching, burning, and severe
blistering on the skin. Notable features of poison senity
include its five almond-shaped leaflets and its aroma. While
being similar to poison ivy and poison oak, the injuries
caused by the urushiol produced by poison senity can be
much more severe because poison senity’s urushiol can take
a longer time to have an effect. As a result, sufferers do not
realize that they have the oils on their skin and can transfer
the poisonous oils to more places on their body. Typical
places are the hands, legs, mouth, neck, and eyes. A person
exposed to poison senity’s urushiol should wash the area
with soap and warm water immediately and seek medical help.
Based on the definition of the imaginary plant poison senity that can
be inferred from the previous paragraph, which of the following
characteristics of a plant must be true for that plant to be a poison
senity, and which must NOT be true for that plant to be a poison
senity? Make only two selections, one in each column.
Must be true Must not be true
Traits of a plant
Blank
Blank
Causes high
fevers
Blank
Blank
Common through
parts of the United
States
Blank
Blank
Grows best in damp
climates
Blank
Blank
Poisonous oil
spreads easily
Blank
Blank
Can be fatal if
swallowed
341
501_06_331-370_501_master.qxd 4/29/13 12:37 PM Page 342
501 GMAT
®
Questions
283.
The following excerpt from a fictitious advertisement discusses a
fictitious type of acne treatment called Clarens.
Clarens users have reported a dramatic improvement in
their acne and skin irritations. Most users prefer Clarens to
other acne treatments. That is because Clarens is specially
formulated to fight acne in all its forms: whiteheads,
blackheads, regular pimples, and the scars that can form
from acne as it heals. The Clarens treatment comes in three
parts. First, a simple tea tree oil facial wash to rid the pores
of dirt. Second, a benzoyl peroxide ointment that can be
spread over the whole face or dabbed on problem areas.
Unlike other acne treatments, the Clarens ointment contains
a stronger dosage of benzoyl peroxide than other brands,
but Clarens doesn’t irritate the skin. Lastly, the Clarens
treatment includes to-go washes that contains salicylic acid.
No other brand on the market offers these special washes.
Based on the definition of the imaginary brand Clarens that can be
inferred from the previous paragraph, which of the following
describes an acne treatment that must be Clarens and which of the
following describes an acne treatment that is NOT Clarens? Make
only two selections, one in each column.
Must be
Clarens
Must NOT be
Clarens
Blank
Blank
Blank
Can be used to wash the face
Blank
Blank
A form of salicylic acid that can
be used outside the home
Blank
Blank
A strict face regimen that must
be adhered to
Blank
Blank
Free of dyes, perfumes, and
chemicals
Blank
Blank
Gentle on the skin
342
501_06_331-370_501_master.qxd 4/29/13 12:37 PM Page 343
501 GMAT
®
Questions
343
284.
Country A presently produces 4 billion tons of carbon emissions
each year. Country B presently produces 5 billion tons of carbon
emissions each year. The carbon emissions produced by both
countries is increasing yearly, each at its own constant rate.
Scientists predict that both countries will maintain their constant
yearly increase in carbon emissions. In addition, scientists predict
that Country A and Country B will produce the same carbon
emissions in 10 years, and Country A will produce more carbon
emissions than Country B after 10 years.
In the following table, identify the rate of increase in carbon
emissions per year for both countries that support the scientists’
predictions. Make only one selection in each column.
Country A Country B
Rate of Carbon Emission Increase
(billions of tons each year)
Blank
Blank
0.05
Blank
Blank
0.15
Blank
Blank
0.50
Blank
Blank
1
Blank
Blank
5
Blank
Blank
15
344
501 GMAT
®
Questions
501_06_331-370_501_master.qxd 4/29/13 12:37 PM Page 344
285. During a period of time when the price of gas is on the rise, Gas
Station X charges $3.40 a gallon and Gas Station Y charges $4.00 a
gallon. Both gas stations are increasing the cost they charge for gas
each week, each at its own constant rate. If both gas stations
continue to increase their costs at their constant rates, they will be
charging the same price for gas four weeks from now, and Gas
Station Y will charge more than Gas Station X in subsequent
weeks.
In the following table, identify the rate of increase per week for the
cost of gas for both gas stations that supports the predictions stated
in the paragraph. Make only one selection in each column.
Gas Station X Gas Station Y
Rate of Cost of Gas Increase
(each week)
Blank
Blank
$0.01
Blank
Blank
$0.05
Blank
Blank
$0.10
Blank
Blank
$0.15
Blank
Blank
$0.20
Blank
Blank
$0.45
501_06_331-370_501_master.qxd 4/29/13 12:37 PM Page 345
501 GMAT
®
Questions
286.
University A currently has 20,000 students enrolled. University B
currently has 22,500 students enrolled. The number of students
enrolling in University A and the number of students enrolling in
University B increases each year, each at its own constant rate. The
deans of the universities project that if each university maintains its
constant rate of student enrollment increase, five years from now
they will both have the same number of students enrolled, and
University A will have more students enrolled than University B in
subsequent years.
In the following table, identify the rate of increase, in student
enrollment per year, for University A and the rate of increase, in
student enrollment per year, for University B that are both
consistent with the deans’ projections. Make only one selection in
each column.
University A University B
Rate of Increase (students
enrolled per year)
Blank Blank
400
Blank Blank
1,000
Blank Blank
1,500
Blank Blank
1,800
Blank Blank
2,200
Blank Blank
2,600
345
501_06_331-370_501_master.qxd 4/29/13 12:37 PM Page 346
501 GMAT
®
Questions
287.
Environmental scientists studying endangered species in a given
region have observed that there are presently 800 members of
Species X in the region and 600 members of Species Y in the
region. The number of each species is decreasing annually, each at
its own constant rate. Environmental scientists predict that both
species will maintain their constant yearly decrease in numbers. In
addition, environmental scientists predict that if both species
maintain their constant rate of decrease, 10 years from now they
will for the first time have the same number of members, and in
subsequent years there will be fewer Species X than Species Y.
In the following table, identify the rate of decrease in the numbers
of each species per year that supports the environmental scientists’
predictions. Make only one selection in each column.
Species X Species Y
Rate of Decrease of Species (in
number of members per year)
Blank Blank
20
Blank Blank
40
Blank Blank
50
Blank Blank
200
Blank Blank
400
Blank Blank
500
346
501_06_331-370_501_master.qxd 4/29/13 12:37 PM Page 347
501 GMAT
®
Questions
288.
Athlete A currently trains 180 minutes each week. Athlete B
currently trains 250 minutes each week. Both athletes A and B are
increasing the number of minutes spent training each week, both
at a different constant rate. If both athletes keep to their constant
rate of training increase, in seven weeks from now, they will be
training for the same number of minutes each week, and in
subsequent weeks, athlete A will train more each week than
athlete B.
In the following table, identify the rate of increase in number of
training minutes each week for both athletes that support the
predictions stated in the last paragraph. Make only one selection in
each column.
Athlete A Athlete B
Rate of T
raining Increase
(number of minutes per week)
Blank Blank
3
Blank Blank
5
Blank Blank
15
Blank Blank
25
Blank Blank
30
Blank Blank
50
347
348
501 GMAT
®
Questions
501_06_331-370_501_master.qxd 4/29/13 12:37 PM Page 348
289. A caterer offers two menus for large events, such as weddings and
conferences. To show the variety of foods the caterer can prepare,
one of the menus has entirely meat-free and vegetarian options
and the other menu includes two fish options. Neither menu
should have more than two hot options. The caterer has already
decided on three out of the four options for both menus. Those
menus are shown here.
*Menu 1* *Menu 2*
Spanish gazpacho (appetizer,
vegetarian, cold)
Thai shrimp soup (appetizer,
hot)
Curried tofu (entree,
vegetarian, hot)
Salmon tacos (entree, hot)
Bananas flambé (dessert,
vegetarian, hot)
Chocolate mousse (dessert,
vegetarian, cold)
Select an option that could be added to both menus. Then, select
an option that could NOT be added to either menu. Make only
two selections, one in each column.
Both menus Neither menu
Option
Blank
Blank
Sesame carrot salad (salad,
vegetarian, cold)
Blank
Blank
Whitefish and tomato salad
(salad, cold)
Blank
Blank
Hot bread pudding (dessert,
vegetarian, hot)
Blank
Blank
Beef spring rolls (entree,
cold)
Blank
Blank
Spinach and wild rice (salad,
vegetarian, hot)
Blank
Blank
Lobster ravioli (entree, hot)
501_06_331-370_501_master.qxd 4/29/13 12:37 PM Page 349
501 GMAT
®
Questions
290.
The staff at the Botanical Garden is planning the garden’s yearly
two-day spring festival intended to celebrate the blooming of the
cherry trees. Five events will be offered each day. To celebrate the
blooming cherry trees, the majority of events scheduled for at least
one day will be located in the Cherry Tree Esplanade in the park.
On the other day of the spring festival, at least four of the events
can be for children or allow children. Neither day should have
more than two events that are just for seniors. The Botanical
Garden staff has already agreed on a festival schedule for eight of
the ten events. That schedule is shown here.
Day 1 Day 2
Children’s Flower Parade (in
Cherry Esplanade, ages 0–12)
Face Painting (in Cherry
Esplanade, ages 0–12)
Using Blossoms in Paintings
(in Cherry Esplanade, ages
5–12)
Yoga for Senior Citizens (in
Cherry Esplanade, seniors)
Jazz and Cocktails (seniors
only)
Singles Mingle with Music
(ages 21 and up)
String Quartet Playing
Mozart (ages 0 and up)
Dancing among Blossoms:
Learn to Waltz (all ages)
349
501_06_331-370_501_master.qxd 4/29/13 12:37 PM Page 350
501 GMAT
®
Questions
Select an event that could be added to the schedule for either day.
Then, select an event that could be added to the schedule for
neither day. Make only two selections, one in each column.
Either day Neither day
Event
Blank
Blank
Lecture on the History of the
Gardens (ages 21 and up)
Blank
Blank
Creating Your Own Garden (all
ages)
Blank
Blank
Making Cherry Ice Cream (in
Cherry Esplanade, all ages)
Blank
Blank
Blossoms: Puppet Show (Main
Theater
, ages 0–12)
Blank
Blank
Senior Walking Tour (in Cherry
Esplanade, seniors)
350
501_06_331-370_501_master.qxd 4/29/13 12:38 PM Page 351
501 GMAT
®
Questions
291.
The American Arts department at a small museum is organizing an
exhibit focused on Georgia O’Keeffe. Her work will be featured in
two rooms of the museum. Both rooms have space to
accommodate five of her paintings. The museum wants a majority
of the paintings in one room to be O’Keeffe paintings that include
skulls or bones. In the other room, the museum wants at least four
of the paintings to be from 1935 or later. Neither room should
have more than two O’Keeffe paintings of buildings of any sort.
The museum has already decided on eight of the paintings for
these two rooms. The plan is shown here.
Room 1 Room 2
“Cow’s Skull: Red, White, and
Blue,” 1931
“Red Hills with Flowers,”
1937
“Church Steeple,” 1930 “Ram’s Head, Blue Morning
Glory,” 1938
“Ranchos Church,” 1930 “Ram’s Head with Holly-
hock,” 1935
“Cow’s Skull with Calico
Roses,” 1931
“Horse’s Skull with White
Rose,” 1931
Select a painting that could work for both rooms. Then, select a
painting that could NOT work in either room. Make only two
selections, one in each column.
Both
rooms
Neither
room
Paintings
Blank
Blank
“Calla Lillies on Red,” 1928
Blank
Blank
“Sky Above Clouds,” 1965
Blank
Blank
“White Rose,” 1927
Blank
Blank
“Church Bell, Ward Colorado,” 1917
Blank
Blank
“Deer’s Skull with Pedernal,” 1936
351
352
501 GMAT
®
Questions
501_06_331-370_501_master.qxd 4/29/13 12:38 PM Page 352
292. A nonprofit literary group in the United States is organizing a day-
long poetry event for the community. Five poets will read in the
morning, and another five poets will read in the afternoon. To
make sure that the poets engage a young audience, the literary
group wants a majority of poets in one part of the event to be born
after 1970. The other part of the event should have at least four
women poets. Neither part of the event should have more than two
poets from other countries. The literary group has already agreed
on a plan for eight of the ten poets. That plan, showing names
along with each poet’s birth year and country of origin, is shown
here.
Morning Afternoon
Matthew Dickman (male,
born 1975, USA)
Tracy K. Smith (female, born
1972, USA)
Kwame Dawes (male, born
1962, Ghana)
Jo Shapcott (female, born
1953, England)
Erika Metiner (female, born
1975, USA)
Coral Bracho (female, born
1951, Mexico)
Aimee Nezhukumatathiln
(female, born 1974, USA)
Amiri Baraka (male, born
1934, USA)
353
501 GMAT
®
Questions
501_06_331-370_501_master.qxd 4/29/13 12:38 PM Page 353
Select a poet who could be added to either the morning or the
afternoon. Then, select a poet who could NOT be added to either
the morning or the afternoon. Make only two selections, one in
each column.
Both morning Neither morning
and afternoon nor afternoon
Poet
Blank
Blank
Rita Dove (female, born
1952, USA)
Blank
Blank
Claribel Alegria (female,
born 1924, Nicaragua)
Blank
Blank
Camille Dungy (female,
born 1972, USA)
Blank
Blank
Natasha Trethewey
(female, born 1966, USA)
Blank
Blank
Kazim Ali (male, born
1971, USA)
501_06_331-370_501_master.qxd 4/29/13 12:38 PM Page 354
Answers—Set 11
354
501 GMAT
®
Questions
275. Column 1, line 1. The question asks about using two variables to
determine the number of joules of electricity used in one hour of
driving in certain circumstances. For column 1, line 2 (X/Y) is
incorrect because the result would be in terms of the number of
hours, not the number of joules. Line 3 (100/X) is incorrect
because this determines the amount of electricity used when
driving 100 kilometers. Line 4 (100/Y) is incorrect because the
answer would be in terms of the number of hours. Line 5 (X/100)
is incorrect because this simply divides the energy efficiency by
100 and does not show the number of joules used in one hour of
driving. Line 6 (Y/100) is incorrect because this divides the speed
by 100 and does not show the number of joules used in one hour
of driving.
Column 2, line 3. The question asks about using two variables to
determine the number of joules of electricity used when driving
100 kilometers in certain circumstances. For column 2, line 1
(Y/X) is incorrect because dividing speed by energy efficiency
doesn’t determine the amount of energy used in 100 kilometers.
Line 2 (X/Y) is incorrect because dividing energy efficiency by
speed doesn’t determine the amount of energy used in 100
kilometers. Line 4 (100/Y) is incorrect because the answer would
be in terms of the number of hours, not number of joules. Line 5
(X/100) is incorrect because dividing the energy efficiency by 100
does not show the number of joules used when driving 100
kilometers. Line 6 (Y/100) is incorrect because dividing the speed
by 100 does not show the number of joules used when driving
100 kilometers.
501_06_331-370_501_master.qxd 4/29/13 12:38 PM Page 355
355
501 GMAT
®
Questions
276. Column 1, line 5. The question asks about using two variables to
determine the number of calories taken in during one minute of
eating hot dogs at a constant rate. For column 1, line 1 (C/H) is
incorrect because dividing the number of calories per hot dog by
the rate at which the contestant ate hot dogs does not result in
finding out the number of calories taken in during one minute.
Line 2 (H/C) is incorrect because dividing the rate at which the
contestant ate hot dogs by the number of calories per hot dog does
not result in finding out the number of calories taken in during
one minute. Line 3 (C
2
) is incorrect because this only involves the
number of calories per hot dog and does not involve a unit of time.
Line 4 (H
2
) is incorrect because this only involves the number of
hot dogs eaten over time and does not involve finding out the
caloric intake. Line 6 (C)(100) is incorrect because this does not
involve time.
Column 2, line 6. The question asks about using two variables to
determine the number of calories taken in if the contestant ate 100
hot dogs at a constant rate. For column 2, line 1 (C/H) is incorrect
because dividing the number of calories per hot dog by the rate at
which the contestant ate hot dogs does not result in finding out the
number of calories taken in from eating 100 hot dogs. Line 2
(H/C) is incorrect because dividing the rate at which the contestant
ate hot dogs by the number of calories per hot dog does not result
in finding out the number of calories taken in from eating 100 hot
dogs. Line 3 (C
2
) is incorrect because this only involves the
number of calories per hot dog and does not involve a specific
number of hot dogs. Line 4 (H
2
) is incorrect because this only
involves the number of hot dogs eaten over time and does not
involve finding out the caloric intake. Line 5 (C)(H ) is incorrect
because it results in finding out the number of calories taken in
from one minute of eating, not the number of calories taken in
from eating 100 hot dogs.
501_06_331-370_501_master.qxd 4/29/13 12:38 PM Page 356
356
501 GMAT
®
Questions
277. Column 1, line 2. The question asks about using two variables to
determine the number of liters of water lost in one hour of hiking
in the woods at a constant speed. For column 1, line 1 (W/S) is
incorrect because dividing the number of meters walked per liter
of water lost by the hiking speed does not result in finding out the
number of liters of water lost. Line 3 (1,000/W ) is incorrect
because dividing 1,000 by the rate at which water is lost does not
involve a unit of time. Line 4 (1,000/S) is incorrect because this
only involves the hiking speed and does not involve a unit of time.
Line 5 (W/1,000) is incorrect because dividing the rate at which
the water is lost by 1,000 diminishes the rate and does not result in
finding out the amount of water lost in one hour. Line 6 (S)(1,000)
is incorrect because dividing the speed by 1,000 diminishes the
speed and does not result in finding out the amount of water lost in
one hour.
Column 2, line 3. The question asks about using two variables to
determine the number of liters of water lost hiking 1,000 meters in
the woods at a constant speed. For column 2, line 1 (W/S) is
incorrect because dividing the number of meters walked per liter
of water lost by the hiking speed does not result in finding out the
number of liters of water lost. Line 2 (S/W ) is incorrect because
dividing the hiking speed by the number of meters walked per liter
of water lost does not involve 1,000 meters. Line 4 (1,000/S) is
incorrect because dividing the distance by the hiking speed doesn’t
result in the number of liters of water lost. Line 5 (W/1,000) is
incorrect because dividing the rate at which the water is lost by
1,000 diminishes the rate and does not result in finding out the
amount of water lost when walking 1,000 meters. Line 6 (S)(1,000)
is incorrect because dividing the speed by 1,000 diminishes the
speed and does not result in finding out the amount of water lost
when walking 1,000 meters.
501_06_331-370_501_master.qxd 4/29/13 12:38 PM Page 357
357
501 GMAT
®
Questions
278. Column 1, line 6. The question asks about using two variables to
determine the number of calories used when a marathon runner
runs for one hour at a constant rate. Column 1, line 1 (500/C) is
incorrect because the question asks about the calories burned in
one hour, not when running 500 meters. Line 2 (500/M ) is
incorrect because the question asks about the calories burned in
one hour, not when running 500 meters. Line 3 (C/500) is
incorrect because the question asks about the calories burned in
one hour, not when running 500 meters. Line 4 (M/500) is
incorrect because the question asks about the calories burned in
one hour, not when running 500 meters. Line 5 (C/M ) is incorrect
because dividing the energy efficiency by the rate does not result in
finding out the number of calories used in one hour.
Column 2, line 1. The question asks about using two variables to
determine the number of calories used when a marathon runner
runs a 500-meter race at a constant rate. For column 2, line 2
(500/M ) is incorrect because the question asks about the number
of calories. Therefore, the distance must be divided by the energy
efficiency, not by the running rate. Line 3 (C/500) is incorrect
because the question asks about the number of calories. Therefore,
the distance must be divided by the energy efficiency, not the
energy efficiency divided by the distance. Line 4 (M/500) is
incorrect because this would result in a number of seconds, not a
number of calories. Line 5 (C/M ) is incorrect because dividing the
energy efficiency by the rate does not result in finding out the
number of calories used when running 500 meters. Line 6 (M/C) is
incorrect because dividing the rate by the energy efficiency does
not result in finding out the number of calories used when running
500 meters.
358
501 GMAT
®
Questions
501_06_331-370_501_master.qxd 4/29/13 12:38 PM Page 358
279. Column 1, line 3. The question asks about determining a trait
that the imaginary pollution tritans has based on the details in the
paragraph. For column 1, line 1 is incorrect because there are no
details to suggest that tritans decrease the number of eggs laid.
Line 2 is incorrect because the paragraph states that scientists are
“researching the source of the tritans,” so this does not support
the idea that tritans come from the water supply. Line 4 is
incorrect because the paragraph is focused on the effects on eggs,
not animals’ bodies. Line 5 is incorrect because the paragraph
states that tritans “weaken the shell of eggs,” not thicken the
eggshells.
Column 2, line 5. The question asks about determining a trait
that the imaginary pollution tritans must NOT have based on the
details in the paragraph. For column 2, line 1 is incorrect because
there are no details in the paragraph to suggest that tritans do or
do not decrease the number of eggs laid. Line 2 is incorrect
because the paragraph states that scientists are “researching the
source of the tritans,” so tritans could come from the water supply.
Line 3 is incorrect because the paragraph states that fewer eggs
survive, so this is a trait of titans. Line 4 is incorrect because there
are no details about this in the paragraph, so it could be a trait
of titans.
359
501 GMAT
®
Questions
501_06_331-370_501_master.qxd 4/29/13 12:38 PM Page 359
280. Column 1, line 1. The question asks about a trait that an
imaginary type of financial exchange called an M.T.B. must have
based on the details in the paragraph. For column 1, line 2 is
incorrect because the paragraph states that there are new
regulations. Line 3 is incorrect because, while M.T.B. exchanges
greater than $250,000 must be acknowledged, being greater than
$250,000 is not necessarily a trait of M.T.B.s. Line 4 is incorrect
because, while M.T.B.s involving IPOs must be disclosed quarterly
(or every three months), this does not necessarily mean that all
M.T.B.s happen every three months. Line 5 is incorrect because
there are no details in the paragraph to suggest that all banks
conduct this type of exchange.
Column 2, line 2. The question asks about a trait that must NOT
describe the imaginary type of financial exchange called an M.T.B.
For column 2, line 1 is incorrect because the paragraph states that
certain types of M.T.B.s “must be disclosed to all shareholders.”
Line 3 is incorrect because the paragraph states that some M.T.B.
exchanges are greater than $250,000. Line 4 is incorrect because
there are no details to determine whether M.T.B. exchanges
happen every three months. Line 5 is incorrect because there are
no details in the paragraph to determine whether all banks conduct
this type of exchange.
501_06_331-370_501_master.qxd 4/29/13 12:38 PM Page 360
360
501 GMAT
®
Questions
281. Column 1, line 4. The question asks about an imaginary type of
disease called spurnit and a trait that spurnit definitely has. For
column 1, line 1 is incorrect because, while the paragraph focuses
on diagnosing the disease in dogs, the paragraph never states that
the disease only occurs in dogs. Line 2 is incorrect because the
paragraph states that spurnit is unlike diseases that “cause an
increase in swelling in the glands.” Line 3 is incorrect because the
paragraph states that detection “can be difficult.” Line 5 is
incorrect because, while the paragraph states that the disease
affects the tongue, the paragraph never describes the transmission
of the disease.
Column 2, line 2. The question asks about an imaginary type of
disease called spurnit and a trait that spurnit definitely does NOT
have. For column 2, line 1 is incorrect because the paragraph
describes diagnosing the disease in dogs. Line 3 is incorrect
because, while the paragraph states that detection “can be
difficult,” it also states that symptoms can manifest and be detected
early. Line 4 is incorrect because the paragraph states that the
disease results in an “elevated level of white blood cells.” Line 5 is
incorrect because, while the paragraph states that the disease
affects the tongue, the paragraph never describes the transmission
of the disease.
501_06_331-370_501_master.qxd 4/29/13 12:38 PM Page 361
361
501 GMAT
®
Questions
282. Column 1, line 4. The question asks about an imaginary type of
plant called poison senity and a characteristic that poison senity
definitely has. For column 1, line 1 is incorrect because the
paragraph never states that poison senity causes fevers. Line 2 is
incorrect because the paragraph states that poison senity “thrives
only in a limited region of the Pacific Northwest.” Line 3 is
incorrect because, while the paragraph states poison senity thrives
in the Pacific Northwest, which tends to be damp, the paragraph
never states that poison senity grows best in damp climates. Line 5
is incorrect because no information is provided in the paragraph
about what happens if poison senity is swallowed.
Column 2, line 2. The question asks about an imaginary type of
plant called poison senity and a characteristic that does NOT
describe poison senity. For column 2, line 1 is incorrect because
poison senity could cause fevers; this information is not provided
in the paragraph. Line 3 is incorrect because poison senity could
thrive in damp climates, but no information specifically regarding
dampness is provided in the paragraph. Line 4 is incorrect because
the paragraph states that the oils from poison senity can transfer to
“more places” on the body, which suggests that the oils transfer
easily. Line 5 is incorrect because poison senity could be fatal if
swallowed, but no information about this is provided in the
paragraph.
501_06_331-370_501_master.qxd 4/29/13 12:38 PM Page 362
362
501 GMAT
®
Questions
283. Column 1, line 2. The question asks about an imaginary type of
acne treatment called Clarens and a description that only pertains to
Clarens. For column 1, line 1 is incorrect because the other acne
treatments or soaps could be used to wash the face, not just Clarens.
Line 3 is incorrect because, while the paragraph suggests that there is
a regimen involved using Clarens, there is no information to support
the idea that Clarens is the only product that involves a regimen.
Line 4 is incorrect because there is no information about “dyes,
perfumes, and chemicals” in the paragraph. Line 5 is incorrect
because, while the paragraph states that Clarens doesn’t irritate the
skin and so could be assumed to be gentle, there is no information to
suggest that other brands are not also gentle on the skin.
Column 2, line 4. The question asks about an imaginary type of
acne treatment called Clarens and a description that does NOT
pertain to Clarens. For column 2, line 1 is incorrect because
Clarens includes a “simple tea tree oil facial wash,” so it could be
used to wash the face. Line 2 is incorrect because Clarens has “to-
go washes that contains salicylic acid,” so this form of salicylic acid
can be used outside the home. Line 3 is incorrect because the
paragraph states that Clarens comes in three parts, which suggests
that there is a regimen. Line 5 is incorrect because the paragraph
states that Clarens doesn’t irritate the skin, which suggests that it is
gentle on the skin.
284. Column 1, line 2. The question asks about the rate of increase in
carbon emissions for Country A that would cause Country A’s
carbon emissions to be the same as Country B’s carbon emissions
in 10 years and greater than Country B’s carbon emissions after 10
years. For column 1, line 1 is incorrect because this would cause
Country A’s carbon emissions to increase by only 0.5 billion tons in
10 years, resulting in Country A’s carbon emissions reaching only
4.5 billion tons. This is not equal to Country B’s carbon emissions.
Line 3 is incorrect because this would cause Country A’s carbon
emissions to increase by 5 billion tons in 10 years, resulting in
Country A’s carbon emissions reaching 9 billion tons. There are no
rates provided for Country B that would cause it to reach 9 billion
tons of carbon emissions in 10 years. Line 4 is incorrect because
this would cause Country A to produce 14 billion tons of carbon
emissions in 10 years. In order for the countries to produce equal
carbon emissions, Country B would have to produce 9 billion tons
501_06_331-370_501_master.qxd 4/29/13 12:38 PM Page 363
363
501 GMAT
®
Questions
more in 10 years, and there are no rates that support that increase
for Country B. Line 5 is incorrect because this would cause
Country A to produce 54 billion tons of carbon emissions in 10
years. In order for the countries to produce equal carbon emissions,
Country B would have to produce 49 billion tons more in 10 years,
and there are no rates that support that increase for Country B.
Line 6 is incorrect because this would cause Country A to produce
154 billion tons of carbon emissions in 10 years. In order for the
countries to produce equal carbon emissions, Country B would
have to produce 159 billion tons more in 10 years, and there are no
rates that support that increase for Country B.
Column 2, line 1. The question asks about the rate of increase in
carbon emissions for Country B that would cause Country B’s
carbon emissions to be the same as Country A’s carbon emissions
in 10 years and less than Country A’s carbon emissions after 10
years. For column 2, line 2 is incorrect because this would cause
Country B’s carbon emissions to increase by 1.5 billion tons in 10
years, resulting in Country B’s carbon emissions reaching 6.5
billion tons. There are no rates in column 2 for Country A that
would cause it to increase by 2.5 billion tons in 10 years. Line 3 is
incorrect because this would cause Country B’s carbon emissions
to increase by 5 billion tons in 10 years, resulting in Country B’s
carbon emissions reaching 10 billion tons. There are no rates in
column 2 for Country A that would cause it to reach 10 billion
tons of carbon emissions in 10 years. Line 4 is incorrect because
this would cause Country B to produce 15 billion tons of carbon
emissions in 10 years. In order for the countries to produce equal
carbon emissions, Country A would have to produce 11 billion
tons more in 10 years, and there are no rates that support that
increase for Country A. Line 5 is incorrect because this would
cause Country B to produce 55 billion tons of carbon emissions in
10 years. In order for the countries to produce equal carbon
emissions, Country A would have to produce 51 billion tons more
in 10 years, and there are no rates that support that increase for
Country A. Line 6 is incorrect because this would cause Country B
to produce 155 billion tons of carbon emissions in 10 years. In
order for the countries to produce equal carbon emissions, Country
A would have to produce 151 billion tons more in 10 years, and
there are no rates that support that increase for Country A.
501_06_331-370_501_master.qxd 4/29/13 12:38 PM Page 364
364
501 GMAT
®
Questions
285. Column 1, line 5. The question asks about the rate of increase in
the cost of gas for Gas Station X that would cause Gas Station X’s
price for gas to be the same as Gas Station Ys price for gas in four
weeks and less than Gas Station Ys price for gas after four weeks.
For column 1, line 1 is incorrect because this would cause Gas
Station X’s price for gas to increase by only $0.04 in 4 weeks,
resulting in Gas Station X’s price for gas being less than Gas
Station Ys price for gas (even if Gas Station Ys price for gas
didn’t increase). Line 2 is incorrect because this would cause Gas
Station X’s price for gas to increase by only $0.20 in 4 weeks,
resulting in Gas Station X’s price for gas being less than Gas
Station Ys price for gas (even if Gas Station Ys price for gas
didn’t increase). Line 3 is incorrect because this would cause Gas
Station X’s price for gas to increase by only $0.40 in 4 weeks,
resulting in Gas Station X’s price for gas being less than Gas
Station Ys price for gas (even if Gas Station Ys price for gas
didn’t increase). Line 4 is incorrect because this would cause Gas
Station X’s price for gas to increase by $0.60, which would cause it
to be equal to Gas Station Ys prices if Gas Station Y did not
increase its prices over four weeks. Line 6 is incorrect because this
would cause Gas Station X’s price of gas to increase by $1.80. In
order for both the gas stations to be charging the same price in 4
weeks, Gas Station Ys price of gas would have to increase by $1.20
in four weeks, and there are no rates that support that increase.
Column 2, line 2. The question asks about the rate of increase in
the cost of gas for Gas Station Y that would cause Gas Station Ys
price for gas to be the same as Gas Station X’s price for gas in four
weeks and greater than Gas Station X’s price for gas after four
weeks. For column 2, line 1 is incorrect because this would cause
Gas Station Ys price for gas to increase by $0.04 in 4 weeks,
resulting in Gas Station Ys price for gas being $4.04. Gas Station
X’s price of gas would have to increase by $0.64 in four weeks, and
there are no rates that support that increase. Line 3 is incorrect
because this would cause Gas Station Ys price for gas to increase
by $0.40 in 4 weeks, resulting in Gas Station Ys price for gas
being $4.40. Gas Station X’s price of gas would have to increase by
$1.00 in four weeks, and there are no rates that support that
increase. Line 4 is incorrect because this would cause Gas Station
501_06_331-370_501_master.qxd 4/29/13 12:38 PM Page 365
365
501 GMAT
®
Questions
Ys price for gas to increase by $0.60 in 4 weeks, resulting in Gas
Station Ys price for gas being $4.60. Gas Station Xs price of gas
would have to increase by $1.20 in four weeks, and there are no
rates that support that increase. Line 5 is incorrect because this
would cause Gas Station Ys price for gas to increase by $0.80 in 4
weeks, resulting in Gas Station Ys price for gas being $4.80. Gas
Station Xs price of gas would have to increase by $1.40 in four
weeks, and there are no rates that support that increase. Line 6 is
incorrect because this would cause Gas Station Ys price for gas to
increase by $1.80 in 4 weeks, resulting in Gas Station Ys price for
gas being $5.80. Gas Station Xs price of gas would have to
increase by $1.40 in four weeks, and there are no rates that support
that increase.
286. Column 1, line 3. The question asks about the rate of increase in
student enrollment per year for University A that would cause
University A and University B to have the same number of
enrolled students in 5 years and for University A to have more
enrolled students than University B after five years. For column 1,
line 1 is incorrect because this would cause University A’s student
enrollment to increase by 2,000 students, resulting in 22,000
students enrolled at University A. This will not equal University
B’s enrollment. Line 2 is incorrect because this would cause
University A’s student enrollment to increase by 5,000, resulting
in 25,000 students enrolled in University A. There are no rates
that would cause University B to also have 25,000 students in five
years. Line 4 is incorrect because this would cause University A’s
student enrollment to increase by 9,000, resulting in 29,000
students enrolled in University A. There are no rates that would
cause University B to also have 29,000 students in five years. Line
5 is incorrect because this would cause University A’s student
enrollment to increase by 11,000, resulting in 31,000 students
enrolled in University A. There are no rates that would cause
University B to also have 31,000 students in five years. Line 6 is
incorrect because this would cause University A’s student
enrollment to increase by 13,000, resulting in 33,000 students
enrolled in University A. There are no rates that would cause
University B to also have 33,000 students in five years.
501_06_331-370_501_master.qxd 4/29/13 12:38 PM Page 366
366
501 GMAT
®
Questions
Column 2, line 2. The question asks about the rate of increase in
student enrollment per year for University B that would cause
University A and University B to have the same number of
enrolled students in 5 years and for University B to have fewer
enrolled students than University A after five years. For column 2,
line 1 is incorrect because this would cause University B’s student
enrollment to increase by 2,000 students, resulting in 24,500
students enrolled at University B. There are no rates to cause
University A to have 24,500 students in five years. Line 3 is
incorrect because this would cause University B’s student
enrollment to increase by 7,500, resulting in 30,000 students
enrolled in University B. There are no rates that would cause
University A’s enrollment to increase by 10,000 students in five
years. Line 4 is incorrect because this would cause University B’s
student enrollment to increase by 9,000, resulting in 31,500
students enrolled in University B. There are no rates that would
cause University A’s enrollment to increase by 11,500 students in
five years. Line 5 is incorrect because this would cause University
B’s student enrollment to increase by 11,000, resulting in 33,500
students enrolled in University B. There are no rates that would
cause University A’s enrollment to increase by 13,500 students in
five years. Line 6 is incorrect because this would cause University
B’s student enrollment to increase by 13,000, resulting in 35,500
students enrolled in University B. There are no rates that would
cause University A’s enrollment to increase by 15,500 students in
five years.
501_06_331-370_501_master.qxd 4/29/13 12:38 PM Page 367
501 GMAT
®
Questions
367
287. Column 1, line 2. The question asks about the rate of decrease in
the numbers of Species X that would cause there to be the same
number of members of Species X and Species Y in 10 years and for
there to be fewer members of Species X than Species Y in
subsequent years. For column 1, line 1 is incorrect because this
would cause Species X to decrease by 200 members in 10 years,
which would equal the number of member of Species Y only if
Species Y did not decrease. Line 3 is incorrect because this would
cause Species X to decrease by 500 members in 10 years, which
would result in there being 300 members of Species X. Species Y
would have to decrease by 300 members in 10 years, and there is
no rate that supports this. Line 4 is incorrect because this would
cause Species X to decrease by 2,000 members in 10 years, which
would result in a negative number and so is not possible. Line 5 is
incorrect because this would cause Species X to decrease by 4,000
members in 10 years, which would result in a negative number and
so is not possible. Line 6 is incorrect because this would cause
Species X to decrease by 5,000 members in 10 years, which would
result in a negative number and so is not possible.
Column 2, line 1. The question asks about the rate of decrease in
the numbers of Species Y that would cause there to be the same
number of members of Species X and Species Y in 10 years and for
there to be more members of Species Y than Species X in
subsequent years. For column 2, line 2 is incorrect because this
would cause Species Y to decrease by 400 members in 10 years,
leaving 200 members of Species Y. There are no rates that would
result in there being 200 members of Species X in ten years. Line 3
is incorrect because this would cause Species Y to decrease by 500
members in 10 years, which would result in there being 100
members of Species Y. Species X would have to decrease by 700
members in 10 years, and there is no rate that supports this. Line 4
is incorrect because this would cause Species Y to decrease by
2,000 members in 10 years, which would result in a negative
number and so is not possible. Line 5 is incorrect because this
would cause Species Y to decrease by 4,000 members in 10 years,
which would result in a negative number and so is not possible.
Line 6 is incorrect because this would cause Species Y to decrease
by 5,000 members in 10 years, which would result in a negative
number and so is not possible.
501_06_331-370_501_master.qxd 4/29/13 12:38 PM Page 368
368
501 GMAT
®
Questions
288. Column 1, line 3. The question asks about the rate of increase in
the number of minutes athlete A trains each week that would cause
athletes A and B to train the same number of minutes in seven
weeks and for athlete A to train more than athlete B in subsequent
weeks. For column 1, line 1 is incorrect because this would cause
athlete A’s training to increase by only 21 minutes in 7 weeks,
resulting in athlete A training for 201 minutes. This is not equal to
athlete B’s training minutes (even if the number of minutes athlete
B trains does not increase). Line 2 is incorrect because this would
cause athlete A’s training to increase by only 35 minutes in 7
weeks, resulting in athlete A training for 215 minutes. This is not
equal to athlete B’s training minutes (even if the number of minutes
athlete B trains does not increase). Line 4 is incorrect because this
would cause athlete A’s training to increase to a total of 355
minutes by 7 weeks. There are no rates that would cause athlete B
to also train 355 minutes per week by 7 weeks. Line 5 is incorrect
because this would cause athlete As training to increase to a total of
390 minutes by 7 weeks. There are no rates that would cause
Athlete B to also train 390 minutes per week by 7 weeks. Line 6 is
incorrect because this would cause athlete As training to increase
to a total of 530 minutes by 7 weeks. There are no rates that would
cause athlete B to also train 530 minutes per week by 7 weeks.
Column 2, line 2. The question asks about the rate of increase in
the number of minutes athlete B trains each week that would cause
athletes A and B to train the same number of minutes in seven
weeks and for athlete B to train for fewer minutes than athlete A in
subsequent weeks. For column 2, line 1 is incorrect because this
would cause athlete B’s training to increase by only 21 minutes in 7
weeks, resulting in athlete B training for 271 minutes. There are
no rates that would cause athlete A to train for 271 minutes per
week after 7 weeks. Line 3 is incorrect because this would cause
athlete B’s training to increase by only 105 minutes in 7 weeks,
resulting in athlete B training for 355 minutes. There are no rates
that would cause athlete A to train for 355 minutes by 7 weeks.
Line 4 is incorrect because this would cause athlete B’s training to
increase to a total of 425 minutes by 7 weeks. There are no rates
that would cause athlete A to also train 425 minutes per week by
7 weeks. Line 5 is incorrect because this would cause athlete B’s
training to increase to a total of 460 minutes by 7 weeks. There are
501_06_331-370_501_master.qxd 4/29/13 12:38 PM Page 369
369
501 GMAT
®
Questions
no rates that would cause athlete A to also train 460 minutes per
week by 7 weeks. Line 6 is incorrect because this would cause
athlete B’s training to increase to a total of 600 minutes by 7 weeks.
There are no rates that would cause athlete A to also train 600
minutes per week by 7 weeks.
289. Column 1, line 1. The question asks about the meal option that
could be added to both menus and fulfill the menu requirements.
For column 1, lines 2 and 4 are incorrect because these would
result in neither menu being vegetarian. Lines 3 and 5 are
incorrect because these would result in both menus having more
than two hot options. Line 6 is incorrect because this would result
in neither menu being vegetarian and both menus having more
than two hot options.
Column 2, line 6. The question asks about the meal option that
could NOT be added to either menu in order to fulfill the menu
requirements. For column 2, line 1 could be added, so this is
incorrect. Lines 2 and 4 would result in Menu 1 not being
vegetarian, but they could be added to Menu 2. Lines 3 and 5
would result in Menu 2 having too many hot options, but they
could be added to Menu 1.
290. Column 1, line 3. The question asks about an event that could be
added to either day of the spring festival schedule and meet the
requirements for the festival. For column 1, lines 1, 2, and 5 are
incorrect because these events would result in neither day having
four events for children. Line 4 is incorrect because this would
result in neither day having a majority of events located in the
Cherry Esplanade.
Column 2, line 1. The question asks about an event that could
NOT be added to either day of the festival schedule as it would
not meet the requirements for the festival. For column 2, line 2 is
incorrect because this event could be added to Day 1 in order to
have four events for children. Line 3 is incorrect because this
would cause both days to have a majority of events in the Cherry
Esplanade. Line 4 is incorrect because this event could be added to
Day 1 in order to have four events for children. Line 5 is incorrect
because this would cause both days to have a majority of events in
the Cherry Esplanade.
501_06_331-370_501_master.qxd 4/29/13 12:38 PM Page 370
370
501 GMAT
®
Questions
291. Column 1, line 5. The question asks about the painting that could
be added to either room that would allow the room to meet the
museum’s requirements. For column 1, lines 1, 2, and 3 are
incorrect because, if any of these paintings were placed in Room 1,
then Room 1 would not have a majority of paintings include skulls
or bones and would not have four paintings from 1935 or later.
Line 4 is incorrect because, if used in Room 1, then Room 1 would
have too many paintings about buildings.
Column 2, line 4. The question asks about the painting that could
NOT work for both rooms and meet the museum’s requirements.
For column 2, lines 1 and 3 are incorrect because these paintings
could be added to Room 2 and a majority of the paintings in Room
2 would still include skulls and bones. Line 2 is incorrect because it
could be added to Room 2 and Room 2 would have four paintings
from 1935 or later. Line 5 is incorrect because it could be added to
Room 1 and a majority of the paintings in Room 1 would include
skulls and bones. Also, it could be added to Room 2 and Room 2
would contain four paintings from 1935 or later.
292. Column 1, line 3. The question asks about the poet who could be
added to the event’s schedule for either morning or afternoon and
fulfill the requirements. For column 1, lines 1, 2, and 4 are
incorrect because if any of these poets were included in the
morning or afternoon sections, then the morning section would
not have four women poets or a majority of poets born after 1970.
Line 5 is incorrect because if this poet were included, then the
morning and afternoon sections would not have four women poets.
Column 2, line 2. The question asks about the poet who could
NOT be added to the event’s schedule for either morning or
afternoon and fulfill the requirements. For column 2, lines 1, 3,
and 4 are incorrect because any of these poets could be added to
the afternoon in order for the afternoon to have four women poets.
Line 5 is incorrect because this poet could be added to the
morning in order for the morning to have a majority of poets born
after 1970.
501_07_371-396_501_master.qxd 4/29/13 12:38 PM Page 371
7
Table Analysis
The table analysis questions in the integrated reasoning portion
of the GMAT will require you to analyze and interpret information from
multiple sources, and to use and apply the data in a variety of ways. The data
displayed in tables will be used to draw conclusions, make inferences, and
identify statements that support or refute the information and relationships
that exist between different sets of information. Specific content knowledge
is not needed; however, being able to identify relationships between, make
inferences, and determine when certain conditions are met based on infor-
mation in a table will be tested in this section. Use of a calculator is per-
mitted on this section of the test.
501_07_371-396_501_master.qxd 4/29/13 12:38 PM Page 372
372
501 GMAT
®
Questions
Test-Taking Tips
Read directions carefully before beginning this portion of the
text. Read through what is expected for the question and
identify what information you will specifically be looking for
within the data.
Understand the format of the question for this section.
Although content-specific knowledge is not necessary, you will
be making conclusions based on information given to you.
Read questions thoroughly and carefully. Read through the
entire question so you are aware of what is necessary to choose
the best option. Be aware of whether the question is asking you
to choose an option that supports or refutes the information in
the table, or whether the question is asking you to choose the
statement that says EXCEPT.
Use testing time wisely. Be sure to pace yourself and not spend
too much time on an individual question. It is often helpful to
eliminate choices before committing to one.
Look for trends in data. The trends that exist in the data will
assist you in making inferences and choosing the best options
for the question.
501_07_371-396_501_master.qxd 4/29/13 12:38 PM Page 373
501 GMAT
®
Questions
Set 12
Now it is time to answer GMAT table analysis practice questions that have
been designed to test your integrated reasoning skills. Good luck!
293.
The following table shows the total number of fast-food
restaurants and Type-2 diabetes statistics for six cities in the
United States with similar square mileage and population size.
Which of the following CANNOT be inferred by the data
represented in the table below?
Fast-Food Restaurants and Population of Type-2 Diabetics
Town
Number of
fast-food
restaurants
% of reported
children
diagnosed
with Type-2
diabetes under
the age of 16
Overall % of
population
with Type-2
diabetes
Rockville 24 8 15
Medford 29 15 19
Pittsfield 21 5 7
Beaver Creek 37 19 23
Greene 40 20 25
Lewistown 28 7 7
a. The city with the most fast-food restaurants will also have the
highest overall occurrence of diabetes.
b. The city with the highest percentage of children with Type-2
diabetes also has the highest number of adults with Type-2
diabetes.
c. The more fast-food restaurants in a city, the more childhood
diabetics it will have.
d. Other factors besides fast-food restaurants, such as culture and
economy, may influence a city’s diabetic population.
373
501_07_371-396_501_master.qxd 4/29/13 12:39 PM Page 374
501 GMAT
®
Questions
294.
Scientists wanted to analyze risk factors for a certain lung ailment.
Some scientists argue that the presence of a certain gene (labeled
gene Lfac) is responsible for a person’s chance of developing the
disease, while others from the group claim the pollution from a ten-
year-old factory in the region is more to blame. The following table
shows 10 patients who have the disease, their level of toxin exposure
based on distance of their homes from the factory, and whether they
have the Lfac gene. Which of the statements can be supported by the
information in the table?
Environmental and Genetic Information on Patients with
Lung Disease
Patient Toxin exposure (%) Gene Lfac present?
1 20 Yes
2 45 No
3 84 No
4 34 Yes
5 62 Yes
6 31 Yes
7 75 No
8 33 Yes
9 77 No
10 58 No
a. The greater the toxin exposure the more likely a person is to get
the disease, whether they have the
LFac gene or not.
b. The disease is caused only by genetic factors.
c. The closer a person lives to the factory, the more likely he or she
is to acquire the disease.
d. It is not possible to determine whether genetics or toxin exposure
increase a person’s chances of developing the disease.
374
501_07_371-396_501_master.qxd 4/29/13 12:39 PM Page 375
501 GMAT
®
Questions
295.
In 2011, 10 different companies reported sales for products they
advertised for on two social media websites. Data was collected for
online sales of products and was determined by customer surveys.
Which of the following statements is true, according to the data in
the table?
Change in Sales of Products 1 and 2, for Companies
1–10 (percent)
Social Media Site #1 Social Media Site #2
Company
Product
1
Product
2
Product
1
Product
2
1 +2.0 +1.7 +3.0 +2.7
2 +2.3 –0.3 +2.5 +1.0
3 –4.6 –2.0 –4.0 –2.1
4 +3.0 +3.2 +2.0 –1.5
5 –2.9 +1.0 –3.6 +0.5
6 –5.0 –2.1 +2.5 +2.5
7 +3.1 –2.0 +1.0 +1.7
8 –0.3 –1.2 +6.2 +4.7
9 +1.0 –1.0 –3.0 –2.2
10 –1.0 –2.5 +4.3 +4.3
a. Product 2 had a higher sales increase than Product 1 on both
websites.
b. Company 1 was not honest when reporting sales of both products.
c. Overall, the second social media site was more effective for
increasing sales of both products.
d. Company 2 was not honest when reporting sales of both products.
375
501_07_371-396_501_master.qxd 4/29/13 12:39 PM Page 376
501 GMAT
®
Questions
296.
The data presented in the following table shows the population of
snakes and the population of rabbits in a meadow ecosystem for a
10-year time period. What can be inferred from the data in the
table?
Rabbit and Snake Populations in a Meadow Ecosystem,
2000–2008
Year Snake population Rabbit population
2000 100 250
2001 120 210
2002 160 185
2003 130 170
2004 110 190
2005 95 215
2006 85 230
2007 90 245
2008 100 235
a. The population of snakes is influenced by the population of
available rabbits.
b. New sources of prey were found by the snakes between 2000
and 2008.
c. There was an increase in other predators of rabbits from 2001
to 2002.
d. If the rabbit population were to double in 2009, the snake pop-
ulation would double as well.
376
501_07_371-396_501_master.qxd 4/29/13 12:39 PM Page 377
501 GMAT
®
Questions
297.
A team of botanists wanted to figure out what sort of conditions
were best for growing a type of flowering plant. The following
table shows the variables that were tested and the results of the
experiment. All other conditions were kept the same, and the
plants came from the same package of seeds. What can we infer are
the best conditions from the data in the table?
Effects of Temperature and Water on Plant Blossoms
Plant Temperature (°F) Amount of water (mL) # of blossoms
1 60 25.0 2
2 60 50.0 2
3 60 75.0 0
4 65 25.0 3
5 65 50.0 4
6 65 75.0 1
7 70 25.0 8
8 70 50.0 10
9 70 75.0 2
10 75 25.0 13
11 75 50.0 10
12 75 75.0 9
13 80 25.0 16
14 80 50.0 12
15 80 75.0 10
a. The plant prefers cool, damp growing conditions.
b. The plant prefers cool, dry growing conditions.
c. The plant prefers warm, damp growing conditions.
d. The plant prefers warm, dry growing conditions.
377
501_07_371-396_501_master.qxd 4/29/13 12:39 PM Page 378
501 GMAT
®
Questions
298.
Five private universities released statistics on the top three college
majors at their schools, graduation rates for each major, and
graduates who reported having a degree-related job in their field
within 12 months of graduation. The following table represents
the information that was released. Which of the following
statements CANNOT be inferred from the data in the table?
Top Three Majors, Graduation Rates, and Graduates with
a Degree-Related Job
University Top three majors
Four-year
graduation
rate within
each major
(%)
Graduates
with a
degree-
related
job (%)
Abracadabra
University
Biology
Chemistry
Zoology
82.5
83.0
87.0
75.0
80.0
80.0
Crescent
Moon
College
Liberal Arts
Finance
Business
Administration
90.0
91.5
90.0
70.0
80.0
75.0
Westfield
University
Women’s Studies
Biology
Liberal Arts
92.0
93.5
95.0
50.0
89.0
70.0
Letterbox
University
Chemistry
Mathematics
Business
Administration
95.0
97.0
95.0
90.0
91.0
80.0
Cranberry
College
Botany
Biology
Zoology
85.0
85.0
83.0
95.0
96.0
88.0
378
501_07_371-396_501_master.qxd 4/29/13 12:39 PM Page 379
379
501 GMAT
®
Questions
a. Crescent Moon College has the most successful networking and
job placement of the universities listed.
b. In the current economic conditions, it is probably more chal-
lenging to get a job in the liberal arts than it is in a science-
related field.
c. Students who require more than four years to graduate will
likely have a harder time finding a job post-college.
d. Those who graduate from Westfield University with degrees
that are not ranked in the table will probably have more success
finding a job within 12 months of graduation.
299. The following table shows the top ten United States imports and
exports for the year 2008. The data in the table supports which of
the following statements?
a. The United States is self-sufficient for many facets of its trans-
portation industry.
b. Passenger cars make up a higher percentage of exports than
imports for the United States.
c. Other nations are dependent on the United States for civilian
aircraft.
d. The United States has little demand for foreign-made clothing.
501_07_371-396_501_master.qxd 4/29/13 12:39 PM Page 380
501 GMAT
®
Questions
Top Ten Imports and Exports for the United States, 2008
Imports Exports
Rank Product
% of
total
imports
Rank Product
% of
total
exports
1 Crude oil 16.3 2 Semicon-
ductors
3.9
8 Cotton
apparel
2.4 5 Automotive
accessories
3.1
3 Medicinal
preparations
3.8 9 Te l e c o m -
munications
equipment
2.6
4 Automotive
accessories
3.1 7 Fuel oil 2.7
5 Other
household
goods
2.9 6 Other
industrial
machines
3.0
2 Passenger
cars
6.0 1 Civilian
aircraft
5.7
6 Computer
accessories
2.8 10 Plastic
materials
2.5
10 Video
equipment
1.9 4 Pharma-
ceutical
prepara-
tions
3.1
7 Petroleum
products
2.5 3 Passenger
cars
3.9
9 Telecom-
munications
equipment
2.1 8 Organic
chemicals
2.6
380
501_07_371-396_501_master.qxd 4/29/13 12:39 PM Page 381
381
501 GMAT
®
Questions
300. A school district in Maryland integrated a state-funded reading
program into its curriculum for four years. The same program was
integrated in a school district in Virginia. The passing rates for
elementary (K–5), middle (6–8), and high school (9–12) are shown
in the following table. Which statement is true about the
implementation of the reading program?
Passing test rates for Virginia and Maryland school districts,
before and after state-funded reading program (percent)
State
Grade Level
K–3 4–6 7–9 10–12
Before
pro-
gram
After
pro-
gram
Before
pro-
gram
After
pro-
gram
Before
pro-
gram
After
pro-
gram
Before
pro-
gram
After
pro-
gram
Virginia 85 78 75 65 82 87 82 89
Maryland 80 78 84 75 79 80 82 85
a. As grade level increased, so did passing rate, both prior to and
after the reading program.
b. The program was unsuccessful in Virginia.
c. The program was unsuccessful in Maryland.
d. The reading program was more effective for older students.
501_07_371-396_501_master.qxd 4/29/13 12:39 PM Page 382
501 GMAT
®
Questions
301.
The following table shows the percent of commuters and the type
of transportation they primarily use, for six countries. Which of
the statements that follows can be supported by the data in the
table?
Common Modes of Transportation for Commuters
Country
Transportation Type (percent)
Bus
Walk/
Bicycle Train Automobile
Brazil 30 50 10 10
Argentina 40 30 15 15
Germany 30 20 25 25
United Kingdom 30 10 40 20
France 20 30 35 15
United States 20 10 10 60
a. Of all the listed countries, pollution is probably lowest in Brazil.
b. South Americans commute with the automobile more fre-
quently than Europeans.
c. Mass transit is the most common way to commute in the
United States.
d. Mass transit is the most common way to commute for the
majority of the countries listed.
382
501_07_371-396_501_master.qxd 4/29/13 12:39 PM Page 383
501 GMAT
®
Questions
302.
A school in New York State has experienced a decrease in
graduation rate for the past six years. The school wants to
determine some of the outside factors that could be playing a role
in the decreasing rates. Behavior reports and juvenile arrests of
students enrolled at the school were the focus of the study. Which
of the following statements can be made, based on the information
in the table?
Graduation Rates, Behavior Reports, and Juvenile Arrests,
2002–2011
Year
Graduation
rate (%)
Behavior
reports (total)
Juvenile arrests
(total)
2002 54 416 5
2003 55 419 5
2004 55 415 4
2005 57 400 4
2006 52 436 8
2007 49 482 11
2008 45 510 16
2009 41 540 20
2010 37 593 25
2011 34 704 31
a. As the total number of behavior reports increases, so do gradua-
tion rates.
b. As graduation rates decrease, behavior reports do the same, but
juvenile arrests increase.
c. Juvenile arrests and behavior reports have been steadily increas-
ing since 2006, and the graduation rate has been steadily
decreasing.
d. Students who do not have behavior issues both in and out of
school make up most of the graduating class each year
.
383
501_07_371-396_501_master.qxd 4/29/13 12:39 PM Page 384
501 GMAT
®
Questions
303.
The following table shows four different cable providers and the
change in their rates for the 2012 subscription year. All four of the
companies have experienced a change in their clientele based on
renewal subscriptions, also shown in the table. What is a
reasonable conclusion based on the information given?
Changes in Cable and Internet Costs for Cable Providers
A, B, C, and D
Cable
provider
% change
in cost of
wireless
Internet
% change in
cost of cable
% of customers
who re-signed
contract for 2012
service year
A +2.0 +5.0 70%
B +4.0 +5.0 65%
C +5.0 +3.0 85%
D +6.0 +2.0 90%
a. The rising cost of cable has more of an influence over whether a
customer will re-sign than the cost of the Internet.
b. The rising cost of Internet service has more of an influence over
whether a customer will re-sign than the cost of cable.
c. Changing Internet and cable costs have little effect on whether
a customer decides to re-sign with the company
.
d. Customers are not likely to renew a subscription for a year of
service if the wireless Internet price increases.
384
501_07_371-396_501_master.qxd 4/29/13 12:39 PM Page 385
501 GMAT
®
Questions
304.
The rate of magazine versus tablet or other device subscription for
three magazines is shown in the following table. Which of the
following statements CANNOT be supported by the information
in the table?
Tablet and Print Magazine Subscriptions for Three
Magazines, 2009–2012 (thousands)
Year
Fashion Forward Sports Weekly Classy Cooking
Print Tablet Print Tablet Print Tablet
2009 25 4 15 2 20 5
2010 28 10 15 4 22 10
2011 32 14 13 7 21 15
2012 35 18 10 10 20 29
a. All three of the magazines have seen an increase in tablet sub-
scriptions for the past four years.
b. People interested in fashion are more likely to get their infor-
mation from a tablet than those interested in cooking.
c. People interested in sports are less likely to have a print sub-
scription than those interested in fashion.
d. People interested in cooking are more likely to have a tablet
subscription than people interested in sports
385
501_07_371-396_501_master.qxd 4/29/13 12:39 PM Page 386
386
501 GMAT
®
Questions
305. The number of new hires for the year 2011 is displayed in the
following table, for four various industries within a county in
Ohio. Which statement is true about the data in the table?
New Hires, Mason County Ohio, 2011
Industry
Number of new
hires
Change in new hire
employment from
year prior
Education 173 +5%
Civil Service 200 +5%
Service Industry 210 +10%
Health Care 205 +5%
a. The healthcare industry hired more new employees for the year
2011 then the civil service industry.
b. There was an increased demand for teachers in 2011.
c. As new hires and employment increase, there is an increased
demand for employees in the service industry.
d. There were more people hired in the service industry and
healthcare than in education and civil service.
501_07_371-396_501_master.qxd 4/29/13 12:39 PM Page 387
501 GMAT
®
Questions
306.
A survey was conducted with groups of people between the ages of
14 and 17 in different states in the United States, measuring how
young people typically spent their time outside of school. The
results of the surveys that were given are displayed in the following
table. Which of the following statements is a reasonable inference
based on the survey results?
How Teenagers Spent Their Free Time, 2010
State
Activities
Tele-
vision
Com-
puter/
Inter-
net use
Sports
or time
outside
Books
and
maga-
zines Work Other
Florida 25% 30% 10% 10% 20% 5%
Vermont 15% 15% 30% 15% 20% 5%
Minnesota 20% 25% 20% 10% 15% 10%
Oregon 15% 15% 25% 15% 25% 5%
Louisiana 20% 30% 20% 5% 15% 10%
Utah 5% 10% 30% 15% 35% 5%
a. Teenagers in northern regions tend to spend more time outside
than teenagers in southern regions.
b. Teenagers in southern states are most likely than teenagers in
northern states to spend time on the Internet in their spare
time.
c. Most people in Vermont get their first job as a teenager.
d. More teenagers spend time watching television than using the
Internet.
387
501_07_371-396_501_master.qxd 4/29/13 12:39 PM Page 388
501 GMAT
®
Questions
307.
A study was conducted to see whether people who drove red cars
were more likely to be written a speeding ticket. The number of
speeding tickets written between 2005 and 2009 for red, black, and
silver cars is displayed in the following table. Which of the
following conclusions can be made, based on the data in the table?
Speeding Tickets Given by Car Color, Monroe County,
2005–2009
Year
Tickets given
to red cars
Tickets given
to black cars
Tickets given
to silver cars
2005 600 500 550
2006 700 400 500
2007 640 450 600
2008 800 600 700
2009 750 550 600
Total tickets
written
3,490 2,500 2,950
a. People driving silver cars are most likely to be given a speeding
ticket.
b. People who drove black cars in 2009 were written the most
speeding tickets.
c. The police wrote more total speeding tickets in 2007 than any
of the other years.
d. People who drive red cars are more likely to get a speeding
ticket in Monroe County than those in black and silver cars.
388
501_07_371-396_501_master.qxd 4/29/13 12:39 PM Page 389
501 GMAT
®
Questions
308.
A laboratory conducted a study on how certain antioxidants
regulate cell division in a variety of groups of the same specimen.
The purpose of the study was to see whether certain antioxidants
can be directly linked to uncontrolled cell division (cancer) in
living organisms, when exposed to cancer-causing agents. The
results of the study are displayed in the following table. What is a
possible inference based on the data the scientists gathered?
Effects of Various Antioxidants on Uncontrolled
Cell Division
Antioxidant
Primary food
source
% of
specimens
who had
uncontrolled
cell division
(control)
% of specimens
who had
uncontrolled
cell division
(experimental)
Alpha-
carotene
Tomatoes,
squash
20.0 12.0
Beta-
carotene
Squash, sweet
potatoes
20.0 11.0
Lutein Spinach, kale 20.0 4.0
Lycopene Cooked red
tomatoes
20.0 13.0
Zeaxanthin Kale, collard
greens
20.0 4.5
a. Lutein and lycopene are the antioxidants that showed the most
beneficial impact on the specimens, so a diet rich in them would
mean a decreased cancer risk.
b. Diets rich in dark green vegetables could mean a decreased
cancer risk.
c. Diets rich in alpha-carotene and beta-carotene will increase the
risk of cancer if exposed to cancer-causing agents.
d. Red and orange vegetables are an excellent source of
zeaxanthin.
389
501_07_371-396_501_master.qxd 4/29/13 12:39 PM Page 390
501 GMAT
®
Questions
309.
The total number of medals for the seven countries who won the
most gold medals at the 2012 Summer Olympic Games is shown
in the following table. Which statement is true about the data in
the table?
Medal Count for the 2012 Summer Olympic Games
Country Gold Silver Bronze Total
United States 46 29 29 104
China 38 27 23 88
Great Britain 29 17 19 65
Russia 24 26 32 82
Korea 13 8 7 28
Germany 11 19 14 44
France 7 11 12 34
a. The country with the most total medals won also had the high-
est medal count for gold, silver, and bronze.
b. The country with the fifth highest gold medal count also had
the fifth highest total medal count.
c. France had the least number of gold medals as well as the low-
est total medal count.
d. Russia had the fourth highest gold and bronze medal counts.
390
501_07_371-396_501_master.qxd 4/29/13 12:39 PM Page 391
501 GMAT
®
Questions
310.
A certain pheromone produced by the female monarch butterfly
has been identified as responsible for their reproductive success.
The same pheromone has been tied to other species of butterflies
but they do not emit it as much as monarchs do, and their
reproductive rates are also significantly lower. Scientists want to
see if increased exposure to the pheromone will be effective in
encouraging mating among an endangered species of butterfly. Six
test groups, with 50 male and 50 female butterflies, were separated
and exposed to different amounts of the pheromone to see if it
increased the mating activity between them. Which of the
following can be inferred from the data in the table?
Mating among Butterflies, Various Pheromone Exposure
Test
group
Amount of pheromone
exposure (%) times
normal amount
Number of butterflies
who mated during
exposure
1 0 0
2 20 12
3 40 36
4 60 74
5 80 94
6 100 90
a. The butterflies would probably mate the most when exposed to
50% pheromone levels.
b. Controlling the amount of pheromone exposure would be an
effective way for scientists to promote reproduction among
these butterflies.
c. An increase in pheromone exposure always leads to increased
mating activity between butterflies.
d. Exposure to controlled levels of pheromones would not encour-
age mating between the endangered butterflies.
391
501_07_371-396_501_master.qxd 4/29/13 12:39 PM Page 392
392
501 GMAT
®
Questions
Answers—Set 12
293. c. The city of Lewistown has 28 fast-food restaurants, which is
four more than Rockville, but has 1% fewer diabetics under the
age of 16, and 8% fewer overall diabetics. Choice a is incorrect
because Greene, the city with the most fast-food restaurants, does
have the highest total occurrence of diabetes. Choice b is incorrect
because Greene, the city with the highest percentage of children
with Type-2 diabetes also has the highest total population with
diabetes. Choice d is incorrect because other factors may
contribute to diabetes, as shown by Lewistown not having the least
number of fast-food restaurants but having the lowest occurrence
of diabetes.
294. d. Since all the patients have the lung disease, and all have some
degree of toxin exposure, and only some have the gene, it is not
possible to tell which one is responsible using this data. Choice a is
incorrect because only some of the patients have the gene, but all
are sick. Choice b is incorrect because the data does not show
conclusive evidence of what causes the disease. Choice c is
incorrect because all of the patients had some degree of exposure.
295. c. For both products, social media site 2 had higher overall sales.
Choice a is incorrect because Product 1 had a higher sales increase
than Product 2 for both websites. Choice b is incorrect because
based on the data, that inference cannot be made. Choice d is
incorrect because based on the data in the table, that inference
cannot be made.
296. a. As the rabbit population increases or decreases, the snake
population shows an opposite trend. Choice b cannot be supported
because there is no data that can support the definite presence of
other prey. Choice c is incorrect because there is no data in the
table that can support the definite presence of other predators.
Choice d is incorrect because the populations do not exactly mimic
each other.
501_07_371-396_501_master.qxd 4/29/13 12:39 PM Page 393
393
501 GMAT
®
Questions
297. d. The plant had the most blossoms at the highest temperature
with the least amount of water. Choice a is incorrect because the
plant had fewer blossoms at the lower temperatures. Choice b is
incorrect because the plant had no blossoms at the lowest
temperature when given the most amount of water. Choice c is
incorrect because the plant had the fewest blossoms under the
wettest conditions for the warmest temperatures.
298. b. The data shows both of the universities that have liberal arts as
a top-three major have 12-month job placement at 70.0%. This is
the lowest percent of job placement besides women’s studies.
Choice a is incorrect because some of the other universities have
higher job placement rates for their top three majors. Choice c is
incorrect because there is no evidence in the table that could lead
to an inference about students who take more than four years to
graduate. Choice d is incorrect because there is no data in the table
that provides any information about unlisted majors; therefore,
an inference about other majors cannot be made based on the
given data.
299. c. Civilian aircraft is the United States’ highest-ranking export.
Choice a is incorrect because the United States’ top export is crude
oil and its second top import is passenger cars. Choice b is
incorrect because passenger cars make up a higher percentage of
imports (6.0%) than exports (3.9%). Choice d is incorrect because
cotton apparel is one of the top imports, at rank number eight.
300. d. Students in the 7–9 and 10–12 groups showed an increase in
reading scores for both states. Choice a is incorrect because fewer
students in the 4–6 age group passed in Virginia than the K–3
group both prior to and after the program was implemented.
Choice b is incorrect because some student groups in Virginia
showed an increase in passing rate after the reading program was
implemented. Choice c is incorrect because some student groups
in Maryland showed an increase in passing rate after the reading
program was implemented.
501_07_371-396_501_master.qxd 4/29/13 12:39 PM Page 394
394
501 GMAT
®
Questions
301. d. Except for Brazil and the United States, over 50% of
commuters from the other four countries rely on the bus and/or
train. Choice a is incorrect because it is an inference that cannot be
directly supported with the evidence in the table. Choice b is
incorrect because the European countries have an overall higher
percentage of commuters using the automobile. Choice c is
incorrect because the majority of Americans (60%) commute by
automobile.
302. c. The graduation rate decreases from 52% to 34% between 2006
and 2011. The behavior reports increase from 436 to 704 and the
juvenile arrests increase from 8 to 31. Choice a is incorrect because
as behavior reports increase, graduation rates decrease. Choice b is
incorrect because as graduation rates decrease, both behavior
reports and juvenile arrests decrease. Choice d is incorrect because
none of the data in the table accounts for factors besides behavior
problems in school and juvenile arrests.
303. a. Although cable provider D increased Internet cost by 6.0% (the
highest) they increased the cost of cable by 2.0% (the lowest) and
they also had the highest percent of customers who re-signed their
contract. Choice b is incorrect because although cable provider A
only increased their Internet service by 2.0%, they increased their
cable cost by 5.0%, but only had 70% of people renew for 2012.
Choice c is incorrect because none of the companies had a 100%
renewal rate for 2012. Choice d is incorrect because all the
companies raised their Internet prices, and all experienced renewal
of contracts.
304. b. Fashion Forward magazine has 18,000 tablet subscriptions, but
Classy Cooking has 29,000. Choice a is incorrect because tablet
subscriptions for the four years given increased for all the
magazines. Choice c is incorrect because Fashion Forward had
35,000 print subscriptions, which is almost twice as many as their
tablet subscriptions. Sports Weekly has an equal number of tablet
and print magazine subscriptions. Choice d is incorrect because
29,000 people have a tablet subscription, while 10,000 have one for
Sports Weekly.
501_07_371-396_501_master.qxd 4/29/13 12:39 PM Page 395
395
501 GMAT
®
Questions
305. d. There were 415 total people hired for healthcare and the
service industry, which is more than the 373 hired in the education
and civil service fields. Choice a is incorrect because the healthcare
field hired 205 new employees, and the civil service field hired 200.
Choice b is incorrect because the education field does not specify
teachers only. Choice c is incorrect because it is not possible to
determine whether there is a correlation between service industry
hires and employment in other fields.
306. b. Based on the results of the survey, 30% of teenagers from
Florida and Louisiana spent time on the Internet, compared to
15% in Vermont and 15% in Oregon, and 25% in Minnesota.
Choice a is incorrect because teenagers in Utah spend the same
amount of time outside as those in Vermont. Choice c is incorrect
because the data in the table does not account for first jobs of older
people in the state. Choice d is incorrect because for all the states
the percentage of teenagers who spend time on the computer is
equal to or higher than the percentage who spend time with
television.
307. d. According to the total number of speeding tickets for each color
of car, red cars had the most tickets written. Choice a is incorrect
because there were 2,950 tickets written to silver cars for the time
period, but 3,490 were written for red cars. Choice b is incorrect
because black cars were written a total of 550 tickets in 2009,
behind both silver and red cars. Choice c is incorrect because
police wrote 1,690 tickets in 2007 but 2,100 in 2008.
308. b. The data shows that green vegetables—kale, spinach, and
collard greens—had the least number of specimens that had
uncontrolled cell division. Choice a is incorrect because the
specimens exposed to lycopene showed a 13.0% likelihood of
acquiring cancer, which is higher than that of zeaxanthin, which
showed a 4.5% cancer occurrence. Choice c is incorrect because
the experimental groups for alpha-carotene and beta-carotene
showed a decrease in uncontrolled cell division when compared to
the control group. Choice d is incorrect because the primary
sources of zeaxanthin are dark green vegetables.
501_07_371-396_501_master.qxd 4/29/13 12:39 PM Page 396
396
501 GMAT
®
Questions
309. a. The United States had 46 gold medals, and 104 medals total,
which is the highest for the data given. Choice b is incorrect
because Korea had the fifth highest number of gold medals, but
was seventh in total medal count. Choice c is incorrect because
France had a total medal count of 34, which is more than Korea
and therefore not the least. Choice d is incorrect because Russia
had the highest number of bronze medals, not the fourth.
310. b. Mating activity increased between the butterflies as the amount
of pheromone exposure increased, except when they were exposed
to 100% of the pheromone, where four less butterflies mated.
However, there was a steady increase in the mating activity
between the butterflies as the amount of exposure increased.
Choice a is incorrect because if the data follows the trend in the
table, a 50% exposure would lead to somewhere between 36 and
74 butterflies mating, and this is less than when they were exposed
to even higher levels. Choice c is incorrect because when the
butterflies were exposed to 100% of the pheromone levels, four
fewer butterflies mated than at 80% exposure. Choice d is
incorrect because more, not less, butterflies mated as the
pheromone levels increased.
501_08_397-414_501_master.qxd 4/29/13 12:40 PM Page 397
8
Multisource
Reasoning
The multisource reasoning section of the GMAT is a test of your
ability to synthesize information from a variety of sources (usually two or
three) to arrive at reasonable inferences and logical conclusions. In today’s
information-rich environment, we are often called on to connect pieces of
data that we receive across a variety of communication types—e-mail, let-
ters, text messages, editorials, manifests and contracts, and so on—as well
as to draw and support suppositions about the ways in which those data
might interact in a wide array of circumstances. To be able to create an
inferential link between such sources and to draw empirically valid con-
clusions from them is a valuable skill in today’s detail-oriented business
environment.
This section presents information broken into segments that are typically
300 words or fewer. Skim through the paragraphs to review the exchange
of information and then assess the questions and answers according to the
presented data. Be sure to read the question stems carefully. A few will require
a single response (e.g., multiple-choice questions), but the majority will
require that you select one of two or three possible responses for each part
of the question (e.g., “Yes or No” responses to three- or four-question seg-
ments). To be credited with a correct response for this latter type, you must
answer all the parts correctly. Please note that questions are independent of
501_08_397-414_501_master.qxd 4/29/13 12:40 PM Page 398
398
501 GMAT
®
Questions
one another, so that you will not be asked to answer one correctly to cor-
rectly answer another. Once you have submitted a response to a given ques-
tion, you cannot return to it, so select your responses with care.
Test-Taking Tips
Maybe the most important thing to keep in mind about the multisource
reasoning section of the GMAT is that the material is loaded with infor-
mation that you will never be asked questions about. Much like the read-
ing comprehension component, multisource reasoning questions are
designed to lure test takers into becoming bogged down in extraneous data
and information. Remember always that you are not being tested on your
ability to memorize information! Keeping this in mind will help you to for-
mulate and enact an effective and efficient approach to the section.
Step One: See the Big Picture
As with the reading comprehension section, a sensible strategy for tackling
multisource reasoning questions is to first skim through the passages to get
a broad sense of the information contained therein. It’s enough at this point
to be able to say, “This is an exchange of offers for the possible acquisition
of a new online business,” or “These are e-mails between a manager and
her human resources officer concerning employee productivity.” You may
want to break the tabs down into very simple statements to help you to
keep track of the flow of information. For example: “Para 1: The manager
thinks her staff is underproducing. Para 2: The HR officer tries to prove
otherwise. Para 3: The manager presents more data to the contrary.” Or
even: “Para 1: Offer. Para 2: Counteroffer. Para 3: Response to coun-
teroffer.” Keep it simple.
You’ll want to keep it simple because, at least for now, getting bogged
down in the fine details (i.e., the nature and specifics of the data pre-
sented)—or, worse, trying to memorize those details—is a waste of your
valuable time. Much, and perhaps most, of the presented information may
prove to be irrelevant. You will not be asked questions about it. More, you
should keep in mind that you will have to return to the passages in order to
answer the questions anyway (and you should never try to answer them
from memory), thus doubling the time you spend engaging the information
in the tabs. See the big picture.
501_08_397-414_501_master.qxd 4/29/13 12:40 PM Page 399
399
501 GMAT
®
Questions
Step Two: Assess the Stem
Now that we have some sense of what the tabs contain, it’s time to carefully
read the question stem to determine exactly what task it is directing us to
do. A few question stems will ask for a single response (e.g., “If Gillian
receives her anticipated yearly bonus, her salary for the month will be . . .”).
Most, however, require multiple responses (e.g., “Does the information in
the articles support each of the following inferences as they appear here?
YES or NO.”). To this end, you will have to read and carefully consider a
number of segments, answering each of them before moving on to the next
question stem. Look for key words in the question stems and answer seg-
ments (e.g., “The manager apparently believes that . . . ,” “The data from
September suggest that . . .”) and let those guide you back to specific details
and data in the multisource reasoning tabs.
Step Three: What Is the Answer?
The best way to answer a question on the GMAT (or, for that matter, any
standardized test) is to find it yourself in the reading, put it in your own
words, and then look for the answer that comes closest in the question’s
answer set. As often as possible—and especially in the case of single-
response question types—be sure to use the process of elimination to
remove unlikely or easily disproven possibilities from the answer set and to
more quickly arrive at a correct response. For common inference question
types, keep in mind that the words “reasonably infer” as they are used on
the GMAT mean something along the lines of “be able to directly prove
through what is written on your screen.” That is, you must be able to point
at the evidence that you use to arrive at your answer. If you are unable to do
so—or if the presented information suggests or supports a broader array of
possible interpretations—take a moment to carefully consider whether such
an answer represents a reasonable or defensible inference. As always, be on
the lookout for extreme language in the answer set. Answers containing
words like must, always, never, no one, and everyone should raise a red flag.
Although such answers are not automatically incorrect, it is rarely the case
that GMAT passages can or will substantiate such absolute positions. Be
extra wary.
Note that once you have answered a question and moved on, you will be
unable to return to it, so be sure you are comfortable with your responses
before pressing ahead.
501_08_397-414_501_master.qxd 4/29/13 12:40 PM Page 400
400
501 GMAT
®
Questions
Set 13
Now it is time to answer GMAT multisource reasoning practice questions
that have been designed to test your integrated reasoning skills. Good luck!
Use the following articles to answer questions 311 through 314.
Article One: Opinion Article in a Popular Political Journal
The incumbent candidate’s protests aside, it is a matter of plain fact that some
47% of the country’s residents currently pay no taxes—the largest share of
nonpayers on record—and therefore live almost entirely off the largess of
those who are now shouldering a greater part of the burden than ever before.
Our concern is that, as this number continues to grow, a greater portion of
the population will become habituated to life on various forms of public assis-
tance and thus less and less likely to seek avenues of employment that would
lead them to leave the rolls of public welfare.
Article Two: Speech Delivered by the Incumbent Candidate
It’s true that more folks are having trouble making ends meet these days, but
what you should keep in mind is that a smaller and smaller group of people
in the top one percent of the income scale takes home a greater share of
wealth than ever before, while people in the bottom half have seen their
incomes frozen essentially since the mid-1970s. When my opponent and his
supporters argue that almost half of taxpayers pay no taxes, you should
remember that many of them pay no taxes because they simply don’t earn
enough money to do so.
Article Three: Assessment by Nonpartisan Tax Policy Think Tank
Although it is true that a greater portion of the public pays no federal income
taxes, almost everyone pays taxes of some kind. These include payroll taxes
(in the case of those drawing income from regular employment), state and
local taxes, fines and fees of various kinds, as well as taxes on gasoline, gro-
ceries, clothing, and most other consumer goods. Meanwhile, it is worth
keeping in mind that federal income tax is the relatively progressive instru-
ment in the national revenue-raising mechanism, whereas other forms of tax-
ation (particularly those at the state and local levels) tend to be markedly
regressive.
501_08_397-414_501_master.qxd 4/29/13 12:40 PM Page 401
401
501 GMAT
®
Questions
311. Consider each of the following statements. Does the information
in the articles support the inference as it appears here?
Yes No The number of taxpayers paying no federal income
taxes is currently the largest on record.
Yes No Making the federal tax code even more progressive
would alleviate much of the repressiveness of the tax
codes at the state and local levels.
Yes No The candidate for local office is unaware of the
relatively progressive nature of the federal tax code.
Yes No The fact that the country’s top earners pay a greater
share of the total national tax burden is proof that
the federal tax code is unusually progressive.
312. Consider each of the following statements. Does the information
in the articles support the inference as it appears here?
Yes No The incumbent candidate is unaware that 47% of
taxpayers currently pay no federal income tax.
Yes No The author of the opinion piece is unaware that the
47% figure he or she employs does not include other
forms of taxation, such as state and local taxes,
payroll taxes, and taxes on consumer goods.
Yes No Members of the top one percent of the income scale
currently pay little or no payroll tax.
Yes No At least some local fees are more regressive than the
federal income tax.
313. It can be reasonably inferred that the Nonpartisan Tax Policy
Think Tank mentions “income from regular employment” to
suggest that
a. some taxpayers are evading legal taxation.
b. there are some forms of income not affected by payroll.
c. the 47% are the only ones who pay payroll taxes.
d. top earners are less likely than others to pay state and local fees.
e. payroll taxes are the most regressive of all those mentioned by
the think tank in its article.
501_08_397-414_501_master.qxd 4/29/13 12:40 PM Page 402
402
501 GMAT
®
Questions
314. For each of the following statements, select All Accept (AA) if,
based on the information provided, it can be inferred that the
authors of the three articles would likely accept that the statement
is true. If not, select Otherwise (O).
AA O 47% of taxpayers currently pay no federal income tax.
AA O Top earners bear an undue burden under the current
tax code.
AA O The distribution of employment income plays at least
some part in the appearance of unbalance in the
federal tax code.
Use the following e-mail correspondence to answer questions 315
through 317.
E-mail from the Convention Coordinator
Our upcoming comic book convention is running into a few unexpected
snags, the most pressing of which is that we appear to have seriously under-
estimated likely attendance. Our idea to promote the attendance of indus-
try professionals (who do not pay an attendance fee, once they are
credentialed) by allowing them to bring along two guests is now threaten-
ing to cause us to overflow our assigned space in the convention center.
There is an additional space available during the three days of the conven-
tion, but acquiring it would threaten our ability to pay our vendors and
force us to rely on greater-than-anticipated volunteer assistance.
E-mail from the Convention Budget Officer
The change in guest policy has created a real problem then. We have cur-
rently budgeted $35,000 for three days to cover a wide array of vendor
services, including security, transportation, and on-site medical personnel
in the event of an accident or sudden illness. It has already been very diffi-
cult to find professionals who are both qualified and available to take on
many of those skilled responsibilities. Worse, securing the use of expanded
convention center space will cost an additional $7,000 over the long week-
end, cutting into our already tight overall budget. Indeed, any additional
expenditures will require the use of yet more (potentially underqualified)
volunteers.
501_08_397-414_501_master.qxd 4/29/13 12:40 PM Page 403
501 GMAT
®
Questions
403
E-mail from the Events Coordinator of the Convention Center
We have considered your proposal for expanded use of the convention cen-
ter facilities and (given the expected size of the convention’s attendance) are
prepared to let you lease the additional space at a 40% discount. More, the
center is willing to offer you the use of its own security personnel, though
only for Friday and Saturday (which together are expected to see far more
traffic than Sunday), and only if you agree to also take on the additional
convention space.
315. Consider each of the following statements. Does the information
in the articles support the inference as it appears here?
Yes No Skilled professionals account for the largest
contingent of the convention’s vendor personnel.
Yes No Given the current budget restrictions, even with the
discount offered by the convention center, it is likely
that the convention will have to somewhat expand its
use of volunteer personnel.
Yes No At least some volunteer personnel are unqualified to
perform certain vendor duties required by the
convention.
Yes No The convention center’s offer of discounted
additional floor space and security personnel will
alleviate most of the “unexpected snags” mentioned
by the Convention Coordinator in her e-mail.
316. The 40% discount on the expanded floor space, if accepted, would
account for what percentage of the convention’s total budget for
vendor services?
a. 15
b. 9
c. 8
d. 11
e. 6
501_08_397-414_501_master.qxd 4/29/13 12:40 PM Page 404
404
501 GMAT
®
Questions
317. For each of the following statements, choose Definitely True (DT)
if the statement is absolutely true. Choose Possibly True (PT) if
the statement can be true but might not necessarily be true all
the time.
DT PT In the past, the comic book convention had a
different policy toward industry professionals and
their guests.
DT PT If every industry professional brought along only
one guest, the convention center would not overflow
its assigned space.
DT PT If the comic book convention eliminates the guest
policy next year, it will not overflow its assigned
space.
Use the following e-mail correspondence to answer questions 318
through 322.
E-mail from Buyer
I recently noticed your listing in Antiques Quarterly magazine and am pre-
pared to make an offer on the ceremonial sword you described. As a seri-
ous collector, however, it behooves me to double-check the authenticity of
my acquisitions and to this end I insist on using experts who are known to
me and who have earned my trust over the years. I find the current authen-
tication open to some question. Because this is a somewhat costly endeavor,
I ask that you consider my offer of $83,000 for the sword, an amount that
will (in part, at least) offset this additional expense.
E-mail from Seller
Thanks so much for getting in touch. Your offer is some 15% less than my
asking price, and (as such) is much less than I’d hoped to get for the sword.
I assure you that this unusual item’s authenticity has been confirmed by its
country’s leading experts on the era. More, weapons from this particular era
are becoming more and more collectible as the ongoing upheaval in its
country of origin has led many of these treasures to be destroyed. It is my
hope that sometime during the next four years the value of the sword will
more than triple. Still, I am willing to part with this exquisite piece of his-
tory if you are willing to increase your offer by an additional 7%. Please
think it over.
501_08_397-414_501_master.qxd 4/29/13 12:40 PM Page 405
405
501 GMAT
®
Questions
318. For each of the following statements, choose Definitely True (DT)
if the statement is absolutely true. Choose Possibly True (PT) if
the statement can be true but might not necessarily be true all
the time.
DT PT The value of the sword will increase during the next
four years.
DT PT The buyer’s expert appraisers are more highly
skilled than any in the employ of the seller.
DT PT The buyer is not entirely certain that the value of
the weapon will triple in the coming four years.
DT PT The cost the buyer will incur to confirm the sword’s
authenticity will exceed the value of 15% of the
seller’s original asking price.
319. According to his message, the seller would be willing to accept an
offer of:
a. $79,500
b. $81,650
c. $88,810
d. $90,100
e. $95,450
320. Consider each of the following statements. Does the information
in the correspondence support the inference as it appears here?
Yes No The buyer has legitimate concerns about the
authenticity of the sword.
Yes No It is unlikely that the experts used by the buyer and
seller overlap.
Yes No There is other evidence that the seller could produce
to convince the buyer to accept the original asking
price.
Yes No The seller does not inhabit the country of origin of
the antique artifact in question.
501_08_397-414_501_master.qxd 4/29/13 12:40 PM Page 406
406
501 GMAT
®
Questions
321. The seller refers to “the ongoing upheaval” in the sword’s country
of origin as evidence to support which of the following?
a. The sword is the last known example of its kind.
b. The buyer’s authenticators lack the expertise to confirm the
sword’s actual value.
c. The seller expects the value of the sword to continue to
increase.
d. The buyer will be unable to acquire a similar artifact at the
price he is offering.
e. The sword’s country of origin is well known for its political
instability.
322. For each of the following statements, select Yes if true. Otherwise,
select No.
Yes No The seller will not be able to afford to sell the sword
at the buyer’s current offer.
Yes No The buyer is not entirely convinced of the reliability
of the seller’s experts.
Yes No Other artifacts from the era in question are known
to exist.
Yes No At least some of the experts on the item’s
authenticity reside in its country of origin.
Use the following to answer questions 323 through 328.
Internal Communication from Sports League
We appear to have reached a serious impasse with the umpires’ union
regarding their contract demands. Besides salary and cost-of-living
increases amounting to 10% distributed evenly over four years, a refine-
ment of the league’s full-time umpire hiring policy, and improved physical
fitness regimes and assessments, the union is demanding that our league
continue to offer a defined-contribution retirement plan to all currently
active umpires through the year 2021. Frankly, we find this latter demand
unacceptable, as it is a benefit enjoyed only by the umpires. We may need
to temporarily vacate certain standing policies related to umpires if we are
to have replacements on the field in time for the regular-season opener.
501_08_397-414_501_master.qxd 4/29/13 12:40 PM Page 407
407
501 GMAT
®
Questions
Internal Communication from Umpires’ Union
We appear to have reached a serious impasse regarding our demands from
the sports league. First, the league is demanding that we permit the hiring
of a greater number of full-time umpires. And, though the league has
agreed in principle to our salary demands (though only to a six percent cost-
of-living increase distributed evenly over four years), the issue of the pen-
sion plan remains, with the league insisting that we accept a more
conventional 401k plan. The cost of the current plan, remarkably, is a mere
$3 million per year and therefore well within the league’s current ability to
fulfill. Still, there has been no significant agreement on this front. We
believe that the league will move soon to hire replacement umpires in hopes
of having them on the field for the regular-season opener in two weeks.
323. For each of the following statements, choose Definitely True (DT) if
the statement is absolutely true. Choose Possibly True (PT) if the
statement can be true but might not necessarily be true all the time.
DT PT The league’s players do not have a defined-pension
plan.
DT PT The replacement umpires will be in place in time
for the league’s regular-season opener.
DT PT The umpires’ union would be willing to accept the
sport league’s current salary counter-offer.
DT PT Prior to this impasse, the league had in place a
physical fitness assessment program for its umpires.
324. According to the information presented by the umpires’ union,
which of the following can be most reasonably inferred about full-
time umpires?
a. There are currently no full-time umpires in the league, but they
will soon outnumber the regular umpires.
b. Full-time umpires currently represent a majority of the
umpires, but will soon represent almost all umpires.
c. At least some of the current roster of umpires are full-time
umpires.
d. None of the full-time umpires has a defined-contribution
retirement plan.
e. The umpires will be unwilling to accept any new hiring of full-
time umpires.
501_08_397-414_501_master.qxd 4/29/13 12:40 PM Page 408
408
501 GMAT
®
Questions
325. For each of the following statements, select Inferable (I) if the
statement is reasonably inferable from the information provided.
Otherwise, select Not Inferable (NI).
I NI The sports league has agreed to at least part of the
umpires’ salary demands.
I NI The sports league brings in more than $3 million a year
in revenue.
I NI None of the umpires would be happy with a 401k
pension plan.
326. The sports league’s mention of vacating “certain standing policies
related to umpires” can be more reasonably said to imply that:
a. The sports league is unconcerned with umpire safety.
b. The sports league will be unwilling to reconsider the umpires’
salary demands.
c. The sports league does not expect the umpires to consider any
new contract proposals for at least two weeks.
d. Certain policies related to umpires sometimes take longer than
two weeks to implement.
e. None of the policies in question apply to full-time umpires.
327. Assuming that the information presented by the umpires’ union is
accurate, the author’s use of the word “remarkable” most
reasonably suggests:
a. The issues in question are not entirely financial in nature.
b. The umpires’ union is surprised that contract negotiations have
not already been resolved in its favor.
c. The league’s insistence on hiring more full-time umpires will
lead to a season-ending strike.
d. The umpires’ union expects many of its members to quit if their
cost-of-living demands are not met.
e. The impasse appears to be the result of a misunderstanding on
the sports league’s part about the nature of its pension plan.
409
501 GMAT
®
Questions
501_08_397-414_501_master.qxd 4/29/13 12:40 PM Page 409
328. Assuming an umpire’s annual salary of $150,000, the difference
between the sports league’s cost-of-living increase and the umpires’
union’s demands amounts to how much per year?
a. $2,500
b. $1,500
c. $3,000
d. $3,100
e. $5,100
501_08_397-414_501_master.qxd 4/29/13 12:40 PM Page 410
410
501 GMAT
®
Questions
Answers—Set 13
311. Yes. The various correspondents are in agreement about this fact,
and therefore it can be reasonably inferred given the available
information.
No. There is not enough information in the passages to support
this prediction of what future actions might be taken by state and
local governments to offset any attempts to add progressivity to
the current tax system.
No. There is not enough information in the passage to support
such a conclusion.
No. There are other explanations as to why this might be the case.
Indeed, the passage suggests that top earners might be paying an
expanded share of total taxes because their total share of the
national income has increased disproportionate to the rest of the
population.
312. No. There is not enough information in the passage to support
such a conclusion.
No. There is not enough information in the passage to support
such a conclusion.
No. There is not enough information in the passage to support
such a conclusion.
Yes. This fact is explicitly stated in the Assessment by the
Nonpartisan Tax Policy Think Tank and can therefore be
supported as a reasonable inference.
313. b. The Nonpartisan Tax Policy Think Tank mentions “income
from regular employment” to suggest a categorical distinction
between types of income that might or might not be exposed to
payroll taxes.
314. AA. All three of the passages appear to accept this information as
fact.
O. Whereas the first passage appears to agree with this statement,
it is clear that the second and third either do not or likely do not.
O. Whereas the second and third passages appear to agree with
this statement, it is clear the third passage likely holds a contrary
view.
501_08_397-414_501_master.qxd 4/29/13 12:40 PM Page 411
501 GMAT
®
Questions
411
315. No. It is clear that skilled professionals constitute a vital part of the
convention’s vendors but there is not enough information
presented in the passages to suggest that they represent the
greatest number of said vendors.
Yes. The e-mail from the Convention Budget Officer explicitly
states that any further expenditures (even discounted ones) will
necessitate the use of greater numbers of volunteers.
Yes. The communications make clear that certain transportation,
security, and medical vendors require professional skills or
credentialing.
No. While it is clear that the convention space issue is significant,
there is no evidence to suggest that it constitutes the greatest
number of possible problems with the convention’s going forward.
316. c. Forty percent of $7,000 is $2,800, which is 8% of the
convention’s total operating budget of $35,000.
317. DT. The communications make clear the policy toward
credentialed professionals is a new one.
PT. Because the number of invited professionals is not known—
nor is some sense of the actual capacity of the convention center in
question—there is not enough information available to fully
substantiate this statement.
PT. There is not enough information available to fully substantiate
this statement.
318. PT. Without much further detail, the seller’s projection that the
sword’s value will increase is not enough to fully substantiate this
statement.
PT. There is not enough information available to fully substantiate
this statement.
DT. The buyer makes clear that he has questions both about the
item’s value and authenticity.
DT. The seller makes it clear that his asking price will only mostly
offset the cost of the authentication.
501_08_397-414_501_master.qxd 4/29/13 12:40 PM Page 412
412
501 GMAT
®
Questions
319. c. The seller makes it clear that he will accept an 8% discount on
an item fully priced at $95,450.
320. No. It is unclear whether the buyer’s concerns are legitimate
inasmuch as there is not enough information about the quality of
his expert authenticators to substantiate such a statement.
Yes. Given the disdain on display in the passages, as well as the
demands of the buyer, it is highly unlikely that the buyer and seller
have any expert authenticators in common.
No. There is not enough information available in the passage to
substantiate this statement.
Yes. The seller makes it clear that he does not currently inhabit
the item’s country of origin.
321. c. The seller mentions “the ongoing upheaval” in the sword’s
country of origin as evidence to support his contention that the
item will continue to gain value.
322. No. There is not enough information available in the passage to
substantiate this statement.
Yes. The buyer makes it clear that he does not entirely trust the
expertise of the seller’s authenticators.
Yes. The seller makes it clear that at least some artifacts from the
era in question have escaped destruction.
Yes. The seller explicitly states that at least some of the experts on
the item’s authenticity reside in its country of origin.
323. DT. The sports league’s communication explicitly states that the
umpires are the only league members to have the defined-
contribution plan.
PT. This is stated as a possibility, though whether it will happen
remains an open question.
PT. There is not enough information available in the passage to
fully substantiate this statement.
DT. The sports league’s communication makes clear reference to
“improved physical fitness regimes and assessments.”
413
501 GMAT
®
Questions
501_08_397-414_501_master.qxd 4/29/13 12:40 PM Page 413
324. c. It is clear from both statements that a number of full-time
umpires are already members of the umpires’ union.
325. I. The umpires’ union makes it clear that the league has, in fact,
acceded to at least some of its demands.
I. The umpires’ union communication makes it clear that the
league brings in more than enough to cover the annual expense of
the umpires’ union’s current pension plan.
NI. There is not enough information available in the passage to
substantiate this statement.
326. d. The statement of the sports league—in conjunction with
information that the regular-season opener is two weeks away—
makes it clear that certain league policies related to umpires
sometimes take longer than that amount of time to implement.
327. a. The fact that the league could easily cover the annual expense
of the umpires’ current pension plan strongly suggests that at least
some of the issues at stake in the current impasse are not financial
in nature.
328. b. Assuming this baseline salary, the difference between the cost-
of-living increases in question distributed evenly over the projected
time period is $6,000, or $1,500 per year.
501_08_397-414_501_master.qxd 4/29/13 12:40 PM Page 414
Blank Page
501_09_415-440_501_master.qxd 4/29/13 12:40 PM Page 415
SECTION 4
GMAT
Quantitative Section
The GMAT quantitative section will measure your ability to reason
mathematically, solve numerical problems, and analyze graphic data. These
are common situations that you will be faced with in the business world.
GMAT quantitative questions fall across three disciplines:
Arithmetic
Elementary algebra
Commonly known concepts of geometry
Most of the concepts tested on the GMAT were probably taught in your
high school math classes.
You will have to answer 37 multiple-choice questions on the GMAT
quantitative section. Of these questions, 15 fall into the data-sufficiency
category and 22 into the problem-solving category. Problem-solving ques-
tions will assess your basic math skills, knowledge of elementary-level con-
cepts, and aptitude for reasoning and solving quantitative problems.
Data-sufficiency questions will assess your ability to interpret a math prob-
lem, identify relevant information, and evaluate whether there is enough
information to solve a problem. Each data-sufficiency question on the
GMAT is accompanied by information and two statements, labeled (1) and
(2). You must decide whether the statements offer sufficient information for
501_09_415-440_501_master.qxd 4/29/13 12:40 PM Page 416
416
501 GMAT
®
Questions
you to answer the question. Then, you’ll choose from the following answer
choices:
Statement (1) ALONE is sufficient, but statement (2) ALONE
is not sufficient.
Statement (2) ALONE is sufficient, but statement (1) ALONE
is not sufficient.
BOTH statements TOGETHER are sufficient, but
NEITHER statement ALONE is sufficient.
EACH statement ALONE is sufficient.
Statements (1) and (2) TOGETHER are NOT sufficient.
501_09_415-440_501_master.qxd 4/29/13 12:40 PM Page 417
9
Arithmetic
Out of the three math topics, arithmetic usually accounts for one-
half of the questions asked in the GMAT quantitative section. This chap-
ter reviews basic arithmetic concepts and formulas, involving:
number terminology
order of operations
absolute value
factoring
fractions
decimals
mean, median, mode, and range
percents
Number Terminology and Properties
Integers are the numbers that you see on a number line. Positive integers are
integers that are larger than zero. Negative integers are smaller than zero.
The result in an addition problem is called the sum. The result in a sub-
traction problem is called the difference.
501_09_415-440_501_master.qxd 4/29/13 12:40 PM Page 418
418
501 GMAT
®
Questions
Adding integers often involves the use of certain properties. The associa-
tive property of addition states that when you add a series of numbers, you can
regroup the numbers any way you’d like:
1 + (9 + 7) = (1 + 9) + 7 = (1 + 7) + 9
The commutative property of addition states that when you add numbers,
order doesn’t matter:
8 + 2 = 2 + 8
The result in a multiplication problem is called a product. The result in a
division problem is called a quotient.
The product of two integers with the same sign (+ and + or and –) is
always positive. The product of two integers with different signs (+ and –)
is always negative. For example:
3 ¥ 4 = 12
–3 ¥ –4 = 12
–3 ¥ 4 = –12
3 ¥ –4 = –12
Likewise, the quotient of two integers with the same sign (+ and + or – and –)
is always positive. The quotient of two integers with the different signs
(+ and –) is always negative. For example:
4 ÷ 2 = 2
–4 ÷ –2 = 2
–4 ÷ 2 = –2
4 ÷ –2 = –2
When multiplying integers, you will often use the same properties you used
with the addition of integers. The associative property of multiplication states
501_09_415-440_501_master.qxd 4/29/13 12:40 PM Page 419
419
501 GMAT
®
Questions
that when you are multiplying a series of numbers, you can regroup the
numbers any way you’d like:
2 ¥ (5 ¥ 9) = (2 ¥ 5) ¥ 9 = (2 ¥ 9) ¥ 5
The commutative property of multiplication states that when you multiply inte-
gers, order doesn’t matter:
6 ¥ 5 = 5 ¥ 6
The Order of Operations
To remember the order of operations with numerical expressions, you can
use a mnemonic, the word PEMDAS.
P Do operations inside parentheses.
E Evaluate terms with exponents.
M/D Do multiplication and division in order from left to right.
A/S Add and subtract terms in order from left to right.
Set 14
Use the order of operations to answer the following questions.
329. 5 + 10 ÷ 5 – 7 =
330. 2 – 6 ¥ 4 ÷ 2 =
331. 9 ÷ 3 + 3 ¥ 8 =
Absolute Value
If you look at a point on a number line, measure its distance from zero, and
consider that value as positive, you have just found the number’s absolute
value. The absolute value of 3, written as |3|, is 3. The absolute value of
–3, written as |–3|, is also 3.
501_09_415-440_501_master.qxd 4/29/13 12:40 PM Page 420
420
501 GMAT
®
Questions
When using the order of operations, the absolute value symbol is treated
at the same level as parentheses. Try this out:
5 ¥ |–13 + 3|
First, evaluate the expression inside the absolute value symbol:
5 ¥ |–10|
Now, evaluate the absolute value:
|–10| = 10, so 5 ¥ 10 = 50
Set 15
Find the absolute values.
332. |156| =
333. |–97| =
334. –|13| =
335. 74 + |–23| =
336. 35 – |–12| =
337. 100 ÷ 5 + |–5 ¥ 3| =
338. 5 ¥ –|9| =
339. 99 ÷ |–33| =
Factoring Numbers
A number is a factor of a second number if it can be divided into the sec-
ond number without leaving a remainder. Let’s look at the factors of 12: 1,
501_09_415-440_501_master.qxd 4/29/13 12:40 PM Page 421
421
501 GMAT
®
Questions
2, 3, 4, 6, and 12. The number 12 can be divided by each of these numbers
without leaving a remainder.
12 ÷ 1 = 12
12 ÷ 2 = 6
12 ÷ 3 = 4
12 ÷ 4 = 3
12 ÷ 6 = 2
12 ÷ 12 = 1
If you start with 1 and the number itself when you write down factor pairs,
you won’t forget any of them. For 12, the factor pairs are as follows:
1 and 12
2 and 6
3 and 4
If a number is a factor of a given number, the given number is divisible by
the factor. A few simple rules will help you quickly determine divisibility
and factor problems.
An integer is divisible by 2 if its ones digit is divisible by 2.
An integer is divisible by 3 if the sum of its digits is divisible
by 3.
An integer is divisible by 4 if its last two digits form a number
divisible by 4.
An integer is divisible by 5 if its ones digit is either 0 or 5.
An integer is divisible by 6 if it is divisible by both 2 and 3.
An integer is divisible by 9 if the sum of its digits is divisible
by 9.
An integer is divisible by 10 if its ones digit is 0.
When an integer greater than 1 has exactly two factors (1 and itself), it is a
prime number. Examples of prime numbers include 2, 3, 5, 7, 11, 13, 17, 19,
23, and 29. Note that the opposite (negative version) of these numbers are
501_09_415-440_501_master.qxd 4/29/13 12:40 PM Page 422
422
501 GMAT
®
Questions
also prime. For example, the factors of –23 are 1, –23, –1, and 23. Thus, –23
is prime because it has exactly two positive factors: 1 and 23.
When an integer greater than 1 has more than two factors, it is called a
composite number.
The numbers 0 and 1 are neither prime nor composite. Zero has an infi-
nite number of factors. The number 1, on the other hand, has one factor—
itself. When a number is expressed as a product of factors that are all prime,
that expression is called the prime factorization of the number.
The greatest of all the factors common to two or more numbers is called
the greatest common factor (GCF).
A multiple of a number is the product of that number and any whole
number. The least of the common multiples of two or more numbers,
excluding 0, is called the least common multiple (LCM).
Set 16
Answer the following questions.
340. What is the prime factorization of 60?
341. Find the greatest common factor of 20 and 30.
342. Find the greatest common factor of 63 and 81.
343. Find the least common multiple of 63 and 81.
Fractions
A proper fraction has a numerator that is smaller than its denominator:
__
9
10
Improper fractions have numerators that are bigger than their denominators:
15
__
7
A mixed number is a number that is represented as a whole number and a
proper fraction. The following are all mixed numbers:
7
__
3
, 8
__
1
, –2
__
5
4 2 6
501_09_415-440_501_master.qxd 4/29/13 12:40 PM Page 423
423
501 GMAT
®
Questions
To change a mixed number into an improper fraction, follow these steps:
1. Multiply the denominator of the fraction by the whole number.
2. Add that sum to the numerator.
3. Put that amount over the original denominator.
A fraction can be considered negative if either its numerator or denomina-
tor is negative. When a fraction receives any type of sign, particularly the
negative sign, it can appear in three different places—in the numerator,
denominator, or right before the fraction.
To add or subtract fractions, the denominators have to match. To add
fractions with like denominators, just add the numerators. To subtract frac-
tions with like denominators, just subtract the numerators.
To n d t h e s u m o r d i f f e r e n c e o f t w o f r a c t i o n s w i t h u n l i k e d e n o m i n a t o r s ,
rename the fractions with a common denominator. Then, add or subtract
and simplify your answer.
To multiply fractions, multiply the numerators, then multiply the
denominators, and finally simplify if possible and necessary.
To divide one fraction by another, you need to flip the second fraction
and then multiply the fractions. This flip of the second fraction is called the
multiplicative inverse of a number or the reciprocal.
Sometimes, you may need to find the greatest fractions or put fractions
in order from least to greatest or from greatest to least. To do this, you
should first give all the fractions a common denominator.
Set 17
Answer the following questions.
344. Convert
19
__
5
to a mixed number.
345. Convert 6
__
5
8
to an improper fraction.
__
4
14
__
3 15
= 346.
501_09_415-440_501_master.qxd 4/29/13 12:40 PM Page 424
424
501 GMAT
®
Questions
Decimals
The decimal system is a way to name numbers based on the powers of 10.
The numbers to the right of the decimal point are fractional equivalents
with denominators that are powers of ten. For example,
0.1 =
__
1
10
0.2 =
__
2
10
0.3 =
__
3
10
Decimal numbers are easy to compare and order, when you remember that
the place value has meaning. In mathematics, 2.4 is the same number as
2.400 because both numbers represent two and four tenths. A whole num-
ber is understood to have a decimal point to the right of the number. For
example, 12 = 12. = 12.0 = 12.000. Each expression represents twelve with
no remainder. To compare decimals, it is best to change each decimal into
an equivalent decimal with the same number of decimal places.
To add or subtract decimal values, line up the decimal points and add or
subtract.
When you are multiplying decimals, first you multiply in the usual fash-
ion, and then count over the proper number of places. This is done by
counting how many places are to the right of the decimal point in each
number you are multiplying.
When you are dividing decimals, you move the decimal point of the div-
idend and divisor the same number of places.
Set 18
Answer the following questions.
347. Order from lower to higher: 0.03, 0.008.
348. 6.4 – 1.3 =
349. 3.1 ¥ 4 =
501_09_415-440_501_master.qxd 4/29/13 12:40 PM Page 425
425
501 GMAT
®
Questions
Mean, Median, Mode, and Range
When you are dealing with sets of numbers, there are certain measures used
to describe the set as a whole. These are called measures of central ten-
dency, and they include mean, median, mode, and range.
Mean is another way of saying average. To find the average, you total up
all the values and then divide by the number of values.
When you are considering a list of values in order (from smallest to
largest), the median is the middle value. If there are two middle values, you
just take their average.
In a list of values, the mode is the number that occurs most often. If two
numbers occur most often, you have two modes. This is called bimodal.
The range indicates how close together the given values are to one
another in a set of data. To find the range, determine the difference between
the largest and the smallest values in the set of data. Subtract the smallest
value from the largest value in the set.
Set 19
350. Find the mean of the following set of data: {32, 34, 34, 35, 37, 38,
34, 42}
351. What is the mode of {71, 68, 71, 77, 65, 68, 72}?
352. The ages at the day camp were as follows: 9, 12, 9, 10, 9, 13, 11, 8,
17, 10. What is the median age?
353. What is the range of the temperatures listed: 43°, 47°, 43°, 52°,
42°, 78°, 84°, 80°?
501_09_415-440_501_master.qxd 4/29/13 12:40 PM Page 426
426
501 GMAT
®
Questions
Probability
Probability is the mathematics of chance. It is a way of calculating how
likely it is that something will happen. It is expressed as the following ratio:
P(event) =
__________________________
number of favorable outcomes
number of total outcomes
The term favorable outcomes refers to the events you want to occur. Total
outcomes refers to all the possible events that could occur.
A probability of zero (0) means that the event cannot occur. A probabil-
ity of 50% is said to be random or chance. A probability of 100% or 1.00
means it is certain to occur. Probabilities can be written in different ways:
As a ratio: 1 out of 2 (1:2)
As a fraction:
__
1
2
As a percent: 50%
As a decimal: 0.5
Percents
When you see a number followed by the percent symbol, think of the per-
cent as a ratio comparing that number to 100. Percents can be expressed in
two different ways:
1. as a fraction (just put the number over 100): 5% =
___
5
100
2. as a decimal (move the decimal point two places to the left):
5% = 0.05
Recall that the word of tells you to multiply. When you take the percent of
a number, you should multiply.
When you take the percent of a percent, all you need to do is multiply:
40% of 20% of 600 =
0.40 of 0.20 of 600 =
0.40 ¥ 0.20 ¥ 600 = 48
501_09_415-440_501_master.qxd 4/29/13 12:40 PM Page 427
427
501 GMAT
®
Questions
Set 20
Now it is time to answer GMAT problem-solving practice questions that
have been designed to test your arithmetic skills. Remember:
Solve the problem and indicate the best of the answer choices
given.
All numbers used are real numbers.
A figure accompanying a problem-solving question is intended
to provide information useful in solving the problem. Figures
are drawn as accurately as possible EXCEPT when it is stated
in a specific problem that the figure is not drawn to scale.
Straight lines may sometimes appear jagged. All figures lie in a
plane unless otherwise indicated.
354. List the following fractions in order from least to greatest.
__
1
,
__
5
,
__
4
,
__
3
__
3
__
1
__
2
2 7 6 4
,
8
,
3
,
5
a.
__
1
,
__
3
,
__
2
8 5
,
__
1
,
__
4
,
__
5
__
3
3 2 6 7
,
4
b.
__
1
,
__
1
,
__
3
,
__
1
,
__
4
,
__
5
,
__
3
2 3 4 5 6 7 8
c.
__
1
2
,
__
1
3
,
__
2
,
__
3
5 4
,
__
3
__
4
,
__
5
8
,
6 7
d.
__
1
,
__
3
3
,
__
2
__
1
__
3
__
5
5
,
__
4
8 2
,
6
,
4
,
7
e.
__
1
,
__
5
__
3
2 7
,
8
,
__
3
,
__
2
,
__
1
4 5 3
,
__
2
5
355. When processing iron ore into stainless steel, high-purity oxygen
is blown through, removing 1.5% of all impurities. Then only
30% of the remaining metal alloy can be used to create rust-free
stainless steel. If 5 kg of stainless steel is desired, how much
starting iron ore is needed?
a. 19.61 kg
b. 16.9 kg
c. 6.73 kg
d. 7.64 kg
e. 16.5 kg
501_09_415-440_501_master.qxd 4/29/13 12:40 PM Page 428
428
501 GMAT
®
Questions
356. The House of Representatives consists of 435 members divided
into two parties. How many Daemocrats are seated in the House if
their seats are equal to 20 more than two-thirds of the Republican
Party seats?
a. 249
b. 186
c. 162
d. 273
e. 229
357. In 2009,TechNet had an average net profit of $7.50 per item sold.
In 2012, the profit decreased to $6.60 per item sold. What is the
percent of profit decrease?
a. 12%
b. 11.3%
c. 8.8%
d. 9%
e. 10%
358. A nutritionist creates the following diet. Four days a week the
nutritionist eats three meals a day, starting with a 450-calorie
breakfast, and adding 100 calories for each subsequent meal. The
other three days the nutritionist eats five smaller meals, each
consisting of 300 calories. At the end of four weeks what is the
average caloric intake per meal?
a. 386
b. 411
c. 394
d. 425
e. 488
501_09_415-440_501_master.qxd 4/29/13 12:41 PM Page 429
501 GMAT
®
Questions
359.
A class is asked to record every book they read during their
summer break. Find the median of the summer reading data set.
Jennifer 2
Roman 14
Javier 21
Sophia 1
Sergio 16
Valerie 5
Lizzette 7
Danny 10
Freddy 9
Michelle 22
Julia 2
Victor 3
a. 8
b. 9
c. 9.3
d. 9.5
e. 10
Set 21
Now it is time to answer GMAT data sufficiency practice questions that
have been designed to test your arithmetic skills. Remember:
All numbers used are real numbers.
A figure accompanying a data sufficiency question will conform
to the information given in the question but will not
429
501_09_415-440_501_master.qxd 4/29/13 12:41 PM Page 430
430
501 GMAT
®
Questions
necessarily conform to the additional information given in
statements (I) and (II).
In data sufficiency problems that ask for the value of a quantity,
the data given in the statement is sufficient only when it is
possible to determine exactly one numerical value for the
quantity.
360. The approximate average temperature for the second week in
March was 75°.
II. The sum of the seven-day forecast was 525.
II. The average temperature from Sunday to Tuesday was 67°.
The rest of the week was 81°.
a. Statement (I) ALONE is sufficient, but statement (II) ALONE
is not sufficient.
b. Statement (II) ALONE is sufficient, but statement (I) ALONE
is not sufficient.
c. BOTH statements TOGETHER are sufficient, but NEITHER
statement ALONE is sufficient.
d. EACH statement ALONE is sufficient.
e. Statements (I) and (II) TOGETHER are NOT sufficient.
361. Is x divisible by five?
II. The sum of the digits is eight.
II. The first digit of x is 3.
a. Statement (I) ALONE is sufficient, but statement (II) ALONE
is not sufficient.
b. Statement (II) ALONE is sufficient, but statement (I) ALONE
is not sufficient.
c. BOTH statements TOGETHER are sufficient, but NEITHER
statement ALONE is sufficient.
d. EACH statement ALONE is sufficient.
e. Statements (I) and (II) TOGETHER are NOT sufficient.
501_09_415-440_501_master.qxd 4/29/13 12:41 PM Page 431
431
501 GMAT
®
Questions
362. Which company had a higher growth in net profit?
II. Company P’s net profits rose by 2.5%.
II. Company Q’s net profits rose by 3.7%.
a. Statement (I) ALONE is sufficient, but statement (II) ALONE
is not sufficient.
b. Statement (II) ALONE is sufficient, but statement (I) ALONE
is not sufficient.
c. BOTH statements TOGETHER are sufficient, but
NEITHER statement ALONE is sufficient.
d. EACH statement ALONE is sufficient.
e. Statements (I) and (II) TOGETHER are NOT sufficient.
363. How many female attendees at a 240-person concert bought a
band T-shirt before the concert?
II. 40% of the attendees were male.
II. 127 T-shirts were sold at the gift shop before the show
started.
a. Statement (I) ALONE is sufficient, but statement (II) ALONE
is not sufficient.
b. Statement (II) ALONE is sufficient, but statement (I) ALONE
is not sufficient.
c. BOTH statements TOGETHER are sufficient, but
NEITHER statement ALONE is sufficient.
d. EACH statement ALONE is sufficient.
e. Statements (I) and (II) TOGETHER are NOT sufficient.
364. A jar is filled with blue and green marbles. What is the probability
that two green marbles are drawn in a row?
II. There are 36 blue marbles in the jar.
II. The probability of choosing a green marble on the first try
is
__
1
4
.
a. Statement (I) ALONE is sufficient, but statement (II) ALONE
is not sufficient.
b. Statement (II) ALONE is sufficient, but statement (I) ALONE
is not sufficient.
c. BOTH statements TOGETHER are sufficient, but
NEITHER statement ALONE is sufficient.
d. EACH statement ALONE is sufficient.
e. Statements (I) and (II) TOGETHER are NOT sufficient.
432
501 GMAT
®
Questions
501_09_415-440_501_master.qxd 4/29/13 12:41 PM Page 432
365. How old is Roy if he is twice as old as his friend Jane, Jane is four
times as old as Timmy, and Timmy is two year older than Ben?
II. The sum of all their ages is 152.
II. Ben is 9.
a. Statement (I) ALONE is sufficient, but statement (II) ALONE
is not sufficient.
b. Statement (II) ALONE is sufficient, but statement (I) ALONE
is not sufficient.
c. BOTH statements TOGETHER are sufficient, but
NEITHER statement ALONE is sufficient.
d. EACH statement ALONE is sufficient.
e. Statements (I) and (II) TOGETHER are NOT sufficient.
366. Annie finished her race before Alexi.
II. Annie ran the first 4 minutes at a 4 mph pace, and finished
the last 8 miles of the race in 60 minutes.
II. Alexi’s top speed was 5.5 mph.
a. Statement (I) ALONE is sufficient, but statement (II) ALONE
is not sufficient.
b. Statement (II) ALONE is sufficient, but statement (I) ALONE
is not sufficient.
c. BOTH statements TOGETHER are sufficient, but
NEITHER statement ALONE is sufficient.
d. EACH statement ALONE is sufficient.
e. Statements (I) and (II) TOGETHER are NOT sufficient.
501_09_415-440_501_master.qxd 4/29/13 12:41 PM Page 433
433
501 GMAT
®
Questions
Answers
Set 14
329. 0. Using the order of operations, first you divide. 5 + 10 ÷ 5 – 7
becomes 5 + 2 – 7. Next, you add/subtract in order from left to
right: 5 + 2 – 7 becomes 7 – 7 = 0.
330. –10. Using the order of operations, first you multiply: 2 – 6 ¥ 4 ÷ 2
becomes 2 – 24 ÷ 2. Next, you divide: 2 – 24 ÷ 2 becomes 2 – 12,
which equals –10.
331. 27. Using the order of operations, first you divide/multiply in
order from left to right: 9 ÷ 3 + 3 ¥ 8 becomes 3 + 3 ¥ 8. Next, you
multiply: 3 + 3 ¥ 8 becomes 3 + 24, which equals 27.
Set 15
332. |156| = 156
333. |–97| = 97
334. –|13| = –13
335. 74 + |–23| = 97
336. 35 – |–12| = 23
337. The absolute value symbol serves as a grouping symbol, and
grouping symbols are evaluated first: |–5 ¥ 3| = 15. Now, divide 5
into 100 to get 20. Finally, add 20 + 15 = 35.
338. –9 is equal to the opposite of the absolute value of 9, or –9: 5 ¥ –9
= –45.
339. –33 is equal to 33. What times 33 equals 99? Your answer should
be 3.
501_09_415-440_501_master.qxd 4/29/13 12:41 PM Page 434
434
501 GMAT
®
Questions
Set 16
340. 2 ¥ 2 ¥ 3 ¥ 5
341. The prime factorization of 20 is 2 ¥ 2 ¥ 5. The prime factorization
of 30 = 2 ¥ 3 ¥ 5. Because 2 and 5 are prime factors of both 20 and
30, the greatest common factor is 2 times 5, which is 10.
342. Factors of 63: 1, 3, 7, 9, 21, 63
Factors of 81: 1, 3, 9, 27, 81
All factors have been listed. The largest one in common between
63 and 81 is 9.
343. Multiples of 63: 63, 126, 189, 252, 315, 378, 441, 504, 567
Multiples of 81: 81, 162, 243, 324, 405, 486, 567
Multiples are listed until one is found in common. The least
common multiple of 63 and 81 is 567.
Set 17
3.
__
4
5
To convert an improper fraction to a mixed number, divide the
numerator by the denominator: 19 ÷ 5 = 3 with 4 left over. You
express the remainder as a fraction. The improper fraction has a
denominator of 5, so the remainder has a denominator of 5:
19 ÷ 5 = 3
__
4
5
.
344.
53
__
8
. To convert a mixed number to an improper fraction, you begin
by multiplying the whole number, 6, by the denominator of the
fraction, 8: 6 ¥ 8 = 48. Next, add to that product the numerator of
the fraction: 48 + 5 = 53. Put this number over the original
denominator, 8, to get the improper fraction
53
__
8
.
345.
__
2
. First convert both fractions to the LCD (
__
4
5 3
=
20
__
15
). Subtract the
numerators and write the result over the denominator. Don’t
forget to reduce! (
20
__
14
__
=
__
6
15 15 15
, which reduces to
__
2
5
).
346.
501_09_415-440_501_master.qxd 4/29/13 12:41 PM Page 435
435
501 GMAT
®
Questions
Set 18
347. The correct answer is 0.008, 0.03. Since 0.008 has three decimal
digits, tack one zero onto the end of 0.03, making it 0.030. To
compare 0.008 to 0.030, just compare 30 to 8. 30 is larger than 8,
so 0.03 is larger than 0.008.
348. 5.1. Line up the numbers so that their decimal points are aligned.
Move the decimal point directly down into the answer and
subtract: 6.4 – 1.3 = 5.1.
349. 12.4. Multiply 31 by 4: 31 ¥ 4 = 124. The answer requires 1
decimal digit because there is a total of 1 decimal digit in 3.1 and 4.
Move the decimal point one place to the left (12.4).
Set 19
350. To find the mean, add all the data values, and divide by the number
of items, which is eight: 32 + 34 + 34 + 35 + 37 + 38 + 34 + 42 =
286; 286 divided by 8 is 35.75.
351. There are two modes for this data set. Both 71 and 68 appear in
the set twice.
352. First, arrange the data into increasing order: 8, 9, 9, 9, 10, 10, 11,
12, 13, 17. There is an even number of data values, so the median
is the mean of the two middle values. The middle values are
10 + 10 = 20, and 20 divided by 2 is 10.
353. The range is the difference between the highest and lowest values
in the set of data. The highest temperature is 84° and the lowest
temperature is 42°: 84° – 42° = 42°.
501_09_415-440_501_master.qxd 4/29/13 12:41 PM Page 436
436
501 GMAT
®
Questions
Set 20
354. a. All fractions can be converted to decimals to verify whether
they are in the correct order form least to greatest.
__
1
3
= 0.33
__
3
8
= 0.375
__
2
5
= 0.4
__
1
2
= 0.5
__
4
6
= 0.67
__
5
7
= 0.71
__
3
4
= 0.75
Choice b is incorrect as it lists the fractions from least to greatest
according to denominator. Choice c is incorrect as it lists the
fractions from least to greatest according to numerator. Choice d is
incorrect as it mixes up the last two fractions,
__
5
and
__
3
7 4
.
355. b. Starting with the 5 kg product, you can work backward to find
the metal alloy (m) middle product and the original total of iron
ore (t).
5 kg = 0.30(m)
m = 16.67 kg
t – 0.015t = m
t – 0.015t = 16.67 kg
0.985t = 16.67 kg
t = 16.9 kg
The rest of the answers are incorrect because they do not set the
equations up correctly (0.015t = 16.66) or they use the wrong
percentile decimal (0.15 instead of 0.015)
501_09_415-440_501_master.qxd 4/29/13 12:41 PM Page 437
437
501 GMAT
®
Questions
356. b. To solve this problem the following equations should be set up
using D for Democrat, R for Republican.
D + R = 435
__
2
3
R + 20 = D
Then substitute the second equation into the first equation.
(
__
2
3
R + 20) + R = 435
__
5
3
R = 415
R = 249
Then substitute R into the first equation to solve for D.
D + (249) = 435
D = 186
Choice a is the number of the Republican seats, not Democrat
seats. Choices c and d are the respective Democrat and Republican
seats if you set your equation up incorrectly and subtracted 20, (
__
2
3
R – 20), instead of adding.
357. a. To properly solve this problem you would need to set up an
equation, using p for percent profit.
7.50 – 7.50(p) = 6.60
–7.5( p) = .9
p = –.12 or –12%
This represents a 12% profit decrease. Choice b is the profit
calculated using this incorrect equation: 7.5 ÷ 6.5 = p.
Choice c is the profit calculated using this incorrect equation:
6.5 ÷ 7.5 = p.
Choice d is incorrect because you just subtracted 7.5 – 6.6 = 0.9.
501_09_415-440_501_master.qxd 4/29/13 12:41 PM Page 438
438
501 GMAT
®
Questions
358. b. To find the average caloric intake per meal during these four
weeks, you have to find the average during a one week cycle.
(450 + 550 + 650) = 1,650 ¥ 4 days = 6,600 (3 meals a day for
4 days = 12 total meals per week)
(300 + 300 + 300 + 300 + 300) = 1,500 ¥ 3 days = 4,500 (5 meals
a day for 3 days = 15 total meals per week)
6,600 + 4,500 = 11,100 total calories at the end of a week
______
11,100
27
= 411 calories per meal
Choice c is incorrect because is it only the average of the two
different meal plans.
(450 + 550 + 650 + 5(300)) ÷ 8 meals = 394
Choice d is incorrect because it is the average of each day averaged
together:
(550 + 300) ÷ 2 = 425
359. a. The median is the number in the middle of the series when the
numbers are arranged in numerical order.
1, 2, 2, 3, 5, 7, 9, 10, 14, 16, 21, 22
Since there are an even amount of numbers, to find the median
you must find the mean of the middle two numbers. The mean of
7 and 9 is 8. Choice b is incorrect because it means you did not
include the second 2 in your series. Choice c is incorrect as it is the
mean of the series. Choice d is incorrect as it is the mean of the
series without including the double 2. Choice e is incorrect
because you forgot to list 2 twice when ordering the numbers from
least to greatest.
Set 21
360. d. Either statement will produce an average temperature of 75°.
For the first statement, if the sum of all seven days is 525, you can
divide by 7 (total days in the week) to find the average of 75°. For
statement (2), you can sum the first three days (Sunday–Tuesday)
and get 201 (67 ¥ 3 = 201) , and then sum the last four days
(Wednesday–Saturday) and get 324 (81 × 4 = 324), add those
together, and divide by 7 to find an average of 75° (201 + 324 =
525 ÷ 7 = 75).
501_09_415-440_501_master.qxd 4/29/13 12:41 PM Page 439
439
501 GMAT
®
Questions
361. c. Both statements are needed together because if x is divisible by
5, the second digit must be either a zero or a 5. If the sum of the
digits is 8, the only combination that would make x divisible by 5
would be 3 and 5 or 8 and 0. The second statement confirms that
the first digit is 3, meaning the second digit has to be 5, so they can
sum to 8.
362. e. It does not matter how much the profits rose if we do not know
the actual profits. 2.5% of 1,000 would be larger than 3.7% of 100.
363. e. Neither statement is sufficient because they do not state how
many females bought T-shirts. The first statement allows you to
determine how many females are present at the concert. The
second statement tells you how many total T-shirts were bought,
but it cannot be assumed that equal ratios of males and females
bought shirts.
364. c. In order to determine the probability, you would need to know
both the total number of marbles and the ratio of blue to green
marbles. Using the information from both statements you can
determine both the total number of marbles and the specific
number of each. If there are 36 blue marbles, and a
__
1
4
chance of
picking a green marble, that means there are 9 green marbles.
(
__
1
4
=
green marbles
=
__
9
____________
blue marbles
36
)
501_09_415-440_501_master.qxd 4/29/13 12:41 PM Page 440
440
501 GMAT
®
Questions
365. d. You can use either statement independently to determine the
age of Roy. To use either statement you would have to set up
algebraic expressions for each variable.
R = 2J
J = 4T
T = B + 2
Given statement (I), you know the sum of all ages,
R + J + T + B = 152
but in order to solve you would have to put the expression in terms
of one variable, say, B:
2(4(B + 2)) + 4(B + 2) + (B + 2) + B = 152, then solve for B:
8B + 16 + 4B + 8 + B + 2 + B = 152
14B + 26 = 152
14B = 126
B = 9
Then, using that, or the information given in statement (II), plug B
back into the original expressions to solve for T, J, and R.
T = (9) + 2
T = 11
J = 4(11)
J = 44
R = 2(44)
R = 88
Roy is 88 years old.
366. c. To determine who won the race, both statements are needed for
comparison. The first statement allows you to calculate Annie’s
average speed. If she ran the first 4 miles at 4 miles per hour, the
last 8 miles in 60 minutes, she ran a total of 12 miles in 120
minutes (or 2 hours).
speed = distance ÷ time
Annie’s speed = 12 miles ÷ 2 hours = 6 miles per hour.
The second statement tells you that Alexi’s fastest pace was 5.5
miles per hour, meaning that could be his highest average speed, if
he ran the entire race at his top speed. With both statements we
can compare the two speeds and confirm that Annie did finish the
race faster if she ran at a higher average speed.
501_10_441-472_501_master.qxd 4/29/13 12:41 PM Page 441
10
Algebra
Algebra questions account for about 25% of the GMAT quantita-
tive section. This chapter reviews key algebra concepts, including:
sequences
exponents
roots
expressions
equations
word problems
inequalities
Sequences
Arithmetic sequences grow by adding or subtracting a constant number to
each term. For example, look at the following series:
9, 13, 17, 21, . . .
501_10_441-472_501_master.qxd 4/29/13 12:41 PM Page 442
442
501 GMAT
®
Questions
Notice that each term is four more than the term that comes before it. This
is an example of an arithmetic series with a common difference of 4. What
will the next term be?
To solve, add 4 to 21:
21 + 4 = 25
Thus, the next term will be 25.
Geometric sequences grow by multiplying each term by a constant num-
ber to get the next term. For example, look at the following series:
5, 25, 125, 625, . . .
Note that each term is five times the prior term. This is an example of a
geometric series with a common ratio of 5. What will the next term be?
Multiply 625 by 5:
625 ¥ 5 = 3,125
Thus, the next term will be 3,125.
Set 22
Find the missing numbers in the following sequences.
367. What is the next number in the following sequence?
3, 16, 6, 12, 12, 8, __
368. What number should come next in the following sequence?
0.2,
__
1
, 0.4,
__
2
, 0.8,
__
4
5 5 5
Exponents
An exponent tells you how many times the base is used as a factor. In the
expression 4
3
, the three is the exponent and the four is the base. The expres-
sion 4
3
shows that four is a factor three times. That means four times four
times four.
501_10_441-472_501_master.qxd 4/29/13 12:41 PM Page 443
443
501 GMAT
®
Questions
When you are multiplying two identical bases, you add the exponents
instead of multiplying them. For example:
x
2
· x
3
=
(x · x)(x · x · x) =
x
5
Set 23
Solve the following problems.
369. 5
2
=
370. 2
3
=
371. a
3
a
4
=
Roots
Yo u k n o w y o u n e e d t o t a k e t h e s q u a r e r o o t o f a n u m b e r w h e n y o u s e e a r a d -
ical sign, which looks like this .
To find the square root of a number, ask yourself: What number when
squared will equal the number under the radical sign?
For example, you know that 2
2
= 4, so
4
= 2. Square roots are relatively
easy to calculate for perfect squares, such as:
4
= 2
9
= 3
16
= 4
25
= 5
36
= 6
Other times, you can approximate the value of a radical by finding out
which two perfect squares it falls between. For example, since
25
= 5 and
36
= 6,
30
must be a number between 5 and 6.
501_10_441-472_501_master.qxd 4/29/13 12:41 PM Page 444
You know that 3 ¥ 3 = 9, so
9
= 3. But, –3 ¥ –3 = 9 as well.
9
has two
solutions: 3 and –3.
However, for the most part, you will be solving for the principle square
root, which will be the positive root.
If a problem is looking for both solutions, the problem will be written
like this: ±
9
.
You can multiply and divide the square roots of different numbers, but
you cannot add or subtract them.
a + b a + b
a b a b
a × b a × b
a a
=
b
b
If you are dealing with the root of the same number, you can combine the
like terms:
a + a = 2 a
4 a
= 2 a
2 a
Set 24
Solve the following problems.
372.
12 × 12 =
1
373.
=
4
374.
9+ 1 6=
375.
91+ 6=
444
501 GMAT
®
Questions
501_10_441-472_501_master.qxd 4/29/13 12:41 PM Page 445
445
Algebraic Expressions
In algebra, letters are often used to represent numbers. These letters are
called variables. The numbers in front of the variable or variables are called
coefficients. Remember:
A coefficient is a factor in an algebraic term, as are the variable
or variables in the term.
Like terms can have different coefficients, but the
configuration of the variables must be the same for the terms
to be alike. For example, 3x and –4x are like terms, but they
are different from 7ax.
Mathematical expressions, like numbers, can be named in different ways.
For example, here are three ways to write the same expression:
x + –3
x + (–3)
x – 3
For purposes of combining like terms, a variable by itself is understood to
mean one of that term:
n = +1n
A term without a sign in front of it is considered to be positive.
To simplify expressions, clear any parentheses, combine like terms by
adding coefficients, and then combine the constants.
When you are asked to evaluate an algebraic expression, you substitute
a number in place of a variable (letter) and then simplify the expression.
Set 25
Simplify the following expressions by combining like terms.
376. 5a + 2a + 7a
501 GMAT
®
Questions
501_10_441-472_501_master.qxd 4/29/13 12:41 PM Page 446
377. 7a + 6b + 3a
378. 4x + 2y x + 3y
379. 27 – 3m + 12 – 5m
380. 7h + 6 + 2w – 3 + h
Simplify the following expressions by combining like terms.
381. Evaluate the expression 2b + a when a = 2 and b = 4.
382. Evaluate the expression a
2
+ 2b + c when a = 2, b = 3, and b = 7.
Equations
An equation is a mathematical tool that helps people solve many real-life
problems. The word equation means two equal expressions. These expressions
could be numbers, such as 6 = 5 + 1, or variables, such as D = rt.
What does it mean to solve an equation? When you find the value of the
variable, you have solved the equation. For example, you have solved the
equation 2x = 10 once you know the value of x.
Here’s the basic rule for solving equations: When you do something to one
side of an equation, you must do the same thing to the other side of the equa-
tion. Yo u ’ l l k n o w y o u h a v e s o lv e d a n e q u a ti o n o n c e th e v a r i a b l e i s a l o n e ( i so -
lated) on one side of the equation and the variable is positive. In the example
x = 5, 5 is not the answer because the x variable is negative. Remember that
x is the same as –1x. To make the variable positive, divide both sides by –1.
Therefore, x = –5.
Solving Equations Using Addition or Subtraction
You can solve equations using addition and subtraction by getting the vari-
able on one side by itself. Think about how you would solve this equation:
x + 4 = 10
446
501 GMAT
®
Questions
501_10_441-472_501_master.qxd 4/29/13 12:41 PM Page 447
447
Your goal is to get the variable x on one side by itself. To get x by itself, you
need to get rid of the 4. If you subtract 4 from 4, the result is 0, and you
have eliminated the 4 to get x on one side by itself. However, if you subtract
4 from the left side of the equation, then you must do the same to the right
side of the equation.
Subtract 4 from both sides of the equation: x + 4 – 4 = 10 – 4
Simplify both sides of the equation: x + 0 = 6
Add 0 to x: x = 6
When you add zero to a number, the number does not change, so
x + 0 = x. When you do this, you are using the additive property of zero,
which states that a number added to zero equals that number. For example,
5 + 0 = 5.
Set 26
Simplify both sides of the following equations.
383. x – 5 = 9
384. a + 6 = 7
385. y – 11 = 8
386. r + 9 = 13
387. d – 7 = 8
388. x – 5 = –6
Solving Equations Using Multiplication or Division
In the equation x + 10 = 2, to get rid of the 10, you would subtract 10. In
the equation x 5 = 6, to get rid of the 5, you would add 5. So, in the equa-
tion 5x = 10, how would you get rid of the 5?
501 GMAT
®
Questions
501_10_441-472_501_master.qxd 4/29/13 12:41 PM Page 448
448
501 GMAT
®
Questions
The opposite of multiplying by 5 is dividing by 5; therefore, you would
solve this equation using division:
5x = 10
5
__
x
5
=
10
__
5
x = 2
When you divide a number by itself, you always get 1, so 5x divided by 5
equals 1x. Remember, 1x is the same as x.
In the equation
__
x
5
= 2, how would you get rid of the 5 so the x will be on
one side by itself? If you multiply
__
x
5
times 5, you will get 1x, which is the
same as x.
Set 27
Solve the following equations.
__
x
389.
2
= 13
__
x
390.
5
= 3
Polynomials
A polynomial is a number, a variable, or a combination of a number and a
variable. A polynomial can be one or more terms. Monomials, binomials,
and trinomials are all polynomials.
A polynomial with one term is called a monomial. To multiply a polyno-
mial with one term (monomial) by a polynomial with more than one term,
use the distributive property. You multiply the term outside the parenthe-
ses by every term inside the parentheses. For example:
2(a + b – 3) = 2a + 2b – 6
3x(x
2
+ 2x) = 3x
3
+ 6x
2
501_10_441-472_501_master.qxd 4/29/13 12:41 PM Page 449
449
A binomial is a polynomial with two terms. To multiply a binomial by a
binomial, you will use a method called FOIL. This process is called FOIL
because you work the problem in this order:
First
Outer
Inner
Last
Example: (x + 2)(x + 3)
Multiply the first terms in each binomial: ([x] + 2)([x] + 3) = x
2
Multiply the two outer terms in each binomial: ([x] + 2)(x + [3]) =
x
2
+ 3x
Multiply the two inner terms in each binomial: (x + [2])([x] + 3) =
x
2
+ 3x + 2x
Multiply the two last terms in each binomial: (x + [2])(x + [3]) =
x
2
+ 3x + 2x + 6
Simplify: = x
2
+ 5x + 6
A bicycle has two wheels; a tricycle has three wheels. Likewise, a bino-
mial has two terms, and a trinomial has three terms. Here’s how you would
multiply a binomial by a trinomial.
(x + 2)( x
2
+ 2x + 1)
To work this problem, you need to multiply each term in the first polyno-
mial with each term in the second polynomial. You are doing exactly that
when you use FOIL.
Multiply x by each term in the second polynomial: x(x
2
+ 2x + 1) =
x
3
+ 2x
2
+ x
Multiply 2 by each term in the second polynomial: 2(x
2
+ 2x + 1) =
2x
2
+ 4x + 2
Simplify: x
3
+ 2x
2
+ x + 2x
2
+ 4x + 2 = x
3
+ 4x
2
+ 5x + 2
Formulas
Formulas are special equations that show relationships between quantities.
For example, you have probably worked with the formula A = lw. This for-
mula tells you to multiply the length times the width of a rectangle to find
501 GMAT
®
Questions
501_10_441-472_501_master.qxd 4/29/13 12:41 PM Page 450
450
501 GMAT
®
Questions
its area. The formula D = rt tells you to multiply the rate by the time to find
the distance traveled.
When you substitute the information you know into a formula, you can
use that to find the information you don’t know. For example, if you travel
55 miles per hour (mph) for 3 hours, how far would you travel? Substitute
what you know into the equation. Then, solve the equation for the variable
you don’t know.
Substitute what you know into the formula: D = 55 ¥ 3
Multiply: D = 165
You would travel 165 miles.
This technique works for any formula, even if the formula is very complex.
Example: Find the interest on a savings account with a balance of
$2,400, when the interest rate is 3% for 3 years. Use the for-
mula I = prt.
I = interest earned
p = amount of money invested
r = interest rate
t = time invested
I = prt
Substitute what you know into the formula:
I = $2,400 ¥ 3% ¥ 3 years
I = 2,400 ¥ .03 ¥ 3
Simplify the equation:
I = $216
Set 28
Answer the following problems by substituting the information that you
know into the formulas.
391. How long would you need to invest $3,000 with an interest rate of
3.5% to earn $630? (Use I = prt)
501_10_441-472_501_master.qxd 4/29/13 12:41 PM Page 451
451
392. How long would it take to travel 300 miles traveling at a speed of
60 mph? (Use D = rt)
Setting Up Equations for Real-World Problems
Equations can be used to solve real-life problems. In an equation, the vari-
able often represents the answer to a real-life problem. For example, sup-
pose you know that you can earn twice as much money this summer as you
did last summer. You made $1,200 last summer. How much will you earn
this summer? You can use the variable x to represent the answer to the
problem. You want to know how much you can earn this summer.
Let x = how much you can earn this summer.
$1,200 = amount earned last summer
x = 2 ¥ $1,200
x = $2,400
Set 29
Answer the following problems.
393. There are twice as many women in your yoga class as there are
men. If there are 18 women in the class, how many men are in the
class?
394. You are going to be working for Success Corporation. You got a
signing bonus of $2,500, and you will be paid $17.50 an hour. If
you are paid monthly, how much will your first paycheck be? Be
sure to include your signing bonus, and assume that you have a 40-
hour work week and that there are 4 weeks in this month.
395. Grace sells real estate and receives a 6% commission for every
residential property that she sells. How much commission would
she receive if she sold a home for $175,000?
396. There are 40 questions on a test. How many questions must you
answer correctly to score 90%?
501 GMAT
®
Questions
501_10_441-472_501_master.qxd 4/29/13 12:41 PM Page 452
Inequalities
An inequality is two numbers or expressions that are connected with an
inequality symbol. The inequality symbols are
< (less than)
> (greater than)
<
(less than or equal to)
(greater than or equal to)
=
/ (not equal to)
Here are some examples of inequalities:
2 < 5 (two is less than five)
9 > 3 (nine is greater than three)
4 <
4 (four is less than or equal to four)
2x + 5
=
/ 11 (2x added to five is not equal to eleven)
You can solve inequalities with variables just like you can solve equations
with variables. Use what you already know about solving equations to solve
inequalities. Like equations, you can add, subtract, multiply, or divide both
sides of an inequality with the same number. In other words, what you do
to one side of an inequality, you must do to the other side.
Example: 2x + 3 < 1
Subtract 3 from both sides of the inequality:
2x + 3 – 3 < 1 – 3
Simplify both sides of the inequality:
2x < –2
Divide both sides of the inequality by 2:
2
__
x
2
<
–2
__
2
452
501 GMAT
®
Questions
501_10_441-472_501_master.qxd 4/29/13 12:41 PM Page 453
453
Simplify both sides of the inequality:
x < –1
The answer for this example is the inequality x < –1. There is an infinite
(endless) number of solutions because every number less than –1 is an
answer. In this problem, the number –1 is not an answer because the
inequality states that your answers must be numbers less than –1.
Did you notice the similarity between solving equations and solving
inequalities? You can see that the previous example was solved using the
same steps you would use if you were solving an equation.
However, there are some differences between solving equations and solv-
ing inequalities. Notice what happens when you multiply or divide an
inequality by a negative number.
2 < 5
–2 ¥ 2 < 5 ¥ –2
–4 < –10
However, –4 is not less than –10. So –4 < –10 is a false statement. To cor-
rect it, you would have to rewrite it as –4 > –10.
You can solve inequalities using the same methods that you use to solve
equations with these exceptions:
When you multiply or divide an inequality by a negative
number, you must reverse the inequality symbol.
The answer to an inequality will always be an inequality.
Set 30
Solve each inequality.
397. x + 3 < 10
398. 2x + 5 < 7
399. –3x < 9
501 GMAT
®
Questions
501_10_441-472_501_master.qxd 4/29/13 12:41 PM Page 454
454
501 GMAT
®
Questions
400. 5x + 1 < 11
401. You are treating a friend to a movie. You will buy 2 tickets and
spend $8 on concessions. If you don’t want to spend more than
$20, how much can you spend on each ticket?
402. You are pricing lawn furniture and plan to buy 4 chairs. You don’t
want to spend more than $120. What is the most you can spend on
one chair?
403. You are going to a restaurant for lunch. You have $15 to spend.
Your beverage is $2.50, and you will leave a $2 tip. How much can
you spend on the entrée?
404. You own a gift shop. You bought a case of candles for $144. There
are 12 candles in a case. What do you need to sell each candle for
to make a profit?
Set 31
Now it is time to answer GMAT problem-solving practice questions that
have been designed to test your algebra skills. Remember:
Solve the problem and indicate the best of the answer choices
given.
All numbers used are real numbers.
A figure accompanying a problem-solving question is intended
to provide information useful in solving the problem. Figures
are drawn as accurately as possible EXCEPT when it is stated
in a specific problem that the figure is not drawn to scale.
Straight lines may sometimes appear jagged. All figures lie in a
plane unless otherwise indicated.
501_10_441-472_501_master.qxd 4/29/13 12:41 PM Page 455
455
501 GMAT
®
Questions
405. Simplify
___________
x
2
+ 7x + 10
_______
x
2
– 25
3x
5
+ 6x
4
÷
15x
8
a.
_________
15x
2
3x
4
(x – 5)
_____
5x
4
b.
x + 5
_____
–5x
4
c.
x –5
d. 5x
4
(x – 5)
____
5x
4
e.
x –5
406. Factor the trinomial 4x
2
–9x –9
a. (4x + 3)(x + 3)
b. (4x + 3)(x – 3)
c. (8x – 6)(x + 3)
d. (4x + 3)(3x + 1)
e. (2x – 3)(2x – 3)
18x
4
y
10
z
6
407. Simplify the expression
________
20x
11
y
3
z
9
7
z
5
a.
_____
y
10x
7
_______
9y
7
x
7
z
5
b.
10
5
c.
___
z
2
y
7
z
5
d.
____
2x
7
9x
7
e.
_____
10y
7
z
5
408. Multiply (x + 3)
2
a. x
2
+ 9x + 9
b. x
2
+ 6x –9
c. x
2
–9
d. x
2
+ 6x + 9
e. x
2
+ 6
409. |3x + 9| < 15
a. x < –2 or x > 8
b. x > –2 and x < 8
c. x < 2 and x > –8
d. x > 2 or x < –8
e. none of the above
501_10_441-472_501_master.qxd 4/29/13 12:41 PM Page 456
456
501 GMAT
®
Questions
410.
3
8+ 8 1
=
a. 13
b. 89
c. 17
d. 33
e. 11
411. Annie has a total of 100 dimes and quarters. If the total value of
the coins is $14.05, how many quarters does she have?
a. 3
b. 40
c. 56
d. 73
e. 27
412. Which equation is part of the process of completing the square for
x
2
– 8x + 4 = 49?
a. (x + 8)
2
= 11
b. (x – 8)
2
= 67
c. (x – 4)
2
= 61
d. (x + 4)
2
= 19
e. (x + 2)
2
= 11
413. The sum of x
2
and 7x is –10. What is one possible answer for x?
a. x = –2
b. x = 5
c. x = 2
d. x = 10
e. x = –7
414. In the following sequence, each term is 6 greater than the previous
term. Which of the following could NOT be a term in the
sequence?
–36, –30, –24, –18, . . .
a. –492
b. 256
c. 344
d. 510
e. 678
501_10_441-472_501_master.qxd 4/29/13 12:41 PM Page 457
457
415. (37)
3
a. 50,653
b. 49,453
c. 50,427
d. 49,867
e. 50,238
Set 32
Now it is time to answer GMAT data sufficiency practice questions that
have been designed to test your algebra skills. Good luck!
416. Is x > y?
II. x and y are both positive integers.
II. x ÷ y > 1
a. Statement (I) ALONE is sufficient, but statement (II) ALONE
is not sufficient.
b. Statement (II) ALONE is sufficient, but statement (I) ALONE
is not sufficient.
c. BOTH statements TOGETHER are sufficient, but NEITHER
statement ALONE is sufficient.
d. EACH statement ALONE is sufficient.
e. Statements (I) and (II) TOGETHER are NOT sufficient.
417. In a sequence, what is the fifth term?
II. The sum of the first term and fourth term is equal to the
fifth term.
II. All numbers are multiples of the first term.
a. Statement (I) ALONE is sufficient, but statement (II) ALONE
is not sufficient.
b. Statement (II) ALONE is sufficient, but statement (I) ALONE
is not sufficient.
c. BOTH statements TOGETHER are sufficient, but NEITHER
statement ALONE is sufficient.
d. EACH statement ALONE is sufficient.
e. Statements (I) and (II) TOGETHER are NOT sufficient.
501 GMAT
®
Questions
501_10_441-472_501_master.qxd 4/29/13 12:41 PM Page 458
458
501 GMAT
®
Questions
418. Are x and y positive integers?
II. xy is positive.
II. x
2
and y
2
are both positive.
a. Statement (I) ALONE is sufficient, but statement (II) ALONE
is not sufficient.
b. Statement (II) ALONE is sufficient, but statement (I) ALONE
is not sufficient.
c. BOTH statements TOGETHER are sufficient, but NEITHER
statement ALONE is sufficient.
d. EACH statement ALONE is sufficient.
e. Statements (I) and (II) TOGETHER are NOT sufficient.
419. Annie and Alexi are running a race. When the race starts Alexi
starts running 9 ft. per second. Annie starts 10 seconds later
because she forgot to stretch. She is running at a pace of 10 ft. per
second. If they keep going at the same pace, who will win?
II. Their distance after 20 seconds is equal.
II. It is a 500 ft. race.
a. Statement (I) ALONE is sufficient, but statement (II) ALONE
is not sufficient.
b. Statement (II) ALONE is sufficient, but statement (I) ALONE
is not sufficient.
c. BOTH statements TOGETHER are sufficient, but NEITHER
statement ALONE is sufficient.
d. EACH statement ALONE is sufficient.
e. Statements (I) and (II) TOGETHER are NOT sufficient.
420. A ball is dropped off a balcony 49 ft. high. How long will it take to
hit the ground?
II. The acceleration of the ball toward the ground is –9.8 m/s.
II. The ball weighs 3 lbs.
a. Statement (I) ALONE is sufficient, but statement (II) ALONE
is not sufficient.
b. Statement (II) ALONE is sufficient, but statement (I) ALONE
is not sufficient.
c. BOTH statements TOGETHER are sufficient, but NEITHER
statement ALONE is sufficient.
d. EACH statement ALONE is sufficient.
e. Statements (I) and (II) TOGETHER are NOT sufficient.
501_10_441-472_501_master.qxd 4/29/13 12:41 PM Page 459
459
501 GMAT
®
Questions
421. Are x and y consecutive integers?
II. x + y = 15
II. x y = 3
a. Statement (I) ALONE is sufficient, but statement (II) ALONE
is not sufficient.
b. Statement (II) ALONE is sufficient, but statement (I) ALONE
is not sufficient.
c. BOTH statements TOGETHER are sufficient, but NEITHER
statement ALONE is sufficient.
d. EACH statement ALONE is sufficient.
e. Statements (I) and (II) TOGETHER are NOT sufficient.
422. Find the values of x and y.
II. 3x + 2 = y – 7
II. 4x – 12y = 36
a. Statement (I) ALONE is sufficient, but statement (II) ALONE
is not sufficient.
b. Statement (II) ALONE is sufficient, but statement (I) ALONE
is not sufficient.
c. BOTH statements TOGETHER are sufficient, but
NEITHER statement ALONE is sufficient.
d. EACH statement ALONE is sufficient.
e. Statements (I) and (II) TOGETHER are NOT sufficient.
423. The sum of the polynomials is 4x + 6. What are the two
polynomials?
II. The difference of the two is 0.
II. The product of the two polynomials is 4x
2
+ 12x + 9.
a. Statement (I) ALONE is sufficient, but statement (II) ALONE
is not sufficient.
b. Statement (II) ALONE is sufficient, but statement (I) ALONE
is not sufficient.
c. BOTH statements TOGETHER are sufficient, but NEITHER
statement ALONE is sufficient.
d. EACH statement ALONE is sufficient.
e. Statements (I) and (II) TOGETHER are NOT sufficient.
501_10_441-472_501_master.qxd 4/29/13 12:41 PM Page 460
460
501 GMAT
®
Questions
424.
Eric was buying cookies and Pop-Tarts. He bought 5 cookies and
10 Pop–Tarts for $12.50. How much were the cookies?
II. Pop-Tarts are $1.00.
II. You can buy 20 cookies and 20 Pop-Tarts for $30.00.
a. Statement (I) ALONE is sufficient, but statement (II) ALONE
is not sufficient.
b. Statement (II) ALONE is sufficient, but statement (I) ALONE
is not sufficient.
c. BOTH statements TOGETHER are sufficient, but NEITHER
statement ALONE is sufficient.
d. EACH statement ALONE is sufficient.
e. Statements (I) and (II) TOGETHER are NOT sufficient.
425. What is the value of x in the following expression?
_______
4a
3
b
4
c
x
12ab
7
c
12
II. The simplified expression has a coefficient of 3 in the
denominator.
II. The resulting exponent of c in the simplified answer is 9.
a. Statement (I) ALONE is sufficient, but statement (II) ALONE
is not sufficient.
b. Statement (II) ALONE is sufficient, but statement (I) ALONE
is not sufficient.
c. BOTH statements TOGETHER are sufficient, but NEITHER
statement ALONE is sufficient.
d. EACH statement ALONE is sufficient.
e. Statements (I) and (II) TOGETHER are NOT sufficient.
426. The expression 4x + 9 – 2(5x + 1) is equal to a number. What is the
number?
II. The number is 9 less than twice the value of x.
II. The number is an integer and is less than x.
a. Statement (I) ALONE is sufficient, but statement (II) ALONE
is not sufficient.
b. Statement (II) ALONE is sufficient, but statement (I) ALONE
is not sufficient.
c. BOTH statements TOGETHER are sufficient, but NEITHER
statement ALONE is sufficient.
d. EACH statement ALONE is sufficient.
e. Statements (I) and (II) TOGETHER are NOT sufficient.
461
501 GMAT
®
Questions
501_10_441-472_501_master.qxd 4/29/13 12:41 PM Page 461
427. Kline spent three hours on the phone with her brother. 20% of
that time was spent talking about work. The rest of the time was
spent talking about her boyfriend, weekend plans, and new kitten.
How much time did she spend talking about her kitten?
II. She spent twice as long talking about her boyfriend as on
work and her weekend plans combined.
II. She spent the same amount of time on the kitten as she did
on her weekend plans.
a. Statement (I) ALONE is sufficient, but statement (II) ALONE
is not sufficient.
b. Statement (II) ALONE is sufficient, but statement (I) ALONE
is not sufficient.
c. BOTH statements TOGETHER are sufficient, but NEITHER
statement ALONE is sufficient.
d. EACH statement ALONE is sufficient.
e. Statements (I) and (II) TOGETHER are NOT sufficient.
501_10_441-472_501_master.qxd 4/29/13 12:41 PM Page 462
462
501 GMAT
®
Questions
Answers
Set 22
367. This sequence actually has two alternating sets of numbers. The
first number is doubled, giving the third number. The second
number has 4 subtracted from it, giving the fourth number.
Therefore, the next number will be 12 doubled, or 24.
368. This is a multiplication sequence with repetition. The decimals
(0.2, 0.4, 0.8) are repeated by fractions with the same values
(
__
1
,
__
2
,
__
4
5 5 5
) and are then multiplied by 2. Thus, the next number
will be 0.8 ¥ 2, or 1.6.
Set 23
369. 5
2
= 5 · 5 = 25
370. 2
3
= 2 · 2 · 2 = 8
371. a
3
a
4
= (a · a · a)(a · a · a · a) = a
3+4
= a
7
Set 24
372. 12. Use the rule
a × b = a × b
to get
12 × 12 = 12 × 12 = 144
= 12.
__
1
a
373. . Use the rule
=
1
2
a
1
/ b
to get
=
1
__
=
2
.
b 4
4
374. 7. Solve and add:
9
+
16
= 3 + 4 = 7
375. 5. Add the numbers under the radical, then take the square
root:
91+ 6= 25
= 5
Set 25
376. Use the associative property of addition: (5a + 2a) + 7a
Add like terms: [5a + 2a = 7a]
Substitute the results into the original expression: (7a) + 7a
Add like terms: 7a + 7a = 14a
The simplified result of the algebraic expression is: 14a
501_10_441-472_501_master.qxd 4/29/13 12:41 PM Page 463
463
501 GMAT
®
Questions
377. Use the commutative property of addition to move like terms
together: 7a + 3a + 6b
Use the associative property for addition: (7a + 3a) + 6b
Add like terms: [(7a + 3a) = 10a]
Substitute: (10a) + 6b
The simplified result of the algebraic expression is: 10a + 6b
378. Change subtraction to addition and change the sign of the term
that follows: 4x + 2y + (–x) + 3y
Use the commutative property of addition to move like terms
together: 4x + (–x) + 2y + 3y
Use the associative property for addition: (4x + –x) + (2y + 3y)
Add like terms: [4x + –x = +3x = 3x]
[2y + 3y = 5y]
Substitute the results into the expression: (4x + –x) + (2y + 3y) =
(3x) + (5y)
The simplified algebraic expression is: 3x + 5y
379. Change subtraction to addition and change the sign of the term
that follows: 27 + –3m + 12 + –5m
Use the commutative property for addition to put like terms
together: 27 + 12 + –3m + –5m
Use the associative property for addition: (27 + 12) + (–3m + –5m)
Add like terms: [27 + 12 = 39]
[–3m + –5m = –8m]
Substitute the results into the expression: (27 + 12) + (–3m + –5m)
= (39) + (–8m)
Rewrite addition of a negative term as subtraction of a positive
term by changing addition to subtraction and changing the sign of
the following term: 39 – +8m = 39 – 8m
The simplified algebraic expression is: 39 – 8m
501_10_441-472_501_master.qxd 4/29/13 12:41 PM Page 464
464
501 GMAT
®
Questions
380. Change subtraction to addition and change the sign of the term
that follows: 7h + 6 + 2w + (–3) + h
Use the commutative property for addition to put like terms
together: 7h + h + 2w + 6 + –3
Use the associative property for addition: (7h + h) + 2w + (6 + –3)
Add like terms: [(7h + h) = 8h]
[(6 + –3) = 3]
Substitute the result into the expression: (8h) + 2w + (3)
The simplified algebraic expression is: 8h + 2w + 3
381. Substitute 2 for the variable a and 4 for the variable b. When the
expression is written as 2b, it means 2 times b: 2b + a
Multiply 2 · 4: 2(4) + 2
Add the numbers: 8 + 2
= 10
382. Substitute 2 for a, 3 for b, and 7 for c: a
2
+ 2b + c
Find the value of 2
2
: (2)
2
+ 2(3) + 7
Multiply 2 · 3: 4 + 2(3) + 7
Add the numbers: 4 + 6 + 7
= 17
Set 26
383. 14. Add 5 to both sides of the equation: x – 5 + 5 = 9 + 5
Simplify both sides of the equation: x + 0 = 14
Add 0 to x: x = 14
384. 1. Subtract 6 from both sides of the equation: a + 6 – 6 = 7 – 6
Simplify both sides of the equation: a + 0 = 1
Add 0 to a: a = 1
385. 19. Add 11 to both sides of the equation: y – 11 + 11 = 8 + 11
Simplify both sides of the equation: y + 0 = 19
Add 0 to y: y = 19
501_10_441-472_501_master.qxd 4/29/13 12:41 PM Page 465
465
501 GMAT
®
Questions
386. –4. Subtract 9 from both sides of the equation: –r + 9 – 9 = 13 – 9
Simplify both sides of the equation: –r + 0 = 4
Add 0 to –r: –r = 4
The value of the variable must be positive, but in this equation, r is
negative. You can make it positive by multiplying both sides of the
equation by –1.
(–1)(–r) = 4(–1)
r = –4
387. 15. Add 7 to both sides: d – 7 + 7 = 8 + 7
d = 15
388. –1. Add 5 to both sides: x – 5 + 5 = –6 + 5
x = –1
Set 27
389. 26. Multiply both sides by 2:
__
x
¥ 2 = 13 ¥
2
2, x = 26.
390. 15. Multiply both sides by 5:
__
x
5
¥ 5 = 3 ¥ 5, x = 15.
Set 28
391. 6 years. Substitute what you know into the formula: $630 = $3,000
¥ 3.5% ¥ t, or 630 = 3,000 ¥ .035t. Simplify the equation:
630 = 105t. Divide both sides of the equation by 105: t =
630
___
105
.
Simplify both sides of the equation: 6 = t. It would take 6 years to
earn $630.
392. 5 hours. Substitute what you know into the formula: 300 = 60t.
Divide both sides of the equation by 60: t =
300
___
60
. Simplify both
sides of the equation: 5 = t. It would take you 5 hours to travel
300 miles.
Set 29
393. 9. Let x = the number of men in the class. Then, 2 times the
number of men is equal to the number of women. You can
represent that in the equation 2x = 18. x = 9; therefore, there are
9 men in the class.
501_10_441-472_501_master.qxd 4/29/13 12:41 PM Page 466
466
501 GMAT
®
Questions
394.
$5,300. Let x = first monthly paycheck. Then, x = 4 ¥ 40 ¥ $17.50
+ $2,500; x = $5,300.
395. $10,500. Grace sells real estate and receives a 6% commission for
every residential property that she sells. To find how much
commission she would receive if she sold a home for $175,000,
label the commission with the variable c, and rewrite 6% as .06:
c = .06 ¥ $175,000; c = $10,500.
396. 36. There are 40 questions on a test. To find out many questions
you must answer correctly to score 90%, rewrite 90% as 90 out of
100 and set up a proportion to see how many points, p, out of 40
are needed:
___
90
=
p
__
. Cross multiply: 40 ¥ 9 = 10p; 360 = 10p.
100
40
Divide both sides by 10: p = 36.
Set 30
397. x < 7. Subtract 3 from both sides of the inequality.
398. x < 1. Subtract 5 from both sides of the inequality. Then, divide
both sides by 2.
399. x > –3. Divide both sides of the inequality by –3. Remember, when
you multiply or divide an inequality by a negative number, you
must reverse the inequality symbol.
400. x < 2. Subtract 1 from both sides of the inequality. Then, divide
both sides by 5.
401. x < $6. 2x + 8 < $20; x < $6. You can spend up to $6 on the tickets.
402. x < $30. 4x < 120; x < $30. The most you can spend on a chair is
$30.
403. x < $10.50. x + $2.50 + $2.00 < $15.00; x < $10.50. You can spend
up to $10.50 on the entrée.
404. x > $12. 12x > $144; x > $12. You need to sell each candle for
> $12.
501_10_441-472_501_master.qxd 4/29/13 12:41 PM Page 467
467
501 GMAT
®
Questions
Set 31
405. c. Start simplifying the problem by changing the second rational
expression to its reciprocal and changing the division symbol
to a multiplication symbol. Factor each numerator and
denominator. The factored form of the rational expression is
___________
(x + 2)(x + 5)
3x
4
(x + 5)
¥
___________
15x
8
(x + 5)(x – 5)
. The (x + 5)s and the (x + 2)s cancel.
Reduce the 15x
8
and 3x
4
by dividing the coefficients by 3 and
subtracting the exponents. Leave the x
4
in the numerator since the
exponent in the numerator is bigger. The other choices have
reducing errors, are still unsimplified, or do not have the correct
terms in the denominator.
406. b. Find the factors of 4x
2
and –9 that multiply and add to make
–9x. 4x and x are both factors of 4x
2
. 3 and –3 are factors of –9.
Testing them in the polynomials (4x + 3)(x – 3) will yield the
answer when multiplied together. The other choices include
switched signs and added values instead of multiplication.
407. a. Simplify the coefficients and subtract the exponents.
18
__
9
20
reduces to
__
10
, and when the powers of x are subtracted they
yield x
7
in the denominator. When the powers of y are subtracted
they yield y
7
in the numerator, and when the powers of z are
subtracted they yield z
5
in the numerator. The other choices
subtracted the coefficients, performed the wrong operation with
the exponents, or have incorrect placement of x in the numerator.
408. d. A squared binomial means you can write out the binomial
multiplied by itself (x + 3)(x + 3). When each term in the first set
of parentheses is multiplied by each term in the second set of
parentheses it yields the answer. x(x) + x(3) + 3(x) + 3(3) = x
2
+ 3x +
3x + 9. Combine like terms to get x
2
+ 6x + 9. The other choices
include squaring each term, multiplying when combining like
terms, or sign errors.
409. c. Start by splitting the absolute value portion into its two possible
outcomes 3x + 9 < 15 and 3x + 9 > –15. Solve each by subtracting 9
from both sides, yielding 3x < 6 and 3x > –24. In both inequalities,
divide by 3, resulting in x < 2 and x > –8. Incorrect choices include
reversed inequalities and incorrect signs.
501_10_441-472_501_master.qxd 4/29/13 12:41 PM Page 468
468
501 GMAT
®
Questions
410. e. The cube root of 8 is 2, and the square root of 81 is 9. 2 + 9 =
11. Choices a, b, and c are incorrect due to incorrect usage of
order of operations and incorrect use of a cubed root.
411. e. Start by setting up two equations. q + d = 100 refers to the
number of quarters added to the number of dimes equaling 100.
25q + 10d = 1,405 refers to the value of the quarters added to the
value of the dimes equaling their combined value of 1,405 cents.
Using the method of linear combination, or elimination, multiply
the second equation by –10 to cancel the ds. Then combine the
two equations –10q – 10d = –1,000 with 25q + 10d = 1,405. This
results is 15q = 405. Divide both sides by 15 to get q = 27. The
incorrect choices are incorrect estimations with guess and check,
and also the value of the dimes.
412. c. Start by subtracting on each side of the equation to isolate the
constants on the right. This will give you x
2
– 8x = 45. Next find
the value that will make x
2
– 8x a perfect square trinomial. Add
16 to both sides, yielding x
2
– 8x + 16 = 61. Factor the left into
(x – 4)(x – 4) and rewrite as a binomial squared. This gives you
(x – 4)
2
= 61. The incorrect choices contain incorrect factoring,
missing negative signs, or incorrect addition and subtraction.
413. a. Start by setting up the equation x
2
+ 7x = –10. Add 10 to both
sides to get x
2
+ 7x + 10 = 0. Factor the left side of the equation
(x + 5)(x + 2) = 0. Using the zero product property, you can assume
that x + 5 = 0 or x + 2 = 0. Solving both equations gives you the
solution set x = –5 and x = –2. The incorrect choices are the values
factored without using the zero product property and a number
seen initially in the problem.
414. c. Because each term in the sequence is 6 greater than the
previous term, every term, positive or negative, in the sequence
must be divisible by six. Choice c is the only solution that is not
evenly divisible by 6. (344 ÷ 6 = 57.333)
415. a. This problem is a lengthy multiplication problem: (37)
3
=
37 ¥ 37 ¥ 37 = 50,653. All the other choices are variations of
multiplication errors.
501_10_441-472_501_master.qxd 4/29/13 12:41 PM Page 469
469
501 GMAT
®
Questions
Set 32
416. b. Statement (II) is the only sufficient statement because if x ÷ y is
greater than 1, it means x must be a larger number that y is able to
go into it at least once. Statement (I) does not provide any
information regarding the relationship between x and y.
417. e. The statements do not provide enough information to
determine the numbers in a sequence. The first statement suggests
that the sequence is increasing by the value of the first term, but it
does not explicitly state the value of the first term. This pattern
would work for any sequence increasing by the first variable
(2, 4, 6, 8, (2 + 8 =10) 10) or (25, 50, 75, 100 (25 + 100 = 125) 125).
The second statement tells you the numbers are all multiples of
the first term but again does not explicitly state the value of the
first term. The previously mentioned sequences could work for
this pattern as well.
418. e. Neither of these statements can strictly determine whether both
x and y are positive. Given the first statement, x and y could either
both be positive, or both be negative. The same goes given the
second statement. Both statements can be deceiving if you forget
that the product of two negative numbers is a positive number.
419. b. If Annie runs at a rate of 9 fps with a 10-second penalty going a
distance of 500 feet, her distance equation will be 500 = 9(T + 10)
based on distance = speed(time). Alexi’s distance equation will be
500 = 10T. When both are solved, we find that Alexi’s time was
50 seconds and Annie’s time was 45, which means Annie won the
race. We cannot use the second piece of information because we
don’t know whether the race lasted for more or less than 2 minutes.
420. a. To find how long (or the time) it will take the ball to hit the
ground, you would need to know its speed. The first statement
gives you the acceleration, and from there you can find the time.
Speed = Distance ÷ Time
–9.8 m/s = 49m ÷ time
49 ÷ 9.8 = 5 seconds
The weight of the ball does not affect the answer, and the
acceleration already takes the weight into account. Therefore, only
the first statement is needed.
501_10_441-472_501_master.qxd 4/29/13 12:41 PM Page 470
470
501 GMAT
®
Questions
421.
b. Statement (I) is not enough because x and y might be 7 and 8.
Statement (II) is sufficient because if you subtract 2 consecutive
integers, the difference should be 1.
422. e. Both statements together are insufficient because if the
equations are manipulated, they become the same, and you cannot
solve for each variable. You end up with only trivial solutions,
which is to say x and y could be anything.
423. d. Statement (I) indicates that since the difference of the two
polynomials is 0, they are the same. So you could take the sum of
them and divide by 2
______
4x + 6
2
= 2x + 3. Statement (II) is sufficient
because the only trinomial that adds to 4x + 6 are the two
binomials (2x + 3)(2x + 3) you get from factoring 4x
2
+ 12x + 9.
424. d. The problem gives you enough to set up the following
equation, 5c + 10p = $12.50. The first statement gives you the value
of variable p, and from there you can solve for c.
5c + 10p = $12.50
5c + 10(1.00) = $12.50
5c + 10 = 12.50
5c = 2.50
c = 0.50
The second statement gives you enough information to set up a
second equation using both variables, p and c, and then you can use
elimination to solve for each variable.
5c + 10p = $12.50
20c + 20p = $30.00
–2(5c + 10p) = –2($12.50)
20c + 20p = $30.00
–10c – 20p = $–25.00
20c + 20p = $ 30.00
10c = $ 5.00
c = 0.50
Either statement (I) or statement (II) provides enough information
independently to solve for the price of the cookies.
501_10_441-472_501_master.qxd 4/29/13 12:41 PM Page 471
471
501 GMAT
®
Questions
425. e. Statement (I) is insufficient because as long as x > 1 there will
be a coefficient in the denominator of 3. Statement (II) is also
insufficient because the exponent of c
9
could be in the numerator
or the denominator, which leaves us two possibilities for the value
of x, 21 and 3.
426. a. Statement (I) provides a way to set up an algebraic expression
that is in terms of x. Replace the number in the original equation
with its equivalent expression; an expression is formed that has a
solution 4x + 9 – 2(5x + 1) = 2x – 9. Statement (II) provides
insufficient information because there are multiple instances in
which the integer could be less than x. If x were 5, the number
would be –23. If x were 6 the number would be –29.
427. c. Statement (I) alone is insufficient because if Kline spent twice
as long talking about her boyfriend than talking about her work
and weekend combined, the amount of time she spent talking with
her boyfriend is still variable, and how long she talked about the
kitten is still unknown. The addition of statement (II) puts
everything in terms of one variable. An expression representing the
time spent talking about her boyfriend is 2(20 + w), where w = time
talked about work. Since time talked about work is the same as
time talked about the kitten, both can be represented as w. An
equation can be set up representing the situation: (20) + (40 + 2w)
+ (w) + (w) = 100, since each is a percent. The simplified expression
is 60 + 4w = 100, and when 60 is subtracted from both sides, the
result is 4w = 40, which when divided by 4 results in w = 10. With
this information, you can tell that 10% of the time Kline talked
about her kitten, and 10% of 3 hours is 18 minutes.
501_10_441-472_501_master.qxd 4/29/13 12:41 PM Page 472
Blank Page
501_11_473-534_501_master.qxd 4/29/13 12:42 PM Page 473
11
Geometry
Geometry questions account for about 25% of the GMAT quan-
titative section. The key to tackling geometry questions is to know the
properties of different shapes and several key formulas, which are reviewed
in this chapter, including:
angles
lines
triangles
quadrilaterals
circles
coordinate geometry
Classifying Angles
In geometry, an angle is formed by two rays with a common endpoint. The
symbol used to indicate an angle is . The two rays are the sides of the
angle. The common endpoint is the vertex of the angle.
501_11_473-534_501_master.qxd 4/29/13 12:42 PM Page 474
Angles that make a square corner are called right angles (see the follow-
ing examples for more details about what makes an angle a right angle). In
drawings, the following symbol is used to indicate a right angle:
Opposite rays are two rays with the same endpoint that form a line. They
form a straight angle. A straight angle has a 180° measure.
D
H
S
An acute angle has a measure between 0° and 90°. Here are two examples
of acute angles.
45
89
A right angle has a 90° measure. The corner of a piece of paper will fit
exactly into a right angle. Here are two examples of right angles.
474
501 GMAT
®
Questions
501_11_473-534_501_master.qxd 4/29/13 12:42 PM Page 475
475
An obtuse angle has a measure between 90° and 180°. Here are two exam-
ples of obtuse angles.
91
170
501 GMAT
®
Questions
Set 33
Use the figure to answer the following practice problems.
J
K
N O
L
M
428. Name three acute angles.
429. Name three obtuse angles.
501_11_473-534_501_master.qxd 4/29/13 12:42 PM Page 476
476
501 GMAT
®
Questions
430. Name two straight angles.
431. Determine the largest angle shown, and write it two ways.
432. If MON measures 27°, then JOK measures ________ degrees.
Congruent Angles and Angle Pairs
When two angles have the same degree measure, they are said to be
congruent.
Names are given to three special angle pairs, based on their relationship
to each other:
Complementary angles: two angles whose sum is 90°.
Supplementary angles: two angles whose sum is 180°.
Vertical angles: two angles that are opposite each other when
two lines cross.
When two lines cross, the adjacent angles are supplementary, and the
sum of all four angles is 360°.
Perpendicular and Parallel Lines
Perpendicular lines are another type of intersecting lines. Perpendicular lines
meet to form right angles. Right angles always measure 90°. In the follow-
ing figure, lines x and y are perpendicular:
y
x
501_11_473-534_501_master.qxd 4/29/13 12:42 PM Page 477
477
Parallel lines lie in the same plane and don’t cross at any point.
l
m
The arrowheads on the lines indicate that they are parallel.
Angle–Pair Problems
Angle–pair problems tend to ask for an angle’s complement or supplement.
Example: If mA = 35°, what is the size of its complement?
To find an angle’s complement, subtract it from 90°:
90° 35° = 55°
Example: If mA = 35°, what is the size of its supplement?
To find an angle’s supplement, subtract it from 180°:
180° 35° = 145°
Set 34
Answer the following problems.
433. What is the complement of a 30° angle?
434. What is the complement of a 5° angle?
501 GMAT
®
Questions
501_11_473-534_501_master.qxd 4/29/13 12:42 PM Page 478
478
501 GMAT
®
Questions
435. What is the supplement of a 5° angle?
436. What is the supplement of a 100° angle?
Triangles
You can classify triangles by the lengths of their sides. Below are three
examples of special triangles called equilateral, isosceles, and scalene triangles.
501_11_473-534_501_master.qxd 4/29/13 12:42 PM Page 479
479
Yo u c a n a l s o c l a s s i f y t r i a n g l e s b y t h e m e a s u r e m e n t s o f t h e i r a n g l e s . H e r e a r e
four examples of special triangles. They are called acute, equiangular, right,
and obtuse triangles.
6
6
5
501 GMAT
®
Questions
501_11_473-534_501_master.qxd 4/29/13 12:42 PM Page 480
480
501 GMAT
®
Questions
Set 35
Classify each triangle described as equilateral, isosceles, or scalene.
437. DABC with AB = 10, BC = 10, and AC = 8.
438. DDEF with DE = 6, EF = 8, and DF = 10.
439. DXYZ with XY = 7, YZ = 7, and XZ = 7.
Classify each triangle described as acute, right, obtuse, or equiangular.
440. DMNO with mM = 130°, mN = 30°, and mO = 20°.
441. DRST with mR = 80°, mS = 45°, and mT = 55°.
442. DGHI with mG = 20°, mH = 70°, and mI = 90°.
Area of a Triangle
To find the area of a triangle, use this formula:
area =
__
1
2
(base ¥ height)
Although any side of a triangle may be called its base, it’s often easiest to use
the side on the bottom. To use another side, rotate the page and view the
triangle from another perspective.
A triangle’s height (or altitude) is represented by a perpendicular line
drawn from the angle opposite the base to the base. Depending on the tri-
angle, the height may be inside, outside, or on the legs of the triangle.
Triangle Rules
Rule 1. The sum of the angles in a triangle is 180°:
A + B + C = 180°
501_11_473-534_501_master.qxd 4/29/13 12:42 PM Page 481
481
501 GMAT
®
Questions
Example: One base angle of an isosceles triangle is 30°. Find the
measure of the vertex angle.
Draw a picture of an isosceles triangle. Drawing it to scale helps. Since it
is an isosceles triangle, draw both base angles the same size (as close to 30°
as you can) and make sure the sides opposite them are the same length.
Label one base angle as 30°.
30˚
30˚
Since the base angles are congruent, label the other base angle as 30°.
There are two steps needed to find the vertex angle:
Add the two base angles together: 30° + 30° = 60°
The sum of all three angles in a triangle is 180°.
To find the measure of the vertex angle, subtract the sum of the two base
angles (60°) from 180°:
180° 60° = 120°
Thus, the measure of the vertex angle is 120°.
Add all 3 angles together to make sure their sum is 180°:
Rule 2. The longest side of a triangle is opposite the largest angle.
This rule implies that the second longest side is opposite the second
largest angle, and the shortest side is opposite the smallest angle.
largest angle
shortest
side
smallest
angle
longest side
501_11_473-534_501_master.qxd 4/29/13 12:42 PM Page 482
482
501 GMAT
®
Questions
Example: In the following triangle, which side is the shortest?
slant
height
height
base
Determine the size of A, the missing angle, by adding the two known
angles and then subtracting their sum from 180°:
90° + 46° = 136°
180° 136° = 44°
Thus A is 44°.
Since A is the smallest angle, side a (opposite A) is the shortest side.
Rule 3. Right triangles have a rule of their own. Using the Pythagorean the-
orem, you can calculate the missing side of a right triangle.
a c
b
a² + b² = c²
(c refers to the hypotenuse)
501_11_473-534_501_master.qxd 4/29/13 12:42 PM Page 483
483
501 GMAT
®
Questions
Example: What is the perimeter of the following triangle?
3 5
Since the perimeter is the sum of the lengths of the sides, we must first find
the missing side. Use the Pythagorean Theorem:
a² + b² = c²
Substitute the given sides for two of the letters. To solve this equation, sub-
tract 9 from both sides:
3
2
+ b
2
= 5
2
9 + b
2
= 25
9 – b
2
= 25 –9
b
2
= 16
Then take the square root of both sides.
b
2
= 16
b = 4
Thus, the missing side has a length of 4 units.
Adding the three sides yields a perimeter of 12:
3 + 4 + 5 = 12
501_11_473-534_501_master.qxd 4/29/13 12:42 PM Page 484
Quadrilaterals
A quadrilateral is four–sided polygon. Three common quadrilaterals are
shown here:
12
Rectangle
4 4
12
4
Square
4 4
4
12
Parallelogram
6 6
12
These quadrilaterals have something in common besides having four
sides:
Opposite sides are the same length and parallel.
Opposite angles are the same size.
484
501 GMAT
®
Questions
501_11_473-534_501_master.qxd 4/29/13 12:42 PM Page 485
501 GMAT
®
Questions
However, each quadrilateral has its own distinguishing characteristics as
given on the table that follows.
Sides Rectangle Square Parallelogram
Angles The horizontal
sides don’t have
to be the same
length as the
vertical sides. All
the angles are
right angles.
All four sides are
the same length.
All the angles
are right angles.
The horizontal
sides don’t have
to be the same
length as the
vertical sides.
The opposite
angles are the
same size, but
they don’t have
to be right
angles. (A paral-
lelogram can
look like a rec-
tangle leaning to
one side.)
The naming conventions for quadrilaterals are similar to those for
triangles:
D
B
A
C
The figure is named by the letters at its four corners, usually in
alphabetical order: rectangle ABCD.
A side is named by the letters at its ends: side AB.
An angle is named by its vertex letter: A.
485
501_11_473-534_501_master.qxd 4/29/13 12:42 PM Page 486
The sum of the angles of a quadrilateral is 360°:
A + B + C + D = 360°
To find the perimeter of a quadrilateral, follow this simple rule:
Perimeter = sum of all four sides
To find the area of a rectangle, square, or parallelogram, use this formula:
Area = base ¥ height
The base is the size of the side on the bottom. The height (or altitude) is the
length of a perpendicular line drawn from the base to the side opposite it.
The height of a rectangle and a square is the same as the length of its ver-
tical side.
Rectangle
height
base
Square
height
base
486
501 GMAT
®
Questions
A parallelogram’s height is not necessarily the same as the length of its ver-
tical side (called the slant height); it is found instead by drawing a perpendi-
cular line from the base to the side opposite it—the length of this line equals
the height of the parallelogram.
501_11_473-534_501_master.qxd 4/29/13 12:42 PM Page 487
487
slant
height
height
base
501 GMAT
®
Questions
The area formula for the rectangle and square may be expressed in an
equivalent form as:
Area = length ¥ width
Example: Find the area of a rectangle with a base of 4 meters and a
height of 3 meters.
Draw the rectangle as close to scale as possible. Label the size of the base
and height.
3
4
Write the area formula, then substitute the base and height numbers into
it:
A = b ¥ h
A = 4 ¥ 3 = 12
Thus, the area is 12 square meters.
Set 36
Answer the following practice problems.
443. All squares are also rectangles.
a. True
b. False
501_11_473-534_501_master.qxd 4/29/13 12:42 PM Page 488
488
501 GMAT
®
Questions
444.
All rectangles are also squares.
a. True
b. False
445. Some rectangles are also squares.
a. True
b. False
446. All squares and rectangles are also parallelograms.
a. True
b. False
447. What is the length of a side of a square room whose perimeter is
58 feet?
448. Find the dimensions of a rectangle with a perimeter of 16 feet
whose long side is 3 times the length of its short side.
449. What is the length in feet of a parking lot that has an area of 8,400
square feet and a width of 70 feet?
Circles
A circle is a set of points that are all the same distance from a given point
called the center.
center
Yo u ar e l ik el y t o c o me a cr os s t h e fo ll ow in g t e rm s w he n de a li ng w it h
circles:
Radius: The distance from the center of the circle to any point on the cir-
cle itself. The symbol r is used for the radius.
501_11_473-534_501_master.qxd 4/29/13 12:42 PM Page 489
489
501 GMAT
®
Questions
Diameter: The length of a line that passes across a circle through the cen-
ter. The diameter is twice the size of the radius. The symbol d is used for
the diameter.
The circumference of a circle is the distance around the circle (compara-
ble to the concept of the perimeter of a polygon). To determine the cir-
cumference of a circle, use either of these two equivalent formulas:
Circumference = 2pr
or
Circumference = pd
The formulas should be written out as (2 ¥p¥r) or (d). It helps to
know that:
r is the radius
d is the diameter
22
p is approximately equal to 3.14, or
__
7
Example: Find the circumference of a circle whose radius is 7 inches.
Draw this circle and write the radius version of the circumference formula
(because you’re given the radius):
7 in.
Substitute 7 for the radius:
C = 2pr
C = 2 ¥p¥7
C = 2 ¥p¥7; C = 14p
Example: What is the diameter of a circle whose circumference is
62.8 centimeters? Use 3.14 for p.
501_11_473-534_501_master.qxd 4/29/13 12:42 PM Page 490
Draw a circle with its diameter and write the diameter version of the cir-
cumference formula (because you’re asked to find the diameter):
C = πd
490
501 GMAT
®
Questions
Substitute 62.8 for the circumference, 3.14 for p, and solve the equation.
62.8 = 3.14 ¥ d
62.8 = 3.14 ¥ 20
The diameter is 20 centimeters.
The area of a circle is the space its surface occupies. To determine the
area of a circle, use this formula:
Area = pr²
The formula can be written out as r ¥ r.
Example: Find the area of the following circle, rounded to the nearest
tenth.
2.3 in.
Write the area formula:
A = pr²
Substitute 2.3 for the radius:
A = 2.3 ¥ 2.3
A = 5.3p
501_11_473-534_501_master.qxd 4/29/13 12:42 PM Page 491
491
Example: What is the diameter of a circle whose area is 9 square cen-
timeters?
501 GMAT
®
Questions
Draw a circle with its diameter (to help you remember that the question
asks for the diameter), then write the area formula. (Note: Refer to Chap-
ter 10 if you need help with square roots.)
A = pr²
Substitute 9p for the area and solve the equation:
9p = pr²
9 = r²
3 = r
Since the radius is 3 centimeters, the diameter is 6 centimeters.
Set 37
Find the approximate circumference of each circle shown or described. Use
3.14 for p.
450.
15 ft.
501_11_473-534_501_master.qxd 4/29/13 12:42 PM Page 492
492
501 GMAT
®
Questions
451.
11 in.
452.
d = 7 m
453. r = 25 m
Find the approximate area for each circle shown or described. Use 3.14
for p.
454.
10 ft.
455.
8 m
456. r = 13 cm
457. d = 24 in.
Surface Area of a Cylinder
When you are looking for the surface area of a cylinder, you need to find
the area of two circles (the bases) and the area of the curved surface that
makes up the side of the cylinder. The area of the curved surface is hard to
visualize when it is rolled up. Picture a paper towel roll. It has a circular top
and bottom. When you unroll a sheet of the paper towel, it is shaped like
a rectangle. The area of the curved surface is the area of a rectangle with the
same height as the cylinder, and the base measurement is the same as the
circumference of the circle base.
501_11_473-534_501_master.qxd 4/29/13 12:42 PM Page 493
493
501 GMAT
®
Questions
Surface area of a cylinder = area of two circles + area of rectangle
= 2pr
2
+ bh
= 2pr
2
+ 2prh
Volume of a Cylinder
You can find the volume of a cylinder by finding the product of the area of
the base and the height of the figure. Of course, the base of a cylinder is a
circle, so you need to find the area of a circle times the height.
Theorem: The volume (V ) of a cylinder is the product of the area of the
base (B) and the height (h).
V = Bh or V = pr
2
h
Volume of a Cone
A cone relates to a cylinder in the same way that a pyramid relates to a
prism. If you have a cone and a cylinder with the same radius and height,
it would take three of the cones to fill the cylinder. In other words, the cone
holds one-third the amount of the cylinder.
V = Bh or V = pr
2
h
V =
__
1
Bh or V =
__
1
pr
2
3
3
h
Coordinate Geometry
The two axes divide the coordinate plane into four regions, which are called
quadrants. The quadrants are numbered counterclockwise beginning with
the upper-right region. The coordinates (x,y) of a point are an ordered pair
of numbers. The first number is the x coordinate. The second number is
the y coordinate. The coordinates of the origin are (0,0).
Each point on the coordinate plane has its own unique ordered pair. You
can think of an ordered pair as an address. Now that you have located a
point, you can also find the coordinates of a point on a graph.
501_11_473-534_501_master.qxd 4/29/13 12:42 PM Page 494
494
501 GMAT
®
Questions
Example: Find the coordinates of each point.
y
C
A
B
x
F
D
E
Solution:
A(2,2)
B(–3,1)
C(0,4)
D(–4,–2)
E(2,–3)
F(5,0)
The distance d between any two points A(x
1
,y
1
) and B(x
2
,y
2
) is
d =
( x
2 2
2
x
1
) + ( y
2
y
1
)
The slope of a line is the measure of its steepness. The slope of a line is
determined by the ratio of its rise to run. When looking at a line on a coor-
dinate grid, always count the run before you count the rise. When a line
points up to the right, it has a positive slope. A line with a negative slope
points up to the left.
Yo u ca n a l s o u s e a f o rm u l a t o d e t e r m i n e t h e s l o p e o f a l i ne c o n t a i n i n g t w o
points, point A(x
1
,y
1
) and point B(x
2
,y
2
). Here is the formula:
y
slope =
2
y
______
1
x
2
x
1
501_11_473-534_501_master.qxd 4/29/13 12:42 PM Page 495
495
Set 38
Use a formula to find the slope of the line through each pair of points.
458. (0,0) and (5,6)
459. (–3,–2) and (–4,–3)
460. (1,–3) and (–1,–3)
461. (5,–6) and (–2,8)
Set 39
Now it is time to answer GMAT data sufficiency practice questions that
have been designed to test your algebra skills. Good luck!
462. The area of a quadrilateral is 81 – 25x
2
; what is the value of x?
II. Its base is 9 – 5x long.
II. The shape is a square.
a. Statement (I) ALONE is sufficient, but statement (II) ALONE
is not sufficient.
b. Statement (II) ALONE is sufficient, but statement (I) ALONE
is not sufficient.
c. BOTH statements TOGETHER are sufficient, but NEITHER
statement ALONE is sufficient.
d. EACH statement ALONE is sufficient.
e. Statements (I) and (II) TOGETHER are NOT sufficient.
501 GMAT
®
Questions
501_11_473-534_501_master.qxd 4/29/13 12:42 PM Page 496
463. Is BD a perpendicular bisector of AC?
II. Angle AC = 180°
II. AB = BC
A B C
D
a. Statement (I) ALONE is sufficient, but statement (II) ALONE
is not sufficient.
b. Statement (II) ALONE is sufficient, but statement (I) ALONE
is not sufficient.
c. BOTH statements TOGETHER are sufficient, but NEITHER
statement ALONE is sufficient.
d. EACH statement ALONE is sufficient.
e. Statements (I) and (II) TOGETHER are NOT sufficient.
496
501 GMAT
®
Questions
501_11_473-534_501_master.qxd 4/29/13 12:42 PM Page 497
497
464. Are angle A and angle B supplementary?
II. Angle A – angle B = 10°
II. LM || ND
L
C
B
M
N
A
O
a. Statement (I) ALONE is sufficient, but statement (II) ALONE
is not sufficient.
b. Statement (II) ALONE is sufficient, but statement (I) ALONE
is not sufficient.
c. BOTH statements TOGETHER are sufficient, but NEITHER
statement ALONE is sufficient.
d. EACH statement ALONE is sufficient.
e. Statements (I) and (II) TOGETHER are NOT sufficient.
501 GMAT
®
Questions
501_11_473-534_501_master.qxd 4/29/13 12:42 PM Page 498
498
501 GMAT
®
Questions
465.
Is the circumference of circle B > 10?
II. Radius of the circle A = 9
II. XY = 2YZ
A
B
X
Y
Z
a. Statement (I) ALONE is sufficient, but statement (II) ALONE
is not sufficient.
b. Statement (II) ALONE is sufficient, but statement (I) ALONE
is not sufficient.
c. BOTH statements TOGETHER are sufficient, but NEITHER
statement ALONE is sufficient.
d. EACH statement ALONE is sufficient.
e. Statements (I) and (II) TOGETHER are NOT sufficient.
501_11_473-534_501_master.qxd 4/29/13 12:42 PM Page 499
499
466. What is the measure of angle A?
II. Angle B and angle C are complementary.
II. Angle B = 30°
A
B
C
a. Statement (I) ALONE is sufficient, but statement (II) ALONE
is not sufficient.
b. Statement (II) ALONE is sufficient, but statement (I) ALONE
is not sufficient.
c. BOTH statements TOGETHER are sufficient, but NEITHER
statement ALONE is sufficient.
d. EACH statement ALONE is sufficient.
e. Statements (I) and (II) TOGETHER are NOT sufficient.
501 GMAT
®
Questions
501_11_473-534_501_master.qxd 4/29/13 12:42 PM Page 500
467. Are the areas of the rectangle and triangle the same?
II. The triangle height is twice the height of the rectangle.
II. The triangle and the rectangle have the same base.
h
h
b b
a. Statement (I) ALONE is sufficient, but statement (II) ALONE
is not sufficient.
b. Statement (II) ALONE is sufficient, but statement (I) ALONE
is not sufficient.
c. BOTH statements TOGETHER are sufficient, but NEITHER
statement ALONE is sufficient.
d. EACH statement ALONE is sufficient.
e. Statements (I) and (II) TOGETHER are NOT sufficient.
Use the following image to answer questions 468 and 469.
Y
A
B C X Z
500
501 GMAT
®
Questions
501_11_473-534_501_master.qxd 4/29/13 12:42 PM Page 501
501
501 GMAT
®
Questions
468.
Are the triangles similar?
II. Angle A = angle X
II. 3AB = XY
a. Statement (I) ALONE is sufficient, but statement (II) ALONE
is not sufficient.
b. Statement (II) ALONE is sufficient, but statement (I) ALONE
is not sufficient.
c. BOTH statements TOGETHER are sufficient, but NEITHER
statement ALONE is sufficient.
d. EACH statement ALONE is sufficient.
e. Statements (I) and (II) TOGETHER are NOT sufficient.
469. Are the triangle areas congruent?
II. They both have an inferior angle of 30°.
II. They both have two sides that are 3 units and 5 units in
length.
a. Statement (I) ALONE is sufficient, but statement (II) ALONE
is not sufficient.
b. Statement (II) ALONE is sufficient, but statement (I) ALONE
is not sufficient.
c. BOTH statements TOGETHER are sufficient, but NEITHER
statement ALONE is sufficient.
d. EACH statement ALONE is sufficient.
e. Statements (I) and (II) TOGETHER are NOT sufficient.
501_11_473-534_501_master.qxd 4/29/13 12:42 PM Page 502
502
501 GMAT
®
Questions
470.
Find the volume of both cubes.
II. a = 4
II. The volume of the small cube is 64.
2a
a
a. Statement (I) ALONE is sufficient, but statement (II) ALONE
is not sufficient.
b. Statement (II) ALONE is sufficient, but statement (I) ALONE
is not sufficient.
c. BOTH statements TOGETHER are sufficient, but NEITHER
statement ALONE is sufficient.
d. EACH statement ALONE is sufficient.
e. Statements (I) and (II) TOGETHER are NOT sufficient.
501_11_473-534_501_master.qxd 4/29/13 12:42 PM Page 503
503
471. Is BC || to DF?
II. Angle J + angle E = 180°
II. Angle H = angle I
C
H
F
J
I
B
E
D
a. Statement (I) ALONE is sufficient, but statement (II) ALONE
is not sufficient.
b. Statement (II) ALONE is sufficient, but statement (I) ALONE
is not sufficient.
c. BOTH statements TOGETHER are sufficient, but NEITHER
statement ALONE is sufficient.
d. EACH statement ALONE is sufficient.
e. Statements (I) and (II) TOGETHER are NOT sufficient.
501 GMAT
®
Questions
501_11_473-534_501_master.qxd 4/29/13 12:42 PM Page 504
472. Is triangle abc an equilateral triangle?
II. Angle a ÷ angle b = 1
II. Angle b – angle a = 0
b
a
c
a. Statement (I) ALONE is sufficient, but statement (II) ALONE
is not sufficient.
b. Statement (II) ALONE is sufficient, but statement (I) ALONE
is not sufficient.
c. BOTH statements TOGETHER are sufficient, but NEITHER
statement ALONE is sufficient.
d. EACH statement ALONE is sufficient.
e. Statements (I) and (II) TOGETHER are NOT sufficient.
473. Find the surface area of a cube.
II. The volume of the cube is 27 cm
3
.
II. The surface area is six times the area of one side.
a. Statement (I) ALONE is sufficient, but statement (II) ALONE
is not sufficient.
b. Statement (II) ALONE is sufficient, but statement (I) ALONE
is not sufficient.
c. BOTH statements TOGETHER are sufficient, but NEITHER
statement ALONE is sufficient.
d. EACH statement ALONE is sufficient.
e. Statements (I) and (II) TOGETHER are NOT sufficient.
504
501 GMAT
®
Questions
501_11_473-534_501_master.qxd 4/29/13 12:42 PM Page 505
505
474. The circumference of a circle is 2xp. What is the value of x?
II. The radius is equal to x.
II. The area of the circle is the same as double the diameter
minus 1.
a. Statement (I) ALONE is sufficient, but statement (II) ALONE
is not sufficient.
b. Statement (II) ALONE is sufficient, but statement (I) ALONE
is not sufficient.
c. BOTH statements TOGETHER are sufficient, but NEITHER
statement ALONE is sufficient.
d. EACH statement ALONE is sufficient.
e. Statements (I) and (II) TOGETHER are NOT sufficient.
Set 40
Now it is time to answer GMAT problem-solving practice questions that
have been designed to test your geometry skills. Good luck!
Solve the problem and indicate the best of the answer choices
given.
All numbers used are real numbers.
A figure accompanying a problem-solving question is intended
to provide information useful in solving the problem. Figures
are drawn as accurately as possible EXCEPT when it is stated
in a specific problem that the figure is not drawn to scale.
Straight lines may sometimes appear jagged. All figures lie in a
plane unless otherwise indicated.
475. Find the slope of the line passing through the two points (3,–5)
(–2,–5).
a. m = 2
b. m = 0
c. m = –2
d. m = 5
e. undefined slope
501 GMAT
®
Questions
501_11_473-534_501_master.qxd 4/29/13 12:42 PM Page 506
506
501 GMAT
®
Questions
476.
Find the y intercept of the equation 3x + 6y = 12
a. 4
b.
__
2
3
c. 3
d. 6
e. 2
477. What is the equation of the following graph?
y
6
5
4
3
2
1
–6 –5 –4 –3 –2 –1 0 1 2 3 4 5 6
x
–1
–2
–3
–4
–5
–6
a. y = –3x + 1
b. y = 3x + 1
c. y =
__
1
3
x + 1
d. y = –
__
1
3
x + 1
e. y = –x + 3
478. Which of the following statements about the lines 3x + 5y = 10 and
–10y = 6x + 50 is true?
a. They have the same y-intercept.
b. They have the same x-intercept.
c. They are parallel.
d. They are perpendicular.
e. The slopes are additive inverses of each other.
501_11_473-534_501_master.qxd 4/29/13 12:42 PM Page 507
507
501 GMAT
®
Questions
479.
Which inequality is represented on the graph shown here?
y
6
5
4
3
2
1
–6 –5 –4 –3 –2 –1 0 1 2 3 4 5 6
x
–1
–2
–3
–4
–5
–6
a. y > –2x + 3
b. y –2x + 3
c. y < –2x + 3
d. y <
–2x + 3
e. y < –3x + 1.5
480. Which ordered pair is a solution to the system of equations?
x + 3 y = 7
x + 2y = 10
a. (,
__
7 13
__
2 4
)
b. (,
__
7 17
__
2 5
)
c. (–2,3)
d. (16,–3)
e. (4,1)
501_11_473-534_501_master.qxd 4/29/13 12:42 PM Page 508
481.
Find the area of the rectangle.
x + 6
2x – 7
a. 2x
2
+ 19x – 42
b. 2x
2
+ 5x – 42
c. 2x
2
– 42
d. 6x – 2
e. 3x – 1
482.
If a public pool is a regular 25 m pool, with a 12 m width and an
overall depth of 8 m, how many gallons are required to fill three-
quarters of the public pool?
1L = 001 m
3
1L = 0.26 gallons
a. 624,000 gallons
b. 468,000 gallons
c. 468 gallons
d. 1,800,000 gallons
e. 2,400 gallons
483. The distance between two coordinates, X and Y, is seven units. A
third coordinate, Z, is five units away from Y. Which of the
following could be the only distance from X to Z?
III. 13
III. 6
III. 1
a. I and II only
b. II only
c. II and III only
d. III only
e. I only
501 GMAT
®
Questions
508
501_11_473-534_501_master.qxd 4/29/13 12:42 PM Page 509
484.
Find the value of x.
42
x
88
a. 46
b. 65
c. 68
d. 70
e. 72
485. If AB = 3x + 14, BC = 2x + 1 and AC = 75, what is the length of AB?
a. 12
b. 68
c. 44
d. 50
e. 13
486. In the three-dimensional coordinate plane, what is the
approximate distance between (3,2,–4) and (7,–8,2)?
a. 11.8
b. 12.3
c. 13.1
d. 15.2
e. 16.5
501 GMAT
®
Questions
509
501_11_473-534_501_master.qxd 4/29/13 12:42 PM Page 510
487. A cylindrical silo and a rectangular silo are both being filled with
400 m
3
of corn. The cylindrical silo has a base with an area of
20 m
2
. The rectangular silo has a base with an area of 10 m
2
.
Assuming that the corn will not overflow in either, which
statement will be true about the corn levels?
a. The cylindrical silo level will be 20 m higher.
b. The cylindrical silo level will be 10 m lower.
c. The rectangular silo level will be 20 m higher.
d. The rectangular silo level will be 40 m higher.
e. The rectangular silo level will be the same as the cylindrical
silo.
488. If triangle ABC is rotated 180° about point C, and then reflected
over the y-axis, what will be the coordinate of B?
5
–2
4
2
a. (–1,–2)
b. (9,1)
c. (9,–1)
d. (–1,2)
e. (–9,–1)
489.
Which list of a triangle’s side lengths is impossible?
a. 4, 7, 4
b. 2, 3, 7
c. 2, 8, 7
d. 5, 6, 7
e. 8, 5, 5
501 GMAT
®
Questions
510
501_11_473-534_501_master.qxd 4/29/13 12:42 PM Page 511
490.
If angle A and angle B are supplementary angles, and the measure
of angle A < 80°, then you can assume that the
a. measure of angle B = 100°.
b. measure of angle B = 10°.
c. measure of angle B < 80°.
d. measure of angle A > 80°.
e. angle A is an obtuse angle.
491.
Find the midpoint of segment MN if point M is (4,6) and point P
is (–2,14)
a. (2,1)
b. (2,20)
c. (6,8)
d. (1,10)
e. (10,12)
492.
What is the measure of angle x?
x
80
25
30
a. x = 124°
b. x = 15°
c. x = 35°
d. x = 30°
e. x = 115°
501 GMAT
®
Questions
511
501_11_473-534_501_master.qxd 4/29/13 12:42 PM Page 512
493. The following net represents which shape?
a. a sphere
b. a cone
c. a dome
d. a cylinder
e. a triangle
501 GMAT
®
Questions
512
501_11_473-534_501_master.qxd 4/29/13 12:42 PM Page 513
494.
Find the area of the shaded region.
10
9
3
2
a. 6
b. 28
c. 48
d. 84
e. 90
495. A spherical model of the planet Earth has a diameter of 6 inches.
The interior is a mantle surrounding a molten core. The core has a
radius of 3 inches. What is the volume of everything outside of the
core?
Volume of a sphere =
__
4
pr
3
3
a. 186p
b. 324p
c. 216p
d. 288p
e. 252p
501 GMAT
®
Questions
513
501_11_473-534_501_master.qxd 4/29/13 12:42 PM Page 514
496.
A pyramid made of rectangular prisms has layers each half the
width of the layer beneath it. The base is a square that is 8 meters
wide. Each level is a meter high. If the outside of the pyramid was
being painted (not including the bottom), what would the surface
area be of the painted part?
1
8
a. 84 m
2
b. 120 m
2
c. 208 m
2
d. 248 m
2
e. 268 m
2
497.
Which string of letters has two lines of symmetry?
a. OLLO
b. MEW
c. TAAT
d. OHO
e. IIBII
498.
If the sum of the interior angles of a triangle is 180° and the sum
of the interior angles of a quadrilateral is 360°, which equation
represents the correlation between the sum of the interior angles (s)
and the number of sides of a polygon (n)?
a. s = 180n
b. s = 180n – 180
c. s = 180(n – 2)
d. s = 90n – 90
e. s = n + 177
514
501 GMAT
®
Questions
501_11_473-534_501_master.qxd 4/29/13 12:42 PM Page 515
499.
Use the Pythagorean theorem to find x. Round your answer to the
nearest tenth.
10
6
x
a.
5
b.
6
c.
8
d.
11.7
e.
12
500.
What is the value of x?
45˚
2x + 5
a.
x = 75
b.
x = 25
c.
x = 135
d.
x = 115
e.
x = 65
515
501 GMAT
®
Questions
501_11_473-534_501_master.qxd 4/29/13 12:42 PM Page 516
501.
What is the tangent of A?
B
20
16
A C
12
a.
__
4
5
b.
__
3
5
c.
__
5
4
d.
__
3
4
e.
__
4
3
516
501 GMAT
®
Questions
501_11_473-534_501_master.qxd 4/29/13 12:42 PM Page 517
Answers
Set 33
428. KOL; JOK; NOM
429. NOK; MOJ; MOL
430. NOL; MOK
431. MOK; KOM
432. 63°. (180° – 27° – 90°)
Set 34
433. 60°. The sum of both angles must equal 90°.
434. 85°. The sum of both angles must equal 90°.
435. 175°. The sum of both angles must equal 180°.
436. 80°. The sum of both angles must equal 180°.
Set 35
437. Isosceles. There are two equal sides.
438. Scalene. No sides are equal.
439. Equilateral. All sides are equal.
440. Obtuse. One angle is greater than 90°.
441. Acute. All angles are less than 90°.
442. Right. Note the right angle.
Set 36
443. a. True. Squares are special rectangles that have four equal sides.
444. b. False. Not all rectangles have four equal sides.
501 GMAT
®
Questions
517
501_11_473-534_501_master.qxd 4/29/13 12:42 PM Page 518
__
__
6
.
6
____
–0
458.
5 5 –0
.
459. 1.
–2 – (–
_______
3)
, or
______
–2 + 3
= 1.
–3 – (–4)
–3 + 4
445.
a. True. The rectangles that are also squares are those that have
four equal sides.
446. a. True. Squares and rectangles are special parallelograms that
have four right angles.
447. 14.5 feet. Because the room is square, divide the total perimeter
by 4:
58
4
= 14.5
448. 2 feet by 6 feet. Use the formula s + s + 3s + 3s. Solve for s:
8s = 16 feet, or s = 2 feet. The length of the short side is 2 feet, so
each long side is 6 feet.
449.
120 feet. 8,400 = length ¥ 70. Solve for the length:
_____
8,400
= length ¥
70
__
70 70
, or 120 feet.
Set 37
450. 47.1 ft. You are told the diameter: 15 ¥ 3.14.
451. 69.08 in. You are told the radius: 2(11) ¥ 3.14.
452. 21.98 m. You are told the diameter: 7 ¥ 3.14.
453. 157 m. You are told the radius: 2(25) ¥ 3.14.
454. 78.5 ft.
2
. You are told the diameter, so you know the radius is 5:
5
2
¥ 3.14.
455. 200.96 m
2
. You are told the radius: 8
2
¥ 3.14.
456. 530.66 cm
2
. You are told the radius: 13
2
¥ 3.14.
457.
452.16 in.
2
. You are told the diameter, so you know the radius is
12: 12
2
¥
3.14.
Set 38
501 GMAT
®
Questions
518
501_11_473-534_501_master.qxd 4/29/13 12:42 PM Page 519
460.
0. 3 – (–3) would result in a zero in the numerator.
461. –2.
8 – (–
______
6)
, or
14
__
= –2.
–2 –5
–7
Set 39
462. c. If the area of the quadrilateral is 81 – 25x
2
and the value of the
base is 9 – 5x, we can divide to find that the height of the
quadrilateral is 9 + 5x. We need to know it is a square because then
we can set the base equal to the height and solve for x. In the
equation 9 – 5x = 9 + 5x, isolate variables and constants by adding
5x to both sides and subtracting 9 from both sides, yielding
0 = 10x. Divide both sides by 10 to get x = 0.
463. e. In order to determine whether line BD and line AC are
perpendicular, you would need to verify that the angle between
them is 90˚. Neither statement gives enough information to
determine the angle between the lines. The first statement only
tells you that line AC is a straight line, and statement (2) tells you
that the lines are the same length.
464. b. The only piece of information you need to know in order to
determine whether angle A and angle B are supplementary, or
when added together are equal to 180°, is whether the two lines
they intersect are parallel, as stated in the second statement. When
given two parallel lines, the alternate interior angles of the
transverse line are always supplementary.
465. c. Statement (1) indicates that the radius of circle A is 9, which
means the circumference of the big circle is 18p or roughly 56.5,
which is insufficient to tell determine whether the radius of circle
B is bigger or smaller than 10. Statement (2) indicates that if
XY = 2YZ, then XZ = 3YZ. The radius of A can be set equal to
3YZ to find the length of YZ. 9 = 3YZ; divide both sides by 3 to get
3 = YZ. This tells us that the circumference of circle B is 6p or
approximately 18.8, which means yes, it is greater than 10.
501 GMAT
®
Questions
519
501_11_473-534_501_master.qxd 4/29/13 12:42 PM Page 520
__
__
466.
c. Inside a triangle, all angles must add up to 180°.
467.
468.
469.
470.
Complementary angles add up to 90°. To determine the value of
angle A, both statement (1) and statement (2) are needed together.
Statement (1) gives the relationship between angles B and C, yet
this statement alone does not let you determine the measure of
angle A. When combined with statement (2), all three angles can
be determined. If angle B is 30°, then angle C must be 60°
(90 – 30), and since the measures of all angles together must equal
180°, angle A is 90°.
c. Area of a rectangle = length ¥ height. Area of a triangle =
1
base ¥ height. To determine whether the area of the rectangle
2
and triangle are the same, you would need both statement (1) and
statement (2). If the triangle height is twice the height of the
rectangle and the bases/lengths are the same, you can set the
equations equal to each other: length ¥ height =
2
1
length ¥
2 height.
e. Statements (1) and (2) together are insufficient because the only
way to prove triangle similarity is to have 2 congruent
corresponding angles. Since these two only have one set of
corresponding angles, they could be 2 right triangles, one with a
side triple the other and 2 other angles completely different.
e. In order to prove triangle congruence, you need to prove that
they share two of the same corresponding sides and an angle that is
in the same position on both of them. The information in
statements (1) and (2) does not state that the sides and angle are in
corresponding locations, so congruency cannot be determined.
d. Because of the direct relationship between the two cubes, either
statement is enough information to calculate the volume of both
cubes. Statement (1) gives the value of variable a, which can be
used for both cubes. Statement (2) gives you the volume of the
smaller cube, and if you work backward you can find the volume
of a.
a
3
= 64
a = 4
Then from this you can find the volume of the second cube.
501 GMAT
®
Questions
520
501_11_473-534_501_master.qxd 4/29/13 12:42 PM Page 521
471.
d. Statement (I) is sufficient because angle J is part of a straight
angle with an interior angle on the parallel transversal. Since the
supplements of J and E are equal, by the converse of the parallel
line transversal theorem (if alternate interior angles of a transversal
are congruent then they are transversing parallel lines) BC is
parallel to DF. Statement (II) is sufficient because H is opposite to
a straight angle and once again by the converse of the parallel line
transversal theorem BC is parallel to DF.
472. e. Statement (I) only implies that angle a equals angle b. This
could make the triangle an isosceles triangle. Statement (II) implies
that if the difference between angle a and angle b is 0, they have
the same measure, which still could possibly be an isosceles
triangle.
473. a. To find the surface area of the cube, you would need to find the
area of each side using the equation area = length ¥ width. Since
this is a cube, you can change the equation to area = length
2
.
Statement (I) gives you the volume, and if you work backward you
will be able to find the length of one side. Volume of a cube =
length
3
.
27 = length
3
3 = length
Substitute this into the area equation and you will be able to solve
for the area of each side and find the total surface area. Statement
(II) does not give enough information to determine the length of
one side needed to find the surface area.
474. b. Given that the circumference of this circle is 2xp, and knowing
that the circumference of a circle is 2pr, then you know that x = r.
This is stated in the first statement, but that does not give you
enough information to determine the value for x. Statement (I) is
insufficient. Statement (II) gives you enough information to find
the value of x, as you should know the equation for both the
circumference and area of a circle.
501 GMAT
®
Questions
521
501_11_473-534_501_master.qxd 4/29/13 12:42 PM Page 522
Set 40
475. b. Substitute the points in the form (x
1
,y
1
) and (x
2
,y
2
) into the
–5 – (–
_______
5)
slope formula. This yields
__
0
3 – (–2)
, which simplifies to
5
, which
equals 0. The incorrect choices contain operation errors and sign
changes.
476. e. To find the y intercept of the line, substitute the value 0 for x.
This resulting equation will be 3(0) + 6y = 12. Simplifying this
makes 6y = 12. When both sides are divided by 2, the result is
y = 2. Choices a, b, c, and d substitute 0 for y or have basic
operation errors.
477. d. The line crosses the y-axis at the point (0,1), which means the
y-intercept (b) is 1. The change in y over the change in x (slope
or m) is –
__
1
3
. Replacing these values in the slope-intercept form of
a line yields the answer y = –
__
1
3
x + 1. The incorrect choices contain
switched slopes and wrong signs for slopes.
478. c. In order to tell which statement is true, the equations must be
put in slope-intercept form to check the y-intercept and slopes.
Doing this results in the equations y = –
__
3
5
x + 3 and y = –
__
3
5
x – 5
respectively. These equations have the same slope, so they will be
parallel. The other choices are incorrect because they don’t have
opposite reciprocal slopes; therefore, they aren’t perpendicular.
The y-intercepts in the equations are 3 and –5; they aren’t the
same, and if you replace y with 0 and solve for x in each equation,
they do not share the same x-intercept.
479. c. Since the area below the line is shaded, the y will be greater
than the other side of the equation. In addition, the line of the
equation is a dashed line, so it will not be greater than or equal to
and will not include the solutions on that line. The incorrect
choices are all variations of reversed inequalities.
501 GMAT
®
Questions
522
501_11_473-534_501_master.qxd 4/29/13 12:43 PM Page 523
480. d. Using the method of linear combination or elimination,
multiply the second equation by –1. Then combine the two
equations x + 3y = 7 and –x – 2y = –10. This results is y = –3.
Substitute the value of y into either of the original equations to
obtain the x-value. x + 3(–3) = 7 when simplified yields x – 9 = 7.
Add 9 to both sides to find that x =16. The ordered pair (16,–3)
reflects the answers y = –3 and x = 16. The incorrect choices are all
the result of addition error or leaving out a negative.
481.
b. Area is equal to the base times the height. Multiply the terms in
each binomial by the terms in the other binomial: x(2x), x(–7),
6(2x) and 6(–7). This yields the expression 2x
2
– 7x + 12x – 42.
Combine terms to get 2x
2
+ 5x – 42. Wrong answers were obtained
through common calculation errors (choice a), not multiplying all
terms (choices c and d), and finding the perimeter (choice e).
482.
b. To start this problem you have to determine the volume of the
pool when it is three-quarters full.
V = l ¥ w ¥ h
V = 25 m ¥
3
12 m ¥
__
4
(8 m)
V = 1800 m
3
Then you need to convert the cubic meters into liters, and then
gallons.
________
1 liter
0.26 gallons
1800 m
3
¥
__________
¥
0.001 m
3
1 liter
= 468,000 gallons
Choice a uses 8 m to find volume, resulting in the overall depth,
not three-quarters. Choices c and d are conversion errors. Choice
c omits the conversion to liters and choice d omits the conversion
to gallons.
483.
b. If X and Y are seven units apart and Z is five units from Y, the
absolute furthest distance between X and Z would be 12 (5 + 7),
and that would only be if angle XYZ was 180°, or they were all
along a straight line. The absolute shortest distance between X and
Z would be 2 (7 – 5), and that would only be if angle XYZ was 0°,
and Z was a point on the line between X and Y. All the other
numbers are not possible.
501 GMAT
®
Questions
523
501_11_473-534_501_master.qxd 4/29/13 12:43 PM Page 524
484.
b. Obtain the length of x by taking the length of each segment,
adding them and dividing by 2 to find the mean value of both
_______
42 + 88 130
segments.
___
2
=
2
= 65. Incorrect answers are incorrect
estimates and the result of subtracting the sides.
485.
d. To find the length of AB, simply set up the equation AB + BC =
AC. Substituting in values you get 3x + 14 + 2x + 1 = 75, then solve
for x. When like terms are combined, the equation becomes
5x + 15 = 75. Using the additive inverse, 5x + 15 – 15 = 75 – 15
yields 5x = 60. Divide both sides by 5 to get x = 12. Substitute the
value of x into the expression for AB: 3(12) + 14. 36 + 14 = 50,
which is the length of AB. Incorrect choices include the value of x,
the length of BC, and an answer with an incorrect operational
error.
486.
a. In order to find the distance between the points, use the
distance formula and take the square root of (x y
2
1
+ x
2
)
2
+ (
1
+ y
2
) +
(z
2
1
+ z
2
) . Replacing the values of x, y, and z with the coordinate
components yields the square root of (7 + 3)
2
+ (–8 + 2)
2
+ (2 – 4)
2
.
Simplifying yields the square root of 10
2
+ (–6)
2
+ (–2)
2
, which
equals the square root of 100 + 36 + 4, which is the square root of
140, which is 11.8. Choices b, c, and d have operational errors
used in the distance formula.
487.
c. The height of each corn level in both silos can be obtained by
using volume = that silo’s base area times its height. The
rectangular silo has a base of 10 cm
2
, so 400 = 10h, which means its
height is 40. The cylindrical silo’s height can be found the same
way, 400 = 20h, which means its height is 20. Hence, the
rectangular silo will be 20 m higher than the cylindrical silo. The
incorrect choices include the difference between the bases, the
height of only one silo, or switching the heights.
488.
e. When triangle ABC is rotated about point C, the new
coordinate of B will become (9,–1). When (9,–1) is reflected over
the y-axis, it will become (–9,–1). The other choices have reflected
over the wrong axis or have the wrong sign.
501 GMAT
®
Questions
524
501_11_473-534_501_master.qxd 4/29/13 12:43 PM Page 525
489. b. The triangle with sides 2, 3, and 7 is impossible to form because
the sides 2 and 3 do not have lengths that could add to more than
the third side. All three sides of the triangle would not connect.
Each other triangle has two sides that add up to more than the
third.
490.
d. If angle A and angle B are supplementary, that means that
A + B = 180. So if angle A is less than 80°, in order to add up to
180 angle B must be greater than 80°. The incorrect choices
include mixups with complementary instead of supplementary,
assuming that A = 80°, and greater than and less than mixups.
491.
d. To find the midpoint of segment MN, one can average each
_______
4 + (–2)
component of the points. Average the x-values
2
and get x = 1.
______
6 + 14
Average the y-components
2
and get y = 10. Together these
yield the coordinate (1,10). The other choices are incorrect
because of operational errors an adding the coordinates instead of
averaging them.
492. b. Find the two other bottom angles in order to find x. The
middle triangle’s interior angles will all add to 180, so find the
missing angle by using the expression 180 – 80 – 25. The bottom
angle will be 75°. Since 30 and 75 and the third angle on the
bottom of the figure make up a straight angle, this is 180°.
Subtract again to find the third angle. 180 – 75 – 30 yields 75.
Two of the angles in the triangle on the left are 75 and 90 degrees.
180 – 75 – 90 gives the value of x, which is 15. The incorrect
choices include other angles in the figure or angles that were
yielded from operational errors.
493. b. When constructed, the circular part of the net becomes the base
of the shape and the round part on the bottom wraps around the
edge of the circle to create the point for the cone.
501 GMAT
®
Questions
525
501_11_473-534_501_master.qxd 4/29/13 12:43 PM Page 526
494.
d. To find the area of the shaded region, the area of the inner
rectangle must be subtracted from the area of the outer rectangle.
The equation is area = length ¥ height, so the area of the outer
rectangle is (10 ¥ 9) = 90, and the area of the inner rectangle is
(2 ¥ 3) = 6. Then, to find only the shaded area subtract 90 – 6 and
the correct answer is 84. Choices a and e represent the area of
either rectangle if it is found alone.
495. e. To find the volume of everything outside the core in the model
of Earth, the volume of the core must be subtracted from the
volume of the total Earth. Find the volume of each by substituting
radius values into the volume formula. The volume of Earth is
__
4
6
3
= 288p. The volume of the core is
__
4
3 3
3
3
= 36p.
288p – 36p = 252p.
496. b. Find the area of each exposed surface face. All the upward-facing
squares will have a combined surface area of a large 8 ¥ 8 square,
so the upward-facing area is 8(8) = 64. The side wall for the base is
8 ¥ 1 and there are four of them, so 8(1)(4). Add this to the side
areas for each level to receive the expression 8(1)(4) + 4(1)(4) +
2(1)(4) = 32 + 16 + 8 = 56. Add the side area (56) with the upward
area (64) to get the total surface area 120 m
2
. The other choices
added extra sides or the bottoms of each shape not visible on the
surface.
497. d. The string of letters OHO is the only choice that has two lines
of symmetry, one vertical that divides it in half in the center of the
H and a horizontal axis that reflects the top to the bottom and runs
through the middle of the letters. Other choices only have one or
no lines of symmetry.
526
501 GMAT
®
Questions
501_11_473-534_501_master.qxd 4/29/13 12:43 PM Page 527
498.
c. A triangle is a 3-sided polygon, and a quadrilateral is a 4-sided
polygon. The difference between the sums of the interior angles of
these shapes is 360° – 180° = 180°. A triangle is the polygon with
the fewest number of sides (there cannot be a 1- or 2-sided
polygon), so it can be reasoned that for each side added to a
polygon above 3, the sum of the interior angles of that polygon
increases by 180°. The equation for the sum of the interior angles
of a polygon must hold true for all polygons, including triangles,
so the equation must subtract the number of sides of the polygon
by 2 before multiplying by 180°. This yields the equation
s = 180(
n – 2), which holds true for the given sums of interior
angles for triangles and quadrilaterals. When the given values are
substituted into the other equations, they are not solutions.
499.
c. When the values are plugged into the Pythagorean theorem,
a
2
+ b
2
= c
2
yields x
2
+ 6
2
= 10
2
. When simplified, it yields x
2
+ 36 =
100. Subtract 36 from both sides to get x
2
= 64. Take the square
root of both sides to find that x = 8. The other choices either
replaced the wrong values in the Pythagorean theorem or are
approximate lengths.
500.
e. Since the bases of a trapezoid are parallel, it can be assumed
that both bottom angles are equal and both top angles are equal,
and since the sum of all interior angles of a quadrilateral are equal
to 360°, the equation can be formed as 2(45) + 2(2x + 5) = 360.
Simplifying yields 90 + 4x + 10 = 360, which simplifies to 4
x + 100
= 360. Subtracting 100 from both sides yields 4x = 260, and when
both sides are divided by 4, x = 65. The other choices assumed that
45 = 2x + 5 or included common operational errors.
501.
e. The tangent of an angle in a right triangle is equal to the
opposite side of that angle divided by the adjacent side to that
angle. The opposite side is 16 and the adjacent is 12. Reduce the
fraction
16
__
4
12
to get
__
3
. The wrong choices have other answers mixed
up with ratios of the adjacent side, the hypotenuse, and the
opposite side.
501 GMAT
®
Questions
527
501_11_473-534_501_master.qxd 4/29/13 12:43 PM Page 528
Blank Page
501_11_473-534_501_master.qxd 4/29/13 12:43 PM Page 531
Notes
501_11_473-534_501_master.qxd 4/29/13 12:43 PM Page 532
Notes
501_11_473-534_501_master.qxd 4/29/13 12:43 PM Page 533
Notes
501_11_473-534_501_master.qxd 4/29/13 12:43 PM Page 534
Notes